JAC Class 12 History Important Questions Chapter 5 यात्रियों के नज़रिए : समाज के बारे में उनकी समझ

Jharkhand Board JAC Class 12 History Important Questions Chapter 5 विचारक, विश्वास और इमारतें : सांस्कृतिक विकास Important Questions and Answers.

JAC Board Class 12 History Important Questions Chapter 5 यात्रियों के नज़रिए : समाज के बारे में उनकी समझ

बहुविकल्पीय प्रश्न (Multiple Choice Questions)

1. इनबतूता का जन्म हुआ था-
(अ) भारत
(ब) ओमान
(स) तुर्की
(द) तैंजियर
उत्तर:
(द) तैंजियर

2. इलबतूता अनेक देशों की यात्रा करने के बाद स्वदेश वापस पहुँचा-
(अ) 1333 ई.
(ब) 1432 ई.
(स) 1354 ई.
(द) 1454 ई.
उत्तर:
(स) 1354 ई.

3. ‘रिहला’ का लेखक कौन था?
(अ) अल-बिरुनी
(ब) हसननिजामी
(स) फिरदौसी
(द) इलबतूता
उत्तर:
(द) इलबतूता

JAC Class 12 History Important Questions Chapter 5 यात्रियों के नज़रिए : समाज के बारे में उनकी समझ

4. फ्रांस्वा बर्नियर कहाँ का निवासी था?
(अ) ब्रिटेन
(ब) फ्रांस
(स) जर्मनी
(द) इटली
उत्तर:
(ब) फ्रांस

5. फ्रांस्वा बर्नियर भारत आया था-
(अ) सोलहवीं सदी
(ब) पन्द्रहवीं सदी
(स) सत्रहवीं सदी
(द) अठारहर्वीं सदी
उत्तर:
(स) सत्रहवीं सदी

6. इन्नबतूता के पाठक किससे पूरी तरह से अपरिचित थे-
(अ) खजूर
(ब) नारियल
(स) केला
(द) अंगूर
उत्तर:
(ब) नारियल

7. इबबतूता ने किस शहर को भारत में सबसे बड़ा बताया है-
(अ) आगरा
(ब) इलाहाबाद
(स) जौनपुर
(द) दिल्ली
उत्तर:
(द) दिल्ली

8. इलबतूता भारत की कौनसी प्रणाली की कार्यकुशलता को देखकर आशचर्यचकित हो गया था-
(अ) जल-निकास प्रणाली
(ब) डाक प्रणाली
(स) गुप्तचर प्रणाली
(द) सुरक्ष प्रणाली
उत्तर:
(ब) डाक प्रणाली

JAC Class 12 History Important Questions Chapter 5 यात्रियों के नज़रिए : समाज के बारे में उनकी समझ

9. ‘ट्रैवल्स इन द मुगल एम्पायर’ नामक ग्रन्थ का रचयिता था-
(अ) मनूची
(ब) सर टॉमस रो
(स) बर्नियर
(द) विलियम हॉकिंस
उत्तर:
(स) बर्नियर

10. किताब-उल-हिन्द के लेखक कौन हैं?
(अ) इब्नबतूता
(ब) बर्नियर
(स) अल-बिरुनी
(द) अब्दुर रज्जाक
उत्तर:
(स) अल-बिरुनी

11. ख्वारिज्म में अल-बिरुनी का जन्म हुआ-
(अ) 1071 ई.
(ब) 933 ई.
(स) 1023 ई.
(द) 973 ई.
उत्तर:
(द) 973 ई.

12. अल-बिरुनी किसके साथ भारत आया?
(अ) मोहम्मद गौरी
(ब) मोहम्मद बिन कासिम
(स) महमूद गजनवी
(द) इनमें से कोई नहीं
उत्तर:
(स) महमूद गजनवी

13. मोहम्मद तुगलक के कहने पर इबबतूता किस देश की यात्रा पर गया?
(अ) अफगानिस्तान
(ब) रूस
(स) नेपाल
(द) चीन
उत्तर:
(द) चीन

14. बर्नियर पेशे से क्या थे?
(अ) तोपची
(ब) चिकित्सक
(स) सुनार
(द) वैज्ञानिक
उत्तर:
(ब) चिकित्सक

JAC Class 12 History Important Questions Chapter 5 यात्रियों के नज़रिए : समाज के बारे में उनकी समझ

15. इब्नबतूता ने अपना यात्रा-वृत्तान्त किस भाषा में लिखा?
(अ) अरबी
(ब) फारसी
(स) हित्रू
(द) उर्दू
उत्तर:
(अ) अरबी

रिक्त स्थानों की पूर्ति कीजिए :
1. यूक्लिड यूनानी ……………..
2. ख्वारिज्म ……………… में स्थित है।
3. सुल्तान महमूद ने ख्वारिज्म पर आक्रमण ……………. ई. में किया।
4. इनबतूता ……………. में स्थलमार्ग से …………. पहुँचा।
5. भारत में पुर्तगालियों का आगमन लगभग ……………….. ई. में हुआ।
6. बर्नियर ने अपनी प्रमुख कृति को फ्रांस के शासक ……………. को समर्पित किया था।
उत्तर:
1. गणितज्ञ
2 . उज्बेकिस्तान
3.1017
4. मध्य एशिया, सिन्ध
5. 1500
6. लुई

अतिलघूत्तरात्मक प्रश्न

प्रश्न 1.
फ्रांस्वा बर्नियर कौन था?
उत्तर:
फ्रांस्वा बर्नियर एक चिकित्सक, राजनीतिक, दार्शानिक और इतिहासकार था।

प्रश्न 2.
अल-बिरुनी का जन्म कब और कहाँ हुआ था?
उत्तर:
अल-बिरुनी का जन्म ख्वारिज्म में सन् 973 में हुआ था।

प्रश्न 3.
15 वीं सदी में भारत की यात्रा करने वाले फारस के दूत का क्या नाम था?
उत्तर:
अब्दुर रज्जाक।

प्रश्न 4.
दसवीं शताब्दी से सत्रहवीं सदी तक भारत की यात्रा करने वाले तीन विदेशी यात्रियों के नाम लिखिए।
(1) अल बिरूनी
(2) इब्नबतूता
(3) फ्रांस्वा
उत्तर:
बर्नियर।

JAC Class 12 History Important Questions Chapter 5 यात्रियों के नज़रिए : समाज के बारे में उनकी समझ

प्रश्न 5.
‘किताब-उल-हिन्द’ का रचयिता कौन था? यह ग्रन्थ किस भाषा में लिखा गया है?
उत्तर:
(1) अल-विरुनी
(2) अरबी भाषा।

प्रश्न 6.
अल-विरुनी द्वारा जिन दो ग्रन्थों का संस्कृत में अनुवाद किया गया, उनके नाम लिखिए।
उत्तर:
(1) पतंजलि का व्याकरण
(2) यूक्लिड के कार्य।

प्रश्न 7.
इब्नबतूता कहाँ का निवासी था?
उत्तर:
इब्नबतूता मोरक्को का निवासी था।

प्रश्न 8.
इब्नबतूता के भारत पहुँचने पर किस सुल्तान ने किस पद पर नियुक्त किया था?
उत्तर:
(1) मुहम्मद बिन तुगलक ने
(2) दिल्ली के काजी (न्यायाधीश) के पद पर

प्रश्न 9.
इब्नबतूता भारत कब पहुँचा और किस मार्ग से पहुंचा?
उत्तर:
इब्नबतूता 1333 ई. में स्थल मार्ग से सिन्ध पहुँचा।

प्रश्न 10.
इब्नबतूता ने अपना यात्रा वृखन्त किस भाषा में लिखा? यह यात्रा वृत्तान्त किस नाम से प्रसिद्ध है?
उत्तर:
(1) अरबी भाषा में
(2) रिहला।

प्रश्न 11.
अब्दुरक समरवन्दी ने भारत में किस भाग की यात्रा की थी और कब?
उत्तर:
1440 के दशक में अब्दुररज्जाक ने दक्षिण भारत की यात्रा की।

प्रश्न 12.
फ्रांस्वा बर्नियर भारत में कितने वर्ष रहा था ?
उत्तर:
फ्रांस्वा बर्नियर 12 वर्ष (1656-1668 ई.) तक भारत में रहा।

प्रश्न 13.
भारतीय समाज को समझने में अल-बिरुनी को कौनसी बाधाओं का सामना करना पड़ा ?
उत्तर:
(1) संस्कृत भाषा की कठिनाई
(2) धार्मिक अवस्था, प्रथाओं में भिन्नता
(3) अभिमान

प्रश्न 14.
इब्नबतूता ने कौनसी दो वानस्पतिक उपजों का रोचक वर्णन किया है, जिनसे उसके पाठक पूरी तरह से अपरिचित थे?
उत्तर:
(1) नारियल
(2) पान।

JAC Class 12 History Important Questions Chapter 5 यात्रियों के नज़रिए : समाज के बारे में उनकी समझ

प्रश्न 15.
इब्नबतूता के अनुसार भारत के दो बड़े शहर कौन से थे?
उत्तर:
(1) दिल्ली
(2) दौलताबाद।

प्रश्न 16.
इब्नबतूता ने भारत की किस प्रणाली की कुशलता का उल्लेख किया है?
उत्तर:
डाक प्रणाली का

प्रश्न 17.
अब्दुररज्जाक ने किस शहर के मन्दिर के शिल्प और कारीगरी को अद्भुत बताया था ?
उत्तर:
मंगलौर शहर से 9 मील के भीतर स्थित मन्दिर

प्रश्न 18.
ऐसे तीन विदेशी यात्रियों के नाम लिखिए जिन्होंने अल बिरूनी और इब्नबतूता के पदचिन्हों का अनुसरण किया।
उत्तर:
(1) अब्दुर रज्जाक
(2) महमूद वली बल्छी
(3) शेख अली हाजिन।

प्रश्न 19.
पेलसर्ट ने भारत की किस सामाजिक समस्या की ओर ध्यान आकृष्ट किया?
उत्तर:
भारत की व्यापक तथा दुःखद गरीबी की समस्या।

प्रश्न 20.
बर्नियर के अनुसार भारत और यूरोप के बीच एक प्रमुख मूल भिन्नता बताइये।
उत्तर:
भारत में निजी भू-स्वामित्व का अभाव।

प्रश्न 21.
अल-विरुनी ने अपनी पुस्तक ‘किताब-उल- हिन्द’ किस भाषा में लिखी?
उत्तर:
अरबी में।

प्रश्न 22.
इब्नबतूता का जन्म कहाँ हुआ था?
उत्तर:
तैंजियर में।

प्रश्न 23.
शरिया का क्या अर्थ है?
उत्तर:
इस्लामी कानून।

प्रश्न 24.
इब्नबतूता ने भारत के लिए कब प्रस्थान
उत्तर:
1332-33 ई. में।

प्रश्न 25.
इब्नबतूता अपने देश वापस कब पहुँचा ?
उत्तर:
1354 ई. में

प्रश्न 26.
1600 ई. के बाद भारत आने वाले दो यूरोपीय यात्रियों के नाम लिखिए।
उत्तर:
(1) ज्यॉँ-बैप्टिस्ट तैर्नियर
(2) मनूकी।

JAC Class 12 History Important Questions Chapter 5 यात्रियों के नज़रिए : समाज के बारे में उनकी समझ

प्रश्न 27.
बर्नियर का यात्रा वृत्तान्त कहाँ और कब प्रकाशित हुआ?
उत्तर:
फ्रांस में 1670-71 में

प्रश्न 28.
जाति व्यवस्था के सम्बन्ध में ब्राह्मणवादी व्याख्या को मानने के बावजूद अल बिरूनी ने किस मान्यता को अस्वीकार किया ?
उत्तर:
अपवित्रता की मान्यता।

प्रश्न 29.
अल बिरूनी ने भारत में प्रचलित वर्ण- व्यवस्था के अन्तर्गत किन चार प्रमुख वर्णों का उल्लेख किया है?
उत्तर:
(1) ब्राह्मण
(2) क्षत्रिय
(3) वैश्य
(4) शूद्र।

प्रश्न 30.
जातिव्यवस्था के विषय में अल-विरुनी का विवरण किन ग्रन्थों पर आधारित था?
उत्तर:
संस्कृत ग्रन्थों पर।

प्रश्न 31.
इब्नबतूता के अनुसार दिल्ली शहर में कितने दरवाजे थे? इनमें से सबसे विशाल दरवाजा कौनसा था ?
उत्तर:
(1) 28 दरवाजे
(2) बदायूँ दरवाजा।

प्रश्न 32.
अब्दुररज्जाक ने किसे ‘विचित्र देश’ बताया था?
उत्तर:
कालीकट बन्दरगाह पर बसे हुए लोगों को।

प्रश्न 33.
बर्नियर द्वारा रचित ग्रन्थ ‘ट्रेवल्स इन द मुगल एम्पायर’ की अपनी किन विशेषताओं के लिए विख्यात है?
अथवा
‘ट्रेवल्स इन द मुगल एम्पायर’ क्या है?
उत्तर:
(1) गहन चिन्तन
(2) गहन प्रेक्षण
(3) आलोचनात्मक अन्तर्दृष्टि

प्रश्न 34.
बर्नियर के अनुसार सत्रहवीं शताब्दी में भारत में जनसंख्या का कितने प्रतिशत भाग नगरों में रहता था?
उत्तर:
लगभग पन्द्रह प्रतिशत।

प्रश्न 35.
इब्नबतूता के अनुसार सुल्तान मुहम्मद बिन तुगलक अमीरों की गतिविधियों की जानकारी प्राप्त करने के लिए किन्हें नियुक्त करता था?
उत्तर:
दासियों को।

प्रश्न 36.
अल बिरूनी द्वारा अपनी कृतियों में जिस विशिष्ट शैली का प्रयोग किया गया, उसे स्पष्ट कीजिये।
उत्तर:
आरम्भ में एक प्रश्न, फिर संस्कृत परम्पराओं पर आधारित वर्णन।

JAC Class 12 History Important Questions Chapter 5 यात्रियों के नज़रिए : समाज के बारे में उनकी समझ

प्रश्न 37.
वर्नियर के विवरणों ने किन दो पश्चिमी विचारकों को प्रभावित किया?
उत्तर:
(1) मॉन्टेस्क्यू
(2) कार्ल मार्क्स।

प्रश्न 38.
वर्नियर ने मुगलकालीन नगरों को क्या कहा है?
अथवा
बर्नियर भारतीय नगरों को किस रूप में देखता है?
उत्तर:
बर्नियर ने मुगलकालीन नगरों को ‘शिविर नगर कहा है।

प्रश्न 39.
बर्नियर ने मुगलकालीन नगरों को शिविर नगर क्यों कहा है?
उत्तर:
क्योंकि ये नगर राजकीय शिविर पर निर्भर थे।

प्रश्न 40.
मुगलकालीन भारत में कौन-कौन से प्रकार के नगर अस्तित्व में थे?
उत्तर:
मुगलकालीन भारत में उत्पादन केन्द्र, व्यापारिक नगर, बन्दरगाह नगर, धार्मिक केन्द्र तीर्थ स्थान आदि नगर अस्तित्व में थे।

प्रश्न 41.
इब्नबतूता के अनुसार भारत में कितने प्रकार की डाक व्यवस्था प्रचलित थी?
उत्तर:
दो प्रकार की डाक व्यवस्था –
(1) अश्व डाक व्यवस्था (उलुक) तथा
(2) पैदल डाक व्यवस्था (दावा)।

प्रश्न 42.
इब्नबतूता के अनुसार ‘ताराबबाद’ क्या था?
उत्तर:
दौलताबाद में पुरुष और महिला गायकों के लिए एक बाजार था, जिसे ‘तारावबाद’ कहते थे।

प्रश्न 43.
इब्नबतूता के अनुसार भारत का सबसे बड़ा शहर कौनसा था ?
उत्तर:
इब्नबतूता के अनुसार दिल्ली भारत का सबसे बड़ा शहर था।

प्रश्न 44.
अल-विरुनी के अनुसार भारत में वर्ण- व्यवस्था का उद्भव किस प्रकार से हुआ?
उत्तर:
अल बिरुनी के अनुसार ब्राह्मण ब्रह्मन् के सिर से, क्षत्रिय कन्धों और हाथों से वैश्य जंघाओं से तथा शूद्र चरणों से उत्पन्न हुए।

प्रश्न 45.
अल बिरुनी किन भाषाओं का ज्ञाता था?
उत्तर:
अल बिरूनी संस्कृत, सीरियाई, फारसी, हिब्रू नामक भाषाओं का ज्ञाता था।

प्रश्न 46.
अल बिरुनी तथा इब्नबतूता किन देशों से और कब भारत आए ?
उत्तर:
अल बिरुनी 11वीं शताब्दी में उज्बेकिस्तान तथा इब्नबतूता 14वीं शताब्दी में मोरक्को से भारत आए थे।

JAC Class 12 History Important Questions Chapter 5 यात्रियों के नज़रिए : समाज के बारे में उनकी समझ

प्रश्न 47.
अल बिरूनी ने अपनी पुस्तक ‘किताब- उल-हिन्द’ में किन विषयों का विवेचन किया है?
अथवा
‘किताब-उल-हिन्द’ पर सक्षिप्त टिप्पणी लिखिए।
अथवा
‘किताब-उल-हिन्द’ क्या है?
उत्तर:
धर्म और दर्शन, त्यौहारों, खगोल विज्ञान, कीमिया, रीति-रिवाजों तथा प्रथाओं, सामाजिक जीवन, माप- तौल, मूर्तिकला, कानून तथा मापतन्त्र विज्ञान ।

प्रश्न 48.
इब्नबतूता के अनुसार किन देशों में किस भारतीय माल की अत्यधिक मांग थी?
उत्तर:
मध्य एशिया तथा दक्षिण-पूर्व एशिया में भारत के सूती कपड़े, महीन मलमल, रेशम, जरी तथा सदन की अत्यधिक मांग थी।

प्रश्न 49.
किस डच यात्री ने भारतीय उपमहाद्वीप की यात्रा की थी और कब?
उत्तर:
(1) पेलसर्ट
(2) सत्रहवीं शताब्दी के आरम्भिक दशकों में

प्रश्न 50.
फ्रांस्वा बर्नियर के अनुसार भारत में किस स्थिति के लोग नहीं थे?
उत्तर;
फ्रांस्वा बर्नियर के अनुसार भारत में मध्य की स्थिति के लोग नहीं थे।

प्रश्न 51.
बर्नियर ने भारत के किस नगर अल्पवयस्क विधवा को सती होते हुए देखा था? उसे किनकी सहायता से चिता स्थल की ओर ले जाया गया?
उत्तर:
(1) लाहौर में
(2) तीन या चार ब्राह्मणाँ तथा एक वृद्ध महिला की सहायता से।

प्रश्न 52.
बर्नियर के अनुसार कौनसे शिल्प भारत में प्रचलित थे?
उत्तर:
गलीचे बनाना, जरी कसीदाकारी कढ़ाई, सोने और चाँदी के वस्त्रों, रेशमी तथा सूती वस्त्रों का निर्माण।

प्रश्न 53.
अल बिरूनी के अनुसार फारस में समाज किन चार वर्गों में विभाजित था?
उत्तर:

  • घुड़सवार और शासक वर्ग
  • भिक्षु, आनुष्ठानिक पुरोहित तथा चिकित्सक
  • खगोलशास्वी तथा अन्य वैज्ञानिक
  • कृषक तथा शिल्पकार।

प्रश्न 54.
दुआ बरबोसा कौन था?
उत्तर:
दुआर्ते बरबोसा एक प्रसिद्ध यूरोपीय लेखक था जिसने दक्षिण भारत में व्यापार और समाज का एक विस्तृत विवरण लिखा ।

प्रश्न 55.
यूरोप के दो यात्रियों के नाम लिखिए जिन्होंने भारतीय कृषकों की गरीबी का वर्णन किया है।
उत्तर:
(1) पेलसर्ट
(2) बर्नियर

JAC Class 12 History Important Questions Chapter 5 यात्रियों के नज़रिए : समाज के बारे में उनकी समझ

प्रश्न 56.
इब्नबतूता भारतीय डाक प्रणाली की कार्यकुशलता देखकर क्यों चकित हुआ? उल्लेख कीजिए।
उत्तर:
डाक व्यवस्था की कार्यकुशलता के कारण व्यापारियों के लिए न केवल लम्बी दूरी तक सूचना और उधार भेजना सम्भव हुआ, बल्कि अल्प सूचना पर माल भेजना भी सम्भव हो गया।

प्रश्न 57.
इब्नबतूता ने मोरक्को जाने से पूर्व किन देशों की यात्रा की थी?
उत्तर:
उत्तरी अफ्रीका, पश्चिम एशिया, मध्य एशिया कुछ भागों, भारतीय उपमहाद्वीप तथा चीन

प्रश्न 58.
1400 से 1800 के बीच भारत की यात्रा करने वाले विदेशी यात्रियों के नाम लिखिए जिन्होंने फारसी में अपने यात्रा-वृत्तान्त लिखे।
उत्तर:
अब्दुररजाक समरकंदी, महमूद वली बल्खी, शेख अली हाजिन

प्रश्न 59.
इब्नबतूता के अनुसार भारत की डाक- प्रणाली क्यों लाभप्रद थी?
उत्तर:
डाक प्रणाली से व्यापारियों के लिए लम्बी दूरी तक सूचना भेजना, उधार भेजना और अल्प सूचना पर माल भेजना सम्भव हो गया।

प्रश्न 60.
पेलसर्ट कौन था?
उत्तर:
पेलसर्ट एक डच यात्री था जिसने सत्रहवीं शताब्दी में भारत की यात्रा की थी।

प्रश्न 61.
बर्नियर के अनुसार मुगल साम्राज्य के स्वरूप की दो त्रुटियों का उल्लेख कीजिए।
उत्तर:
(1) मुगल साइट भिखारियों’ और ‘क्रूर लोगों’ का राजा था
(2) इसके शहर विनष्ट तथा खराब हवा से दूषित थे।

प्रश्न 62.
बर्नियर ने किस जटिल सामाजिक सच्चाई का उल्लेख किया है?
उत्तर:
(1) सम्पूर्ण विश्व से बड़ी मात्रा में बहुमूल्य धातुओं का भारत में आना
(2) भारत में एक समृद्ध व्यापारिक समुदाय का अस्तित्व।

प्रश्न 63.
बर्नियर ने भारत की कृषि की किन दो विशेषताओं का उल्लेख किया है?
उत्तर:
(1) देश के विस्तृत भू-भाग का अधिकांश भाग अत्यधिक उपजाऊ था
(2) भूमि पर खेती अच्छी होती थी।

प्रश्न 64.
बर्नियर ने मुगलकालीन नगरों को किसकी संज्ञा दी है और क्यों?
उत्तर:
(1) शिविर नगर
(2) क्योंकि ये नगर अपने अस्तित्व के लिए राजकीय शिविर पर निर्भर थे।

प्रश्न 65.
बर्नियर के अनुसार पश्चिमी भारत में व्यापारियों के समूह क्या कहलाते थे? उनके मुखिया को क्या कहते घे?
उत्तर:
(1) पश्चिमी भारत में व्यापारियों के समूह महाजन कहलाते थे।
(2) उनके मुखिया सेठ कहलाते थे।

प्रश्न 66.
बर्नियर के अनुसार अन्य शहरी समूहों में व्यावसायिक वर्ग में कौन-कौन लोग सम्मिलित थे?
उत्तर:

  1. चिकित्सक
  2. अध्यापक
  3. अधिवक्ता
  4. चित्रकार
  5. वास्तुविद
  6. संगीतकार
  7. सुलेखक।

प्रश्न 67.
इब्नबतूता के अनुसार दासों की सेवाओं को विशेष रूप से किस कार्य में उपयोग किया जाता था?
उत्तर:
दास पालकी या डोले में पुरुषों और महिलाओं को ले जाने का कार्य करते थे।

प्रश्न 68.
इब्नबतूता के अनुसार अधिकांश दासियाँ अ किस प्रकार प्राप्त की जाती थीं?
उत्तर:
अधिकांश दासियों को आक्रमणों और अभियानों के दौरान बलपूर्वक प्राप्त किया जाता था

JAC Class 12 History Important Questions Chapter 5 यात्रियों के नज़रिए : समाज के बारे में उनकी समझ

प्रश्न 69.
विदेशी यात्री अब्दुरज्जाक ने कालीकट बन्दरगाह पर बसे हुए लोगों को क्या बताया था?
उत्तर:
अब्दुरम्नांक ने कालीकट बन्दरगाह पर बसे हुए लोगों को एक विचित्र देश’ बताया था।

प्रश्न 70.
“कृषकों को इतना निचोड़ा जाता है कि पेट भरने के लिए उनके पास सूखी रोटी भी मुश्किल से बचती है।” यह कथन किसका है?
उत्तर:
यह कथन पेलसर्ट नामक एक डच यात्री का

प्रश्न 71.
बर्नियर ने भारत में पाई जाने वाली सती प्रथा का विवरण क्यों दिया?
उत्तर:
क्योंकि महिलाओं से किया जाने वाला बर्ताव प्रायः पश्चिमी तथा पूर्वी समाजों के बीच भिन्नता का प्रतीक माना जाता था।

प्रश्न 72.
मुहम्मद बिन तुगलक के दूत के रूप में किस विदेशी यात्री को मंगोल शासक के पास चीन जाने का आदेश दिया गया और कब दिया गया?
उत्तर:
(1) इब्नबतूता को
(2) 1342 ई. में

प्रश्न 73.
अल बिरूनी ने संस्कृत भाषा की किन विशेषताओं का उल्लेख किया।
उत्तर:
(1) शब्दों तथा विभक्तियों दोनों में संस्कृति की पहुँच विस्तृत है।
(2) एक ही वस्तु के लिए कई शब्द प्रयुक्त होते हैं।

प्रश्न 74.
बर्नियर ने अपने वृतान्त में भारत को किसके रूप में दिखाया है?
उत्तर:
बर्नियर ने भारत को यूरोप के प्रतिलोम के रूप में अथवा फिर यूरोप का विपरीत जैसा दिखाया है।

प्रश्न 75.
दासों को सामान्यतः किस कार्य के लिए प्रयुक्त किया जाता था?
उत्तर:
दासों को सामान्यतः घरेलू श्रम के लिए ही प्रयुक्त किया जाता था।

प्रश्न 76.
बर्नियर ने सती प्रथा के बारे में क्या लिखा है?
उत्तर:
कुछ महिलाएँ प्रसन्नतापूर्वक मृत्यु को गले लगा लेती थीं, अन्यों को मरने के लिए बाध्य किया जाता था।

प्रश्न 77.
बर्नियर के अनुसार ‘शिविर नगर’ क्या थे?
उत्तर:
बर्नियर के अनुसार ‘शिविर नगर’ वे थे जो अपने अस्तित्व और बने रहने के लिए राजकीय शिविर पर निर्भर थे।

JAC Class 12 History Important Questions Chapter 5 यात्रियों के नज़रिए : समाज के बारे में उनकी समझ

प्रश्न 78.
बर्नियर के अनुसार भारत और यूरोप के बीच मूल भिन्नता क्या थी?
उत्तर:
बर्नियर के अनुसार भारत और यूरोप के बीच मूल भिन्नता भारत में निजी भू-स्वामित्व का अभाव था।

प्रश्न 79.
किस सुल्तान ने नसीरुद्दीन नामक धर्मोपदेशक से प्रसन्न होकर उसे एक लाख के तथा दो सौ दाम दिये ?
उत्तर:
मुहम्मद बिन तुगलक ने।

प्रश्न 80.
इब्नबतूता ने भारत की किन बातों का विशेष रूप से वर्णन किया है?
उत्तर:
इब्नबतूता ने डाक व्यवस्था, पान तथा नारियल का विशेष रूप से वर्णन अपने ग्रन्थ ‘रिहला’ में किया है।

प्रश्न 81.
इब्नबतूता ने भारत के किस शहर को सबसे बड़ा कहा है?
उत्तर:
दिल्ली।

प्रश्न 82.
बर्नियर के ग्रन्थ का क्या नाम है?
उत्तर:
ट्रेवल्स इन द मुगल एम्पावर।

प्रश्न 83.
ताराबबाद किसे कहा जाता है?
उत्तर:
दौलताबाद में पुरुष तथा महिला गायकों के लिए बाजार होता था; जिसे तारावबाद कहा जाता था।

प्रश्न 84.
बर्नियर ने मुगल सेना के साथ कहाँ की न यात्रा की थी?
उत्तर:
कश्मीर

प्रश्न 85.
बर्नियर जब भारत आया उस समय यूरोप में कौनसा युग गतिमान था?
उत्तर:
बर्नियर भारत में सत्रहवीं शताब्दी में आया था, कि उस समय लगभग सम्पूर्ण यूरोप में पुनर्जागरण का काल था।

प्रश्न 86.
किस यात्री ने सुल्तान मुहम्मद तुगलक को भेंट में देने के लिए घोड़े, अँट तथा दास खरीदे ?
उत्तर:
मोरक्को निवासी इब्नबतूता ने।

प्रश्न 87.
यह तर्क किसने दिया कि भारत में ही उपनिवेशवाद से पहले अधिशेष का अधिग्रहण राज्य द्वारा होता था?
उत्तर:
कार्ल मार्क्स।

लघुत्तरात्मक प्रश्न

प्रश्न 1.
लगभग दसवीं सदी से सत्रहवीं सदी तक के काल में लोगों के यात्राएँ करने के क्या उद्देश्य थे?
उत्तर:
लगभग दसवीं सदी से सत्रहवीं सदी तक के काल में महिलाओं और पुरुषों के यात्राएं करने के निम्नलिखित उद्देश्य थे –

  1. कार्य की तलाश में
  2. आपदाओं से बचाव के लिए
  3. व्यापारियों सैनिकों, पुरोहितों और तीर्थयात्राओं के रूप में
  4. साहस की भावना से प्रेरित होकर।

प्रश्न 2.
विदेशी यात्रियों की कौनसी बात उनके यात्रा-वृत्तान्तों को अधिक रोचक बनाती है?
उत्तर:
पूर्ण रूप से भिन्न सामाजिक तथा सांस्कृतिक पृष्ठभूमि से आने के कारण ये विदेशी यात्री दैनिक गतिविधियों तथा प्रथाओं के प्रति अधिक सावधान रहते थे। देशज लेखकों के लिए ये सभी विषय सामान्य थे, जो वृत्तान्तों में उल्लिखित करने योग्य नहीं थे दृष्टिकोण में यही भिन्नता ही उनके यात्रा वृत्तान्तों को अधिक रोचक बनाती है।

JAC Class 12 History Important Questions Chapter 5 यात्रियों के नज़रिए : समाज के बारे में उनकी समझ

प्रश्न 3.
अल-बिरुनी के यात्रा-वृत्तान्त लिखने के उद्देश्य लिखिए।
उत्तर:
अल- विरुनी के यात्रा-वृत्तान्त लिखने के निम्नलिखित उद्देश्य थे –
(1) उन लोगों की सहायता करना जो हिन्दुओं से धार्मिक विषयों पर चर्चा करना चाहते थे।
(2) ऐसे लोगों के लिए सूचना का हिन्दुओं के साथ सम्बद्ध होना चाहते थे।

प्रश्न 4.
अल-विरुनी ग्रन्थों का अनुवाद करने में क्यों सक्षम था? स्पष्ट कीजिए उसके द्वारा अनुवादित ग्रन्थों के नाम लिखिए।
उत्तर:
अल बिरूनी कई भाषाओं में दक्ष था जिनमें सीरियाई, फारसी, हिब्रू तथा संस्कृत शामिल हैं। इसलिए वह भाषाओं की तुलना तथा ग्रन्थों का अनुवाद करने में सक्षम रहा। उसने अनेक संस्कृत ग्रन्थों का अरबी में अनुवाद किया। संग्रह करना जो उसने पतंजलि के व्याकरण ग्रन्थ का भी अरबी भाषा में अनुवाद किया। उसने अपने ब्राह्मण मित्रों के लिए यूनानी गणित यूक्लिड के कार्यों का संस्कृत में अनुवाद किया।

प्रश्न 5.
किताब-उल-हिन्द’ के बारे में आप क्या जानते हैं ?
उत्तर:
अल बिरूनी ने अरबी भाषा में अपनी पुस्तक ‘किताब-उल-हिन्द’ लिखी। इसकी भाषा सरल और स्पष्ट है। यह एक विस्तृत ग्रन्थ है जो अस्सी अध्यायों में विभाजित है। इस ग्रन्थ में भारतीय धर्म और दर्शन त्योहारों, खगोल- विज्ञान, कीमिया, रीति-रिवाजों तथा प्रथाओं, सामाजिक- जीवन, भार-तौल तथा मापन विधियों, मूर्तिकला, कानून, मापतन्त्र विज्ञान आदि विषयों का विवेचन किया गया है।

प्रश्न 6.
इब्नबतूता अकेला ही विश्व यात्रा पर क्यों निकल पड़ा? उस समय उसकी क्या आयु थी? वह अपने घर वापस कब पहुँचा ?
उत्तर:
इब्नबतूता के वृत्तान्त से ज्ञात होता है कि वह अपने जन्म स्थान जियर से अकेला ही अपनी यात्रा पर निकल पड़ा। उसके मन में लम्बे समय से प्रसिद्ध पुण्य स्थानों को देखने की तीव्र इच्छा थी इसलिए उसने किसी कारणों में शामिल होने की प्रतीक्षा नहीं की और अकेला ही घर से निकल पड़ा। उस समय इब्नबतूता की आयु बाईस वर्ष थी। वह 1354 में अपने घर वापस पहुँच गया।

प्रश्न 7.
अल बिरूनी को भारत का यात्रा-वृत्तान्त लिखने में किन कठिनाइयों का सामना करना पड़ा?
उत्तर:
(1) अल बिरूनी के अनुसार पहली कठिनाई भाषा थी। उसके अनुसार संस्कृत, अरबी और फारसी से इतनी भिन्न थी कि विचारों और सिद्धान्तों को एक ही भाषा से दूसरी में अनुवादित करना सरल नहीं था।
(2) दूसरी कठिनाई धार्मिक अवस्था और प्रथाओं में भिन्नता थी उसे इन्हें समझने के लिए वेदों, पुराणों आदि की सहायता लेनी पड़ी।
(3) अल-बिरुनी के अनुसार तौसरी कठिनाई भारतीयों का जातीय अभिमान था।

प्रश्न 8.
अपनी श्रेणी के अन्य यात्रियों से इब्नबतूता किन बातों में अलग था?
उत्तर:
इब्नबतूता पुस्तकों के स्थान पर यात्राओं से प्राप्त अनुभव को अपनी जानकारी का अधिक महत्त्वपूर्ण स्त्रोत मानता था। उसे यात्राएँ करने का बड़ा शौक था और उसने नये-नये देशों तथा लोगों के विषय में जानने के लिए | दूर-दूर के क्षेत्रों तक की यात्रा की 1332-33 ई. में भारत के लिए प्रस्थान करने से पूर्व वह मक्का, सीरिया, इराक, फारस, यमन, ओमान तथा पूर्वी अफ्रीका के कई तटीय व्यापारिक बन्दरगाहों की यात्राएँ कर चुका था।

प्रश्न 9.
इब्नबतूता के तत्कालीन सुल्तान मुहम्मद- बिन तुगलक के साथ सम्बन्धों का वर्णन कीजिए।
उत्तर:
1333 में इब्नबतूता दिल्ली पहुंचा। मुहम्मद- बिन तुगलक इब्नबतूता की विद्वता से बझ प्रभावित हुआ और उसे दिल्ली का काली अथवा न्यायाधीश नियुक्त किया। उसने इस पद पर कई वर्षों तक कार्य किया। कुछ कारणों से सुल्तान इब्नबतूता से नाराज हो गया और उसे कारागार में कैद कर दिया गया। परन्तु कुछ समय बाद सुल्तान की नाराजगी दूर हो गई और उसने 1342 में इब्नबतूता को अपने दूत के रूप में चीन के शासक के पास भेजा।

प्रश्न 10.
“इब्नबतूता एक हठीला यात्री था।” व्याख्या कीजिए।
उत्तर:
इब्नबतूता अपनी सुन का पक्का था उसे यात्राएँ करने का बहुत शौक था। वह लम्बी यात्राओं के दौरान होने वाली कठिनाइयों से हतोत्साहित नहीं होता था। उसने उत्तर-पश्चिमी अफ्रीका में अपने निवास स्थान मोरक्को जाने से पूर्व कई वर्ष उत्तरी अफ्रीका, पश्चिमी एशिया, मध्य एशिया के भागों, भारतीय उपमहाद्वीप तथा चीन की यात्रा की थी। उसके वापिस लौटने पर मोरक्को के शासक ने उसकी कहानियों को दर्ज करने के निर्देश दिए।

JAC Class 12 History Important Questions Chapter 5 यात्रियों के नज़रिए : समाज के बारे में उनकी समझ

प्रश्न 11.
“बर्नियर द्वारा प्रस्तुत ग्रामीण समाज का चित्रण सच्चाई से बहुत दूर था।” स्पष्ट कीजिए।
उत्तर:
बर्नियर द्वारा प्रस्तुत ग्रामीण समाज का चित्रण सच्चाई से बहुत दूर था सोलहवीं तथा सत्रहवीं शताब्दी में ग्रामीण समाज में चारित्रिक रूप से बड़े पैमाने पर सामाजिक और आर्थिक विभेद था। एक ओर बड़े जमींदार थे और दूसरी ओर ‘अस्पृश्य’ भूमि विहीन श्रमिक इन दोनों के बीच में बड़ा किसान था जो किराए के श्रम का प्रयोग करता था और माल उत्पादन में जुटा रहता था कुछ छोटे किसान भी थे, जो बड़ी कठिनाई से गुजरे योग्य उत्पादन कर पाते थे।

प्रश्न 12.
बर्नियर ने शहरी समूहों में किन व्यावसायिक वर्गों का उल्लेख किया है?
उत्तर:
बर्नियर के अनुसार शहरी समूहों में चिकित्सक (हकीम अथवा वैद्य), अध्यापक (पंडित या मुल्ला), अधिवक्ता (वकील), चित्रकार, वास्तुविद्, संगीतकार, सुलेखक आदि व्यावसायिक वर्ग थे। कई लोग राजकीय संरक्षण पर आश्रित थे तथा कई अन्य लोग संरक्षकों या भीड़-भाड़ वाले बाजार में सामान्य लोगों की सेवा द्वारा अपना जीवनयापन करते थे।

प्रश्न 13.
इब्नबतूता के विवरण के अनुसार दासों में काफी विभेद था। स्पष्ट कीजिए।
उत्तर:
इब्नबतूता के विवरण से ज्ञात होता है कि दाखों में काफी विभेद था। सुल्तान की सेवा में कार्यरत कुछ दासियाँ संगीत और गायन में निपुण थीं। इब्नबतूता सुल्तान की बहिन की शादी के अवसर पर उनके प्रदर्शन से बड़ा आनन्दित हुआ। सुल्तान अपने अमीरों की गतिविधियों पर नजर रखने के लिए दासियों को भी नियुक्त करता था। अधिकतर दासों का प्रयोग घरेलू श्रम के लिए किया जाता था घरेलू श्रम करने वाले दासों, दासियों की कीमत बहुत कम होती थी।

प्रश्न 14.
इब्नबतूता के अनुसार सुल्तान किस प्रकार अमीरों पर दासियों द्वारा नजर रखता था?
उत्तर:
इब्नबतूता के अनुसार सुल्तान की यह आदत थी कि हर बड़े या छोटे अमीर के साथ एक दास को रखता जो उनकी मुखबिरी करता था। वह इन अमीरों के घरों में महिला सफाई कर्मचारियों की भी नियुक्ति करता था। दासियों के पास जो भी सूचनाएँ होती थीं, वे इन महिला सफाई कर्मचारियों को दे देती थीं। अधिकांश दासियों को हमलों और अभियानों के दौरान बलपूर्वक प्राप्त किया जाता था।

प्रश्न 15.
अल बिरूनी के प्रारम्भिक जीवन का वर्णन कीजिए।
अथवा
अल-विरुनी के विषय में आप क्या जानते हैं ? उत्तर- अल बिरूनी का जन्म आधुनिक उज्बेकिस्तान में स्थित ख्वारिज्म में सन् 973 में हुआ था। अल बिरुनी ने उच्च कोटि की शिक्षा प्राप्त की। वह सीरियाई, फारसी, हिब्रू, संस्कृत आदि कई भाषाओं का ज्ञाता था। 1017 ई. में महमूद गजनवी ने ख्वारिज्म पर आक्रमण किया और अल बिरूनी सहित यहाँ के कई विद्वानों को अपने साथ अपनी राजधानी गजनी ले गया। उसने अपना शेष जीवन गजनी में ही बिताया। 70 वर्ष की आयु में उसकी मृत्यु हो गई।

प्रश्न 16.
गजनी में रहते हुए अल बिरूनी की भारत के प्रति रुचि कैसे विकसित हुई?
उत्तर:
आठवीं शताब्दी से ही संस्कृत में रचित खगोल- विज्ञान, गणित और चिकित्सा सम्बन्धी कार्यों का अरबी भाषा में अनुवाद होने लगा था। पंजाब के गजनवी साम्राज्य का भाग बन जाने के पश्चात् स्थानीय लोगों से हुए सम्पर्को से आपसी विश्वास और समझ का वातावरण बना। अल- विरुनी ने ब्राह्मण पुरोहितों तथा विद्वानों के साथ कई वर्ष व्यतीत किए और संस्कृत, धर्म तथा दर्शन का ज्ञान प्राप्त किया जिससे अल बिरुनी की भारत के प्रति रुचि विकसित हुई।

प्रश्न 17.
‘हिन्दू’, ‘हिन्दुस्तान’ तथा ‘हिन्दवी’ शब्दों का प्रचलन किस प्रकार हुआ?
उत्तर:
‘हिन्दू’ शब्द लगभग छठी पांचवीं शताब्दी ईसा पूर्व में प्रयुक्त होने वाले एक प्राचीन फारसी शब्द से निकला था, जिसका प्रयोग सिन्धु नदी (Indus) के पूर्व के क्षेत्र के लिए होता था। अरबी लोगों ने इस फारसी शब्द का प्रयोग करना जारी रखा। इस क्षेत्र को ‘अल-हिन्द’ तथा यहाँ के निवासियों को ‘हिन्दी’ कहा। कालान्तर में तुर्की ने सिन्धु से पूर्व में रहने वाले लोगों को ‘हिन्दू’, उनके निवास क्षेत्र को ‘हिन्दुस्तान’ तथा उनकी भाषा को ‘हिन्दवी’ की संज्ञा दी।

JAC Class 12 History Important Questions Chapter 5 यात्रियों के नज़रिए : समाज के बारे में उनकी समझ

प्रश्न 18.
अल बिरुनी के लेखन कार्य की विशेषताओं का विवेचन कीजिए।
उत्तर:

  1. अल बिरुनी ने लेखन में अरबी भाषा का प्रयोग किया था।
  2. अल बिरूनी ने सम्भवतः अपने ग्रन्थ उपमहाद्वीप के सीमान्त क्षेत्रों में रहने वाले लोगों के लिए लिखे थे।
  3. वह संस्कृत, पालि तथा प्राकृत ग्रन्थों के अरबी भाषा में हुए अनुवादों से परिचित था।
  4. इन ग्रन्थों की लेखन सामग्री शैली के विषय में अल बिरूनी का दृष्टिकोण आलोचनात्मक था वह उनमें सुधार करना चाहता था।

प्रश्न 19.
“इब्नबतूता की यात्राएँ कठिन तथा जोखिम भरी हुई थीं।” उदाहरण देते हुए स्पष्ट कीजिए।
उत्तर:
इब्नबतूता के अनुसार चौदहवीं शताब्दी में यात्रा करना अधिक कठिन, जोखिम भरा कार्य और असुरक्षित था इब्नबतूता को कई बार डाकुओं के समूहों के आक्रमणों का सामना करना पड़ा था। फिर भी राजमार्ग असुरक्षित थे। जब वह मुल्तान से दिल्ली की यात्रा कर रहा था, डाकुओं ने उसके कारवाँ पर आक्रमण किया जिसके फलस्वरूप उसके कई साथी यात्री मारे गए। जो यात्री बच गए थे, वे भी बुरी तरह से घायल हो गए थे। इनमें इब्नबतूता भी सम्मिलित था।

प्रश्न 20.
इब्नबतूता के श्रुतलेखों को लिखने के लिए नियुक्त किए गए इब्नजुजाई ने अपनी प्रस्तावना में क्या वर्णन किया है?
उत्तर:
इब्नजुजाई ने अपनी प्रस्तावना में लिखा है कि राजा ने इब्नबतूता को निर्देश दिया कि वह अपनी यात्रा में देखे गए शहरों का तथा रोचक घटनाओं का वृतान्त लिखवाएँ। इसके साथ ही वह विभिन्न देशों के जिन शासकों से मिले, उनके महान साहित्यकारों के तथा उनके धर्मनिष्ठ सन्तों के विषय में भी बताएँ। इस आदेश के अनुसार इब्नबतूता ने इन सभी विषयों पर एक कथानक लिखवाया। इसके अतिरिक्त इब्नबतूता ने कई प्रकार के असाधारण विवरण भी दिए।

प्रश्न 21.
अल बिरूनी और इब्नबतूता के पदचिन्हों का अनुसरण करने वाले यात्रियों का वर्णन कीजिए।
उत्तर:
1400 से 1800 के बीच भारत आने वाले अनेक यात्रियों ने अल बिरूनी और इब्नबतूता के पदचिन्हों का अनुसरण किया। इनमें अब्दुररज्जाक समरकंदी, महमूद वली बल्खी तथा शेख अली हानि उल्लेखनीय हैं। अब्दुरस्ज्जाक ने 1440 के दशक में दक्षिण भारत की यात्रा की तथा महमूद वली बल्खी ने 1620 के दशक में व्यापक रूप से यात्राएं की थीं। शेख अली हाजिन ने 1740 के दशक में उत्तर भारत की यात्रा की थी।

प्रश्न 22.
बर्नियर ने भारत में जो देखा, उसकी तुलना यूरोप से की। इसका मूल्यांकन कीजिए।
अथवा
बर्नियर द्वारा दी गई पूर्व और पश्चिम की तुलना का वर्णन कीजिए।
अथवा
“बर्नियर प्रायः भारत में जो देखता था, उसकी तुलना यूरोपीय स्थिति से ही करता था।” स्पष्ट कीजिए।
उत्तर:
बर्नियर प्राय: भारत में जो देखता था, उसकी तुलना यूरोपीय स्थिति से करता था वह यूरोप की सर्वश्रेष्ठता प्रतिपादित करना चाहता था यूरोपीय स्थितियों के मुकाबले में वह भारत की स्थितियों को दयनीय दर्शाना चाहता था। यही कारण है कि लगभग प्रत्येक दृष्टान्त में बर्नियर ने भारत की स्थिति को यूरोप में हुए विकास की तुलना में दयनीय बताया। यद्यपि उसका आकलन हम्मेसटीक नहीं था, फिर भी जब उसके कार्य प्रकाशित हुए). (बर्नियर के वृत्तांत अत्यधिक प्रसिद्ध हुए।

प्रश्न 23.
बर्नियर द्वारा मुगल सेना के कश्मीर कूच का वर्णन कीजिए। वह अपने साथ कौनसी वस्तुएँ ले गया था? उससे क्या अपेक्षा की जाती थी?
उत्तर:
बर्नियर मुगल सेना के कश्मीर कूच के सम्बन्ध में लिखता है कि इस देश की प्रथा के अनुसार उससे दो अच्छे तुर्कमान घोड़े देखने की अपेक्षा की जाती थी। वह अपने साथ एक शक्तिशाली पारसी ऊँट तथा चालक, अपने घोड़ों के लिए एक साईस, एक खानसामा तथा एक सेवक भी रखता था। उसे एक तम्बू एक दरी, एक छोटा बिस्तर, एक तकिया, एक विछौना, चमड़े के मेजपोश कुछ अंगोछे, झोले, जाल आदि वस्तुएँ दी गई थीं उसने चावल, मीठी रोटी, नींबू, चीनी आदि वस्तुएँ अपने साथ रखी थीं।

प्रश्न 24.
संस्कृत भाषा के विषय में अल बिरूनी के विचार व्यक्त कीजिए।
उत्तर:
अल बिरूनी के अनुसार संस्कृत भाषा को सीखना एक कठिन कार्य है क्योंकि अरबी भाषा की भाँति ही, शब्दों तथा विभक्तियों, दोनों में ही संस्कृत भाषा की पहुँच बहुत विस्तृत है। इसमें एक ही वस्तु के लिए कई शब्द, मूल तथा व्युत्पन्न दोनों प्रयुक्त होते हैं। इसमें एक ही शब्द का प्रयोग कई वस्तुओं के लिए होता है, जिन्हें अच्छी तरह से समझने के लिए विभिन्न विशेषक संकेत पदों के माध्यम से एक-दूसरे से पृथक किया जाना आवश्यक है।

प्रश्न 25.
अल बिरूनी द्वारा वर्णित फारस के चार सामाजिक वर्गों का उल्लेख कीजिए।
उत्तर:
अल बिरूनी के अनुसार प्राचीन फारस में समाज चार वर्गों में विभाजित था ये चार वर्ग थे –
(1) घुड़सवार और शासक वर्ग
(2) भिक्षु, आनुष्ठानिक पुरोहित
(3) चिकित्सक, खगोलशास्त्री तथा अन्य वैज्ञानिक और
(4) कृषक तथा शिल्पकार अल बिरुनी यह दर्शाना चाहता था कि ये सामाजिक वर्ग केवल भारत तक ही सीमित नहीं थे, बल्कि ये अन्य देशों में भी थे।

JAC Class 12 History Important Questions Chapter 5 यात्रियों के नज़रिए : समाज के बारे में उनकी समझ

प्रश्न 26.
“ जाति-व्यवस्था के सम्बन्ध में ब्राह्मणवादी व्याख्या को स्वीकार करने के बावजूद, अल बिरूनी ने अपवित्रता की मान्यता को अस्वीकार कर दिया।” स्पष्ट कीजिए।
उत्तर:
अल बिरूनी ने लिखा है कि प्रत्येक वह वस्तु जो अपवित्र हो जाती है, अपनी पवित्रता की मूल स्थिति को पुनः प्राप्त करने का प्रयास करती है और सफल होती है। सूर्य वायु को स्वच्छ करता है और समुद्र में नमक पानी को गंदा होने से बचाता है अल बिरुनी जोर देकर कहता है कि यदि ऐसा नहीं होता, तो पृथ्वी पर जीवन असम्भव हो जाता। उसके अनुसार जाति-व्यवस्था में शामिल अपवित्रता की अवधारणा प्रकृति के नियमों के विरुद्ध थी।

प्रश्न 27.
” जाति व्यवस्था के विषय में अल-बिरुनी का विवरण संस्कृत ग्रन्थों पर आधारित था।” स्पष्ट कीजिए।
उत्तर:
जाति व्यवस्था के विषय में अल बिरुनी का विवरण संस्कृत ग्रन्थों के अध्ययन से पूर्ण रूप से प्रभावित था। इन ग्रन्थों में ब्राह्मणों के दृष्टिकोण से जाति-व्यवस्था को संचालित करने वाले नियमों का प्रतिपादन किया गया था परन्तु वास्तविक जीवन में यह व्यवस्था इतनी कठोर नहीं थी। उदाहरणार्थ, अन्त्यजों (जाति-व्यवस्था से परे रहने वाले लोग) से प्रायः यह अपेक्षा की जाती थी कि वे किसानों और जमींदारों के लिए सस्ता श्रम प्रदान करें ये आर्थिक तन्त्र में सम्मिलित थे।

प्रश्न 28.
अल बिरूनी द्वारा उल्लिखित भारत की वर्ण-व्यवस्था का वर्णन कीजिए।
उत्तर:

  1. ब्राह्मण-ब्राह्मणों की जाति सबसे ऊँची थी। ब्राह्मण ब्रह्मन् के सिर से उत्पन्न हुए थे। इन्हें सबसे उत्तम माना जाता था।
  2. क्षत्रिय ब्राह्मणों के बाद दूसरी जाति क्षत्रियों की थी जिनका जन्म ब्रह्मन् के कन्धों और हाथों से हुआ • था। उनका दर्जा ब्राह्मणों से अधिक नीचा नहीं था।
  3. वैश्य क्षत्रियों के बाद वैश्य आते हैं। इनका जन्म ब्रह्मन् की जंघाओं से हुआ था।
  4. शूद्र इनका उद्भव ब्रह्मन् के चरणों से हुआ था।

प्रश्न 29.
“इब्नबतूता में अनजाने को जानने की लालसा कूट-कूटकर भरी हुई थी।” स्पष्ट कीजिए । अपरिचित को रेखांकित करने का उसका क्या उद्देश्य था?
उत्तर:
इब्नबतूता ने भारत में व्यापक यात्राएँ कीं और अपरिचित को जानने का भरसक प्रयास किया। उसके वृत्तान्तों में धर्मनिष्ठ लोगों, क्रूर और दयालु शासकों, सामान्य पुरुषों तथा महिलाओं और उनके जीवन की कहानियाँ शामिल थीं। इब्नबतूता को इन कहानियों में जो भी कुछ अपरिचित लगा था, उसे उसने विशेष रूप से रेखांकित किया ताकि श्रोता अथवा पाठक सुदूर देशों के वृत्तान्तों से पूर्ण रूप से प्रभावित हो सकें।

प्रश्न 30.
इब्नबतूता ने नारियल का वर्णन किस प्रकार किया है?.
उत्तर:
इब्नबतूता के अनुसार नारियल के वृक्ष स्वरूप में सबसे अनोखे और प्रकृति में सबसे आश्चर्यजनक वृक्षों में से हैं। ये बिल्कुल खजूर के वृक्ष जैसे दिखते हैं। इनमें केवल एक अन्तर है कि नारियल से काष्ठफल प्राप्त होता तथा दूसरे से खजूर नारियल मानव सिर से मेल खाता हैं क्योंकि इसमें भी दो आँखें तथा एक मुख है और अन्दर का भाग हरा होने पर मस्तिष्क जैसा दिखता है। इससे जुड़ा रेशा बालों जैसा दिखाई देता है वे इससे रस्सी बनाते हैं।

प्रश्न 31.
इब्नबतूता द्वारा पान का वर्णन किस प्रकार किया गया है? उसने पान का वर्णन क्यों किया? पान का किस प्रकार प्रयोग किया जाता था?
उत्तर:
इब्नबतूत ने पान का वर्णन इसलिए किया क्योंकि इससे उसके पाठक पूरी तरह से अपरिचित थे। इब्नबतूता के अनुसार पान का कोई फल नहीं होता और इसे केवल इसकी पत्तियों के लिए ही उगाया जाता था पान को अंगूर लता की तरह ही उगाया जाता था। पान के प्रयोग करने की विधि यह थी कि इसे खाने से पहले सुपारी ली जाती थी। इसके छोटे- छोटे टुकड़ों को मुंह में रखकर चबाया जाता था। इसके पश्चात् पान की पत्तियों के साथ इन्हें चबाया जाता था।

प्रश्न 32.
इब्नबतूता ने भारतीय शहरों के सम्बन्ध में जो लिखा है, उस पर प्रकाश डालिए।
अथवा
इब्नबतूता ने भारतीय शहरों का किस प्रकार वर्णन किया है?
उत्तर:
इब्नबतूता के अनुसार भारतीय शहर पनी आबादी वाले तथा समृद्ध थे, परन्तु ये कभी-कभी युद्धों तथा अभियानों में विनष्ट हो जाते थे। अधिकांश शहरों में भीड़-भीड़ वाली सड़कें तथा चमक-दमक वाले और रंगीन बाजार थे जो विविध प्रकार की वस्तुओं से भरे रहते थे। दिल्ली एक बड़ा शहर था जिसकी आबादी बहुत अधिक थी तथा यह भारत में सबसे बड़ा शहर था। दौलताबाद (महाराष्ट्र में ) भी कम नहीं था और आकार में दिल्ली को चुनौती देता था।

प्रश्न 33.
इब्नबतूता के अनुसार भारतीय शहरों के बाजारों की क्या विशेषताएँ थीं?
उत्तर:
इब्नबतूता के अनुसार भारतीय शहरों के बाजार चमक-दमक वाले तथा रंगीन थे जहाँ विविध प्रकार की वस्तुएँ उपलब्ध रहती थीं ये बाजार केवल आर्थिक विनिमय के स्थान ही नहीं थे बल्कि ये सामाजिक तथा आर्थिक गतिविधियों के केन्द्र भी थे। अधिकांश बाजारों में एक मस्जिद तथा एक मन्दिर होता था और उनमें से कम से कम कुछ में तो नर्तकों, संगीतकारों एवं गायकों के सार्वजनिक प्रदर्शन के स्थान भी निर्धारित थे।

JAC Class 12 History Important Questions Chapter 5 यात्रियों के नज़रिए : समाज के बारे में उनकी समझ

प्रश्न 34.
इब्नबतूता द्वारा उल्लिखित दिल्ली ( देहली) शहर की विशेषताओं का वर्णन कीजिए।
उत्तर:
दिल्ली बड़े क्षेत्र में फैला पनी जनसंख्या वाला शहर था। शहर के चारों ओर बनी प्राचीर अतुलनीय थी। इसके अन्दर रात्रि के पहरेदार तथा द्वारपालों के कक्ष थे। प्राचीरों के अन्दर अनेक भंडार गृह बने हुए थे इस शहर के अट्ठाईस द्वार थे जिन्हें ‘दरवाजा’ कहा जाता है। इनमें से ‘बदायूँ दरवाजा’ सबसे विशाल था मांडवी दरवाजे के भीतर एक अनाज मंडी थी तथा गुल दरवाजे की बगल में एक फलों का बगीचा था।

प्रश्न 35.
इब्नबतूता के वृत्तान्त से भारतीय कृषि, व्यापार और वाणिज्य के बारे में क्या जानकारी मिलती है?
उत्तर:
इब्नबतूता के अनुसार भारतीय कृषि के अत्यधिक उत्पादनकारी होने का कारण भूमि का उपजाऊपन था। इस वजह से किसान वर्ष में दो फसलें उगाते थे। भारतीय उपमहाद्वीप व्यापार तथा वाणिज्य के अन्तर एशियाई तन्त्रों से अच्छी तरह से जुड़ा हुआ था भारतीय माल की मध्य तथा दक्षिण-पूर्वी एशिया, दोनों में बहुत मांग थी, जिससे शिल्पकार तथा व्यापारी बहुत लाभ कमाते थे भारतीय सूती कपड़े, महीन मलमल, रेशम, जी तथा साटन की बहुत अधिक माँग थीं।

प्रश्न 36.
इब्नबतूता ने दौलताबाद के संगीत बाजार का क्या वृत्तान्त दिया है? अपने वर्णन में इब्नबतूता ने इन गतिविधियों को उजागर क्यों किया?
उत्तर:
इब्नबतूता के अनुसार दौलताबाद में पुरुष और महिला गायकों के लिए एक बाजार था जिसे ‘ताराववाद’ कहते थे। यहाँ अनेक दुकानें थीं जिन्हें कालीनों से सजाया गया था। दुकान के मध्य में एक झूला था, जिस पर गायिका बैठती थी। बाजार के मध्य में एक विशाल गुम्बद खड़ा था, जिसमें कालीन बिछाए गए थे। इस बाजार में इबादत के लिए मस्जिदें बनी हुई थीं।

प्रश्न 37.
अपने वर्णन में इब्नबतूता ने इन गतिविधियों का उल्लेख क्यों किया?
उत्तर:
हमारे विचार में इब्नबतूता ने अपने विवरण में इन गतिविधियों का वर्णन इसलिए किया था क्योंकि इस प्रकार के संगीत के बाजारों से उसके पाठक अपरिचित थे। अतः उसने इन गतिविधियों का वर्णन किया ताकि उसके पाठक इन वृत्तान्तों से प्रभावित हो सकें।

प्रश्न 38.
अब्दुररज्जाक ने अपने यात्रा-वृत्तान्त में दक्षिण भारत का किस प्रकार वर्णन किया है?
उत्तर:
1440 के दशक में लिखा गया अब्दुरज्जाक का यात्रा-वृत्तान्त संवेगों और अवबोधनों का एक रोचक मिश्रण है। उसने केरल में कालीकट (आधुनिक कोलीकोड) बन्दरगाह पर जो देखा, उसे प्रशंसनीय नहीं माना। उसने लिखा है कि “यहाँ ऐसे लोग बसे हुए थे, जिनकी कल्पना उसने कभी भी नहीं की थी।” इन लोगों को उसने एक ‘विचित्र देश’ बताया।

प्रश्न 39.
अब्दुररज्जाक द्वारा वर्णित मंगलौर के मन्दिर का विवरण प्रस्तुत कीजिए।
उत्तर:
अब्दुररज्याक ने लिखा है कि मंगलौर से 9 मील के भीतर ही उसने एक ऐसा पूजा स्थल देखा जो सम्पूर्ण विश्व में अतुलनीय है। यह वर्गाकार था तथा चार द्वार मंडपों के साथ काँसे से ढका हुआ था प्रवेश-द्वार के द्वार-मंडप में सोने की बनी एक मूर्ति थी जो मानव- आकृति जैसी तथा आदमकद थी इसकी दोनों आँखों में काले रंग के माणिक इतनी चतुराई से लगाए गए थे कि ऐसा लगता था मानो वह देख सकती हों। यह शिल्प और कारीगरी अद्भुत थी।

प्रश्न 40.
इब्नबतूता ने भारत की डाक व्यवस्था को संचार की एक अनूठी प्रणाली क्यों बताया है?
अथवा
मध्यकालीन भारत में डाक प्रणाली पर प्रकाश डालिए।
उत्तर:
इब्नबतूता भारत की डाक प्रणाली की कार्यकुशलता देखकर बड़ा आश्चर्यचकित हुआ। इससे व्यापारियों के लिए न केवल लम्बी दूरी तक सूचना और उधार भेजना सम्भव हुआ बल्कि अल्प सूचना पर माल भेजना भी सम्भव हो गया। डाक प्रणाली इतनी कुशल थी कि जहाँ सिन्ध से दिल्ली की यात्रा में पचास दिन लगते थे, वहीं गुप्तचरों की सूचनाएँ सुल्तान तक इस डाक- व्यवस्था के द्वारा केवल पाँच दिनों में पहुँच जाती थीं।

JAC Class 12 History Important Questions Chapter 5 यात्रियों के नज़रिए : समाज के बारे में उनकी समझ

प्रश्न 41.
सोलहवीं तथा सत्रहवीं शताब्दियों में भारत में आने वाले तीन यूरोपीय यात्रियों का उल्लेख कीजिए।
उत्तर:

  1. सोलहवीं शताब्दी में दुआर्ते बरबोसा नामक यूरोपीय यात्री ने दक्षिण भारत में व्यापार और समाज का एक विस्तृत विवरण लिखा।
  2. सत्रहवीं शताब्दी में ज्यों वैप्टिस्ट तैवर्नियर नामक एक फ्रांसीसी जौहरी ने भारत की कम से कम 6 बार यात्रा की वह भारत की व्यापारिक स्थितियों से बड़ा प्रभावित था।
  3. सत्रहवीं शताब्दी में इतालवी चिकित्सक मनूकी भारत आए और यहीं बस गए।

प्रश्न 42.
फ्रांस्वा बर्नियर का संक्षिप्त परिचय दीजिए।
उत्तर:
फ्रांस्वा बर्नियर फ्रांस का निवासी था। यह एक चिकित्सक, राजनीतिक दार्शनिक तथा एक इतिहासकार था। यह मुगल साम्राज्य में अवसरों की तलाश में भारत आया था। वह 1656 से 1668 ई. तक भारत में बारह वर्ष तक रहा और मुगल दरबार से घनिष्ठ सम्बन्ध बनाए रखे। प्रारम्भ में उसने मुगल सम्राट शाहजहाँ के ज्येष्ठ पुत्र दाराशिकोह के चिकित्सक के रूप में कार्य किया तथा बाद में एक मुगल अमीर दानिशमन्द खान के साथ कार्य किया।

प्रश्न 43.
डच यात्री पेलसर्ट ने भारत में व्याप्त व्यापक गरीबी का किस प्रकार वर्णन किया है?
उत्तर:
पेलसर्ट नामक एक हच यात्री भारत के लोगों में व्याप्त गरीबी को देखकर बड़ा आश्चर्यचकित था। उसने लिखा है कि “लोग इतनी अधिक और दुःखद गरीबी में रहते थे कि उनके जीवन को मात्र नितान्त अभाव के घर और कठोर कष्ट दुर्भाग्य के आवास के रूप में चित्रित किया जा सकता है।” पेलसर्ट के अनुसार, “कृषकों को इतना अधिक निचोड़ा जाता था कि पेट भरने के लिए उनके पास सूखी रोटी भी कठिनाई से बचती थी।”

प्रश्न 44.
“बर्नियर का उद्देश्य यूरोप की श्रेष्ठता को दर्शाना तथा भारतीय स्थितियों को दयनीय बताना था।” स्पष्ट कीजिए।
उत्तर:
बर्नियर निरन्तर मुगलकालीन भारत की तुलना तत्कालीन यूरोप से करता रहा और प्रायः यूरोप की श्रेष्ठता को दर्शाता रहा। वह यूरोपीय प्रथाओं, रीति-रिवाज और प्रशासनिक व्यवस्था को श्रेष्ठ दर्शाना चाहता था तथा भारतीय परिस्थितियों को दयनीय बताना चाहता था। उसने भारत को यूरोप के प्रतिलोम के रूप में दिखाया है। उसने भारत में जो भिताएँ अनुभव की उन्हें भी पदानुक्रम के अनुसार क्रमबद्ध किया ताकि भारत पश्चिमी संसार को निम्न कोटि का लगे।

प्रश्न 45.
भूस्वामित्व के सम्बन्ध में वर्नियर के विचारों को संक्षेप में स्पष्ट कीजिए।
अथवा
बर्नियर के अनुसार भूमि स्वामित्व के प्रश्न पर भारत और यूरोप के बीच क्या भिन्नता थी? राजकीय भूस्वामित्व राज्य तथा उसके निवासियों के लिए क्यों हानिकारक था?
उत्तर:
बर्नियर के अनुसार भारत और यूरोप के बीच मूल भिन्नताओं में से एक भारत में निजी भूस्वामित्व का अभाव था। बर्नियर निजी स्वामित्व का समर्थक था। उसके अनुसार भूमि पर राजकीय स्वामित्व राज्य तथा उसके निवासियों, दोनों के लिए हानिकारक था उसने यह महसूस किया कि मुगल साम्राज्य में सम्राट सम्पूर्ण भूमि का स्वामी था जो इसे अपने अमीरों में बांटता था और इसके अर्थव्यवस्था तथा समाज के लिए विनाशकारी परिणाम होते थे।

प्रश्न 46.
बर्नियर के अनुसार राजकीय भूस्वामित्व राज्य के निवासियों के लिए क्यों विनाशकारी था?
उत्तर:
अनियर के अनुसार राजकीय भूस्वामित्व के कारण, भूधारक अपने बच्चों को भूमि नहीं दे सकते थे। इसलिए वे उत्पादन के स्तर को बनाए रखने और उसमें वृद्धि के लिए प्रयास नहीं करते थे। निजी भूस्वामित्व के अभाव ने बेहतर भूधारकों को पनपने से रोका। इसी वजह से कृषि का विनाश हुआ, किसानों को अत्यधिक उत्पीड़न का सामना करना पड़ा तथा समाज के सभी वर्गों के जीवन स्तर में लगातार पतन की स्थिति उत्पन्न हुई।

प्रश्न 47.
बर्नियर द्वारा वर्णित भारतीय ग्रामीण क्षेत्रों के कृषकों की दशा का वृत्तान्त प्रस्तुत कीजिये।
उत्तर:
बर्नियर के अनुसार भारत के कई ग्रामीण अंचल रेतीली भूमियाँ या बंजर पर्वत थे। यहाँ की खेती अच्छी नहीं थी और इन क्षेत्रों की आबादी भी कम थी। यहाँ श्रमिकों के अभाव में कृषि योग्य भूमि का एक बड़ा भाग कृषि विहीन रह जाता था। कई श्रमिक गवर्नरों द्वारा किये गए अत्याचारों के फलस्वरूप मर जाते थे। गरीबों को न केवल जीवन निर्वहन के साधनों से वंचित कर दिया जाता था, बल्कि उनके बच्चों को दास बना लिया जाता था।

JAC Class 12 History Important Questions Chapter 5 यात्रियों के नज़रिए : समाज के बारे में उनकी समझ

प्रश्न 48.
“बर्नियर भारतीय समाज को दरिद्र लोगों के जनसमूह से बना वर्णित करता है।” स्पष्ट कीजिए। उत्तर बर्नियर के अनुसार भारतीय समाज दरिद्र लोगों के समरूप जनसमूह से बना था। यह वर्ग एक अत्यन्त अमीर तथा शक्तिशाली शासक वर्ग के द्वारा अधीन बनाया जाता था। शासक वर्ग के लोग अल्पसंख्यक होते थे गरीबों में सबसे गरीब तथा अमीरों में सबसे अमीर व्यक्ति के बीच नाममात्र को भी कोई सामाजिक समूह या वर्ग नहीं था बर्नियर दृढ़तापूर्वक कहता है कि “भारत में मध्या की स्थिति के लोग नहीं हैं।”

प्रश्न 49.
बर्नियर ने मुगल साम्राज्य को जिस रूप में देखा, उसका वर्णन कीजिए।
उत्तर:
बर्नियर के अनुसार मुगल साम्राज्य का राजा भिखारियों तथा क्रूर लोगों का राजा था मुगल साम्राज्य के शहर और नगर विनष्ट तथा ‘खराब वायु’ से दूषित थे और इसके खेत ‘शाड़ीदार’ तथा ‘घातक दलदल’ से परिपूर्ण।। इसका केवल एक ही कारण था-राजकीय भूस्वामित्व।

प्रश्न 50.
बर्नियर ने मुगल साम्राज्य को भूमि का एकमात्र स्वामी बताया है। क्या इसकी पुष्टि मुगल साक्ष्यों धे से होती है?
उत्तर:
एक भी सरकारी मुगल दस्तावेज यह नहीं दर्शाता कि राज्य ही भूमि का एकमात्र स्वामी था। उदाहरण के लिए अकबर के काल के सरकारी इतिहासकार अबुल फजल ने भूमि राजस्व को ‘राजत्व का पारिश्रमिक’ बताया है जो राजा द्वारा अपनी प्रजा को सुरक्षा प्रदान करने के बदले की गई माँग लगती है, न कि अपने स्वामित्व वाली भूमि पर लगान कुछ यूरोपीय यात्री ऐसी मांगों को लगान मानते थे परन्तु वास्तव में यह न तो लगान था, न ही भूमिकर, बल्कि उपज पर लगने वाला कर था।

प्रश्न 51.
“बर्नियर द्वारा प्रस्तुत भारतीय ग्रामीण समाज का चित्रण सच्चाई से बहुत दूर था।” स्पष्ट कीजिए।
उत्तर:
बर्नियर द्वारा प्रस्तुत भारतीय ग्रामीण समाज का चित्रण सच्चाई से बहुत दूर था सोलहवीं तथा सहव शताब्दी में ग्रामीण समाज में चारित्रिक रूप से बड़े पैमाने पर सामाजिक और आर्थिक विभेद था। एक ओर बड़े जमींदार थे, जो भूमि पर उच्चाधिकारों का उपभोग करते थे तथा दूसरी ओर अस्पृश्य भूमिहीन श्रमिक (बलाहार) थे। इन दोनों के बीच में बड़ा किसान था तथा साथ ही कुछ छोटे किसान भी थे, जो बड़ी कठिनाई से अपने गुजारे लायक उत्पादन कर पाते थे।

प्रश्न 52.
बर्नियर ने अपने वृत्तान्त में किस अधिक जटिल सामाजिक सच्चाई का उल्लेख किया है?
उत्तर:
बर्नियर लिखता है कि शिल्पकारों को अपने उत्पादों की वृद्धि के लिए कोई प्रोत्साहन नहीं दिया जाता था क्योंकि समस्त लाभ राज्य द्वारा ही प्राप्त कर लिया जाता था। इसलिए उत्पादन सर्वत्र पतनोन्मुख था। इसके साथ ही बनियर ने यह भी स्वीकार किया है कि सम्पूर्ण विश्व से बड़ी मात्रा में बहुमूल्य धातुएँ भारत में आती थीं क्योंकि उत्पादों का सोने और चांदी के बदले निर्यात होता था बर्नियर ने एक समृद्ध व्यापारिक समुदाय के अस्तित्व का भी उल्लेख किया है।

प्रश्न 53.
बर्नियर ने यूरोपीय राजाओं को मुगल ढाँचे का अनुसरण करने पर क्या चेतावनी दी है? उसने सर्वनाश के दृश्य का चित्रण किस प्रकार किया है?
उत्तर:
बर्नियर के अनुसार यूरोपीय राज्य इस प्रकार अच्छी तरह से जुते और बसे हुए इतनी अच्छी प्रकार से निर्मित तथा इतने समृद्ध नहीं रह जायेंगे, जैसा कि लोग उन्हें देखते हैं वे शीघ्र ही रेगिस्तान तथा निर्जन स्थानों के, भिखारियों तथा क्रूर लोगों के राजा बनकर रह जायेंगे जैसे | कि मुगल शासक हम उन महान शहरों और नगरों को खराब वायु के कारण न रहने योग्य अवस्था में पाएँगे। विनाश की स्थिति में टीले और झाड़ियाँ अथवा पातक दलदल से भरे खेत ही रह जायेंगे।

प्रश्न 54.
बर्नियर ने अपने वृत्तान्त में भारतीय कृषि तथा शिल्प उत्पादन की उन्नत स्थिति का किस प्रकार वर्णन किया है?
उत्तर:
बर्नियर ने लिखा है कि देश का अधिकांश भू- भाग अत्यधिक उपजाऊ है। उदाहरण के लिए बंगाल राज्य चावल, मकई, रेशम कपास तथा नील के उत्पादन में से आगे है। यहाँ के शिल्पकार आलसी होते हुए भी गलीचों, जरी, कसीदाकारी कढ़ाई, सोने और चाँदी के स्वों तथा विभिन्न प्रकार के रेशमी एवं सूती वस्त्रों निर्माण का कार्य करने में संलग्न रहते हैं विश्व के सभी भागों में संचलन के बाद सोना और चाँदी कुछ सीमा तक खो जाता है।

प्रश्न 55.
बर्नियर द्वारा उल्लिखित राजकीय कारखानों की कार्यप्रणाली का वर्णन कीजिए।
उत्तर:
बर्नियर के अनुसार कई स्थानों पर बड़े कक्ष दिखाई देते थे, जिन्हें कारखाना अथवा शिल्पकारों की कार्यशाला कहते थे एक कक्ष में कसीदाकार एक मास्टर के निरीक्षण में कार्यरत रहते थे। एक अन्य कक्ष में सुनार कार्यरत थे। तीसरे कक्ष में चित्रकार तथा चौधे में प्रलाक्षा रस का रोगन लगाने वाले कार्यरत थे।

पाँचवें कक्ष में बढ़ई, खरादी, दर्जी तथा जूते बनाने वाले तथा हठे कक्ष में रेशम, जरी तथा बारीक मलमल का काम करने वाले कार्यरत थे। प्रश्न 56. बर्नियर के अनुसार ‘मुगलकालीन शहर’ ‘शिविर नगर’ थे। स्पष्ट कीजिए। जाता था। शासक वर्ग के लोग अल्पसंख्यक होते थे गरीबों में सबसे गरीब तथा अमीरों में सबसे अमीर व्यक्ति के बीच नाममात्र को भी कोई सामाजिक समूह या नहीं था। बर्नियर दृढ़तापूर्वक कहता है कि “भारत में की स्थिति के लोग नहीं हैं।”

JAC Class 12 History Important Questions Chapter 5 यात्रियों के नज़रिए : समाज के बारे में उनकी समझ

प्रश्न 49.
बर्नियर ने मुगल साम्राज्य को जिस रूप देखा, उसका वर्णन कीजिए।
उत्तर:
बर्नियर के अनुसार मुगल साम्राज्य का राज भिखारियों तथा क्रूर लोगों का राजा था मुगल साम्राज्य के शहर और नगर विनष्ट तथा ‘खराब वायु’ से दूषित और इसके खेत ‘शाड़ीदार’ तथा ‘घातक दलदल से परिपूर्ण थे। इसका केवल एक ही कारण था राजकीय भूस्वामित्व।

प्रश्न 50.
बर्नियर ने मुगल साम्राज्य को भूमि का एकमात्र स्वामी बताया है। क्या इसकी पुष्टि मुगल साक्ष्यों से होती है?
उत्तर:
एक भी सरकारी मुगल दस्तावेज यह नहीं दर्शाता कि राज्य ही भूमि का एकमात्र स्वामी था। उदाहरण के लिए अकबर के काल के सरकारी इतिहासकार अबुल फजल ने भूमि राजस्व को ‘राजत्व का पारिश्रमिक’ बताया है जो राजा द्वारा अपनी प्रजा को सुरक्षा प्रदान करने के बदले की गई माँग लगती है, न कि अपने स्वामित्व वाली भूमि पर लगान कुछ यूरोपीय यात्री ऐसी माँगों को लगान मानते थे परन्तु वास्तव में यह न तो लगान था, न ही भूमिकर, बल्कि उपज पर लगने वाला कर था।

प्रश्न 51.
” बर्नियर द्वारा प्रस्तुत भारतीय ग्रामीण समाज का चित्रण सच्चाई से बहुत दूर था।” स्पष्ट कीजिए।
उत्तर:
बर्नियर द्वारा प्रस्तुत भारतीय ग्रामीण समाज का चित्रण सच्चाई से बहुत दूर था सोलहवीं तथा सत्रहवीं शताब्दी में ग्रामीण समाज में चारित्रिक रूप से बड़े पैमाने पर सामाजिक और आर्थिक विभेद था। एक ओर बड़े जमींदार थे, जो भूमि पर उच्चाधिकारों का उपभोग करते थे तथा दूसरी ओर अस्पृश्य भूमिहीन श्रमिक (बलाहार) थे। इन दोनों के बीच में बड़ा किसान था तथा साथ ही कुछ छोटे किसान भी थे, जो बड़ी कठिनाई से अपने गुजारे लायक उत्पादन कर पाते थे।

प्रश्न 52.
बर्नियर ने अपने वृत्तान्त में किस अधिक जटिल सामाजिक सच्चाई का उल्लेख किया है?
उत्तर:
बर्नियर लिखता है कि शिल्पकारों को अपने उत्पादों की वृद्धि के लिए कोई प्रोत्साहन नहीं दिया जाता था क्योंकि समस्त लाभ राज्य द्वारा ही प्राप्त कर लिया जाता था। इसलिए उत्पादन सर्वत्र पतनोन्मुख था। इसके साथ ही बनियर ने यह भी स्वीकार किया है कि सम्पूर्ण विश्व से बड़ी मात्रा में बहुमूल्य धातुएँ भारत में आती थीं क्योंकि उत्पादों का सोने और चांदी के बदले निर्यात होता था बर्नियर ने एक समृद्ध व्यापारिक समुदाय के अस्तित्व का भी उल्लेख किया है।

प्रश्न 53.
बर्नियर ने यूरोपीय राजाओं को मुगल ढाँचे का अनुसरण करने पर क्या चेतावनी दी है? उसने सर्वनाश के दृश्य का चित्रण किस प्रकार किया है?
उत्तर:
बर्नियर के अनुसार यूरोपीय राज्य इस प्रकार अच्छी तरह से जुते और बसे हुए इतनी अच्छी प्रकार से निर्मित तथा इतने समृद्ध नहीं रह जायेंगे, जैसा कि लोग उन्हें देखते हैं वे शीघ्र ही रेगिस्तान तथा निर्जन स्थानों के, भिखारियों तथा क्रूर लोगों के राजा बनकर रह जायेंगे जैसे कि मुगल शासक हम उन महान शहरों और नगरों को खराब बायु के कारण न रहने योग्य अवस्था में पाएँगे। विनाश की स्थिति में टीले और झाड़ियाँ अथवा पातक दलदल से भरे खेत ही रह जायेंगे।

प्रश्न 54.
बर्नियर ने अपने वृत्तान्त में भारतीय कृषि तथा शिल्प उत्पादन की उन्नत स्थिति का किस प्रकार वर्णन किया है?
उत्तर:
बर्नियर ने लिखा है कि देश का अधिकांश भू- भाग अत्यधिक उपजाऊ है। उदाहरण के लिए बंगाल राज्य चावल, मकई, रेशम कपास तथा नील के उत्पादन में मिल से आगे है। यहाँ के शिल्पकार आलसी होते हुए भी गलीचों, जरी कसीदाकारी कढ़ाई, सोने और चांदी के यस्व तथा विभिन्न प्रकार के रेशमी एवं सूती वस्त्रों के निर्माण का कार्य करने में संलग्न रहते हैं। विश्व के सभी भागों में संचलन के बाद सोना और चाँदी भारत में आकर कुछ सीमा तक खो जाता है।

प्रश्न 55.
बर्नियर द्वारा उल्लिखित राजकीय कारखानों की कार्यप्रणाली का वर्णन कीजिए।
उत्तर:
बर्नियर के अनुसार कई स्थानों पर बड़े कक्ष दिखाई देते थे, जिन्हें कारखाना अथवा शिल्पकारों की कार्यशाला कहते थे। एक कक्ष में कसीदाकार एक मास्टर के निरीक्षण में कार्यरत रहते थे। एक अन्य कक्ष में सुनार कार्यरत थे। तीसरे कक्ष में चित्रकार तथा पौधे में प्रलाक्षा रस का रोगन लगाने वाले कार्यरत थे। पाँचवें कक्ष में बढ़ई, खरादी, दर्जी तथा जूते बनाने वाले तथा हठे कक्ष में रेशम, जरी तथा बारीक मलमल का काम करने वाले कार्यरत थे।

प्रश्न 56.
बर्नियर के अनुसार ‘मुगलकालीन शहर’ “शिविर नगर’ थे स्पष्ट कीजिए।
अथवा
बर्नियर के वृत्तान्त से उभरने वाले शहरी केन्द्रों के चित्र पर चर्चा कीजिये। बर्नियर भारतीय शहरों को किस रूप में देखता है?
उत्तर:
बनिंवर ने ‘मुगलकालीन शहरों’ को ‘शिविर नगर’ कहा है। शिविर नगरों से उसका अभिप्राय उन नगरों से था, जो अपने अस्तित्व और बने रहने के लिए राजकीय शिविरों पर निर्भर थे। उसका विचार था कि ये नगर राजकीय दरबार के आगमन के साथ अस्तित्व में आते थे तथा दरबार के कहीं और चले जाने के बाद ये तेजी से विलुप्त हो जाते थे। उसके अनुसार इन नगरों की सामाजिक और आर्थिक नींव व्यावहारिक नहीं होती थी और ये राजकीय संरक्षण पर आश्रित रहते थे।

प्रश्न 57.
बर्नियर के अनुसार “मुगलकालीन व्यापारी प्रायः सुदृढ़ सामुदायिक अथवा बन्धुत्व के सम्बन्धों से जुड़े होते थे।” स्पष्ट कीजिए।
उत्तर:
बर्नियर के अनुसार मुगलकाल में व्यापार प्रायः सुदृद् सामुदायिक अथवा वन्धुत्व के सम्बन्धों में जुड़े होते थे और अपनी जाति तथा व्यावसायिक संस्थाओं के माध्यम से संगठित रहते थे। पश्चिमी भारत में ऐसे समूहों को ‘महाजन’ कहा जाता था और उनका मुखिया ‘सेठ’ कहलाता था। अहमदाबाद जैसे शहरी केन्द्रों में सभी महाजनों का सामूहिक प्रतिनिधित्व व्यापारिक समुदाय के मुखिया द्वारा होता था, जिसे ‘नगर सेठ’ कहा जाता था।

JAC Class 12 History Important Questions Chapter 5 यात्रियों के नज़रिए : समाज के बारे में उनकी समझ

प्रश्न 58.
इब्नबतूता ने तत्कालीन दास-दासियों की स्थिति का क्या विवरण प्रस्तुत किया है?
अथवा
इब्नबतूता द्वारा दास प्रथा के सम्बन्ध में दिए गए साक्ष्यों का विवेचन कीजिये।
उत्तर:
इब्नबतूता के अनुसार पुरुष तथा महिला दास बाजारों में खुले आम बेचे जाते थे और नियमित रूप से भेंट में दिए जाते थे। दासों का प्रायः घरेलू श्रम के लिए ही प्रयोग किया जाता था। ये लोग और महिलाओं को ले जाते थे पालकी या डोले में पुरुषों घरेलू श्रम में लगे हुए दासों का मूल्य बहुत कम होता था इसलिए अधिकांश परिवार कम से कम एक या दो दासों को रख पाने में समर्थ थे। सुल्तान की सेवा में लगी हुई कुछ दासियाँ संगीत और गायन में निपुण थीं।

प्रश्न 59.
“भारतीय महिलाओं का जीवन सती प्रथा के अलावा कई और चीजों के चारों ओर घूमता था। ” स्पष्ट कीजिए।
उत्तर:
महिलाओं का जीवन सती प्रथा के अलावा कई और चीजों के चारों ओर घूमता भी था उनका श्रम कृषि तथा कृषि के अलावा होने वाले उत्पादन, दोनों में महत्त्वपूर्ण था। व्यापारिक घरानों की महिलाएँ व्यापारिक गतिविधियों में भाग लेती थीं। वे कभी-कभी वाणिज्यिक विवादों को न्यायालय के सामने भी ले जाती थीं। अतः यह सम्भव नहीं लगता है कि महिलाओं को उनके घरों के विशेष स्थानों तक परिसीमित कर रखा जाता था।

प्रश्न 60.
‘बर्नियर द्वारा किया गया भारत का चित्रण द्वि-विपरीतता के नमूने पर आधारित है।” स्पष्ट कीजिये।
उत्तर:
बर्नियर द्वारा किया गया भारत का चित्रण द्वि-विपरीतता के नमूने पर आधारित है। उसने भारत को यूरोप के प्रतिलोम के रूप में दिखाया है या फिर यूरोप के विपरीत देश के रूप में दर्शाया है। उसने जो भिन्नताएँ महसूस कीं, उन्हें भी पदानुक्रम के अनुसार क्रमबद्ध किया जिससे भारत यूरोपीय देशों को निम्न कोटि का प्रतीत हो।

प्रश्न 61.
बर्नियर ने सती प्रथा का विस्तृत विवरण देना क्यों उचित समझा ?
उत्तर:
बर्नियर भारत में प्रचलित उन सभी प्रथाओं का वर्णन करना चाहता था जिससे भारत यूरोपीय देशों की तुलना में एक निम्न कोटि का देश प्रतीत होता हो उस समय सभी समकालीन यूरोपीय यात्रियों तथा लेखकों के लिए महिलाओं से किया जाने वाला व्यवहार प्रायः पश्चिमी तथा पूर्वी देशों के बीच भिन्नता का एक महत्त्वपूर्ण संकेतक माना जाता था इसलिए बर्नियर ने सती प्रथा का वर्णन करना उचित समझा।

प्रश्न 62.
बर्नियर ने लाहौर में एक अल्पवयस्क विधवा के सती होने का किस प्रकार विवरण दिया है?
उत्तर:
लाहौर में बर्नियर ने एक अत्यन्त सुन्दर अल्पवयस्क विधवा को सती होते हुए देखा। यह बालिका बुरी तरह से रो रही थी परन्तु उसे कुछ ब्राह्मण लोगों तथा एक वृद्ध महिला की सहायता से उस अनिच्छुक पीड़िता को बलपूर्वक सती-स्थल की ओर ले जाया गया। उसे लकड़ियों पर बिठाया गया, उसके हाथ और पैर बांध दिए गए ताकि वह भाग न जाए और इस स्थिति में उस निर्दोष विधवा को जीवित जला दिया गया।

प्रश्न 63.
बर्नियर के विवरणों ने फ्रांसीसी दार्शनिक मान्टेस्क्यू को किस प्रकार प्रभावित किया?
उत्तर:
बर्नियर के विवरणों से प्रभावित होकर फ्रांसीसी दार्शनिक मॉन्टेस्क्यू ने वर्नियर के वृतान्त का प्रयोग प्राच्य निरंकुशवाद के सिद्धान्त को विकसित करने में किया। इस सिद्धान्त के अनुसार एशिया (प्राच्य अथवा पूर्व) में शासक अपनी प्रजा के ऊपर अपने प्रभुत्व का उपभोग करते थे, जिसे दासता तथा गरीबी की स्थितियों में रखा जाता था। इस तर्क का आधार यह था कि सम्पूर्ण भूमि पर राजा का स्वामित्व होता था।

प्रश्न 64.
बर्नियर के विवरणों से कार्ल मार्क्स किस प्रकार प्रभावित हुआ?
उत्तर:
कार्ल मार्क्स ने लिखा कि भारत ( तथा अन्य एशियाई देशों में ) उपनिवेशवाद से पहले अधिशेष का अधिग्रहण राज्य द्वारा होता था। इससे एक ऐसे समाज का उद्भव हुआ जो बड़ी संख्या में स्वतन्त्र तथा आन्तरिक रूप से समतावादी ग्रामीण समुदायों से बना था। इन ग्रामीण समुदायों पर राजकीय दरबार का नियंत्रण होता था और जब तक अधिशेष की आपूर्ति बिना किसी बाधा के जारी रहती थी इनकी स्वायत्तता का सम्मान किया जाता था।

प्रश्न 65.
इब्नबतूता भारत की डाक प्रणाली को देखकर चकित क्यों हो गया?
उत्तर:
इब्नबतूता भारत की डांक प्रणाली को देखकर चकित हो गया क्योंकि भारत की डाक प्रणाली इतनी कुशल थी कि जहाँ सिन्ध से दिल्ली यात्रा में पचास दिन लगाते थे, वहीं सुल्तान तक गुप्तचरों की खबर मात्र पाँच दिनों में ही पहुँच जाती थी। इसके अतिरिक्त इससे व्यापारियों के लिए न केवल लम्बी दूरी तक सूचना भेजी जा सकती श्री बल्कि अल्प सूचना पर माल भी भेजा जा सकता था।

JAC Class 12 History Important Questions Chapter 5 यात्रियों के नज़रिए : समाज के बारे में उनकी समझ

प्रश्न 66.
‘ट्रैवल्स इन द मुगल एम्पायर’ नामक ग्रन्थ में बर्नियर भारत को पश्चिमी जगत की तुलना में अल्प- विकसित व निम्न श्रेणी का दर्शाना चाहता था। स्पष्ट कीजिए।
उत्तर:
‘ट्रैवल्स इन द मुगल एम्पायर में बनिंबर की भारतीय उपमहाद्वीप की यात्राओं का वर्णन है। यह ग्रन्थ वर्नियर की गहन आलोचनात्मक चिन्तन दृष्टि का उदाहरण है, लेकिन बर्नियर का यह दृष्टिकोण पूर्वाग्रह से प्रेरित हैं तथा एकपक्षीय है। बर्नियर ने मुगलकालीन इतिहास को भारत की भौगोलिक, सामाजिक, आर्थिक परिस्थितियों को ध्यान में न रखकर एक वैश्विक ढाँचे में ढालने का प्रयास किया। वह यूरोप के परिप्रेक्ष्य में मुगलकालीन इतिहास की तुलना निरन्तर करने का प्रयास करता रहा। बर्नियर के अनुसार यूरोप की प्रशासनिक व्यवस्था, यूरोप की सामाजिक- आर्थिक स्थिति भारत से कहीं बेहतर है। बास्तव में उसका भारत चित्रण पूरी तरह प्रतिकूलता पर आधारित है।

प्रश्न 67.
इब्नबतूता ने भारतीय शहरों में क्या विशेषताएं देखीं? संक्षिप्त टिप्पणी लिखिए।
उत्तर:
इब्नबतूता ने भारतीय शहरों को उन्नत अवस्था में पाया। उसके अनुसार जो लोग जीवन को समग्रता से जीने की आकांक्षा रखते हैं, जिनके पास कला-कौशल, दृढ़ इच्छा-शक्ति और पर्याप्त साधन हैं, उनके लिए आगे बढ़ने हेतु भारत के शहरों में व्यापक अवसर मौजूद थे। भारतीय शहरों की आबादी धनी थी व शहरों में समृद्धि झलकती थी। परन्तु युद्ध आदि की विभीषिका का दुष्परिणाम शहरों को कभी-कभी भुगतना पड़ता था।

इब्नबतूता के अनुसार शहर के बाजार चहल-पहल तथा चमक-दमक से भरपूर विभिन्न प्रकार की व्यापारिक वस्तुओं से भरे रहते थे। भारत में दिल्ली एक विपुल आबादी वाला सबसे बड़ा शहर था महाराष्ट्र में स्थित दौलताबाद भी दिल्ली से किसी प्रकार कमतर नहीं था। बाजारों में केवल व्यापारिक गतिविधियाँ ही नहीं अन्य धार्मिक तथा सामाजिक गतिविधियाँ भी होती थीं। इब्नबतूता का यह वर्णन भारतीय शहरों की विकसित समृद्धि का व्यापक उदाहरण है।

प्रश्न 68.
अब्दुर रज्जाक द्वारा लिखित यात्रा वृत्तान्त संवेगों और अवबोधनों का एक रोचक मिश्रण है, कैसे?
उत्तर:
1440 के दशक में लिखा गया अब्दुर रजाक का यात्रा वृत्तान्त उसके शब्दों में, “यहाँ ऐसे लोग बसे हुए थे जिनकी कल्पना मैंने कभी नहीं की।” 1440 के दशक में अब्दुर रजाक केरल के कालीकट बन्दरगाह पर पहुँचा। इन लोगों को देखकर उसके मुँह से ‘विचित्र देश का सम्बोधन निकला। लेकिन कालान्तर में अपनी पुनः भारत यात्रा के दौरान वह मंगलौर आया और यहाँ पश्चिमी पाट को पार कर उसने एक मन्दिर देखा, जिसे देखकर वह बहुत अधिक प्रभावित हुआ। मन्दिर की स्थापत्य कला को देखकर वह मन्त्रमुग्ध रह गया और ‘विचित्र देश’ की छवि उसके मस्तिष्क से निकल गई।

प्रश्न 69.
पेलसर्ट नामक डच यात्री की भारत यात्रा का वर्णन संक्षेप में कीजिए।
उत्तर:
पेलसर्ट नामक एक डच यात्री ने सत्रहवीं शताब्दी के आरम्भिक दशकों में भारतीय उपमहाद्वीप की यात्रा की थी। बर्नियर की ही तरह वह भी लोगों में व्यापक गरीबी देखकर अचम्भित था “लोग इतनी अधिक तथा दुःखद गरीबी में रहते हैं कि इनके जीवन को मात्र नितान्त अभाव के पर तथा कठोर कष्ट दुर्भाग्य के आभास के रूप में चित्रित अथवा ठीक प्रकार से वर्णित किया जा सकता है।” राज्य को उत्तरदायी ठहराते हुए वह कहता है: ‘कृषकों को इतना अधिक निचोड़ा जाता है कि पेट भरने के लिए उनके पास सूखी रोटी भी मुश्किल से बचती है।”

प्रश्न 70.
भारतीय महिलाओं की मध्यकालीन सामाजिक स्थिति के बारे में यूरोपीय लेखकों के विचार क्या थे?
उत्तर:
विभिन्न यूरोपीय यात्री जो भारत आये उन्होंने सामाजिक परिस्थितियों के अन्तर्गत मध्यकालीन भारतीय महिलाओं की स्थिति पर अपने विचार व्यक्त किए हैं। इन विचारकों ने पाया कि भारतीय महिलाओं से किया जाने वाला व्यवहार पश्चिमी समाजों से सर्वथा भिन्न था। बर्नियर ने इस सन्दर्भ में भारत की क्रूरतम अमानवीय सती प्रथा का उदाहरण दिया है जो भारत के अतिरिक्त विश्व के किसी भी देश में नहीं थी।

पस्तु सती प्रथा के अतिरिक्त यूरोपीय लेखकों ने महिलाओं के सामाजिक जीवन के अन्य पक्षों पर भी अपना ध्यान केन्द्रित किया है। वे केवल घरों की चारदीवारी तक ही सीमित नहीं रहती थीं। कृषि तथा कृषि से सम्बन्धित कार्यों जैसे पशुपालन में उनके श्रम का महत्त्व था। व्यापारिक परिवारों की महिलाएँ व्यापारिक गतिविधियों में भागीदारी रखती थीं। कुछ महिलाएँ प्रशासनिक कार्यों में भी भाग लेती थीं।

JAC Class 12 History Important Questions Chapter 5 यात्रियों के नज़रिए : समाज के बारे में उनकी समझ

निबन्धात्मक प्रश्न

प्रश्न 1.
भारत के 10वीं से 17वीं सदी तक के इतिहास के पुनर्निर्माण में विदेशी यात्रियों के विवरणों का क्या योगदान है ? उदाहरण सहित स्पष्ट कीजिए।
उत्तर:
विदेशी यात्रियों के वृत्तान्त निश्चित रूप से इतिहास के निर्माण में सहायक होते हैं। यही स्थिति हम 10वीं से 17वीं शताब्दी के मध्य भी देखते हैं। इस काल में अनगिनत यात्री भारत आए तथा अपनी समझ के अनुसार भारत का विवरण प्रस्तुत किया। इस तथ्य को हम निम्न विन्दुओं के माध्यम से समझ सकते हैं-
(1) अधिकांश यात्री भिन्न-भिन्न देशों, भिन्न-भिन्न आर्थिक तथा सामाजिक परिदृश्य से आए थे।

(2) स्थानीय लेखक भारत की तत्कालीन परिस्थितियों का विवरण करने में रुचि नहीं रखते थे; जबकि इन विदेशी लेखकों ने भारत की तत्कालीन आर्थिक, सामाजिक तथा सांस्कृतिक गतिविधियों पर विशेष विवरण दिए हैं।

(3) 10वीं शताब्दी के आस-पास भारतीय विद्वान विदेशों के साथ सम्बन्ध बनाने तथा उनके विषय में जानने .में अधिक रूचि नहीं रखते थे। इसके फलस्वरूप ये तुलनात्मक अध्ययन करने में असमर्थ थे।

(4) विदेशी यात्रियों ने अपने विवरण में उन तथ्यों को अधिक महत्त्व दिया है; जो उन्हें विचित्र जान पड़ते थे। इससे उनके विवरण में रोचकता आ जाती है।

(5) विदेशी यात्रियों के विवरण तत्कालीन राजदरबार के क्रियाकलापों, धार्मिक विश्वास तथा स्थापना की प्रमुख विशेषताओं पर प्रकाश डालते हैं। इससे इतिहास निर्माण में अत्यधिक सहायता प्राप्त होती है। उपर्युक्त बिन्दुओं को हम पृथक्-पृथक् यारियों के विवरण के साथ समझ सकते हैं।

ये विवरण निम्नलिखित –
1. अल बिरूनी का विवरण अल-विरुनी का विवरण निम्नलिखित बिन्दुओं के अन्तर्गत किया गया है –

  • अल-विरुनी ने अपनी रचनाओं में भारत में व्याप्त जाति-व्यवस्था तथा वर्ण व्यवस्था का वर्णन किया है।
  • अल बिरूनी ने भारतीय समाज की रूढ़िवादिता को भी अपनी किताबों में दर्शाया है।
  • अल बिरूनी ने भारत के ज्योतिष, खगोल विज्ञान तथा गणित की विस्तारपूर्वक चर्चा की है।

2. इब्नबतूता का विवरण इससे सम्बन्धित मुख्य बिन्दु निम्न प्रकार है –

  • इब्नबतूता ने भारतीय डाक प्रणाली की प्रशंसा की है तथा उससे हमें परिचित भी कराया।
  • इब्नबतूता की रचनाओं द्वारा हमें यह पता चलता है कि उस समय भारत में नारियल तथा पान की खेती का व्यापक प्रचलन था।
  • इब्नबतूता ने दासों का भी विस्तारपूर्वक विवरण दिया है; जिससे यह ज्ञात होता है कि तत्कालीन समाज तथा राजनैतिक व्यवस्था में दासों की महत्वपूर्ण भूमिका थी।
  • इब्नबतूता की रचनाओं में भारतीय बाजारों तथा व्यापारियों की सम्पन्नता का उल्लेख भी मिलता है। जिससे यह पता चलता है कि तत्कालीन भारत आर्थिक रूप से समृद्ध था।

3. बर्नियर का विवरण बर्नियर द्वारा रचित ‘ट्रैवल्स इन द मुगल एम्पायर’ में दिया गया विवरण निम्न बिन्दुओं द्वारा किया जा सकता है-

  • बर्नियर के अनुसार भारतीयों को निजी भू-स्वामित्व का अधिकार प्राप्त नहीं है।
  • बर्नियर की रचनाओं में सती प्रथा का विवरण मिलता है।
  • बर्नियर ने मुगल सेना के साथ कश्मीर यात्रा की थी तथा उस यात्रा का विस्तृत विवरण दिया है। उस विवरण से यह ज्ञात होता है कि उस समय यात्रा में किन सामानों की आवश्यकता होती थी। इस प्रकार स्पष्ट होता है कि समकालीन यात्रियों के विवरण इतिहास को समझने तथा उसका निर्माण करने में अत्यधिक सहायक होते हैं।

प्रश्न 2.
अल बिरूनी के प्रारम्भिक जीवन का वर्णन करते हुए ‘किताब-उल-हिन्द’ की विशेषताओं पर प्रकाश डालिए।
उत्तर:
अल बिरूनी का प्रारम्भिक जीवन अल बिरूनी का जन्म आधुनिक उज्बेकिस्तान में स्थिता ख्वारिज्म में सन् 973 में हुआ था। ख्वारिज्म शिक्षा का एक प्रसिद्ध केन्द्र था। अल बिरूनी ने उस समय उपलब्ध सबसे अच्छी शिक्षा प्राप्त की। वह एक उच्च कोटि का विद्वान था तथा सीरियाई फारसी, हिब्रू और संस्कृत भाषाओं का ज्ञाता था। यद्यपि वह यूनानी भाषा का जानकार नहीं था, फिर भी वह प्लेटो तथा अन्य यूनानी दार्शनिकों की रचनाओं से परिचित था जिनका उसने अरबी अनुवादों के माध्यम से अध्ययन किया था। 1017 ई. में महमूद गजनवी ने ख्वारिज्म पर आक्रमण किया और यहाँ से कई विद्वानों तथा कवियों को अपने साथ अपनी राजधानी गजनी ले गया।

अल बिरुनी भी उनमें से एक था। वह एक बन्धक के रूप में गजनी आया था, परन्तु धीरे-धीरे उसकी गजनी शहर में रुचि बढ़ने लगी। उसने अपना शेष जीवन गजनी में ही बिताया। 70 वर्ष की आयु में गजनी में अल बिरुनी की मृत्यु हो गयी। भारत के प्रति रुचि बढ़ना-गजनी में रहते हुए अल- बिरनी की भारत के प्रति रुचि बढ़ने लगी। पंजाब को गलनी साम्राज्य में सम्मिलित करने के बाद स्थानीय लोगों से सम्पर्कों से आपसी विश्वास तथा समझ का वातावरण बना। अल-विरुनी ने ब्राह्मण पुरोहितों तथा विद्वानों के साथ कई वर्ष व्यतीत किए और संस्कृत, धर्म तथा दर्शन का ज्ञान प्राप्त किया।

JAC Class 12 History Important Questions Chapter 5 यात्रियों के नज़रिए : समाज के बारे में उनकी समझ

‘किताब-उल-हिन्द’ की विशेषताएं –
(1) विविध विषयों का विवेचन-किताब-उल- हिन्द अरबी में लिखी गई अल बिरूनी की प्रसिद्ध रचना है इसकी भाषा सरल और स्पष्ट है। यह एक विस्तृत ग्रन्थ है जिसमें धर्म और दर्शन, त्यौहारों, खगोल विज्ञान, कीमिया, रीति-रिवाजों तथा प्रथाओं, सामाजिक-जीवन, भार तौल, मापन विधियों, मूर्तिकला, कानून, मापतन्त्र विज्ञान आदि विषयों का विस्तृत विवेचन किया गया है। यह ग्रन्थ अस्सी अध्यायों में विभाजित है।

(2) विशिष्ट शैली अल-बिहनी ने प्रत्येक अध्याय में एक विशिष्ट शैली का प्रयोग किया है, जिसमें आरम्भ में एक प्रश्न होता था, फिर संस्कृतवादी परम्पराओं पर आधारित वर्णन था और अन्त में अन्य संस्कृतियों के साथ तुलना की गई थी। यह लगभग एक ज्यामितीय संरचना है।

(3) स्पष्टता तथा पूर्वानुमेयता यह ग्रन्थ अपनी स्पष्टता तथा पूर्वानुमेयता के लिए प्रसिद्ध है।

(4) भारतीय ग्रन्थों के अरबी भाषा में अनुवादों से परिचित अल बिरूनी ने सम्भवतः अपनी रचनाएँ भारतीय उपमहाद्वीप के सीमान्त क्षेत्रों में रहने वाले लोगों के लिए लिखी थीं। अल बिरुनी संस्कृत, पालि तथा प्राकृत ग्रन्थों के अरबी भाषा में अनुवादों तथा रूपान्तरणों से परिचित था।

(5) समालोचनात्मक दृष्टिकोण- इन ग्रन्थों की लेखन सामग्री तथा शैली के विषय में अल-बिरुनी का दृष्टिकोण समालोचनात्मक था और निश्चित रूप से वह उनमें सुधार करना चाहता था।

प्रश्न 3.
इब्नबतूता के प्रारम्भिक जीवन एवं उसकी यात्राओं का वर्णन कीजिए।
उत्तर:
इब्नबतूता का प्रारम्भिक जीवन इब्नबतूता मोरक्को का निवासी था उसका जन्म 1304 में तैंजियर नामक नगर के एक सम्मानित एवं शिक्षित परिवार में हुआ था। उसका परिवार इस्लामी कानून अथवा शरिया पर अपनी विशेषज्ञता के लिए प्रसिद्ध था अपने परिवार की परम्परा के अनुसार इब्नबतूता ने कम आयु में ही साहित्यिक तथा शास्वारूढ़ शिक्षा प्राप्त की। उसका यात्रा वृत्तान्त ‘रिला’ के नाम से प्रसिद्ध है।

(1) इब्नबतूता की यात्राएँ – अन्य विदेशी यात्रियों के विपरीत, इब्नबतूता पुस्तकों के स्थान पर यात्राओं से प्राप्त अनुभव को ज्ञान का अधिक महत्त्वपूर्ण स्रोत मानता था। इब्नबतूता को यात्राएँ करने का बहुत शौक था इसलिए वह नए-नए देशों तथा लोगों के विषय में जानकारी प्राप्त करने के लिए दूर-दूर के प्रदेशों तक में गया। 1332-33 ई. में भारत के लिए प्रस्थान करने से पहले वह मक्का की तीर्थ यात्राएँ और सीरिया, इराक, फारस, यमन, ओमान तथा पूर्वी अफ्रीका के अनेक तटीय व्यापारिक बन्दरगाहों की यात्राएँ कर चुका था।

(2) भारत की यात्रा – मध्य एशिया के मार्ग से होते हुए इब्नबतूता 1333 ई. में स्थल मार्ग से सिन्ध पहुँचा। उसने दिल्ली के सुल्तान मुहम्मद बिन तुगलक के बारे में सुन रखा था कि वह कला और साहित्य का उदार संरक्षक है अतः उसकी ख्याति से आकर्षित हो इब्नबतूता ने मुल्तान और कच्छ होते हुए दिल्ली को ओर प्रस्थान किया। मुहम्मद बिन तुगलक इब्नबतूता की विद्वता से बड़ा प्रभावित हुआ और उसे दिल्ली का काजी अथवा न्यायाधीश नियुक्त किया। वह इस पद पर कई वर्ष तक रहा।

(3) चीन के राजदूत के रूप में इब्नबतूता की नियुक्ति 1342 ई. में इब्नबतूता को मंगोल शासक के पास सुल्तान के दूत के रूप में चीन जाने का आदेश दिया गया। उसने दिल्ली से चीन के लिए प्रस्थान किया और मध्य भारत के रास्ते मालाबार तट की ओर बढ़ा। मालाबार से वह मालद्वीप गया तथा वहाँ वह अठारह महीनों तक काजी के पद पर रहा। अन्ततः उसने लंका जाने का निश्चय किया। बाद में वह एक बार पुनः मालाबार तट तथा मालद्वीप गया। चीन जाने के अपने कार्य को पुनः शुरू करने से भा पहले वह बंगाल तथा असम भी गया।

प्रश्न 4.
लगभग 1500 ई. के बाद भारत की यात्रा करने वाले यूरोपीय लेखकों का वर्णन कीजिए।
उत्तर:
लगभग 1500 ई. के बाद भारत की यात्रा करने वाले यूरोपीय लेखक लगभग 1500 ई. में भारत में पुर्तगालियों के आगमन के बाद उनमें से अनेक लोगों ने भारतीय सामाजिक रीति- रिवाजों तथा धार्मिक प्रथाओं के विषय में विस्तृत वृत्तान्त लिखे। जेसुइट राबट नोबिली भी एक ऐसा ही लेखक था जिसने भारतीय ग्रन्थों का यूरोपीय भाषाओं में अनुवाद भी किया।

JAC Class 12 History Important Questions Chapter 5 यात्रियों के नज़रिए : समाज के बारे में उनकी समझ

(1) दुआतें बरबोसा- दुआर्ते बरबोसा यूरोप का एक प्रसिद्ध लेखक था जिसने दक्षिण भारत में व्यापार और समाज का एक विस्तृत विवरण लिखा 1600 ई. के बाद भारत में आने वाले डच अंग्रेज और फ्रांसीसी यात्रियों की संख्या बढ़ने लगी थी।

(2) फ्रांसिस्को पेलसर्ट-फ्रांसिस्को पेलसर्ट एक डच यात्री था जिसने सत्रहवीं शताब्दी के आरम्भिक दशकों में उपमहाद्वीप की यात्रा की थी। वह यहाँ के लोगों में व्यापक गरीबी देखकर आश्चर्यचकित था। उसने कृषकों की अत्यन्त दयनीय दशा को मार्मिक चित्रण किया है।

(3) ज्यों-बैप्टिस्ट तैवर्नियर ज्यों-बैप्टिस्ट तैवर्नियर एक फ्रांसीसी जौहरी था जिसने कम से कम छः बार भारत की यात्रा की यह विशेष रूप से भारत की व्यापारिक स्थितियों से बहुत प्रभावित था। उसने भारत की तुलना ईरान और ओटोमन साम्राज्य से की।

(4) मनूकी मनूकी एक इतालवी चिकित्सक था। वह कभी भी यूरोप वापस नहीं गया और भारत में ही बस गया।

(5) फ्रांस्वा बर्नियर-फ्रांस का निवासी फ्रांस्वा बनियर एक चिकित्सक, राजनीतिक दार्शनिक तथा एक इतिहासकार था। वह अन्य लोगों की भांति मुगल साम्राज्य में अवसरों की तलाश में आया था। वह 1656 से 1668 तक भारत में बारह वर्ष तक रहा और मुगल दरबार से निकटता से जुड़ा रहा-पहले सम्राट शाहजहाँ के ज्येष्ठ पुत्र दारा शिकोह के चिकित्सक के रूप में तथा बाद में मुगल दरबार के एक आर्मीनियाई अमीर दानिशमंद खान के साथ एक बुद्धिजीवी तथा वैज्ञानिक के रूप में।

प्रश्न 5.
अल बिरुनी के यात्रा-वृत्तान्त का आलोचनात्मक वर्णन कीजिए।
उत्तर:
अल बिरूनी का यात्रा वृत्तान्त अल बिरूनी एक उच्च कोटि का विद्वान था। वह भारत में कई वर्षों तक रहा। उसने ब्राह्मण पुरोहितों तथा विद्वानों के साथ कई वर्ष बिताये और संस्कृत, धर्म तथा दर्शन का ज्ञान प्राप्त किया। अल बिरुनी की लेखन शैली की विशेषताएँ-अल- बिरुनी की लेखन शैली की प्रमुख विशेषताएँ निम्नलिखित किया।

  1. अल बिरूनी ने लेखन में अरबी भाषा का प्रयोग
  2. उसने सम्भवतः अपनी कृतियाँ उपमहाद्वीप के सीमान्त क्षेत्रों में रहने वाले लोगों के लिए लिखी थीं।
  3. वह संस्कृत, पालि तथा प्राकृत ग्रन्थों के अरबी भाषा में अनुवादों तथा रूपान्तरणों से परिचित था।
  4. इन ग्रन्थों की लेखन सामग्री शैली के विषय में उसका दृष्टिकोण आलोचनात्मक था और निश्चित रूप से वह उनमें सुधार करना चाहता था।

अल बिरूनी द्वारा अपवित्रता की मान्यता को स्वीकार करना-यद्यपि अल बिरुनी जाति-व्यवस्था के सम्बन्ध में ब्राह्मणवादी व्याख्या को मानता था, फिर भी उसने अपवित्रता की मान्यता को अस्वीकार कर दिया। उसने लिखा कि प्रत्येक यह वस्तु जो अपवित्र हो जाती है, अपनी पवित्रता की मूल स्थिति को पुनः प्राप्त करने का प्रयास करती है और सफल होती है। उसका कहना था कि जाति व्यवस्था में संलग्न अपवित्रता की अवधारणा प्रकृति के नियमों के विरुद्ध थी। भारत में प्रचलित वर्ण-व्यवस्था का विवरण अल- बिरूनी ने भारत में प्रचलित वर्ण व्यवस्था का उल्लेख अग्र प्रकार से किया है –

  • ब्राह्मण-ब्राह्मणों की जाति सबसे ऊँची थी। हिन्दू ग्रन्थों के अनुसार ब्राह्मण ब्रह्मन् के सिर से उत्पन्न हुए थे। हिन्दू ब्राह्मणों को मानव जाति में सबसे उत्तम मानते हैं।
  • क्षत्रिय अल-विरुनी के अनुसार ऐसी मान्यता थी कि क्षत्रिय ब्रह्मन् के कंधों और हाथों से उत्पन्न हुए थे। उनका दर्जा ब्राह्मणों से अधिक नीचा नहीं है।
  • वैश्य क्षत्रियों के बाद वैश्य आते हैं। वैश्य ब्रह्मन् की जंघाओं से उत्पन्न हुए थे।
  • शूद्र इनका जन्म ब्रह्मन् के चरणों से हुआ था।

अल बिरुनी के अनुसार अन्तिम दो वर्णों में अधिक अन्तर नहीं है। परन्तु इन वर्गों के बीच भिन्नता होने पर भी ये शहरों और गाँवों में मिल-जुलकर रहते हैं। जाति व्यवस्था के बारे में अल बिरूनी का विवरण संस्कृत ग्रन्थों पर आधारित होना जाति व्यवस्था के बारे में अल बिरुनी का विवरण संस्कृत ग्रन्थों के अध्ययन से पूर्णतया प्रभावित था। इन ग्रन्थों में ब्राह्मणों के दृष्टिकोण से जाति व्यवस्था का संचालन करने वाले नियमों का प्रतिपादन किया गया था। परन्तु वास्तविक जीवन में यह व्यवस्था इतनी कठोर नहीं थी।

प्रश्न 6.
इब्नबतूता द्वारा किए गए दिल्ली तथा दौलताबाद के वर्णन प्रस्तुत कीजिए।
अथवा
इब्नबतूता द्वारा वर्णित दिल्ली का संक्षिप्त विवरण दीजिये।
अथवा
दिल्ली के विशेष संदर्भ में भारतीय नगरों के बारे में इब्नबतूता के वृत्तान्तों की व्याख्या कीजिये।
उत्तर:
1. इब्नबतूता द्वारा किया गया दिल्ली का वर्णन –
(1) दिल्ली की बनावट इनवतृत के अनुसार दिल्ली बड़े क्षेत्र में फैला पनी जनसंख्या वाला शहर है। शहर के चारों ओर बनी प्राचीर अतुलनीय है दीवार की चौड़ाई ग्यारह हाथ (एक हाथ लगभग 20 इंच के बराबर) है और इसके भीतर रात्रि के पहरेदार तथा द्वारपालों के कक्ष हैं।

प्राचीरों के अन्दर खाद्य सामग्री, हथियार, बारूद, प्रक्षेपास्त्र तथा पेरेबन्दी में प्रयुक्त होने वाली मशीनों के संग्रह के लिए भंडार गृह बने हुए थे। प्राचीर में खिड़कियाँ बनी हैं जो शहर की ओर खुलती हैं तथा इन्हीं खिड़कियों के द्वारा प्रकाश भीतर आता है। प्राचीर का निचला भाग पत्थर से बना है तथा ऊपरी भाग ईंटों से निर्मित है।

(2) शहर के द्वार दिल्ली शहर के 28 द्वार हैं जिन्हें दरवाजा कहा जाता है और इनमें से बदायूँ दरवाजा सबसे विशाल है। मांडवी दरवाजे के भीतर एक अनाज मंडी है, गुल-दरवाजे की बगल में एक फलों का बगीचा है।

(3) कब्रगाह दिल्ली शहर में एक उत्तम कब्रगाह है, जिसमें बनी कब्रों के ऊपर गुम्बद बनाई गई है और गुम्बद विहीन कब्रों पर मेहराब बने हुए हैं। कब्रगाह में कदाकार चमेली तथा जंगली गुलाब जैसे फूल उगाए जाते हैं और फूल सभी ऋतुओं में खिले रहते हैं।

JAC Class 12 History Important Questions Chapter 5 यात्रियों के नज़रिए : समाज के बारे में उनकी समझ

2. इब्नबतूता द्वारा दौलताबाद का विवरण
(1) पुरुष और महिला गायकों का बाजार- इब्नबतूता के अनुसार दौलताबाद में पुरुष और महिला गायकों के लिए एक बाजार है, जिसे ‘तारामबाद’ कहते हैं यहाँ बहुत-सी दुकानें हैं और प्रत्येक दुकान में एक ऐसा दरवाजा है, जो मालिक के आवास में खुलता है। दुकानें कालीनों से सुसज्जित हैं और दुकान के मध्य में झूला है, जिस पर गायिका बैठती है।

(2) विशाल गुम्बद बाजार के मध्य में एक विशाल गुम्बद खड़ा है जिसमें कालीन बिछे हुए हैं और यह खूब सजाया गया है। इसमें प्रत्येक गुरुवार प्रातः काल की उपासना के बाद संगीतकारों के प्रमुख अपने सेवकों और दासों के साथ स्थान ग्रहण करते हैं। गायिकाएँ एक के बाद एक झुंडों में उनके समक्ष आकर सूर्यास्त का गीत गाती हैं और नृत्य करती हैं जिसके पश्चात् वे चले जाते हैं।

(3) मस्जिदें इब्नबतूता के अनुसार इस बाजार में इबादत के लिए मस्जिदें बनी हुई हैं जब भी कोई हिन्दू शासक इस बाजार से गुजरता था, वह गुम्बद में उत्तर कर आता था और गायिकाएँ उसके समक्ष गान प्रस्तुत करती थीं। यहाँ तक कि अनेक मुस्लिम शासक भी ऐसा ही करते थे।

प्रश्न 7.
इब्नबतूता द्वारा वर्णित भारत की डाक व्यवस्था का वर्णन कीजिए।
उत्तर:
इब्नबतूता द्वारा वर्णित भारत की डाक व्यवस्था इब्नबतूता के अनुसार व्यापारियों को प्रोत्साहित करने के लिए राज्य विशेष उपाय करता था। लगभग सभी व्यापारिक मार्गों पर सराय तथा विश्राम गृह स्थापित किए गए थे। इब्नबतूता भारत की डाक व्यवस्था की कार्यकुशलता देखकर बड़ा चकित हुआ।

इससे व्यापारियों के लिए न केवल लम्बी दूरी तक सूचना भेजना और उधार भेजना सम्भव हुआ, बल्कि इससे अल्पसूचना पर माल भेजना भी आसान हो गया। इब्नबतूता के अनुसार डाक प्रणाली इतनी कुशल थी कि जहाँ सिन्ध से दिल्ली की यात्रा में पचास दिन लगते थे, वहीं गुप्तचरों की सूचनाएं सुल्तान तक इस डाक व्यवस्था के द्वारा केवल पाँच दिनों में पहुँच जाती थीं।

डाक व्यवस्था इब्नबतूता के अनुसार भारत में दो प्रकार की डाक व्यवस्था थी –
(1) अश्व डाक व्यवस्था तथा
(2) पैदल डाक व्यवस्था।
(1) अश्व डाक व्यवस्था अश्व डाक व्यवस्था को ‘उल्लुक’ कहा जाता था। यह हर चार मील की दूरी पर स्थापित राजकीय घोड़ों द्वारा चालित होती थी।
(2) पैदल डाक व्यवस्था पैदल डाक व्यवस्था में प्रति मील तीन चौकियाँ होती थीं, जिन्हें ‘दावा’ कहा जाता था। यह एक मील का एक तिहाई होता था। हर | तीन मील पर घनी आबादी वाला एक गाँव होता था, जिसके बाहर तीन मण्डप होते थे, जिनमें लोग कार्य शुरू करने के लिए तैयार बैठे रहते थे। उनमें से प्रत्येक के पास दो हाथ लम्बी एक छड़ी होती थी, जिसके ऊपर ताँबे की घंटियाँ लगी होती थीं।

जब सन्देशवाहक शहर से यात्रा आरम्भ करता था, तो एक हाथ में पत्र तथा दूसरे में घटियाँ वाली छड़ लिए वह यथाशक्ति तेज भागता था जब मंडप में बैठे लोग घंटियों की आवाज सुनते थे, तो तैयार हो जाते थे। जैसे ही सन्देशवाहक उनके निकट पहुँचता था, उनमें से एक पत्र ले लेता था और वह छड़ी हिलाते हुए पूरी शक्ति से दौड़ता था, जब तक वह अगले दावा तक नहीं पहुँच जाता। पत्र के अपने गन्तव्य स्थान तक पहुँचने तक यही प्रक्रिया चलती रहती थी यह पैदल डाक व्यवस्था अश्व डाक व्यवस्था से अधिक तीव्र होती थी। इसका प्रयोग प्रायः खुरासान के फलों के परिवहन के लिए होता था, जिन्हें भारत में बहुत पसन्द किया जाता था।

प्रश्न 8.
इब्नबतूता द्वारा वर्णित भारतीय यात्रा- वृत्तांत का विवेचन कीजिए।
उत्तर:
इब्नबतूता का भारतीय यात्रा वृत्तांत इब्नबतूता के भारतीय यात्रा वृत्तांत का विवेचन निम्नलिखित बिन्दुओं के अन्तर्गत किया जा सकता है –
(1) नारियल – इब्नबतूता के अनुसार नारियल एक अनोखा तथा विस्मयकारी वृक्ष था। यह खजूर के वृक्ष जैसा दिखता था। इन दोनों वृक्षों में एक ही अन्तर था-नारियल से काष्ठफल प्राप्त होता था तथा दूसरे से खजूर नारियल के वृक्ष का फल मानव सिर से मेल खाता था। लोग नारियल के रेशे से रस्सी बनाते थे।

(2) पान इब्नबतूता के अनुसार पान को अंगूर लता की तरह ही उगाया जाता था। पान का कोई वृक्ष नहीं होता था और इसे केवल इसकी पत्तियों के लिए ही उगाया जाता था। इसे
खाने से पहले सुपारी ली जाती थी। इसके छोटे- छोटे टुकड़ों को मुँह में रखकर चनाया जाता था। इसके बाद पान की पत्तियों के साथ इन्हें चबाया जाता था।

(3) भारतीय शहर इब्नबतूता के अनुसार भारतीय शहर घनी आबादी वाले तथा समृद्ध थे परन्तु कभी-कभी युद्धों तथा अभियानों के दौरान नष्ट हो जाते थे। अधिकांश शहरों में भीड़-भाड़ वाली सड़कें तथा चमक-दमक वाले और रंगीन बाजार थे। ये बाजार विभिन्न प्रकार की वस्तुओं से भरे रहते थे। इब्नबतूता के अनुसार दिल्ली बहुत अधिक आबादी वाला शहर था। वह भारत में सबसे बड़ा शहर था दौलताबाद (महाराष्ट्र में भी कम नहीं था तथा आकार में दिल्ली को चुनौती देता था।’ शहरों में वस्त्र उद्योग उन्नत अवस्था में था। विदेशों में भारतीय सूती कपड़े, महीन मलमल, रेशम, जरी तथा साटन की अत्यधिक मांग थी।

(4) डाक व्यवस्था इब्नबतूता के अनुसार भारतीय डाक प्रणाली इतनी कुशल थी कि जहाँ सिन्ध से दिल्ली की यात्रा में पचास दिन लगते थे, वहीं गुप्तचरों की सूचनाएँ सुल्तान तक इस डाक व्यवस्था के द्वारा केवल पाँच दिनों में ही पहुँच जाती थीं। इब्नबतूता के अनुसार भारत में दो प्रकार की डाक व्यवस्था थी –

  • अश्व डाक व्यवस्था तथा
  • पैदल डाक व्यवस्था।

(5) महिलाएँ दासियाँ सती तथा अमिक- इब्नबतूता के अनुसार बाजारों में दास अन्य वस्तुओं की तरह खुले आम बेचे जाते थे और नियमित रूप से भेंट स्वरूप दिए जाते थे। दासों में विभेद – इब्नबतूता के अनुसार दासों में काफी विभेद था। सुल्तान की सेवा में कार्यरत कुछ दासियाँ संगीत और गायन में निपुण थीं।

इसके अतिरिक्त सुल्तान अपने अमीरों पर नजर रखने के लिए दासियों को भी नियुक्त करता था। घरेलू श्रम के लिए दासों का प्रयोग इब्नबतूता के अनुसार दासों का प्रायः घरेलू श्रम के लिए प्रयोग किया जाता था। ये लोग पालकी या डोले में पुरुषों और महिलाओं को ले जाते थे। घरेलू श्रम के काम में लगे हुए दासों एवं दासियों की कीमत बहुत कम होती थी। अतः अधिकांश परिवार एक-दो दास तो रखते ही थे।

JAC Class 12 History Important Questions Chapter 5 यात्रियों के नज़रिए : समाज के बारे में उनकी समझ

प्रश्न 9.
विदेशी यात्रियों द्वारा वर्णित भारतीय महिलाओं, दास-दासियों एवं सती प्रथा का विवरण प्रस्तुत कीजिये।
अथवा
इब्नबतूता एवं बर्नियर द्वारा वर्णित भारतीय महिलाओं, दास-दासियों एवं सती प्रथा का विवरण प्रस्तुत कीजिये।
उत्तर:
विदेशी यात्रियों द्वारा वर्णित दास- दासियों एवं सती प्रथा का विवरण –
(1) दास-दासियाँ – इब्नबतूता के अनुसार भारतीय बाजारों में दास अन्य वस्तुओं की भाँति खुले आम बेचे जाते थे और नियमित रूप से भेंटस्वरूप दिए जाते थे। सिन्ध पहुँचने पर इब्नबतूता ने सुल्तान मुहम्मद बिन तुगलक के लिए भेंट स्वरूप घोड़े, ऊँट तथा दास खरीदे। मुल्तान पहुँचने पर उसने गवर्नर को किशमिश तथा बादाम के साथ एक दास और घोड़ा भेंट के रूप में दिए। इब्नबतूता के अनुसार मुहम्मद बिन तुगलक ने नसीरुदीन नामक धर्मोपदेशक के प्रवचन से प्रसन्न होकर उसे एक लाख टके तथा तथा दो सौ दास दिए थे।

(2) दासों में विभेद इब्नबतूता के अनुसार दासों में काफी विभेद था। सुल्तान की सेवा में कार्यरत कुछ दासियाँ संगीत और गायन में निपुण थीं। इब्नबतूता सुल्तान की बहन की शादी के अवसर पर उनके प्रदर्शन से खूब आनन्दित हुआ।

(3) अमीरों की गतिविधियों की जानकारी के लिए दासियों की नियुक्ति इनवता के अनुसार सुल्तान मुहम्मद बिन तुगलक अपने अमीरों की गतिविधियों पर नजर रखने के लिए दासियों को नियुक्त करता था।

(4) दासों का घरेलू श्रम के लिए प्रयोग करना- इब्नबतूता के अनुसार दासों को सामान्यतः घरेलू श्रम के लिए ही प्रयुक्त किया जाता था। ये लोग पालकी या डोले में महिलाओं को ले जाते थे। दासों का मूल्य, विशेष रूप से उन दासियों का मूल्य, जिनका प्रयोग घरेलू श्रम के लिए किया जाता था, बहुत कम होता था। इसी वजह से अधिकांश परिवार कम से कम एक या दो दास तो रखते ही थे।

(5) सती प्रथा बर्नियर के अनुसार भारत में सती प्रथा प्रचलित थी। यद्यपि कुछ महिलाएँ प्रसन्नता से चिता में जल कर मर जाती थीं, परन्तु अनेक विधवाओं को मरने के लिए बाध्य किया जाता है। बर्नियर ने लाहौर में एक बारह वर्षीय विधवा की बलि का उल्लेख करते हुए लिखा है कि “इस बालिका को उसकी इच्छा के विरुद्ध जीवित जला दिया था। यह बालिका काँपते हुए बुरी तरह रो रही थी परन्तु तीन या चार ब्राह्मणों तथा एक बूढ़ी महिला की सहायता से उस अनिच्छुक बालिका को जबरन चिता स्थल की ओर ले जाया गया, उसे लकड़ियों पर बिठाया गया और उसके हाथ तथा पैर बाँध दिए गए ताकि वह भाग न जाए। इस प्रकार इस निर्दोष बालिका को जीवित जला दिया गया।”

प्रश्न 10.
“बर्नियर के विवरणों ने अठारहवीं शताब्दी से पश्चिमी विचारकों को प्रभावित किया।” विवेचना कीजिए।
उत्तर:
बर्नियर के विवरणों द्वारा पश्चिमी विचारकों को प्रभावित करना बर्नियर के विवरणों ने अठारहवीं शताब्दी से निम्नलिखित पश्चिमी विचारकों को प्रभावित किया –

(1) मान्टेस्क्यू-फ्रांस का प्रसिद्ध दार्शनिक मान्टेस्क्यू बर्नियर के विवरणों से बड़ा प्रभावित हुआ। उसने बर्नियर के वृत्तांत का प्रयोग प्राच्य निरंकुशवाद के सिद्धान्त को विकसित करने में किया। इस सिद्धान्त के अनुसार एशिया (प्राच्य अथवा पूर्व) में शासक अपनी प्रजा के ऊपर असीम प्रभुत्व का उपभोग करते थे तथा प्रजा को दासता तथा गरीबी की स्थितियों में रखा जाता था। इस तर्क का आधार यह था कि सम्पूर्ण भूमि पर राजा का स्वामित्व होता था तथा निजी सम्पत्ति अस्तित्व में नहीं थी। इस दृष्टिकोण के अनुसार राजा और उनके अमीर वर्ग को छोड़कर प्रत्येक व्यक्ति कठिनाई से गुजारा कर पाता था।

(2) कार्ल मार्क्स प्रसिद्ध साम्यवादी विचारक कार्ल मार्क्स भी बर्नियर के विवरणों से प्रभावित हुआ। उसने उन्नीसवीं सदी के इस विचार को एशियाई उत्पादन शैली के सिद्धान्त के रूप में और आगे बढ़ाया। उसने यह तर्क प्रस्तुत किया कि भारत तथा अन्य एशियाई देशों में उपनिवेशवाद से पहले राज्य अधिशेष का अधिग्रहण कर लेता था। इसके फलस्वरूप एक ऐसे समाज का प्रादुर्भाव हुआ जो काफी स्वायत्त तथा आन्तरिक से समताण से बना था। इन ग्रामीण समुदायों पर से ( का नियन्त्रण होता था। जब तक अधिशेष की आपूर्ति निरन्तर जारी रहती थी इनकी स्वायत्तता का सम्मान किया जाता था। यह एक निष्क्रिय प्रणाली मानी जाती थी।

ग्रामीण समाज का चित्रण सच्चाई से दूर होना- परन्तु ग्रामीण समाज का यह चित्रण सच्चाई से बहुत दूर था सोलहवीं तथा सत्रहवीं शताब्दी में ग्रामीण समाज में चारित्रिक रूप से बड़े पैमाने पर सामाजिक और आर्थिक विभेद था। एक ओर बड़े जमींदार थे जो भूमि पर उच्चाधिकारों का उपभोग करते थे और दूसरी ओर ‘अस्पृश्य’ भूमिविहीन श्रमिक इन दोनों के बीच में बड़ा किसान था जो किराए के श्रम का प्रयोग करता था और माल उत्पादन में जुटा रहता था। इसके साथ ही कुछ छोटे किसान भी थे जो कठिनाई से ही अपने गुजारे योग्य उत्पादन कर पाते थे।

JAC Class 12 History Important Questions Chapter 5 यात्रियों के नज़रिए : समाज के बारे में उनकी समझ

प्रश्न 11.
फ्रांस्वा बर्नियर द्वारा की गई ‘पूर्व और पश्चिम की तुलना को उल्लेखित कीजिए।
अथवा
बर्नियर द्वारा वर्णित भारतीय यात्रा वृत्तान्त का विवेचन कीजिए।
अथवा
“बर्नियर का ग्रन्थ ‘ट्रेवल्स इन मुगल एम्पायर अपने गहन प्रेक्षण, आलोचनात्मक अन्तर्दृष्टि तथा गहन चिन्तन के लिए उल्लेखनीय है।” व्याख्या कीजिए।
उत्तर:
(1) फ्रांस्वा बर्नियर द्वारा की गई ‘पूर्व और पश्चिम’ की तुलना बर्नियर प्राय: भारत में जो देखता था, उसकी तुलना यूरोपीय स्थिति से करता था। वह यूरोपीय रा संस्कृति की श्रेष्ठता का प्रबल समर्थक था यूरोपीय स्थितियों के मुकाबले में वह भारत की स्थितियों को दयनीय दर्शाना चाहता था। यही कारण है कि लगभग प्रत्येक दृष्टान्त में बर्नियर ने भारत की स्थिति को यूरोप में हुए विकास की अ तुलना में दयनीय बताया।

(2) भूमि स्वामित्व का प्रश्न बर्नियर के अनुसार भारत और यूरोप के बीच मूल भिन्नताओं में से एक भारत में निजी भूस्वामित्व का अभाव था। बर्नियर के अनुसार भूमि = पर राजकीय स्वामित्व राज्य तथा उसके निवासियों, दोनों के लिए हानिकारक था। उसका विचार था कि मुगल साम्राज्य में सम्राट समस्त भूमि का स्वामी था जो इसे अपने अमीरों में बांटता था। इसके अर्थव्यवस्था और समाज दोनों के लिए विनाशकारी परिणाम होते थे।

(3) किसानों की दयनीय दशा-बर्नियर के अनुसार ग्रामीण अंचलों में रहने वाले कृषकों की दशा बड़ी दयनीय थी। यहाँ की खेती अच्छी नहीं थी और श्रमिकों के अभाव में कृषि योग्य भूमि का एक बड़ा भाग भी कृषि -विहीन रह जाता था। कई श्रमिक गवर्नरों के द्वारा किए गए अत्याचारों के कारण मौत के मुंह में चले जाते थे। अपने स्वामियों की माँगों को पूरा न करने के कारण अनेक किसानों को उनके गुजारा करने के साधनों से वंचित कर दिया जाता था तथा उनके बच्चों को दास बना लिया जाता था।

(4) मुगल साम्राज्य का स्वरूप- बर्नियर के अनुसार मुगल साम्राज्य का राजा ‘भिखारियों और क्रूर लोगों का राजा था। मुगल राज्य के शहर और नगर विनष्ट तथा ‘खराब वायु’ से दूषित थे और इसके खेत ‘झाड़ीदार’ तथा ‘घातक दलदल से भरे हुए थे। इसका मात्र एक ही कारण था राजकीय भू-स्वामित्व।

(5) एक अधिक जटिल सामाजिक सच्चाई एक ओर वर्नियर कहता है कि भारतीय शिल्पकारों के पास अपने उत्पादों के विस्तार के लिए कोई प्रोत्साहन नहीं था क्योंकि समस्त लाभ राज्य के खजाने में चला जाता था। इसलिए उत्पादन सर्वत्र पतनोन्मुख था परन्तु इसके साथ ही बर्नियर यह भी स्वीकार करता है कि सम्पूर्ण विश्व से बड़ी माश में बहुमूल्य धातुएँ भारत में आती थीं क्योंकि उत्पादों का सोने और चाँदी के बदले निर्यात होता था. भारत में एक समृद्ध व्यापारिक समुदाय भी था।

(6) भारतीय शहर बर्नियर के अनुसार मुगलकालीन शहर ‘शिविर नगर’ थे। शिविर नगरों से उसका अभिप्राय उन नगरों से था, जो अपने अस्तित्व और बने रहने के लिए राजकीय शिविर पर निर्भर थे उसका विचार था कि ये नगर राजकीय दरबार के आगमन के साथ अस्तित्व में आते थे और इसके अन्यत्र चले जाने के बाद तेजी से विलुप्त हो जाते थे।

(7) व्यावसायिक वर्ग अन्य शहरी समूहों में व्यावसायिक वर्ग जैसे चिकित्सक (हकीम एवं वैद्य), अध्यापक (पंडित या मुल्ला), अधिवक्ता (वकील), चित्रकार, वास्तुविद् संगीतकार, सुलेखक आदि सम्मिलित थे।

JAC Class 12 History Important Questions Chapter 5 यात्रियों के नज़रिए : समाज के बारे में उनकी समझ

प्रश्न 12.
इब्नबतूता और अल बिरूनी के भारत यात्रा-वृत्तान्तों की तुलना कीजिए।
उत्तर:
इब्नबतूता और अल बिरूनी के भारत यात्रा वृत्तान्तों की तुलना इब्नबतूता तथा अल बिरुनी के भारत यात्रा वृत्तान्तों की तुलना निम्न प्रकार से की जा सकती है-

(1) भिन्न-भिन्न कालों से सम्बन्धित यात्रा वृत्तान्त- अल बिरुनी का या वृत्तान्त ग्यारहवीं शताब्दी के भारतीय सामाजिक एवं सांस्कृतिक जीवन से सम्बन्धित है, जबकि इब्नबतूता का यात्रा-वृत्तान्त चौदहवीं शताब्दी के भारतीय सामाजिक एवं सांस्कृतिक जीवन से सम्बन्धित है।

(2) विषय अल बिरूनी ने अपनी प्रसिद्ध रचना ‘किताब-उल-हिन्द’ में भारतीय धर्म और दर्शन, त्योहारों, खगोल विज्ञान, रीति-रिवाज तथा प्रथाओं, सामाजिक जीवन, 7 भार तौल तथा मापन विधियों, मूर्तिकला, कानून, मापतन्त्र विज्ञान आदि विषयों का विवेचन किया है। इब्नबतूता ने न अपनी प्रसिद्ध रचना ‘रिहला’ में भारतीय सामाजिक तथा सांस्कृतिक जीवन, डाक व्यवस्था, भारतीय शहरों, बाजारों नारियल तथा पान, दास-दासियों, सती प्रथा, भारत की बा जलवायु लोगों के रहन-सहन, वेशभूषा, कृषि व्यापार आदि न विषयों का विवेचन किया है।

(3) भारतीय लोगों के धर्म, दर्शन और विज्ञान के के बारे में वर्णन करना अल बिरुनी संस्कृत भाषा का था। उसने यहाँ के लोगों के दर्शन, धर्म, विज्ञान और ना विचारों के ग्रन्थों का अच्छा ज्ञान प्राप्त कर लिया था। उसके द्वारा वर्णित भारत की जाति व्यवस्था का वर्णन उसके संस्कृत ग्रन्थों के अध्ययन से पूर्णतया प्रभावित था। परन्तु से इब्नबतूता संस्कृत और अन्य भारतीय भाषाओं, यहाँ के क रीति-रिवाजों, दर्शन आदि से अपरिचित थे।

(4) साहित्यिक ग्रन्थों एवं यात्राओं से अर्जित अनुभव को महत्त्वपूर्ण स्रोत मानना अल बिरूनी न साहित्यिक ग्रन्थों से प्राप्त अनुभव को ज्ञान का अधिक वाय महत्त्वपूर्ण खोत मानता था, परन्तु इब्नबतूता साहित्यिक नए ग्रन्थों के स्थान पर चाशओं से अर्जित अनुभव को महत्त्वपूर्ण नगर स्त्रोत मानता था।

(5) उद्देश्य अल बिरुनी के बाश-वृत्तान्त के उद्देश्य ये –

  • उन लोगों के लिए सहायक जो हिन्दुओं से धार्मिक विषयों पर चर्चा करना चाहते थे तथा
  • ऐसे लोगों के लिए एक सूचना का संग्रह जो उनके साथ सम्बद्ध होना चाहते थे इब्नबतूता के भारत यात्रा के वृत्तान्त का उद्देश्य अपरिचित वस्तुओं, राज्यों, घटनाओं आदि से अपने देशवासियों को परिचित कराना था। वह चाहता था कि श्रोता अथवा पाठक सुदूर देशों के वृत्तान्तों से पूरी तरह से प्रभावित हो सकें। इसी कारण इब्नबतूता ने पान और नारियल डाक- व्यवस्था, भारतीय शहरों के वैभव आदि के बारे में विस्तार से लिखा।

प्रश्न 13.
एक ओर बर्नियर ने भारतीय समाज को दरिद्र लोगों के समरूप जनसमूह से निर्मित वर्णित किया है, तो दूसरी ओर उसने एक अधिक सामाजिक आर्थिक सच्चाई को भी उजागर किया है। विवेचना कीजिये।
उत्तर:
भारतीय समाज दरिद्र लोगों के जनसमूह के रूप में बर्नियर ने भारतीय समाज को दरिद्र लोगों के जनसमूह के रूप में दर्शाते हुए लिखा है कि भारत के बड़े ग्रामीण अंचलों में से कई अंचल रेतीली भूमियाँ या बंजर पर्वत है। यहाँ की खेती भी अच्छी नहीं है प्रान्तीय गवर्नरों के अत्याचारों के कारण अनेक गरीब मजदूर मर जाते हैं। जब गरीब लोग अपने भू-स्वामियों की मांगों को पूरा नहीं कर पाते, तो उन्हें न केवल जीवन निर्वहन के साधनों से वंचित कर दिया जाता है, बल्कि उनके बच्चों को दास भी बना लिया जाता है। विवश होकर बहुत से गरीब किसान गाँव छोड़कर चले जाते हैं।

(1) बहुमूल्य धातुओं का भारत में आना एक अधिक जटिल सामाजिक और आर्थिक सच्चाई को दर्शाना- दूसरी ओर बर्नियर भारत में व्याप्त एक अधिक प्रसिद्ध सामाजिक और आर्थिक सच्चाई को दर्शाते हुए लिखता है कि शिल्पकारों के पास अपने उत्पादों को उत्तम बनाने के लिए कोई प्रोत्साहन नहीं था, क्योंकि सारा मुनाफा राज्य को होता था फिर भी सम्पूर्ण विश्व से बड़ी मात्रा में बहुमूल्य धातुएँ भारत में आती थीं क्योंकि उत्पादों का सोने और चाँदी के बदले निर्यात होता था। इससे देश में सोना और चाँदी इकट्ठा होता था। बर्नियर के अनुसार भारत में एक समृद्ध व्यापारिक समुदाय का भी अस्तित्व था जो लम्बी दूरी के विनिमय में संलग्न था।

(2) उपजाऊ भूमि और उन्नत शिल्प-बर्नियर ने लिखा है कि भारत का एक बड़ा भू-भाग अत्यन्त उपजाऊ है। बंगाल मित्र से न केवल चावल, मकई तथा जीवन की अन्य आवश्यक वस्तुओं के उत्पादन में बल्कि रेशम कपास, नील आदि के उत्पादन में भी आगे है भारत के अनेक हिस्सों में खेती अच्छी होती है। यहाँ के शिल्पकार अनेक | वस्तुओं के उत्पादन में संलग्न रहते हैं। ये शिल्पकार गलीचों, जरी कसीदाकारी, कढ़ाई, सोने और चांदी के वस्त्रों तथा विभिन्न प्रकार के रेशम और सूती वस्त्रों के निर्माण का कार्य करते हैं। रेशमी तथा सूती वस्त्रों का प्रयोग केवल भारत में ही नहीं होता, अपितु ये विदेशों में भी निर्यात किये जाते हैं।

(3) भारत में सोना और चाँदी का संग्रह होना- बर्नियर ने यह भी लिखा है कि सम्पूर्ण विश्व के सभी भागों में संचलन के बाद सोना और चाँदी का भारत में आकर कुछ सीमा तक संग्रह हो जाता है।

JAC Class 12 History Important Questions Chapter 5 यात्रियों के नज़रिए : समाज के बारे में उनकी समझ

प्रश्न 14.
अल बिरूनी ने ब्राह्मणवादी व्यवस्था की अपवित्रता की मान्यता को क्यों अस्वीकार कर दिया? क्या जाति व्यवस्था के नियमों का पालन पूर्ण कठोरता से किया जाता था? अल बिरूनी ने भारत की वर्ण व्यवस्था का वर्णन किस प्रकार किया है?
उत्तर:
यद्यपि अल – विरुनी ने भारतीय सामाजिक व्यवस्था में ब्राह्मणों द्वारा निर्मित जाति व्यवस्था को स्वीकार किया और उसकी मान्यता के लिए अन्य देशों के समुदायों में इस व्यवस्था के प्रतिरूपों के उदाहरणों को भी प्रस्तुत किया, फिर भी वह अपवित्रता की मान्यता को स्वीकार न कर अल बिरुनी ने लिखा है कि, “हर वह वस्तु जो अपवित्र हो जाती है, अपनी खोई हुई पवित्रता को पुनः पाने का प्रयास करती है और सफल होती है सूर्य हवा को स्वच्छ करता है और समुद्र में नमक पानी को गन्दा होने से बचाता है।” अल-विरुनी जोर देकर कहता है कि यदि ऐसा नहीं होता तो पृथ्वी पर जीवन असम्भव हो जाता। उसके अनुसार अपवित्रता की अवधारणा प्राकृतिक नियमों के विरुद्ध है।

अल बिरूनी ने जाति व्यवस्था के सम्बन्ध में जो भी विवरण दिया है; वह पूर्णतया संस्कृत ग्रन्थों के अध्ययन से प्रभावित है। जिन नियमों का वर्णन इन ग्रन्थों में ब्राह्मणवादी जाति-व्यवस्था को संचालित करने हेतु किया गया है; वह वास्तविक रूप में समाज में उतनी कठोरता से लागू नहीं थी। इनमें लचीलापन था उदाहरण हेतु परित्यक्त लोग जिन्हें अंत्यज कहते थे जो इस जाति व्यवस्था में शामिल नहीं थे, आर्थिक तन्त्र में उन्हें भी शामिल किया गया था। भले ही उनसे सस्ता श्रम प्राप्त करने के लिए ऐसा किया जाता हो।
अल- विरुनी की भारत की सामाजिक व्यवस्था की जानकारी प्राचीन भारतीय संस्कृत ग्रन्थों पर आधारित थी।

इसी आधार पर अल बिरूनी ने भारत की वर्ण व्यवस्था का वर्णन निम्न प्रकार से किया है –

  • ब्राह्मण अल बिरूनी लिखता है कि ब्राह्मण सबसे सर्वोच्च वर्ण था क्योंकि हिन्दू ग्रन्थों की मान्यताओं के अनुसार इनकी उत्पत्ति आदि देव ब्रह्मा के मुख से हुई और मुख का स्थान सबसे उच्च है; इसी कारण हिन्दू जाति में से सबसे उच्च माने जाते हैं।
  • क्षत्रिय क्षत्रियों की उत्पत्ति आदि देव ब्रह्मा के कन्धों व हाथों से मानी गई है। मुख के बाद द्वितीय स्थान कन्धों व याँहों का है अतः उन्हें वर्ण व्यवस्था में ब्राह्मणों से कुछ नीचे द्वितीय स्थान पर रखा गया।
  • वैश्य वैश्य वर्ण की उत्पत्ति ब्रह्मा के उदर व जंघा भाग से मानी गई इसलिए इन्हें तीसरे स्थान पर रखा गया।
  • शूद्र-शूद्र वर्ण की उत्पत्ति ब्रह्मा के चरणों से मानी गयी है, अतः वैश्य और शूद्रों के बीच अल बिरुनी अधिक अन्तर नहीं मानता था। अल बिरुनी के अनुसार, यद्यपि वर्ग-भेद तो था फिर भी सभी लोग एक साथ एक ही शहर या गांव में समरसता के साथ रहते थे।

JAC Class 12 History Important Questions Chapter 4 विचारक, विश्वास और इमारतें : सांस्कृतिक विकास

Jharkhand Board JAC Class 12 History Important Questions Chapter 4 विचारक, विश्वास और इमारतें : सांस्कृतिक विकास Important Questions and Answers.

JAC Board Class 12 History Important Questions Chapter 4 विचारक, विश्वास और इमारतें : सांस्कृतिक विकास

बहुविकल्पीय प्रश्न (Multiple Choice Questions)

1. साँची का स्तूप किस राज्य में स्थित है –
(अ) उत्तर प्रदेश
(स) मध्य प्रदेश
(ब) बिहार
(द) गुजरात
उत्तर:
(स) मध्य प्रदेश

2. ऋग्वेद का संकलन कब किया गया?
(अ) 2500-2000 ई. पूर्व
(ब) 2000-1500 ई. पूर्व
(स) 1000-500 ई. पूर्व
(द) 1500 से 1000 ई. पूर्व
उत्तर:
(द) 1500 से 1000 ई. पूर्व

3. महावीर स्वामी से पहले कितने शिक्षक (तीर्थंकर) हो चुके थे?
(अ) 24
(स) 14
(ब) 20
(द) 23
उत्तर:
(द) 23

JAC Class 12 History Important Questions Chapter 4 विचारक, विश्वास और इमारतें : सांस्कृतिक विकास

4. बौद्धसंघ में सम्मिलित होने वाली प्रथम भिक्षुणी थी –
(अ) महामाया
(ब) पुन्ना
(स) महाप्रजापति गोतमी
(द) सुलक्षणी
उत्तर:
(स) महाप्रजापति गोतमी

5. जिन ग्रन्थों में बुद्ध की शिक्षाओं का संकलन किया गया है, उन्हें कहा जाता है –
(अ) बौद्धग्रन्थ
(स) महावंश
(ब) दीपवंश
(द) त्रिपिटक
उत्तर:
(द) त्रिपिटक

6. किस ग्रन्थ में बौद्ध भिक्षुओं और भिक्षुणियों के लिए नियम संकलित किए गए हैं?
(अ) अभिधम्मपिटक
(ब) विनयपिटक
(स) सुत्तपिटक
(द) बुद्धचरित
उत्तर:
(ब) विनयपिटक

7. अमरावती के स्तूप की खोज सर्वप्रथम कब हुई ?
(अ) 1896
(ब) 1796
(स) 1717
(द) 1817
उत्तर:
(ब) 1796

8. साँची की खोज कब हुई?
(अ) 1818 ई.
(ब) 1718 ई.
(स) 1918 ई.
(द) 1878 ई.
उत्तर:
(अ) 1818 ई.

9. किस अंग्रेज लेखक ने साँची पर लिखे अपने महत्त्वपूर्ण ग्रन्थों को सुल्तान जहाँ बेगम को समर्पित किया?
(अ) जेम्स टॉड
(ब) जॉर्ज थॉमस
(स) जॉन मार्शल
(द) जरथुस्व
उत्तर:
(स) जॉन मार्शल

10. किस दार्शनिक का सम्बन्ध चूनान से है?
(अ) सुकरात
(ख) खुंगत्सी
(स) बुद्ध
(द) कनिंघम
उत्तर:
(अ) सुकरात

11. समकालीन बौद्ध ग्रन्थों में हमें कितने सम्प्रदायों या चिन्तन परम्पराओं की जानकारी मिलती है?
(अ) 10
(स) 94
(ब) 24
(द) 64
उत्तर:
(द) 64

JAC Class 12 History Important Questions Chapter 4 विचारक, विश्वास और इमारतें : सांस्कृतिक विकास

12. किस देश से दीपवंश एवं महावंश जैसे क्षेत्र विशेष के बौद्ध इतिहास का सम्बन्ध है-
(अ) भारत
(स) चीन
(ब) नेपाल
(द) श्रीलंका
उत्तर:
(द) श्रीलंका

13. त्रिपिटक की रचना कब हुई थी?
(अ) बुद्ध के जन्म से पूर्व
(ब) बुद्ध के महापरिनिर्वाण प्राप्त करने के पश्चात्
(स) बुद्ध के जीवन काल में
(द) इनमें से कोई नहीं
उत्तर:
(ब) बुद्ध के महापरिनिर्वाण प्राप्त करने के पश्चात्

14. महात्मा बुद्ध की शिक्षाओं का संकलन निम्नलिखित में से किसमें है?
(अ) सुत्तपिटक
(ब) विनय पिटक
(स) जातक
(द) इनमें से कोई नहीं
उत्तर:
(अ) सुत्तपिटक

15. महात्मा बुद्ध का जन्म हुआ था –
(अ) कपिलवस्तु
(ब) लुम्बिनी
(स) सारनाथ
(द) बोधगया
उत्तर:
(ब) लुम्बिनी

रिक्त स्थानों की पूर्ति कीजिए।

1. भोपाल की नवाब शाहजहाँ बेगम की आत्मकथा ………….. है।
2. शाहजहाँ बेगम की उत्तराधिकारी ……………. थीं।
3. ………… और …………… जैसे जटिल यज्ञ सरदार और राजा किया करते थे।
4. महात्मा बुद्ध के दर्शन से जुड़े विषय …………. पिटक में आए।
5. ज्यादातर पुराने बौद्ध ग्रन्थ …………… भाषा में हैं।
6. बौद्ध धर्म के पूर्व एशिया में फैलने के पश्चात् …………. और ………… जैसे तीर्थयात्री बौद्ध ग्रन्थों की खोज में चीन से भारत आए।
7. वे महापुरुष जो पुरुषों और महिलाओं को जीवन की नदी के पार पहुँचाते हैं उन्हें …………….. कहते हैं।
उत्तर:
1. ताज-उल- इकबाल
2. सुल्तान जहाँ बेगम
3. राजसूय, अश्वमेध
4. अभिधम्म
5. पालि
6. फा- शिएन, श्वैन त्सांग
7. तीर्थंकर

अतिलघूत्तरात्मक प्रश्न

प्रश्न 1.
महात्मा बुद्ध ने अपना प्रथम उपदेश कहाँ दिया?
उत्तर:
महात्मा बुद्ध ने अपना प्रथम उपदेश सारनाथ में दिया।

प्रश्न 2.
जैन दर्शन की सबसे महत्त्वपूर्ण अवधारणा क्या है?
उत्तर:
जैन दर्शन के अनुसार सम्पूर्ण संसार प्राणवान है

JAC Class 12 History Important Questions Chapter 4 विचारक, विश्वास और इमारतें : सांस्कृतिक विकास

प्रश्न 3.
मन्दिर स्थापत्य कला में गर्भगृह एवं शिखर से आप क्या समझते हैं?
उत्तर:
(1) गर्भगृह- मन्दिर का चौकोर कमरा
(2) शिखर- गर्भगृह के ऊपर ढाँचा।

प्रश्न 4.
बौद्ध संघ की संचालन पद्धति को स्पष्ट कीजिये।
उत्तर:
बौद्ध संघ की संचालन पद्धति गणों और संघों की परम्परा पर आधारित थी। लोग बातचीत के द्वारा एकमत होते थे।

प्रश्न 5.
बुद्ध के जीवन से जुड़े बोधगया एवं सारनाथ नामक स्थानों का महत्त्व बताइये।
उत्तर:
(1) बोधगया में. बुद्ध ने ज्ञान प्राप्त किया था।
(2) सारनाथ में बुद्ध ने प्रथम उपदेश दिया था।

प्रश्न 6.
यदि आप भोपाल की यात्रा करेंगे, तो वहाँ किस बौद्धकालीन स्तूप को देखना चाहेंगे?
उत्तर:
साँची के स्तूप को।

प्रश्न 7.
महात्मा बुद्ध का बचपन का नाम क्या था?
उत्तर:
सिद्धार्थ

प्रश्न 8.
साँची का स्तूप कहाँ स्थित है?
उत्तर:
साँची का स्तूप मध्य प्रदेश की राजधानी भोपाल से 20 मील उत्तर-पूर्व में स्थित साँची कनखेड़ा नामक एक गाँव में स्थित है।

प्रश्न 9.
थेरीगाथा बौद्ध ग्रन्थ किस पिटक का हिस्सा है?
उत्तर:
सुत्तपिटक।

प्रश्न 10.
कैलाशनाथ मन्दिर कहाँ स्थित है?
उत्तर:
महाराष्ट्र में।

JAC Class 12 History Important Questions Chapter 4 विचारक, विश्वास और इमारतें : सांस्कृतिक विकास

प्रश्न 11.
महावीर स्वामी का बचपन का नाम क्या
उत्तर:
वर्धमान।

प्रश्न 12.
ऐसे तीन स्थानों का उल्लेख कीजिये, जहाँ स्तूप बनाए गए थे।
उत्तर:
(1) भरहुत
(2) साँची
(3) अमरावती।

प्रश्न 13.
साँची के स्तूप को आर्थिक अनुदान देने वाले दो शासकों के नाम लिखिए।
उत्तर:
(1) शाहजहाँ बेगम
(2) सुल्तानजहाँ बेगम।

प्रश्न 14.
बुद्ध की शिक्षाएँ किन ग्रन्थों में संकलित हैं?
उत्तर:
त्रिपिटक में

प्रश्न 15.
साँची के स्तूप को किसने संरक्षण प्रदान किया?
उत्तर:
साँची के स्तूप को भोपाल के शासकों ने संरक्षण प्रदान किया जिनमें शाहजहाँ बेगम एवं सुल्तानजहाँ बेगम प्रमुख हैं।

प्रश्न 16.
“इस देश की प्राचीन कलाकृतियों की लूट होने देना मुझे आत्मघाती और असमर्थनीय नीति लगती है।” यह कथन किस पुरातत्त्ववेत्ता का है?
उत्तर:
एच. एच. कोल।

प्रश्न 17.
ईसा पूर्व प्रथम सहस्राब्दी का काल विश्व इतिहास में एक महत्त्वपूर्ण मोड़ क्यों माना जाता है ?
उत्तर:
क्योंकि इस काल में ईरान में जरस्थुस्व, चीन में खुंगस्ती, यूनान में सुकरात, प्लेटो, अरस्तू एवं भारत में महावीर व बुद्ध जैसे चिन्तकों का उदय हुआ।

प्रश्न 18.
अजन्ता की गुफाएँ कहाँ स्थित हैं?
उत्तर:
महाराष्ट्र में

प्रश्न 19.
पौराणिक हिन्दू धर्म में किन दो प्रमुख देवताओं की पूजा प्रचलित थी?
उत्तर:
(1) विष्णु
(2) शिव।

प्रश्न 20.
सबसे प्राचीन कृत्रिम गुफाओं का निर्माण किस शासक ने करवाया था ?
उत्तर:
अशोक ने आजीविक सम्प्रदाय के सन्तों हेतु निर्माण करवाया था।

JAC Class 12 History Important Questions Chapter 4 विचारक, विश्वास और इमारतें : सांस्कृतिक विकास

प्रश्न 21.
धेरी का क्या अभिप्राय है?
उत्तर:
थेरी का अर्थ है ऐसी महिलाएँ जिन्होंने निर्वाण प्राप्त कर लिया हो।

प्रश्न 22.
बौद्ध धर्म की दो शिक्षाएँ बताइये।
उत्तर:
(1) विश्व अनित्य है।
(2) यह संसार दुःखों का घर है।

प्रश्न 23.
जैन धर्म की दो शिक्षाएँ बताइये।
उत्तर:
(1) जीवों के प्रति अहिंसा
(2) कर्मवाद और पुनर्जन्म में विश्वास।

प्रश्न 24.
बौद्ध संघ में सम्मिलित होने वाली प्रथम महिला कौन थी?
उत्तर:
बुद्ध की उपमाता महाप्रजापति गोतमी।

प्रश्न 25.
जैन धर्म के महापुरुष क्या कहलाते थे?
उत्तर:
तीर्थंकर।

प्रश्न 26.
जैन धर्म के 23वें तीर्थकर कौन थे?
उत्तर:
पार्श्वनाथ

प्रश्न 27.
जैन दर्शन की सबसे महत्त्वपूर्ण अवधारणा क्या है?
उत्तर:
जैन दर्शन की सबसे महत्त्वपूर्ण अवधारणा यह है कि सम्पूर्ण विश्व प्राणवान है।

प्रश्न 28.
राजसूय एवं अश्वमेध यज्ञों का अनुष्ठान किनके द्वारा कराया जाता था ?
उत्तर:
ब्राह्मण पुरोहितों द्वारा

प्रश्न 29.
समकालीन बौद्ध ग्रन्थों में कितने सम्प्रदायों एवं चिन्तन परम्पराओं का उल्लेख मिलता है?
उत्तर:
64 सम्प्रदाय या चिन्तन परम्पराओं का।

प्रश्न 30.
कुटागारशालाओं का शाब्दिक अर्थ बताइये।
उत्तर:
नुकीली छत वाली झोंपड़ी।

JAC Class 12 History Important Questions Chapter 4 विचारक, विश्वास और इमारतें : सांस्कृतिक विकास

प्रश्न 31.
शिक्षक अपने दार्शनिक विचारों की चर्चा कहाँ करते थे?
उत्तर:
कुटागारशालाओं या उपवनों में।

प्रश्न 32.
महावीर स्वामी तथा बुद्ध ने किसका विरोध किया?
उत्तर:
वेदों के प्रभुत्व का।

प्रश्न 33.
चीनी यात्री फा-शिएन तथा श्वैन-त्सांग ने भारत की यात्रा क्यों की?
उत्तर:
बौद्ध ग्रन्थों की खोज के लिए।

प्रश्न 34.
प्रमुख नियतिवादी बौद्ध भिक्षु कौन थे?
उत्तर:
मक्खलि गोसाल।

प्रश्न 35.
प्रमुख भौतिकवादी बौद्ध दार्शनिक कौन थे?
उत्तर:
अजीत केसकम्बलिन।

प्रश्न 36.
महावीर स्वामी से पहले कितने तीर्थकर हो चुके थे?
उत्तर:
231

प्रश्न 37.
अमरावती के स्तूप की खोज कब हुई ?
उत्तर:
1796 ई. में

JAC Class 12 History Important Questions Chapter 4 विचारक, विश्वास और इमारतें : सांस्कृतिक विकास

प्रश्न 38.
विष्णु के कितने अवतारों की कल्पना की गई ?
उत्तर:
दस अवतारों की।

प्रश्न 39.
बराबर (बिहार) की गुफाओं का निर्माण किसने करवाया था ?
उत्तर:
अशोक ने।

प्रश्न 40.
अशोक ने किस सम्प्रदाय के सन्तों के लिए बराबर की गुफाओं का निर्माण करवाया था?
उत्तर:
आजीविक सम्प्रदाय के सन्तों के लिए।

प्रश्न 41.
कैलाशनाथ के मन्दिर कहाँ स्थित हैं?
उत्तर:
एलोरा (महाराष्ट्र) में।

प्रश्न 42.
कैलाशनाथ के मन्दिर का निर्माण कब करवाया गया था?
उत्तर:
आठवीं शताब्दी में।

प्रश्न 43.
यूनानी शैली से प्रभावित मूर्तियाँ किन क्षेत्रों से प्राप्त हुई हैं?
उत्तर:
तक्षशिला और पेशावर से।

प्रश्न 44.
प्रतीकों द्वारा बुद्ध की स्थिति किस प्रकार दिखाई जाती है? दो उदाहरण दीजिये।
उत्तर:
बुद्ध के ‘अध्यान की दशा’ को ‘रिक्त स्थान’ तथा ‘महापरिनिर्वाण’ को ‘स्तूप’ के द्वारा दिखाया जाता है।

प्रश्न 45.
ऐसे चार सामाजिक समूहों के नाम बताइये जिनमें से बुद्ध के अनुयायी आए।
उत्तर:
(1) राजा
(2) धनवान
(3) गृहपति तथा
(4) सामान्य जन

प्रश्न 46.
बुद्ध द्वारा गृह त्याग के क्या कारण थे?
उत्तर:
नगर का भ्रमण करते समय एक वृद्ध व्यक्ति, एक रोगी, एक लाश और संन्यासी को देख कर बुद्ध ने घर त्याग दिया।

प्रश्न 47.
लुम्बिनी एवं कुशीनगर नामक स्थानों का महत्त्व बताइये।
उत्तर:
लुम्बिनी में बुद्ध का जन्म हुआ। कुशीनगर में बुद्ध ने निर्वाण प्राप्त किया। ये बौद्ध धर्म के पवित्र स्थान हैं।

प्रश्न 48.
जैन धर्म के अनुसार कर्मफल से मुक्ति कैसे पाई जा सकती है ?
उत्तर:
त्याग और तपस्या के द्वारा।

प्रश्न 49.
आपकी दृष्टि में बौद्ध धर्म के तेजी से प्रसार के क्या कारण थे? किन्हीं दो कारणों का उल्लेख कीजिये।
उत्तर:
(1) लोग समकालीन धार्मिक प्रथाओं से असन्तुष्ट थे।
(2) बौद्ध धर्म ने जाति प्रथा का विरोध किया, सामाजिक समानता पर बल दिया।

प्रश्न 50.
हीनयान और महायान बौद्ध सम्प्रदायों में अन्तर स्पष्ट कीजिये।
उत्तर:
महायानी बुद्ध को देवता मान कर उनकी पूजा करते थे, परन्तु हीनयानी बुद्ध को अवतार नहीं मानते थे।

JAC Class 12 History Important Questions Chapter 4 विचारक, विश्वास और इमारतें : सांस्कृतिक विकास

प्रश्न 51.
साँची का स्तूप कहाँ स्थित है?
उत्तर:
साँची का स्तूप भोपाल से बीस मील दूर उत्तर- पूर्व में स्थित साँची कनखेड़ा नामक एक गाँव में स्थित है।

प्रश्न 52.
ईसा पूर्व प्रथम सहस्राब्दी में विश्व में किन प्रसिद्ध चिन्तकों का उद्भव हुआ?
उत्तर:
ईरान में जरथुस्व, चीन में खुंगत्सी, यूनान में सुकरात, प्लेटो और अरस्तू तथा भारत में महावीर और बुद्ध का उद्भव हुआ।

प्रश्न 53.
ऋग्वेद का संकलन कब किया गया?
उत्तर:
ग्वेद का संकलन 1500 से 1000 ई. पूर्व में किया गया।

प्रश्न 54.
नियतिवादियों और भौतिकवादियों में क्या अन्तर था?
उत्तर:
नियतिवादियों के अनुसार सब कुछ पूर्व निर्धारित है परन्तु भौतिकवादियों के अनुसार दान, यज्ञ या चढ़ावा निरर्थक हैं।

प्रश्न 55.
कर्म के चक्र से मुक्ति के लिए क्या किया जाना आवश्यक है?
उत्तर:
कर्म के चक्र से मुक्ति के लिए त्याग और तपस्या किया जाना आवश्यक है।

प्रश्न 56.
जैन धर्म के पाँच व्रतों का उल्लेख कीजिए।
उत्तर:
(1) हत्या न करना
(2) चोरी न करना
(3) झूठ न बोलना
(4) ब्रह्मचर्य
(5) धन संग्रह न करना।

प्रश्न 57.
बुद्ध के संदेश भारत के बाहर किन-किन देशों में फैले ? नाम लिखिए।
अथवा
बौद्ध धर्म का प्रसार किन देशों में हुआ?
उत्तर:
बौद्ध धर्म का प्रसार सम्पूर्ण उपमहाद्वीप मध्य एशिया, चीन, कोरिया, जापान, श्रीलंका, म्यांमार, थाइलैंड तथा इंडोनेशिया में हुआ।

प्रश्न 58.
नगर का भ्रमण करते समय किन को देखकर सिद्धार्थ ने घर त्यागने का निश्चय कर लिया?
उत्तर:
एक वृद्ध व्यक्ति, एक रोगी, एक लाश और एक संन्यासी को देखकर सिद्धार्थ ने घर त्यागने का निश्चय कर लिया।

प्रश्न 59.
बौद्ध धर्म के अनुसार ‘संघ’ से क्या अभिप्राय है?
उत्तर:
युद्ध द्वारा स्थापित संघ भिक्षुओं की एक संस्था थी जो धम्म के शिक्षक बन गए।

प्रश्न 60.
संघ के भिक्षुओं की दैनिक चर्या का उल्लेख कीजिए।
उत्तर:
(1) भिक्षु सादा जीवन बिताते थे।
(2) वे उपासकों से भोजन दान पाने के लिए एक कटोरा रखते थे।

JAC Class 12 History Important Questions Chapter 4 विचारक, विश्वास और इमारतें : सांस्कृतिक विकास

प्रश्न 61.
बौद्ध धर्म के शीघ्र प्रसार के दो कारण बताइये।
उत्तर:
(1) लोग समकालीन धार्मिक प्रथाओं से असन्तुष्ट थे
(2) बौद्ध धर्म सामाजिक समानता पर बल देता था।

प्रश्न 62.
‘चैत्य’ किसे कहते हैं?
उत्तर:
शवदाह के बाद शरीर के कुछ अवशेष टीलों पर सुरक्षित रख दिए जाते थे। ये टीले चैत्य कहे जाने लगे।

प्रश्न 63.
बुद्ध के जीवन से जुड़े चार स्थानों का उल्लेख कीजिए।
उत्तर:
(1) लुम्बिनी ( बुद्ध का जन्म स्थान)
(2) बोधगया (ज्ञान प्राप्त होना)
(3) सारनाथ (प्रथम उपदेश देना)
(4) कुशीनगर (निर्वाण प्राप्त करना)।

प्रश्न 64.
स्तूप’ किसे कहते हैं?
उत्तर:
कुछ पवित्र स्थानों पर बुद्ध से जुड़े कुछ अवशेष जैसे उनकी अस्थियाँ गाड़ दी गई थीं। ये टीले स्तूप कहलाये।

प्रश्न 65.
किन लोगों के द्वारा स्तूपों को दान दिया जाता था?
उत्तर:
(1) राजाओं के द्वारा (जैसे सातवाहन वंश के राजा)
(2) शिल्पकारों तथा व्यापारियों की श्रेणियों द्वारा
(3) महिलाओं और पुरुषों के द्वारा।

प्रश्न 66.
स्तूप की संरचना के प्रमुख तत्त्वों का उल्लेख कीजिए।
उत्तर:
(1) मिट्टी का टीला (अंड)
(2) हर्मिका (उन्जे जैसा ढाँचा)
(3) यष्टि (हर्मिका से निकला मस्तूल)
(4) वेदिका (टीले के चारों ओर बनी वेदिका)।

प्रश्न 67.
महायान मत में बोधिसत्तों की अवधारणा को स्पष्ट कीजिए।
उत्तर:
बोधिसता परम करुणामय जीव थे जो अपने सत्कार्यों से पुण्य कमाते थे और इससे दूसरों की सहायता करते थे।

प्रश्न 68.
महायान मत से आप क्या समझते हैं?
उत्तर:
महायान मत में बुद्ध और बोधिसत्तों की मूर्तियों की पूजा की जाती थी।

प्रश्न 69.
हीनयान या थेरवाद’ से क्या अभिप्राय है?
उत्तर:
पुरानी बौद्ध परम्परा के अनुवायी स्वयं को धेरवादी कहते थे। वे पुराने, प्रतिष्ठित शिक्षकों के बताए रास्ते पर चलते थे।

प्रश्न 70.
कैलाशनाथ मन्दिर के बारे में आप क्या जानते हैं?
उत्तर:
कैलाशनाथ मन्दिर एलोरा (महाराष्ट्र) में स्थित है यह सारा ढाँचा एक चट्टान को काट कर तैयार किया गया है।

प्रश्न 71.
जैन धर्म के दो सिद्धान्तों का उल्लेख कीजिए
उत्तर:
(1) जीवों के प्रति अहिंसा
(2) जन्म और पुनर्जन्म का चक्र कर्म के द्वारा निर्धारित होना।

JAC Class 12 History Important Questions Chapter 4 विचारक, विश्वास और इमारतें : सांस्कृतिक विकास

प्रश्न 72.
जैन विद्वानों ने किन भाषाओं में अपने ग्रन्थों की रचना की?
उत्तर:
जैन विद्वानों ने प्राकृत, संस्कृत, तमिल आदि भाषाओं में अपने ग्रन्थों की रचना की।

प्रश्न 73.
बौद्ध धर्म के दो सिद्धान्तों का उल्लेख कीजिये।
उत्तर:
(1) विश्व अनित्य है और निरन्तर बदल रहा है।
(2) विश्व आत्माविहीन है क्योंकि यहाँ कुछ भी स्थायी या शाश्वत नहीं है।

प्रश्न 74.
स्तूप क्या हैं?
उत्तर:
स्तूप बुद्ध धर्म से जुड़े पवित्र टीले हैं। इनमें बुद्ध के शरीर के अवशेष अथवा उनके द्वारा प्रयोग की गई किसी वस्तु को गाड़ा गया था।

प्रश्न 75.
चैत्य क्या थे?
उत्तर:
शवदाह के पश्चात् बौद्धों के शरीर के कुछ अवशेष टीलों पर सुरक्षित रख दिए जाते थे। अन्तिम संस्कार से जुड़े इन टीलों को चैत्य कहा जाता था।

प्रश्न 76.
ऋग्वेद में किन सूक्तियों का संग्रह है?
उत्तर:
ऋग्वेद में इन्द्र, अग्नि, सोम आदि देवताओं की स्तुति से सम्बन्धित सूक्तियों का संग्रह है।

प्रश्न 77.
नियतिवादी कौन थे?
उत्तर:
नियतिवादी वे लोग थे जो विश्वास करते थे कि सुख और दुःख पूर्व निर्धारित मात्रा में माप कर दिए गए हैं।

प्रश्न 78.
भौतिकवादी कौन थे?
उत्तर:
भौतिकवादी लोग यह मानते थे कि दान, यज्ञ या चढ़ावा निरर्थक हैं। इस दुनिया या दूसरी दुनिया का अस्तित्व नहीं होता।

JAC Class 12 History Important Questions Chapter 4 विचारक, विश्वास और इमारतें : सांस्कृतिक विकास

प्रश्न 79.
‘सन्तचरित्र’ से क्या अभिप्राय है?
उत्तर:
संतचरित्र किसी संत या धार्मिक महापुरुष की जीवनी है। संतचरित्र संत की उपलब्धियों का गुणगान करते हैं।

प्रश्न 80.
विनयपिटक तथा सुत्तपिटक में किन शिक्षाओं का संग्रह था ?
उत्तर:
(1) विनयपिटक में बौद्ध मठों में रहने वाले लोगों के लिए नियमों का संग्रह था।
(2) सुत्तपिटक में महात्मा बुद्ध की शिक्षाओं का संग्रह था।

प्रश्न 81.
अभिधम्मपिटक’ से आप क्या समझते हैं ?
उत्तर:
अभिधम्मपिटक’ नामक ग्रन्थ में बौद्ध दर्शन से सम्बन्धित सिद्धान्तों का संग्रह था।

प्रश्न 82.
श्रीलंका के इतिहास पर प्रकाश डालने वाले बौद्ध ग्रन्थों का उल्लेख कीजिये।
उत्तर:
‘दीपवंश’ (द्वीप का इतिहास) तथा ‘महावंश’ (महान इतिहास) से श्रीलंका के इतिहास पर प्रकाश पड़ता है।

प्रश्न 83.
स्तूप को संस्कृत में क्या कहा जाता है?
उत्तर:
स्तूप को संस्कृत में टीला कहा जाता है।

प्रश्न 84.
कौनसा अनूठा ग्रन्थ सुत्तपिटक का हिस्सा
उत्तर:
थेरीगाथा ग्रन्थ सुत्तपिटक का हिस्सा है।

प्रश्न 85.
बिना अलंकरण वाले प्रारम्भिक स्तूप कौन- कौनसे हैं?
उत्तर:
साँची और भरहुत स्तूप बिना अलंकरण वाले प्रारम्भिक स्तूप हैं।

प्रश्न 86.
जैन धर्म की सबसे महत्वपूर्ण अवधारणा क्या है?
उत्तर:
जैन धर्म के अनुसार सम्पूर्ण संसार प्राणवान है; पत्थर, चट्टान और जल में भी जीवन होता है। प्रश्न 87. अमरावती का स्तूप किस राज्य में है? उत्तर- अमरावती का स्तूप गुंटूर (आन्ध्र प्रदेश) में स्थित है।

प्रश्न 88.
जेम्स फर्ग्युसन ने वृक्ष और सर्प पूजा का केन्द्र किसे माना ?
उत्तर:
जेम्स फर्ग्युसन ने वृक्ष और सर्प पूजा का केन्द्र साँची को माना।

JAC Class 12 History Important Questions Chapter 4 विचारक, विश्वास और इमारतें : सांस्कृतिक विकास

प्रश्न 89.
वराह ने किसकी रक्षा की थी?
उत्तर:
वराह ने पृथ्वी की रक्षा की थी।

प्रश्न 90.
बौद्धों का सबसे विशाल और शानदार स्तूप कौनसा था ?
उत्तर:
बौद्धों का सबसे विशाल और शानदार स्तूप अमरावती का स्तूप है।

प्रश्न 91.
भक्ति से क्या आशय है?
उत्तर:
भक्ति एक प्रकार की आराधना है। इसमें उपासक एवं ईश्वर के मध्य के रिश्ते को प्रेम तथा समर्पण का रिश्ता माना जाता है।

लघुत्तरात्मक प्रश्न

प्रश्न 1.
बौद्ध धर्म के व्यावहारिक पक्ष के बारे में सुत्तपिटक के उद्धरण पर प्रकाश डालिए।
उत्तर:
(1) मालिक को अपने नौकरों और कर्मचारियों की पाँच प्रकार से देखभाल करनी चाहिए उनकी क्षमता के अनुसार उन्हें काम देकर उन्हें भोजन और मजदूरी देकर, बीमार पड़ने पर उनकी परिचर्या करने, उनके साथ स्वादिष्ट भोजन बाँट कर और समय-समय पर उन्हें छुट्टी देकर।
(2) कुल के लोगों को पाँच तरह से श्रमणों और ब्राह्मणों की देखभाल करनी चाहिए अनुराग द्वारा, सदैव पुस् घर खुले रखकर तथा उनकी दिन-प्रतिदिन की आवश्यकताओं दिन की पूर्ति करके।

प्रश्न 2.
बौद्ध एवं जैन धर्म में कितनी समानता थी?
उत्तर:

  1. दोनों धर्म कर्मवाद और पुनर्जन्मवाद में विश्वास करते हैं।
  2. दोनों धर्म अहिंसा के सिद्धान्त में विश्वास करते की में हैं।
  3. दोनों धर्म अनीश्वरवादी हैं।
  4. दोनों धर्मों ने यज्ञों, बहुदेववाद और कर्मकाण्डों गी का विरोध किया।
  5. दोनों धर्म निर्वाण प्राप्त करने पर बल देते हैं।
  6. दोनों धर्म निवृत्तिमार्गी हैं और संसार त्याग पर बल देते हैं।

प्रश्न 3.
साँची का स्तूप यूरोप के लोगों को विशेषकर में रुचिकर क्यों लगता है?
उत्तर:
भोपाल से बीस मील उत्तर पूर्वी की ओर एक पहाड़ी की तलहटी में साँची का स्तूप स्थित है। इस स्तूप की पत्थर की वस्तुएँ, बुद्ध की मूर्तियाँ तथा प्राचीन तोरणद्वार आदि यूरोप के लोगों को विशेषकर रुचिकर लगते हैं जिनमें मेजर अलेक्जैंडर कनिंघम एक हैं। मेजर अलेक्जेंडर कनिंघम ने इस स्थान के चित्र बनाए। उन्होंने यहाँ के अभिलेखों को पढ़ा और गुम्बदनुमा ढाँचे के बीचों-बीच खुदाई की। उन्होंने इस खोज के निष्कर्षो को एक अंग्रेजी पुस्तक में लिखा।

प्रश्न 4.
साँची का पूर्वी तोरणद्वार भोपाल राज्य से बाहर जाने से कैसे बचा रहा?
उत्तर:
साँची के स्तूप का पूर्वी तोरणद्वार सबसे अच्छी दशा में था अतः फ्रांसीसियों ने सांची के पूर्वी तोरणद्वार को फ्रांस के संग्रहालय में प्रदर्शित करने के लिए शाहजहाँ बेगम से इसे फ्रांस ले जाने की अनुमति माँगी। अंग्रेजों ने भी इसे इंग्लैण्ड ले जाने का प्रयास किया। सौभाग्वश फ्रांसीसी और अंग्रेज दोनों ही इसकी प्रतिकृतियों से सन्तुष्ट हो गए, जो बड़ी सावधानीपूर्वक प्लास्टर से बनाई गई थीं। इस प्रकार मूल कृति भोपाल राज्य में अपने स्थान पर ही बनी रही।

प्रश्न 5.
भोपाल के शासकों ने साँची स्तूप के संरक्षण के लिए क्या उपाय किये?
उत्तर:

  1. भोपाल के शासकों शाहजहाँ बेगम तथा उनकी उत्तराधिकारी सुल्तानजहाँ बेगम ने साँची के स्तूप के रख-रखाव के लिए प्रचुर धन का अनुदान किया।
  2. सुल्तानजहाँ बेगम ने वहाँ पर एक संग्रहालय और अतिथिशाला बनाने के लिए अनुदान दिया।
  3. जान मार्शल द्वारा साँची के स्तूप पर लिखी गई पुस्तक के प्रकाशन में भी सुल्तानजहाँ बेगम ने अनुदान दिया।
  4. साँची के स्तूप को भोपाल राज्य में बनाए रखने में शाहजहाँ बेगम ने योगदान दिया।

प्रश्न 6.
” ईसा पूर्व प्रथम सहस्राब्दी का काल विश्व इतिहास में एक महत्त्वपूर्ण मोड़ माना जाता है।” व्याख्या कीजिए।
उत्तर:
इस काल में ईरान में जरथुस्त्र, चीन में खंगत्सी, यूनान में सुकरात, प्लेटो और अरस्तू तथा भारत में महावीर, गौतम बुद्ध एवं कई अन्य चिन्तकों का उदय हुआ। उन्होंने जीवन के रहस्यों को समझने का प्रयास किया। उन्होंने मानव तथा विश्व व्यवस्था के बीच सम्बन्ध को भी समझने का प्रयास किया। इसी काल में गंगा घाटी में नये राज्य और शहर उभर रहे थे और सामाजिक एवं आर्थिक जीवन में कई प्रकार के परिवर्तन हो रहे थे जिन्हें ये चिन्तक समझने का प्रयास कर रहे थे।

JAC Class 12 History Important Questions Chapter 4 विचारक, विश्वास और इमारतें : सांस्कृतिक विकास

प्रश्न 7.
प्राचीन युग में प्रचलित यज्ञों की परम्परा का वर्णन कीजिए।
उत्तर:
पूर्व वैदिक काल में यज्ञों की परम्परा प्रचलित थी यज्ञों के समय ऋग्वैदिक देवताओं की स्तुति सूतों का न उच्चारण किया जाता था और लोग पशु, पुत्र, स्वास्थ्य, दीर्घ आयु आदि के लिए प्रार्थना करते थे आरम्भिक यह सामूहिक नेरूप से किए जाते थे। बाद में (लगभग 1000 ई. पूर्व से 500 ई. पूर्व) कुछ यज्ञ घरों के स्वामियों द्वारा किए जाते थे राजसूष और अश्वमेध जैसे जटिल यज्ञ सरदार और राजा ही किया करते थे।

प्रश्न 8.
वैदिक परम्परा के अन्तर्गत तथा वैदिक र परम्परा के बाहर छठी शताब्दी ई. पूर्व में उठे प्रश्नों और विवादों का उल्लेख कीजिए।
उत्तर:
छठी शताब्दी ई. पूर्व में लोग जीवन का अर्थ, मृत्यु के पश्चात् जीवन की सम्भावना और पुनर्जन्म के बारे ट में जानने के लिए उत्सुक थे क्या पुनर्जन्म अतीत के कर्मों । के कारण होता था? इस प्रकार के प्रश्नों पर खूब बाद- ही विवाद होता था। चिन्तक परम यथार्थ की प्रकृति को समझने और प्रकट करने में संलग्न थे। वैदिक परम्परा से बाहर के पण कुछ दार्शनिक यह प्रश्न उठा रहे थे कि सत्य एक होता है या अनेक। कुछ लोग यज्ञों के महत्त्व के बारे में विचार कर था रहे थे।

प्रश्न 9.
छठी शताब्दी ई. पूर्व में चिन्तकों में होने वाले वाद-विवादों तथा चर्चाओं की विवेचना कीजिए।
उत्तर:
छठी शताब्दी ई.पूर्व में विभिन्न सम्प्रदाय के शिक्षक एक स्थान से दूसरे स्थान पर घूम-घूमकर अपने दर्शन या विश्व के विषय में अपने दृष्टिकोण को लेकर एक-दूसरे से तथा सामान्य लोगों से तर्क-वितर्क करते थे। इस प्रकार की चर्चाएँ कुटागारशालाओं या ऐसे उपवनों में होती थीं जहाँ घुमक्कड़ चिन्तक ठहरा करते थे। यदि एक शिक्षक अपने प्रतिद्वन्द्वी को अपने तर्कों से सहमत कर लेता था, तो वह अपने अनुयायियों के साथ उसका शिष्य बन जाता था।

प्रश्न 10.
छठी शताब्दी ई. पूर्व में शिक्षक वैदिक धर्म के किन सिद्धान्तों पर प्रश्न उठाते थे?
उत्तर:
छठी शताब्दी ई. पूर्व में अनेक शिक्षक वेदों के प्रभुत्व पर प्रश्न उठाते थे इन शिक्षकों में महावीर स्वामी तथा बुद्ध भी सम्मिलित थे। उन्होंने यह विचार भी प्रकट किया कि जीवन के दुःखों से मुक्ति का प्रयास प्रत्येक व्यक्ति स्वयं कर सकता था। यह बात ब्राह्मणवाद से बिल्कुल भिन्न थी क्योंकि ब्राह्मणवाद की यह मान्यता थी कि किसी व्यक्ति का अस्तित्व उसकी जाति और लिंग से निर्धारित होता था।

प्रश्न 11.
आत्मा की प्रकृति और सच्चे यज्ञ के बारे में उपनिषदों में क्या कहा गया है?
उत्तर:
(1) आत्मा की प्रकृति छान्दोग्य उपनिषद में आत्मा की प्रकृति के आरे में कहा गया है कि “यह आत्मा धान या यव या सरसों या बाजरे के बीज की गिरी से भी छोटी है मन के अन्दर छुपी यह आत्मा पृथ्वी से भी विशाल, क्षितिज से भी विस्तृत, स्वर्ग से भी बड़ी है। और इन सभी लोकों से भी बड़ी है।”

(2) सच्चा यज्ञ ही एक यज्ञ है। बहते यह (पवन) जो बह रहा है, निश्चय बहते यह सबको पवित्र करता है। इसलिए यह वास्तव में यज्ञ है।

प्रश्न 12.
बौद्ध धर्म की शिक्षाओं के प्रसार में चीनी और भारतीय विद्वानों के योगदान का वर्णन कीजिये।
उत्तर:
जब बौद्ध धर्म पूर्वी एशिया में फैल गया तब फा-शिएन और श्वेन त्सांग जैसे चीनी यात्री बौद्ध ग्रन्थों की खोज में भारत आए ये पुस्तकें वे अपने देश ले गए, जहाँ विद्वानों ने इनका अनुवाद किया। भारत के बौद्ध शिक्षक भी अनेक देशों में गए बुद्ध की शिक्षाओं का प्रसार करने हेतु वे कई ग्रन्थ अपने साथ ले गए। कई सदियों तक ये पाण्डुलिपियाँ एशिया के विभिन्न देशों में स्थित बौद्ध- विहारों में संरक्षित थीं।

प्रश्न 13.
त्रिपिटकों पर एक संक्षिप्त टिप्पणी लिखिए।
उत्तर:
बौद्धों के धार्मिक सिद्धान्त त्रिपिटकों में संकलित –

  1. विनयपिटक – इसमें संघ या बौद्ध मठों में रहने वाले बौद्ध भिक्षुओं और भिक्षुणियों के आचरण सम्बन्धी नियमों का वर्णन है।
  2. सुत्तपिटक – इसमें महात्मा बुद्ध की शिक्षाओं का संग्रह है।
  3. अभिधम्मपिटक – इसमें बौद्ध दर्शन से जुड़े विषय संकलित हैं।

प्रश्न 14.
नियतिवादियों के सिद्धान्तों का वर्णन कीजिए।
उत्तर:
(1) नियतिवादियों के अनुसार सब कुछ पूर्व निर्धारित है। सुख और दुःख पूर्व निर्धारित मात्रा में माप कर दिए गए हैं। इन्हें संसार में बदला नहीं जा सकता। इन्हें बढ़ाया या घटाया नहीं जा सकता।
(2) बुद्धिमान लोग यह विश्वास करते हैं कि वे सद्गुणों तथा तपस्या द्वारा अपने कर्मों से मुक्ति प्राप्त कर लेंगे। मूर्ख लोग उन्हीं कार्यों को करके मुक्ति प्राप्त करने की आशा करते हैं नहीं है।

JAC Class 12 History Important Questions Chapter 4 विचारक, विश्वास और इमारतें : सांस्कृतिक विकास

प्रश्न 15.
भौतिकवादियों के सिद्धान्तों का विवेचन कीजिए।
उत्तर:
(1) संसार में दान, यज्ञ या चढ़ावा जैसी कोई चीज नहीं है।
(2) मनुष्य चार तत्वों से बना है जब वह मरता है, तब मिट्टी वाला अंश पृथ्वी में जल वाला अंश जल में, गर्मी वाला अंश आग में तथा साँस का अंश वायु में वापिस मिल जाता है और उसकी इन्द्रियाँ अंतरिक्ष का हिस्सा बन जाती हैं।
(3) दान देने का सिद्धान्त मूर्खों का सिद्धान्त है, यह खोखला झूठ है। मूर्ख हो या विद्वान् दोनों ही कट कर नष्ट हो जाते हैं। मृत्यु के बाद कुछ नहीं बचता।

प्रश्न 16.
जैन धर्म की प्रमुख शिक्षाओं का वर्णन कीजिये।
उत्तर:
(1) सम्पूर्ण विश्व प्राणवान है। पत्थर चट्टान, जल आदि में भी जीवन है।
(2) जीवों के प्रति अहिंसा का पालन करना चाहिए। मनुष्यों, जानवरों, पेड़-पौधों, कीड़े-मकोड़ों को नहीं मारना चाहिए।
(3) जन्म और पुनर्जन्म का चक्र कर्म के द्वारा निर्धारित होता है।
(4) पाँच व्रतों का पालन करना चाहिए –

  • हत्या न करना
  • चोरी नहीं करना
  • झूठ न बोलना
  • ब्रह्मचार्य
  • धन संग्रह न करना।

प्रश्न 17.
गौतम बुद्ध की जीवनी का संक्षेप में वर्णन कीजिए।
अथवा
गीतम बुद्ध पर एक संक्षिप्त टिप्पणी लिखिए।
अथवा
गौतम बुद्ध के जीवन का वर्णन कीजिये।
उत्तर:
बुद्ध के बचपन का नाम सिद्धार्थ था। वह शाक्य कबीले के सरदार के पुत्र थे। एक दिन नगर का भ्रमण करते समय सिद्धार्थ को एक रोगी, वृद्ध, मृतक तथा संन्यासी के दर्शन हुए जिससे उनका संसार के प्रति वैराग्य और बढ़ गया। अतः सिद्धार्थ महल त्याग कर सत्य की खोज में निकल गए। प्रारम्भ में उन्होंने 6 वर्ष तक कठोर तपस्या की। अन्त में उन्होंने एक वृक्ष के नीचे बैठकर चिन्तन करना शुरू किया और सच्चा ज्ञान प्राप्त किया। इसके बाद वह अपनी शिक्षाओं का प्रचार करने लगे।

प्रश्न 18.
बौद्ध धर्म की शिक्षाओं का वर्णन कीजिए।
अथवा
गौतम बुद्ध के उपदेशों का वर्णन कीजिये।
उत्तर:
(1) बौद्ध दर्शन के अनुसार विश्व अनित्य है और निरन्तर बदल रहा है। यह आत्माविहीन है क्योंकि यहाँ कुछ भी स्थायी अथवा शाश्वत नहीं है।
(2) इस क्षणभंगुर संसार में दुःख मनुष्य के जीवन का अन्तर्निहित तत्त्व है।
(3) घोर तपस्या और विषयासक्ति के बीच मध्यम मार्ग का अनुसरण करते हुए मनुष्य दुनिया के दुःखों से मुक्ति पा सकता है।
(4) बुद्ध की मान्यता थी कि समाज का निर्माण मनुष्यों ने किया था, न कि ईश्वर ने

प्रश्न 19.
‘बौद्ध संघ’ पर एक संक्षिप्त टिप्पणी लिखिए।
उत्तर:
बुद्ध ने अपने शिष्यों के लिए ‘संघ’ की स्थापना की संघ बौद्ध भिक्षुओं की एक ऐसी संस्था थी जो धम्म के शिक्षक बन गए। ये भिक्षु एक सादा जीवन बिताते थे। प्रारम्भ में केवल पुरुष ही संघ में सम्मिलित हो सकते थे, बाद में महिलाओं को भी संघ में सम्मिलित होने की अनुमति दे दी गई। कई स्वियों जो संघ में आईं, वे धम्म की उपदेशिकाएँ बन गई। संघ में सभी को समान दर्जा प्राप्त था। संघ की संचालन पद्धति गणों और संघ की परम्परा पर आधारित थी।

JAC Class 12 History Important Questions Chapter 4 विचारक, विश्वास और इमारतें : सांस्कृतिक विकास

प्रश्न 20.
संघ में रहने वाले भिक्षुओं का जीवन कैसा था?
उत्तर:
संघ में रहने वाले बौद्ध भिक्षु सादा जीवन बिताते थे। उनके पास जीवनयापन के लिए अत्यावश्यक वस्तुओं के अतिरिक्त कुछ नहीं होता था। वे दिन में केवल एक बार उपासकों से भोजनदान पाने के लिए एक कटोरा रखते थे। वे दान पर निर्भर थे, इसलिए उन्हें भिक्खु कहा जाता था। संघ में रहते हुए वे बौद्ध ग्रन्थों का अध्ययन करते थे।

प्रश्न 21.
महात्मा बुद्ध ने महिलाओं को भी संघ में सम्मिलित होने की अनुमति क्यों दी?
उत्तर:
प्रारम्भ में केवल पुरुष ही बौद्ध संघ में सम्मिलित हो सकते थे परन्तु कालान्तर में अपने प्रिय शिष्य आनन्द के अनुरोध पर महात्मा बुद्ध ने स्त्रियों को भी संघ में सम्मिलित होने की अनुमति प्रदान कर दी। बुद्ध की उपमाता महाप्रजापति गोतमी संघ में सम्मिलित होने वाली पहली भिक्षुणी थी। संघ में आने वाली कई स्त्रियाँ धम्म की उपदेशिकाएँ बन गई। आगे चलकर वे घेरी बनीं, जिसका अर्थ है- निर्वाण प्राप्त करने वाली महिलाएँ।

प्रश्न 22.
बुद्ध के अनुयायी किन सामाजिक वर्गों से सम्बन्धित थे?
उत्तर:
बुद्ध के अनुयायियों में कई सामाजिक वर्गों के लोग सम्मिलित थे। इनमें राजा, धनवान, गृहपति, व्यापारी, सामान्यजन कर्मकार, दास, शिल्पी आदि शामिल थे। इनमें स्त्री और पुरुष दोनों सम्मिलित थे। एक बार बौद्ध संघ में आ जाने पर सभी को बराबर माना जाता था क्योंकि भिक्षु और भिक्षुणी बनने पर उन्हें अपनी पुरानी पहचान को त्यागना पड़ता था।

प्रश्न 23.
भिक्षुओं और भिक्षुणियों के लिए निर्धारित नियमों का वर्णन कीजिए।
उत्तर:
(1) जब कोई भिक्षु एक नया कम्बल या गलीचा बनाएगा, तो उसे इसका प्रयोग कम से कम छः वर्षों तक करना पड़ेगा।
(2) यदि कोई भिक्षु किसी गृहस्थ के घर जाता है और उसे भोजन दिया जाता है, तो वह दो से तीन कटोरा भर ही स्वीकार कर सकता है।
(3) यदि बिहार में ठहरा हुआ भिक्षु प्रस्थान के पहले अपने बिस्तर को नहीं समेटता है, तो उसे अपराध स्वीकार करना होगा।

प्रश्न 24.
बौद्ध धर्म के तेजी से प्रसार होने के क्या कारण थे?
उत्तर:
(1) लोग समकालीन धार्मिक प्रथाओं से असन्तुष्ट थे।
(2) बौद्ध धर्म ने जन्म पर आधारित वर्ण व्यवस्था का विरोध किया और सामाजिक समानता पर बल दिया।
(3) बौद्ध धर्म में अच्छे आचरण और मूल्यों को महत्त्व दिया गया। इससे स्त्री और पुरुष इस धर्म की ओर आकर्षित हुए।
(4) बौद्ध धर्म ने निर्बल लोगों के प्रति दयापूर्ण और मित्रतापूर्ण व्यवहार को महत्त्व दिया।

प्रश्न 25.
चैत्य से क्या अभिप्राय है?
उत्तर:
अत्यन्त प्राचीनकाल से ही लोग कुछ स्थानों को पवित्र मानते थे ऐसे स्थानों पर जहाँ प्रायः विशेष वनस्पति होती थी, अनूठी चट्टानें थीं या आश्चर्यजनक प्राकृतिक सौन्दर्य था वहाँ पवित्र स्थल बन जाते थे ऐसे कुछ स्थलों पर एक छोटी-सी वेदी भी बनी रहती थी, जिन्हें कभी-कभी चैत्य कहा जाता था शवदाह के बाद शरीर के कुछ अवशेष टीलों पर सुरक्षित रख दिए जाते थे। अन्तिम संस्कार से जुड़े टीले चैत्य के रूप में जाने गए।

प्रश्न 26.
बौद्ध साहित्य में वर्णित कुछ चैत्यों का उल्लेख कीजिये।
उत्तर:
बौद्ध साहित्य में कई चैत्यों का उल्लेख मिलता है। इसमें बुद्ध के जीवन से सम्बन्धित स्थानों का भी उल्लेख है, जैसे लुम्बिनी (जहाँ बुद्ध का जन्म हुआ), बोधगया (जहाँ बुद्ध ने ज्ञान प्राप्त किया), सारनाथ (जहाँ उन्होंने उपदेश दिया) और कुशीनगर (जहाँ बुद्ध ने निब्बान प्राप्त किया। धीरे-धीरे ये समस्त स्थान पवित्र स्थल बन गए और यहाँ अनेक चैत्य बनाए गए।

प्रश्न 27.
स्तूपों का निर्माण क्यों किया जाता था?
उत्तर:
स्तूप बनाने की परम्परा बुद्ध से पहले की रही होगी। परन्तु वह बौद्ध धर्म से जुड़ गई। चूंकि स्तूपों में ऐसे अवशेष रहते थे, जिन्हें पवित्र समझा जाता था। इसलिए समूचा स्तूप ही बुद्ध और बौद्ध धर्म के प्रतीक के रूप में प्रतिष्ठित हुआ। अशोक ने बुद्ध को अवशेषों के हिस्से प्रत्येक महत्त्वपूर्ण नगर में बाँट कर उनके ऊपर स्तूप बनाने का आदेश दिया था।

JAC Class 12 History Important Questions Chapter 4 विचारक, विश्वास और इमारतें : सांस्कृतिक विकास

प्रश्न 28.
स्तूपों की संरचना का वर्णन कीजिये।
उत्तर:
(1) स्तूप का जन्म एक गोलाई लिए हुए मिट्टी के टीले से हुआ जिसे बाद में अंड कहा गया।
(2) अंड के ऊपर एक ‘हर्मिका’ होती थी। वह छज्जे जैसा ढांचा होता था।
(3) हर्मिका से एक मस्तूल निकलता था, जिसे ‘वष्टि’ कहते थे, जिस पर प्रायः एक छत्री लगी होती थी।
(4) टीले के चारों ओर एक वेदिका होती थी जो पवित्र स्थल को सामान्य दुनिया से अलग करती थी।

प्रश्न 29.
साँची और भरहुत के स्तूपों का वर्णन कीजिए।
उत्तर:
साँची और भरहुत के स्तूप बिना अलंकरण के हैं, उनमें केवल पत्थर की वेदिकाएँ तथा तोरणद्वार हैं। ये पत्थर की वेदिकाएँ किसी बाँस के या काठ के घेरे के समान थीं और चारों दिशाओं में खड़े तोरणद्वार पर खूब नक्काशी की गई थी। उपासक पूर्वी तोरणद्वार से प्रवेश करके टीले को दाई ओर देखते हुए दक्षिणावर्त परिक्रमा करते थे, मानो वे आकाश में सूर्य के पथ का अनुकरण कर रहे हों। बाद में स्तूप के टीले पर भी अलंकरण और नक्काशी की जाने लगी।

प्रश्न 30.
स्तूप किन लोगों के दान से बनाए गए ?
उत्तर:
(1) स्तूपों के निर्माण के लिए कुछ दान राजाओं के द्वारा दिए गए।
(2) कुछ दान शिल्पकारों और व्यापारियों की श्रेणियों द्वारा दिए गए।
(3) साँची के एक तोरण द्वार का भाग हाथीदांत का काम करने वाले शिल्पकारों के दान से बनाया गया था
(4) स्तूपों के निर्माण के लिए बौद्ध भिक्षुओं और भिक्षुणियों ने भी दान दिया।
(5) स्तूपों के निर्माण के लिए सैकड़ों महिलाओं और पुरुषों के द्वारा दान दिए गए।

प्रश्न 31.
अमरावती स्तूप की खोज किस प्रकार की गई ?
उत्तर:
1796 में एक स्थानीय राजा को अचानक अमरावती के स्तूप के अवशेष मिल गये। उन्होंने उसके पत्थरों से एक मन्दिर बनवाने का निश्चय किया। कुछ वर्षों बाद कालिन मेकेंजी नामक एक अंग्रेज अधिकारी इस क्षेत्र से गुजरे। उन्हें वहाँ नई मूर्तियाँ मिलीं और उन्होंने उनका चित्रांकन किया। 1854 ई. में गुन्टूर के कमिश्नर ने अमरावती की यात्रा की। वे वहाँ से कई मूर्तियाँ और उत्कीर्ण पत्थर मद्रास ले गए। उन्होंने पश्चिमी तोरणद्वार को भी खोज निकाला।

प्रश्न 32.
साँची क्यों बच गया, जबकि अमरावती नष्ट हो गया?
उत्तर:
सम्भवतः अमरावती की खोज थोड़ी पहले हो गई थी। 1818 में जब साँची की खोज हुई, इसके तीन तोरणद्वार तब भी खड़े थे। चौथा तोरणद्वार वहीं पर गिरा हुआ था। उस समय भी यह सुझाव आया कि तोरणद्वारों को पेरिस या लन्दन भेज दिया जाए। परन्तु कई कारणों से साँची का स्तूप वहीं बना रहा और आज भी बना हुआ है। दूसरी ओर अमरावती का महाचैत्व अब केवल एक छोटा- सा टीला है जिसका सम्पूर्ण गौरव नष्ट हो चुका है।

प्रश्न 33.
इतिहासकार किसी मूर्तिकला की व्याख्या लिखित साक्ष्यों के साथ तुलना के द्वारा करते हैं। स्पष्ट कीजिए।
उत्तर:
प्रस्तुत चित्र पहली बार देखने पर तो इस मूर्तिकला के अंश में फूस की झोंपड़ी तथा पेड़ों वाले ग्रामीण दृश्य का चित्रण दिखाई देता है परन्तु कला इतिहासकार इसे ‘वेसान्तर जातक’ से लिया गया एक दृश्य बताते हैं यह कहानी एक ऐसे दानी राजकुमार के बारे में है जो अपना सर्वस्व एक ब्राह्मण को सौंप कर स्वयं अपनी पत्नी तथा बच्चों के साथ वन में रहने के लिए चला गया। इस उदाहरण से स्पष्ट है कि प्राय: इतिहासकार किसी मूर्तिकला की व्याख्या लिखित साक्ष्यों के साथ तुलना के द्वारा करते हैं। (पाठ्यपुस्तक का चित्र 4.13, पेज 100)

प्रश्न 34.
“कई प्रारम्भिक मूर्तिकारों ने बुद्ध को मानव के रूप में न दिखाकर उनकी उपस्थिति प्रतीकों के माध्यम से दर्शाने का प्रयास किया।” स्पष्ट कीजिए।
उत्तर:
कई प्रारम्भिक मूर्तिकारों ने बुद्ध को मानव के रूप में न दिखाकर उनकी उपस्थिति प्रतीकों के माध्यम से दर्शाने का प्रयास किया। उदाहरण के लिए ‘रिक्त स्थान’ बुद्ध के ध्यान की दशा तथा ‘स्तूप’ ‘महापरिनिब्बान’ के प्रतीक बन गए। चक्र बुद्ध द्वारा सारनाथ में दिए गए पहले उपदेश का प्रतीक था। पेड़ बुद्ध के जीवन की एक पटना का प्रतीक था ऐसे प्रतीकों को समझने के लिए यह आवश्यक है कि इतिहासकार इन कलाकृतियों के निर्माताओं की परम्पराओं से परिचित हो ।

JAC Class 12 History Important Questions Chapter 4 विचारक, विश्वास और इमारतें : सांस्कृतिक विकास

प्रश्न 35.
साँची स्तूप के तोरणद्वार के किनारे पर एक वृक्ष पकड़कर झूलती हुई महिला की मूर्ति से किस लोक परम्परा का बोध होता है?
उत्तर:
प्रारम्भ में विद्वानों को इस मूर्ति के महत्त्व के बारे में कुछ असमंजस था। इस मूर्ति से त्याग और तपस्या के भाव प्रकट नहीं होते थे। परन्तु लोक साहित्यिक परम्पराओं के अध्ययन के द्वारा वे इस निष्कर्ष पर पहुँचे कि यह संस्कृत भाषा में वर्णित ‘शालभंजिका’ की मूर्ति है। लोक परम्परा के अनुसार लोग यह मानते थे कि इस स्वी द्वारा हुए जाने से वृक्षों में फूल खिल उठते थे और फल होने लगते थे। यह एक शुभ प्रतीक माना जाता था और इसी कारण स्तूप के अलंकरण में यह प्रयुक्त हुआ ।

प्रश्न 36.
“साँची की मूर्तियों में पाए गए कई प्रतीक या चिह्न लोक परम्पराओं से उभरे थे।” व्याख्या कीजिए।
उत्तर:
साँची की मूर्तियों में पाए गए कई प्रतीक या चिह्न लोक परम्पराओं से उभरे थे। उदाहरण के लिए, साँची में जानवरों के कुछ सुन्दर उत्कीर्णन पाए गए हैं। इन जानवरों में हाथी, घोड़े, बन्दर और गाय-बैल सम्मिलित हैं। ऐसा प्रतीत होता है कि यहाँ पर लोगों को आकर्षित करने के लिए जानवरों का उत्कीर्णन किया गया था। इसके अतिरिक्त जानवरों का मनुष्यों के गुणों के प्रतीक के रूप में प्रयोग किया जाता था। उदाहरणार्थ, हाथी शक्ति और ज्ञान के प्रतीक माने जाते थे।

प्रश्न 37.
चित्र 4.19 पृष्ठ 102 पर चित्रित कमलदल और हाथियों के बीच एक महिला की मूर्ति से किस लोक परम्परा के बारे में जानकारी मिलती है?
उत्तर:
इस चित्र में हाथी महिला के ऊपर जल छिड़क रहे हैं, जैसे वे उनका अभिषेक कर रहे हो कुछ इतिहासकार इस महिला को बुद्ध की माँ माया मानते हैं तो दूसरे इतिहासकार इसे एक लोकप्रिय देवी गजलक्ष्मी मानते हैं। गजलक्ष्मी सौभाग्य लाने वाली देवी थी, जिन्हें प्राय: हाथियों से सम्बन्धित किया जाता है। यह भी सम्भव है कि इन उत्कीर्णित मूर्तियों को देखने वाले उपासक इसे माया और गजलक्ष्मी दोनों से ही जोड़ते थे।

प्रश्न 38.
अजन्ता की चित्रकारी की प्रमुख विशेषताओं का वर्णन कीजिए।
उत्तर:
प्राचीन काल में चित्रकारी जैसे सम्प्रेषण के अन्य माध्यमों का भी प्रयोग किया जाता था। इन चित्रों में जो सबसे अच्छी दशा में बचे हुए हैं, वे गुफाओं की दीवारों पर बने चित्र हैं। इनमें अजन्ता (महाराष्ट्र) की चित्रकारी अत्यन्त प्रसिद्ध है। अजन्ता के चित्रों में जातकों की कथाएँ चित्रित हैं। इनमें राजदरबार का जीवन, शोभायात्राएँ, काम करते हुए स्वी- पुरुष और त्यौहार मनाने के चित्र दिखाए गए हैं अजन्ता के कुछ चित्र अत्यन्त स्वाभाविक एवं सजीव लगते हैं।

प्रश्न 39.
बौद्धमत में महायान के विकास का वर्णन कीजिए।
अथवा
महायान बौद्ध धर्म के विकास का संक्षेप में वर्णन कीजिये।
उत्तर:
ईसा की प्रथम शताब्दी के पश्चात् यह विश्वास किया जाने लगा कि बुद्ध लोगों को मुक्ति दिलवा सकते थे। साथ-साथ बोधिसत्त की अवधारणा भी विकसित होने लगी। बोधिसत्तों को परम दयालु जीव माना गया जो अपने सत्कार्यों से पुण्य कमाते थे परन्तु वे इस पुण्य का प्रयोग दूसरों का कल्याण करने में करते थे। इस प्रकार बौद्धमत में बुद्ध और बोधिसत्वों की मूर्तियों की पूजा इस परम्परा का एक महत्त्वपूर्ण अंग बन गई। इस नई परम्परा को ‘महायान’ कहा गया।

प्रश्न 40.
पौराणिक हिन्दू धर्म के उदय का वर्णन कीजिए।
उत्तर:
हिन्दू धर्म में वैष्णव एवं शैव परम्पराएँ सम्मिलित हैं। वैष्णव परम्परा में विष्णु को सबसे महत्वपूर्ण देवता माना जाता है और शैव परम्परा में शैव को परमेश्वर माना जाता है। इनके अन्तर्गत एक विशेष देवता की पूजा को विशेष महत्व दिया जाता था। इस प्रकार की आराधना में उपासना और ईश्वर के बीच का सम्बन्ध प्रेम और समर्पण का सम्बन्ध माना जाता था। इसे भक्ति कहते हैं।

JAC Class 12 History Important Questions Chapter 4 विचारक, विश्वास और इमारतें : सांस्कृतिक विकास

प्रश्न 4.
‘अवतारवाद की अवधारणा’ से आप क्या समझते हैं ?
उत्तर:
वैष्णववाद में दस अवतारों की कल्पना की गई है यह माना जाता था कि पापियों के बढ़ते प्रभाव के कारण जब संसार में अराजकता, अव्यवस्था और विनाश की स्थिति आ जाती थी, तब विश्व की रक्षा के लिए भगवान अलग-अलग रूपों में अवतार लेते थे। सम्भवतः पृथक् पृथक् अवतार देश के भिन्न-भिन्न भागों में लोकप्रिय थे। इन समस्त स्थानीय देवताओं को विष्णु का रूप मान लेना एकीकृत धार्मिक परम्परा के निर्माण का एक महत्त्वपूर्ण तरीका था।

प्रश्न 42.
“वैष्णववाद में अनेक अवतारों के इर्द- गिर्द पूजा पद्धतियाँ विकसित हुई।” व्याख्या कीजिये।
अथवा
वैष्णववाद में अनेक अवतारों के इर्द-गिर्द पूजा – पद्धतियाँ किस प्रकार विकसित हुई? विवेचना कीजिये।
उत्तर:
वैष्णववाद में कई अवतारों को मूर्तियों के रूप में दिखाया गया है। अन्य देवताओं की मूर्तियों का भी निर्माण हुआ। शिव को उनके प्रतीक लिंग के रूप में बनाया जाता था। परन्तु उन्हें कई बार मनुष्य के रूप में भी दिखाया गया है। ये समस्त चित्रण देवताओं से जुड़ी हुई मिश्रित अवधारणाओं पर आधारित थे। उनकी विशेषताओं और प्रतीकों को उनके शिरोवस्त्र, आभूषणों, आयुषों (हथियार और हाथ में धारण किए गए अन्य शुभ अस्त्र) और बैठने की शैली से दर्शाया जाता था।

प्रश्न 43.
प्रारम्भिक मन्दिरों की विशेषताओं का वर्णन कीजिए।
उत्तर:
(1) शुरू के मन्दिर एक चौकोर कमरे के रूप में थे जिन्हें गर्भगृह कहा जाता था। इनमें एक दरवाजे से उपासक मूर्ति की पूजा करने के लिए भीतर प्रविष्ट हो
सकता था।
(2) धीरे-धीरे गर्भगृह के ऊपर एक ऊँचा ढाँचा बनाया जाने लगा जिसे शिखर कहा जाता था।
(3) मन्दिर की दीवारों पर प्रायः भित्तिचित्र उत्कीर्ण किए जाते थे।
(4) बाद में मन्दिरों में विशाल सभा स्थल, ऊंची दीवारों और तोरणों का भी निर्माण किया जाने लगा।
(5) कुछ पहाड़ियों को काट कर कुछ गुफा मन्दिर भी बनाये गये।

प्रश्न 44.
उन्नीसवीं शताब्दी के यूरोपीय विद्वान यूनानी शैली से प्रभावित भारतीय मूर्तियों से क्यों प्रभावित हुए?
उत्तर:
उन्नीसवीं शताब्दी के यूरोपीय विद्वानों ने मूर्तियों की तुलना प्राचीन यूनान की कला परम्परा से की। वे बुद्ध और बोधिसतों की मूर्तियों की खोज से काफी उत्साहित हुए। इसका कारण यह था कि ये मूर्तियाँ यूनानी मूर्तिकला से प्रभावित थीं। ये मूर्तियाँ यूनानी मूर्तियों से काफी मिलती- जुलती थीं। चूँकि वे विद्वान यूनानी परम्परा से परिचित थे, इसलिए उन्होंने इन्हें भारतीय मूर्तिकला का सर्वश्रेष्ठ नमूना माना।

प्रश्न 45.
कृत्रिम गुफाएँ बनाने की परम्परा का वर्णन कीजिए।
उत्तर:
प्राचीनकाल में कुछ मन्दिर पहाड़ियों को काट कर खोखला कर कृत्रिम गुफाओं के रूप में बनाए गए थे। कृत्रिम गुफाएँ बनाने की परम्परा काफी प्राचीन थी। सबसे प्राचीन कृत्रिम गुफाएँ ई. पूर्व तीसरी सदी में अशोक के आदेश से बराबर ( बिहार ) की पहाड़ियों में आजीविक सम्प्रदाय के संतों के लिए बनाई गई थीं कृत्रिम गुफा बनाने का सबसे विकसित रूप हमें आठवीं शताब्दी के कैलाशनाथ के मन्दिर में दिखाई देता है। इसे पूरी पहाड़ी काटकर बनाया गया था।

प्रश्न 46.
पुरातत्त्ववेत्ता एच. एच. कोल प्राचीन कलाकृतियों के बारे में क्या सोच रखते थे?
उत्तर:
पुरातत्त्ववेत्ता एच. एच. कोल प्राचीन कलाकृतियों को उठा ले जाने के विरुद्ध थे। वे इस लूट को आत्मघाती मानते थे। उनका मानना था कि संग्रहालयों में मूर्तियों की प्लास्टर कलाकृतियाँ रखी जानी चाहिए जबकि असली कृतियाँ खोज के स्थान पर ही रखी जानी चाहिए।

प्रश्न 47.
जैन धर्म के अहिंसावादी स्वरूप के विषय में आप क्या जानते हैं? स्पष्ट कीजिए।
उत्तर:
हॉपकिन्स नामक विद्वान ने कहा था कि जैन धर्म कीट-पतंगों का भी पोषण करता है, पूर्णतया सत्य है। क्योंकि जैन धर्म में अहिंसा पर अत्यधिक बल दिया गया है। विश्व के अस्तित्व के लिए प्रत्येक अणु महत्त्वपूर्ण है। इसके लिए जैन दार्शनिकों ने मत दिए कि व्यक्ति को मन, कर्म तथा वचन से पूर्ण अहिंसावादी होना चाहिए। मनुष्य को जाने-अनजाने में भी प्राणियों के प्रति किसी भी हिंसावादी गतिविधि का सम्पादन नहीं करना चाहिए। इसी के फलस्वरूप अनेक जैन सन्त मुँह पर पट्टी बाँधते हैं; जिससे उनके श्वास लेते समय कोई कीटाणु उनके मुँह में न चला जाए।

JAC Class 12 History Important Questions Chapter 4 विचारक, विश्वास और इमारतें : सांस्कृतिक विकास

प्रश्न 48.
गौतम बुद्ध एक महान् समाज सुधारक थे। स्पष्ट कीजिए।
उत्तर:
बौद्ध धर्म का उद्भव एक क्रान्तिकारी धर्म के रूप में हुआ था। इस धर्म के प्रवर्तक मध्यमार्गी बुद्ध थे। गौतम बुद्ध की विचारधारा, उपदेश तथा कार्यप्रणाली अत्यन्त सरल थी। गौतम बुद्ध ने जातिवाद विशेषकर जन्म पर आधारित जातिवाद का घोर विरोध किया तथा जन्म के स्थान पर कर्म को व्यक्ति की पहचान बताया।

एक स्थान पर गौतम बुद्ध अपने शिष्य आनन्द से कहते हैं कि साधक की जाति क्या पूछता है कर्म पूछ गौतम बुद्ध की यह विचारधारा तत्कालीन अतिवादी तथा ब्राह्मणवादी व्यवस्था के विरुद्ध जनसामान्य की सबसे बड़ी आवश्यकता थी। बुद्ध ने जनता की इस आवश्यकता को समझा और ऐसी कर्मकाण्डीय ब्राह्मणवादी व्यवस्था का विरोध किया; जिसमें जनसामान्य का कोई स्थान नहीं था।

प्रश्न 49.
बौद्ध संघों की स्थापना क्यों की गयी?
उत्तर:
बौद्ध धर्म की व्यापक लोकप्रियता के कारण शिष्यों की संख्या तीव्रता से बढ़ने लगी। शिष्यों में ही कुछ ऐसे शिष्य जिनका आत्मिक विकास उच्च था; वे धम्म के शिक्षक बन गये। संघ की स्थापना धम्म के शिक्षकों के की गयी संघ में सदाचरण और नैतिकता पर बल दिया जाता था। हेतु प्रारम्भ में महिलाओं का संघ में प्रवेश वर्जित था परन्तु बाद में महात्मा बुद्ध ने महिलाओं को भी संघ में प्रवेश की अनुमति दे दी। महाप्रजापति गौतमी जो कि बुद्ध की उपमाता थीं, संघ में प्रवेश करने वाली पहली महिला थीं।

प्रश्न 50.
जैन तथा बौद्ध धर्म में क्या भिन्नताएँ हैं? संक्षेप में लिखिए।
उत्तर:
जैन तथा बौद्ध धर्म में अनेक समानताएँ थीं परन्तु कुछ असमानताएँ भी थीं। प्रमुख भिन्नताओं का विवरण निम्नलिखित है –

  1.  जैन धर्म आत्मवादी है परन्तु बौद्ध धर्म अनात्मवादी
  2. जैनों के साहित्य को आगम कहा जाता है; बौद्धों के साहित्य को त्रिपिटक कहा जाता है।
  3. जैन धर्म अत्यधिक अहिंसावादी है बौद्ध धर्म मध्यमार्गी है।
  4. जैन धर्म के अनुसार मोक्ष मृत्यु के पश्चात् ही सम्भव है वहीं बौद्ध धर्म के अनुसार इसी जन्म में निर्वाण सम्भव है।
  5. जैन धर्म में तीर्थंकरों की उपासना होती है परन्तु बौद्ध धर्म में बुद्ध और बोधिसत्वों की आराधना की जाती है।

प्रश्न 51.
बौद्ध मूर्तिकला की प्रतीकात्मक पद्धति क्या थी? इन प्रतीकों को समझ पाना एक दुष्कर कार्य क्यों था?
उत्तर:
बौद्ध मूर्तिकला की मूर्तियों को स्पष्ट रूप से समझ पाना एक दुष्कर कार्य इसलिए है कि इतिहासकार केवल इस बात का अनुमान ही लगा सकते हैं कि मूर्ति बनाते समय मूर्तिकार का दृष्टिकोण क्या था इतिहासकारों के ‘अनुमान के अनुसार आरम्भिक मूर्तिकारों ने महात्मा बुद्ध को मनुष्य के रूप में न दिखाकर उन्हें प्रतीकों के माध्यम से प्रकट करने का प्रयास किया है। चित्र में दिखाए गए रिक्त स्थान को इतिहासकार बुद्ध की ध्यान अवस्था के रूप में बताते हैं; क्योंकि ध्यान की महादशा में अन्तर में रिक्तता की अनुभूति होती है स्तूप को महानिर्वाण की दशा के रूप में व्याख्यायित किया है।

महानिर्वाण का अर्थ है- विराट में समा जाना चक्र को बुद्ध द्वारा सारनाथ में दिए गए पहले प्रवचन का प्रतीक माना गया है; इसके अनुसार यहीं से बौद्ध धर्म के प्रचार-प्रसार का चक्र घूमा पेड़ का अर्थ मात्र पेड़ के रूप में नहीं बल्कि वह बीज की पूर्ण परिपक्वता का प्रतीक है एक बीज जिसकी सम्भावना वृक्ष बनने की है वह अपनी पूर्णता को प्राप्त हुआ इसी प्रकार बुद्ध अपने जीवन में सम्पूर्णता को प्राप्त हुए।

प्रश्न 52.
साँची के प्रतीक लोक-परम्पराओं से जुड़े थे; संक्षिप्त विवेचना कीजिए।
उत्तर:
साँची की मूर्तियों में प्राप्त उत्कीर्णन में लोक परम्परा से जुड़े हुए बहुत से प्रतीकों का चिशंकन है। मूर्तियों को आकर्षक और सुन्दर दर्शाने हेतु विविध प्रतीकों जैसे हाथी, घोड़ा, बन्दर, गाय, बैल आदि जानवरों का उत्कीर्णन जीवन्त रूप से किया गया है। हाथी को शक्ति तथा ज्ञान का प्रतीक कहा गया है। इसी प्रकार एक स्वी तोरणद्वार के किनारे एक पेड़ पकड़कर झूल रही है। यह शालभंजिका की मूर्ति है, लोक परम्परा में शाल भंजिका को शुभ प्रतीक माना जाता है। वामदल और हाथियों के बीच एक महिला को एक अन्य मूर्ति में दिखाया गया है।

हाथी उस महिला के ऊपर जल छिड़क रहे हैं; जैसे उसका अभिषेक कर रहे हैं। इस महिला को बुद्ध की माँ माना जाता है। कुछ इतिहासकारों के अनुसार यह महिला सौभाग्य की देवी गजलक्ष्मी है। इतिहासकारों का इस सम्बन्ध में अपना दृष्टिकोण है। सर्पों का उत्कीर्णन भी कई स्तम्भों पर पाया जाता है। इस प्रतीक को भी लोक परम्परा से जुड़ा हुआ माना जाता है। प्रारम्भिक इतिहासकार जेम्स फर्गुसन के अनुसार साँची में वृक्षों और सर्पों की पूजा की जाती थी। वे बौद्ध साहित्य से अनभिज्ञ थे, उन्होंने उत्कीर्णित मूर्तियों का अध्ययन करके अपना यह निष्कर्ष निकाला था।

JAC Class 12 History Important Questions Chapter 4 विचारक, विश्वास और इमारतें : सांस्कृतिक विकास

प्रश्न 53.
अतीत से प्राप्त चित्रों की क्या विशेषताएँ थीं?
उत्तर:
अजन्ता और एलोरा की गुफाओं में बने भित्ति चित्र सम्पूर्ण विश्व के आकर्षण का केन्द्र है। चित्रकारी भी मूर्तिकला की भाँति सम्प्रेषण का एक माध्यम है। जातक की कथाओं का चित्रण बहुत ही सुन्दर ढंग से अजन्ता के चित्रों में दिखाया गया है। इन कथाओं में राजदरबार का चित्रण, शोभा यात्राएं, त्यौहारों का उत्सव और कार्य करते हुए पुरुषों और महिलाओं का चित्रांकन है। यह चित्र अत्यन्त ही सुन्दर और सजीव है हर्षोल्लास, उमंग, प्रसन्नता, प्रेम की भावनाओं की अभिव्यक्ति इतनी कुशलता से और जीवन्तता से की गई है कि यह लगता है कि चित्र बोल पड़ेंगे। कलाकारों ने इन्हें त्रिविम रूप से चित्रित किया गया है; इसके लिए आभा भेद तकनीक का प्रयोग करके इन्हें सजीवता प्रदान की है।

प्रश्न 54.
हीनयान तथा महायान में मुख्य अन्तरों को समझाइए।
उत्तर:
हीनयान तथा महायान में अनेक मूलभूत अन्तर हैं। इनमें से प्रमुख अन्तरों का विवरण निम्नलिखित है –

  1. हीनयान प्राचीन, रूढ़िवादी तथा मूल मत है जबकि महायान बौद्ध मत का संशोधित तथा सरल रूप है जो चतुर्थ बौद्ध संगीति में प्रकाश में आया।
  2. हीनयानी बुद्ध की स्तुति प्रतीकों के माध्यम से करते हैं वहीं महायानी मूर्ति पूजा में विश्वास करते हैं।
  3. हीनयान मत रूढ़िवादी है जबकि महायान अत्यधिक सरल मत है।
  4. हीनयान में बुद्ध की स्तुति की जाती है; वहीं महायान में बुद्ध के साथ-साथ बोधिसत्वों की आराधना भी की जाती है।
  5. हीनयान ज्ञान को महत्त्व देता है जबकि महायान करुणा को।

प्रश्न 55.
प्राचीन भारतीय कला की पृष्ठभूमि और महत्त्व को 19वीं सदी के यूरोपीय विद्वान प्रारम्भ में क्यों नहीं समझ सके ? उनकी समस्या का निराकरण किस प्रकार हुआ?
उत्तर:
प्रत्येक देश की धार्मिक आस्थाओं, धारणाओं, परम्पराओं आदि में अन्तर होते हैं उनके सोचने और समझने के ढंग और प्रतिमान अलग-अलग होते हैं। इसलिए यूरोपीय विद्वानों ने जब प्राचीन भारतीय मूर्तिकला की यह मूर्तियाँ जो कई हाथों, कई सिरों या अर्द्ध मानव के रूप में निर्मित थीं देखीं तो उन्हें यह विचित्र प्रतीत हुई। आराध्य देवों के यह रूप उनकी कल्पना से परे थे। फिर भी इन आरम्भिक यूरोपीय विद्वानों ने इन विभिन्न रूपों वाली आराध्य देवों की मूर्तियों को समझने हेतु प्रयास किए।

यूरोपीय विद्वानों ने प्राचीन यूनानी कला परम्परा की पृष्ठभूमि को आधार बनाकर इन मूर्तियों की यूनानी मूर्तियों से तुलना की तथा उन्हें समझने का प्रयास किया। लेकिन जब उन्होंने बौद्ध धर्म की कला परम्परा, बौद्ध धर्म की बुद्ध और बोधसत्व की मूर्तियाँ देखीं तो वे बहुत प्रोत्साहित हुए। इन मूर्तियों की उत्कृष्टता को देखकर हैरान रह गए, उन्हें लगा ये मूर्तियाँ यूनानी प्रतिमानों के अनुरूप हैं। इस प्रकार इस मूर्तिकला की अनजानी व अपरिचित पृष्ठभूमि और इसके अपरिचित महत्त्व को उन्होंने परिचित यूनानी मूर्तिकला के आधार पर समझने का प्रयास किया।

निबन्धात्मक प्रश्न

प्रश्न 1.
साँची के स्तूप के संरक्षण में भोपाल की शासिकाओं के योगदान का वर्णन कीजिये।
उत्तर:
साँची के स्तूप के संरक्षण में भोपाल की शासिकाओं का योगदान साँची के स्तूप के संरक्षण में भोपाल की शासिकाओं के योगदान का वर्णन निम्नलिखित बिन्दुओं के अन्तर्गत किया जा सकता है –
(1) साँधी के स्तूप के रख-रखाव के लिए धन का अनुदान देना- भोपाल की शासिकाओं- शाहजहाँ बेगम तथा उनकी उत्तराधिकारी सुल्तान जहाँ बेगम की पुरातात्विक स्थलों के संरक्षण में बड़ी रुचि थी। उन्होंने साँची के विश्व प्रसिद्ध स्तूप के रख-रखाव के लिए प्रचुर धन का अनुदान दिया।

(2) जान मार्शल द्वारा रचित पुस्तक के प्रकाशन के लिए अनुदान देना जान मार्शल ने साँची स्तूप पर एक महत्वपूर्ण ग्रन्थ की रचना की। उन्होंने सुल्तानजहाँ की पुरातात्विक स्थलों के प्रति रुचि को देखते हुए अपना प्रसिद्ध ग्रन्थ सुल्तानजहाँ बेगम को समर्पित किया। सुल्तानजहाँ बेगम ने इस ग्रन्थ के विभिन्न खण्डों के प्रकाशन हेतु धन का अनुदान दिया।

(3) संग्रहालय और अतिथिशाला का निर्माण करवाया सुल्तानजहाँ बेगम ने स्तूप स्थल पर एक संग्रहालय तथा अतिथिशाला के निर्माण के लिए प्रचुर धन का अनुदान दिया।

(4) साँची के स्तूप को भोपाल राज्य में सुरक्षित रखना – उन्नीसवीं शताब्दी के यूरोपीय लोगों में साँची के स्तूप को लेकर बड़ी रुचि थी। प्रारम्भ में फ्रांसीसियों ने सबसे अच्छी दशा में बचे साँची के पूर्वी तोरणद्वार को फ्रांस के संग्रहालय में प्रदर्शित करने हेतु शाहजहाँ बेगम से फ्रांस ले जाने की अनुमति माँगी।

इसके बाद अंग्रेजों ने भी साँची के पूर्वी तोरणद्वार को इंग्लैण्ड ले जाने की अनुमति माँगी। परन्तु शाहजहाँ बेगम ने उन्हें साँची के स्तूप की प्लास्टर प्रतिकृतियाँ देकर सन्तुष्ट कर दिया। इस प्रकार शाहजहाँ बेगम के प्रयासों के परिणामस्वरूप साँची के स्तूप की मूलकृति भोपाल राज्य में अपने स्थान पर ही सुरक्षित रही। इस प्रकार यह स्तूप समूह बना रहा है तो इसके पीछे भोपाल की बेगमों के विवेकपूर्ण निर्णयों की बड़ी भूमिका रही है।

प्रश्न 2.
बौद्ध ग्रन्थ किस प्रकार तैयार और संरक्षित किए जाते थे?
उत्तर:
बौद्ध ग्रंथों को तैयार एवं संरक्षित करना बौद्ध धर्म के संस्थापक महात्मा बुद्ध अन्य शिक्षकों की भाँति चर्चा और वार्तालाप करते हुए मौखिक शिक्षा देते थे। स्त्री, पुरुष और बच्चे इन प्रवचनों को सुनते थे और इन पर चर्चा करते थे। बुद्ध की शिक्षाओं को उनके जीवनकाल में लिखा नहीं गया। उनकी मृत्यु के बाद पांचवीं चौथी सदी ई. पूर्व में उनके शिष्यों ने वरिष्ठ श्रमणों की एक सभा वैशाली में आयोजित की। वहाँ पर ही उनकी शिक्षाओं का संकलन किया गया। इन संग्रहों को ‘त्रिपिटक’ कहा जाता था।

(1) त्रिपिटक त्रिपिटक का शाब्दिक अर्थ है-भिन्न प्रकार के ग्रन्थों को रखने के लिए ‘तीन टोकरियाँ’ त्रिपिटक तीन हैं-

  • विनयपिटक – इसमें संघ या बौद्ध मठों में रहने वाले बौद्ध भिक्षुओं और भिक्षुणियों के लिए आचरण सम्बन्धी नियमों का संग्रह है।
  • सुत्तपिटक – इसमें बुद्ध की शिक्षाओं का संग्रह है।
  • अभिधम्मपिटक इसमें बौद्धधर्म के दार्शनिक सिद्धान्तों का विवेचन है।

प्रत्येक पिटक के अन्दर कई ग्रन्थ होते थे-बाद के युगों में बौद्ध विद्वानों ने इन ग्रन्थों पर टीकाएँ लिखीं।

(2) दीपवंश तथा महावंश-जब बौद्ध धर्म का श्रीलंका जैसे नए क्षेत्रों में प्रसार हुआ, तब दीपवंश ( द्वीप का इतिहास) तथा महावंश (महान इतिहास) जैसे क्षेत्र विशेष के बौद्ध इतिहास को लिखा गया। इनमें से कई रचनाओं में बुद्ध की जीवनी लिखी गई है। अधिकांश पुराने ग्रन्थ पालि में हैं। कालान्तर में संस्कृत में भी ग्रन्थों की रचना की गई।

(3) बौद्ध ग्रन्थों का संरक्षण जब बौद्ध धर्म का पूर्वी एशिया में प्रसार हुआ, तब फा-शिएन और श्वैन-त्सांग जैसे तीर्थयात्री बौद्धग्रन्थों की खोज में चीन से भारत आए थे। वे अनेक बौद्ध ग्रन्थ अपने देश ले गए जहाँ विद्वानों ने इनका अनुवाद किया। भारत के बौद्ध शिक्षक भी दूर-दराज के देशों में गए। बुद्ध की शिक्षाओं का प्रचार-प्रसार करने के लिए वे अनेक ग्रन्थ भी अपने साथ ले गए। कई शताब्दियों तक ये पांडुलिपियाँ एशिया के भिन्न-भिन्न देशों में स्थित बौद्ध- विहारों में संरक्षित थीं। पालि, संस्कृत, चीनी और तिब्बती भाषाओं में लिखे इन ग्रन्थों से आधुनिक अनुवाद तैयार किए गए हैं।

प्रश्न 3.
नियतिवादियों एवं भौतिकवादियों के विचारों की विवेचना कीजिये।
उत्तर:
नियतिवादियों के विचार मक्खलिगोसाल नामक व्यक्ति नियतिवादियों के प्रमुख दार्शनिक थे। नियतिवादियों के प्रमुख विचार निम्नलिखित थे
(1) सब कुछ पूर्व निर्धारित है नियतिवादियों के अनुसार सब कुछ पूर्व निर्धारित है। सुख और दुःख पूर्व निर्धारित मात्रा में माप कर दिए गए हैं। स्न्नों संका में बदला नहीं जा सकता। इन्हें बढ़ाया या घटाया नहीं जा सकता। जैसे धागे का गोला फेंक देने पर लुढ़कते लुढ़कते अपनी पूरी लम्बाई तक खुलता जाता है, उसी प्रकार मूर्ख और विद्वान दोनों ही पूर्व निर्धारित मार्ग से होते हुए दुःखों का निदान करेंगे।

(2) कर्म – मुक्ति की निरर्थक आशा करना- नियतिवादियों का कहना है कि बुद्धिमान लोग यह विश्वास करते हैं कि वे अपने सद्गुणों एवं तपस्या के बल पर कर्म मुक्ति प्राप्त करेंगे। इसी प्रकार मूर्ख लोग उन्हीं कार्यों को सम्पन्न करके शनैः-शनैः कर्म मुक्ति प्राप्त करने की आशा करते हैं। परन्तु उनका यह सोचना गलत है तथा दोनों में से कोई भी कुछ नहीं कर सकता। इसलिए लोगों के भाग्य में जो कुछ लिखा है, उसे उन्हें भोगना ही पड़ेगा।

भौतिकवादियों के विचार अजीत केसकंबलिन नामक व्यक्ति भौतिकवादियों के प्रमुख दार्शनिक थे भौतिकवादियों के प्रमुख विचार निम्नलिखित है-
(1) संसार में दान, यज्ञ या चढ़ावा जैसी कोई वस्तु नहीं होती। इस दुनिया या दूसरी दुनिया जैसी किसी वस्तु का कोई अस्तित्व नहीं होता।
(2) मनुष्य चार तत्त्वों से बना होता है जब उसकी मृत्यु होती है, तब मिट्टी वाला अंश पृथ्वी में, जल वाला अंश जल में, गर्मी वाला अंश आग में तथा साँस का अंश वायु में वापस मिल जाता है और उसकी इन्द्रियाँ अन्तरिक्ष का भाग बन जाती हैं।
(3) दान देने की बात मूर्खों का सिद्धान्त है, यह सिद्धान्त असत्य है। कुछ मूर्ख एवं विद्वान दोनों ही कट कर नष्ट हो जाते हैं। मृत्यु के बाद कोई नहीं बचता।

JAC Class 12 History Important Questions Chapter 4 विचारक, विश्वास और इमारतें : सांस्कृतिक विकास

प्रश्न 4.
महात्मा बुद्ध की जीवनी का वर्णन कीजिए।
उत्तर:
महात्मा बुद्ध की जीवनी महात्मा बुद्ध बौद्ध धर्म के संस्थापक और प्रवर्तक थे। बुद्ध का जन्म 563 ई. पू. में कपिलवस्तु के निकट लुम्बिनी नामक वन में हुआ था। इनके पिता का नाम शुद्धोधन था, जो शाक्य गणराज्य के प्रधान थे। इनकी माता का नाम महामाया (मायादेवी) था। बुद्ध के बचपन का नाम सिद्धार्थ था।

16 वर्ष की आयु में सिद्धार्थ का विवाह यशोधरा नामक एक सुन्दर कन्या से किया गया। कुछ समय बाद उनके यहाँ पुत्र ने जन्म लिया, जिसका नाम राहुल रखा गया। परन्तु सिद्धार्थ की संसार से विरक्ति बढ़ती गई। महाभिनिष्क्रमण-नगर-दर्शन हेतु विभिन्न अवसरों पर बाहर जाते हुए सिद्धार्थ ने एक वृद्ध व्यक्ति, रोगी, मृतक एवं संन्यासी को देखा जिन्हें देखकर उन्हें संसार से विरक्ति हो गई। अन्त में 29 वर्ष की आयु में सिद्धार्थ अपनी पत्नी, अपने पुत्र तथा राजकीय वैभव को छोड़कर ज्ञान की खोज में निकल पड़े।

यह घटना ‘महाभिनिष्क्रमण’ के नाम से प्रसिद्ध है। ज्ञान की प्राप्ति प्रारम्भ में सिद्धार्थ ने वैशाली के ब्राह्मण विद्वान आलारकालाम तथा राजगृह के विद्वान् उद्रक रामपुत्त से ज्ञान प्राप्त करने का प्रयास किया, परन्तु उनकी जिज्ञासा शान्त नहीं हुई। इसके बाद वे उरुवेल के जंगल में अपने पाँच ब्राह्मण साथियों के साथ कठोर तपस्या करने लगे, परन्तु यहाँ भी उनके हृदय को शान्ति नहीं मिली। उनका शरीर सूख सूख कर काँटा हो गया, परन्तु उन्हें ज्ञान की प्राप्ति नहीं हुई। अतः उन्होंने भोजन आदि ग्रहण करना शुरू कर दिया जिससे नाराज होकर उनके पाँचों साथी सिद्धार्थ का साथ छोड़कर वहाँ से चले गए।

अन्त में सिद्धार्थ एक पीपल के वृक्ष के नीचे ध्यान की अवस्था में बैठ गए। सात दिन अखण्ड समाधि में लीन रहने के बाद वैशाखी पूर्णिमा की रात को उन्हें ‘ज्ञान’ प्राप्त हुआ, उन्हें सत्य के दर्शन हुए और वे ‘बुद्ध’ कहलाने लगे। इस घटना को ‘सम्बोधि’ कहा जाता है। जिस वृक्ष के नीचे सिद्धार्थ को ज्ञान प्राप्त हुआ, उसे ‘बोधिवृक्ष’ कहा जाने लगा और गया ‘बोधगया’ के नाम से प्रसिद्ध हुआ। धर्म का प्रचार ज्ञान प्राप्त करने के पश्चात् बुद्ध ने सारनाथ पहुँचकर उन पाँच ब्राह्मणों को उपदेश दिया, जो उन्हें छोड़कर चले आए थे। ये पाँचों ब्राह्मण बुद्ध के अनुयायी बन गए। इस घटना को ‘धर्मचक्र प्रवर्तन’ कहते हैं। इसके बाद बुद्ध ने काशी, कोशल, मगध, वज्जि प्रदेश, मल्ल, वत्स आदि में अपने धर्म की शिक्षाओं का प्रचार किया।

उनके अनुयायियों की संख्या दिन-प्रतिदिन बढ़ती गई। उनके अनुयायियों में अनेक राजा, व्यापारी, ब्राह्मण, विद्वान, सामान्यजन कर्मकार, दास, शिल्पी आदि सम्मिलित थे। महापरिनिर्वाण लगभग 45 वर्ष तक बुद्ध अपने धर्म का प्रचार करते रहे। अन्त में 483 ई. पू. में 80 वर्ष की आयु में कुशीनगर में बुद्ध का देहान्त हो गया। बौद्ध परम्परा के अनुसार यह घटना ‘महापरिनिर्वाण’ कहलाती है।

JAC Class 12 History Important Questions Chapter 4 विचारक, विश्वास और इमारतें : सांस्कृतिक विकास

प्रश्न 5.
बुद्ध की शिक्षाओं का विवेचन कीजिए। अथवा
उत्तर:
बौद्ध धर्म के प्रमुख सिद्धान्तों का वर्णन कीजिए। बुद्ध की शिक्षाएँ (बौद्ध धर्म के सिद्धान्त ) बुद्ध की प्रमुख शिक्षाएँ निम्नलिखित हैं –
(1) विश्व अनित्य है – बौद्ध दर्शन के अनुसार विश्व अनित्य है और निरन्तर परिवर्तित हो रहा है। यह आत्माविहीन है क्योंकि यहाँ कुछ भी स्थायी या शाश्वत नहीं है।
(2) मध्यम मार्ग – बुद्ध का कहना था कि न तो मनुष्य को घोर तपस्या करनी चाहिए, न ही अधिक भोग- विलास में लिप्त रहना चाहिए।
(3) भगवान का होना या न होना अप्रासंगिक- बौद्ध धर्म की प्रारम्भिक परम्पराओं में भगवान का होना या न होना अप्रासंगिक था।
(4) समाज का निर्माण मनुष्यों द्वारा किया जाना- बुद्ध मानते थे कि समाज का निर्माण मनुष्यों ने किया था, न कि ईश्वर ने
(5) व्यक्तिगत प्रवास पर बल बुद्ध के अनुसार व्यक्तिगत प्रयास से सामाजिक परिवेश को बदला जा सकता था।
(6) व्यक्ति केन्द्रित हस्तक्षेप तथा सम्यक् कर्म पर बल बुद्ध ने जन्म-मृत्यु के चक्र से मुक्ति, आत्म-ज्ञान और निर्वाण के लिए आत्मकेन्द्रित हस्तक्षेप और सम्यक् कर्म पर बल दिया।
(7) निर्वाण बौद्ध धर्म के अनुसार निर्वाण का अर्थ था अहं और इच्छा का समाप्त हो जाना जिससे गृह त्याग करने वालों के दुःख के चक्र का अन्त हो सकता था।
(8) चार आर्य सत्य बौद्ध धर्म के अनुसार चार था। आर्य सत्य निम्नलिखित हैं –

  • दुःख – यह संसार दुःखमय है। संसार में सर्वत्र दुःख ही दुःख है।
  • दुःख समुदय-दुःख और कष्टों का कारण तृष्णा है।
  • दु:ख निरोध- तृष्णा नष्ट कर देने से दुःखों से. मुक्ति प्राप्त हो सकती है।
  • दु:ख निरोध का मार्ग-तृष्णा के विनाश के लिए अष्टांगिक मार्ग का अनुसरण करना चाहिए।

(9) अष्टांगिक मार्ग-अष्टांगिक मार्ग की मुख्य आठ बातें निम्नलिखित हैं –

  • सम्यक् दृष्टि (सत्य, विश्वास या दृष्टिकोण)
  • सम्यक् संकल्प (सत्य संकल्प या विचार)
  • सम्यक् वाणी (मधुर एवं सत्य वचन बोलना )
  • सम्यक् कर्मान्त (सदाचारपूर्ण आचरण करना)
  • सम्यक् आजीव (सदाचारपूर्ण साधनों से जीविका का निर्वाह करना)
  • सम्यक् व्यायाम ( निरन्तर सद्प्रयास )
  • सम्यक् स्मृति (दुर्बलताओं की निरन्तर स्मृति)
  • सम्यक् समाधि (चित्त की एकाग्रता)।

(10) स्वावलम्बन पर बल बुद्ध का कहना था कि मनुष्य स्वयं अपने भाग्य का निर्माता है।

प्रश्न 6.
बौद्ध संघ पर एक संक्षिप्त निबन्ध लिखिए।
अथवा
बुद्ध के अनुयायियों के बारे में आप क्या जानते हैं ?
उत्तर:
बौद्ध संघ (बुद्ध के अनुयायी ) महात्मा बुद्ध के उपदेशों से प्रभावित होकर उनके अनुयायियों की संख्या बढ़ती चली गई। धीरे-धीरे उनके शिष्यों का दल तैयार हो गया। उन्होंने अपने शिष्यों के लिए बौद्ध संघ की स्थापना की संघ ऐसे बौद्ध भिक्षुओं की एक संस्था थी, जो धम्म के शिक्षक बन गए। ये बौद्ध भिक्षु एक सादा जीवन व्यतीत करते थे। उनके पास जीवनयापन के लिए अत्यावश्यक वस्तुओं के अलावा कुछ नहीं होता था। वे दिन में एक बार भोजन करते थे। इसके लिए वे उपासकों से भोजन दान प्राप्त करने के लिए एक कटोरा रखते थे। चूंकि वे दान पर निर्भर थे, इसलिए उन्हें भिक्षु कहा जाता था।

(1) महिलाओं को संघ में सम्मिलित करना प्रारम्भ में केवल पुरुष ही संघ में सम्मिलित हो सकते थे परन्तु बाद में महिलाओं को भी संघ में सम्मिलित होने की अनुमति दे दी गई। बौद्ध ग्रन्थों से ज्ञात होता है कि अपने प्रिय शिष्य आनन्द के अनुरोध पर बुद्ध ने महिलाओं को संघ में सम्मिलित होने की अनुमति प्रदान कर दी। बुद्ध की उपमाता महाप्रजापति गोतमी संघ में सम्मिलित होने वाली प्रथम भिक्षुणी थी। संघ में सम्मिलित होने वाली कई स्त्रिय धम्म की उपदेशिकाएँ बन गई। कालान्तर में ये धेरी बनीं, जिसका अर्थ है-ऐसी महिलाएँ जिन्होंने निर्वाण प्राप्त कर लिया हो।

(2) बुद्ध के अनुयायियों का विभिन्न सामाजिक वर्गों से सम्बन्धित होना बुद्ध के अनुयायी विभिन्न सामाजिक वर्गों से सम्बन्धित थे। इनमें राजा, धनवान, गृहपति और सामान्य जन कर्मकार, दास, शिल्पी सभी सम्मिलित थे। संघ में सम्मिलित होने वाले सभी भिक्षुओं तथा भिक्षुणियों को बराबर माना जाता था क्योंकि भिक्षु बनने पर उन्हें अपनी पुरानी पहचान को त्याग देना पड़ता था।

JAC Class 12 History Important Questions Chapter 4 विचारक, विश्वास और इमारतें : सांस्कृतिक विकास

(3) संघ की संचालन पद्धति संघ की संचालन पद्धति गणों और संघों की परम्परा पर आधारित थी। इसके अन्तर्गत लोग वार्तालाप के द्वारा एकमत होने का प्रयास करते थे एकमत न होने पर मतदान द्वारा निर्णय लिया जाता था।

प्रश्न 7.
स्तूपों की खोज किस प्रकार हुई ? अमरावती तथा साँची की नियति बताइये।
अथवा
स्तूपों की खोज किस प्रकार हुई ? साँची क्यों बच गया, जबकि अमरावती नष्ट हो गया?
उत्तर:
स्तूपों की खोज बौद्ध कला में स्तूपों का महत्त्वपूर्ण स्थान है। साँची और अमरावती के स्तूप तत्कालीन वास्तुकला के उत्कृष्ट नमूने हैं।
(1) साँची के स्तूप की खोज-साँची का स्तूप एक पहाड़ी के ऊपर बना हुआ है जो मुकुट जैसा दिखाई देता है 1818 ई. में साँची की खोज हुई।

(2) अमरावती के स्तूप की खोज- 1796 ई. में स्थानीय राजा एक मन्दिर का निर्माण करना चाहते थे। उन्हें अचानक अमरावती के स्तूप के अवशेष मिल गए। उन्होंने इसके पत्थरों का प्रयोग करने का निश्चय किया। कुछ वर्षों के बाद कॉलिन मेकेंजी नामक एक अंग्रेज अधिकारी को इस क्षेत्र से गुजरने का अवसर मिला। यद्यपि उन्होंने यहाँ कई मूर्तियाँ प्राप्त की और उनका विस्तृत चिशंकन भी किया, परन्तु उनकी रिपोर्ट कभी प्रकाशित नहीं हुई।

(3) गुन्टूर के कमिश्नर द्वारा अमरावती की यात्रा करना- 1854 ई. में गुन्टूर (आन्ध्र प्रदेश) के कमिश्नर ने अमरावती की यात्रा की। उन्होंने कई मूर्तियों तथा उत्कीर्ण पत्थरों को एकत्रित किया और वे उन्हें मद्रास ले गए।

(4) अमरावती के स्तूप के उत्कीर्ण पत्थरों को विभिन्न स्थानों पर ले जाना-1850 के दशक में अमरावती के स्तूप के उत्कीर्ण पत्थरों को भिन्न-भिन्न स्थानों पर ले जाया गया। कुछ उत्कीर्ण पत्थर कोलकाता में एशियाटिक सोसायटी ऑफ बंगाल पहुंचे, तो कुछ पत्थर मद्रास के इण्डिया ऑफिस पहुँचे। कुछ पत्थर तो लन्दन तक पहुँच गए। कई अंग्रेज अधिकारियों ने अपने बागों में अमरावती की मूर्तियाँ स्थापित कीं।

(5) साँची क्यों बच गया, जबकि अमरावती नष्ट हो गया – सम्भवतः अमरावती की खोज थोड़ी पहले हो गई थी तब तक विद्वान इस बात के महत्त्व को नहीं समझ पाए थे कि किसी पुरातात्विक अवशेष को उठाकर ले जाने की बजाय खोज की जगह पर ही संरक्षित करना बड़ा महत्त्वपूर्ण था। 1818 में सांची की खोज हुई उस समय तक भी इस स्तूप के तीन तोरणद्वार खड़े थे।

चौथा तोरण द्वार वहीं पर गिरा हुआ था। इसके अतिरिक्त टीला भी अच्छी दशा में था। उस समय कुछ विदेशियों ने यह सुझाव दिन था कि तोरणद्वारों को पेरिस या लन्दन भेज दिया जाए। परन्तु कई कारणों से साँची का स्तूप वहीं बना रहा और आज भी बना हुआ है। दूसरी ओर अमरावती का महाचैत्य अब केवल एक छोटा सा टीला है, जिसका सम्पूर्ण गौरव नष्ट हो चुका है।

प्रश्न 8.
पौराणिक हिन्दू धर्म की विशेषताओं का वर्णन कीजिए।
उत्तर:
पौराणिक हिन्दू धर्म की विशेषताएँ पौराणिक हिन्दू धर्म की विशेषताओं का वर्णन अग्रानुसार
(1) पौराणिक हिन्दू धर्म का उदय पौराणिक हिन्दू धर्म में भी मुक्तिदाता की कल्पना विकसित हो रही थी। इस पौराणिक हिन्दू धर्म में दो परम्पराएँ प्रमुख थीं –

  • वैष्णववाद तथा
  • शैववाद वैष्णववाद में विष्णु को सबसे प्रमुख देवता माना जाता है और शैववाद में शिव परमेश्वर माने गए हैं।

(2) अवतारवाद – वैष्णववाद में कई अवतारों के चारों ओर पूजा पद्धतियाँ विकसित हुई इस परम्परा के अन्दर दस अवतारों की कल्पना की गई है। लोगों में यह मान्यता प्रचलित थी कि पापियों के बढ़ते प्रभाव के कारण जब संसार में अराजकता, अव्यवस्था और विनाश की स्थिति उत्पन्न हो जाती थी, तब संसार की रक्षा के लिए भगवान अलग-अलग रूपों में अवतार लेते थे।

(3) अवतारों को मूर्तियों में दिखाना- कई अवतारों को मूर्तियों के रूप में दिखाया गया है। अन्य देवताओं की मूर्तियों का भी निर्माण किया गया। शिव को उनके प्रतीक लिंग के रूप में बनाया जाता था परन्तु उन्हें कई बार मनुष्य के रूप में भी दर्शाया गया है।

(4) पुराणों की कहानियाँ – इन मूर्तियों के उत्कीर्णन का अर्थ समझने के लिए इतिहासकारों को इनसे जुड़ी हुई कहानियों से परिचित होना पड़ता है। कई कहानियाँ प्रथम सहस्राब्दी के मध्य से ब्राह्मणों द्वारा रचित पुराणों में पाई जाती हैं। इनमें देवी-देवताओं की भी कहानियाँ हैं। प्रायः इन्हें संस्कृत श्लोकों में लिखा गया था। इन्हें ऊंची आवाज में पढ़ा जाता था जिसे कोई भी सुन सकता था।

यद्यपि महिलाओं और शूद्रों (हरिजनों) को वैदिक साहित्य पढ़ने – सुनने की अनुमति नहीं थी. परन्तु वे पुराणों को सुन सकते थे। पुराणों की अधिकांश कहानियाँ लोगों के आपसी मेल- मिलाप से विकसित हुई। पुजारी, व्यापारी और सामान्य स्त्री-पुरुष एक स्थान से दूसरे स्थान पर आते-जाते हुए, अपने विश्वासों और विचारों का आदान-प्रदान करते थे। उदाहरण के लिए, वासुदेव कृष्ण मथुरा क्षेत्र के महत्त्वपूर्ण देवता थे। कई शताब्दियों के दौरान उनकी पूजा देश के दूसरे प्रदेशों में भी प्रचलित हो गई।

JAC Class 12 History Important Questions Chapter 4 विचारक, विश्वास और इमारतें : सांस्कृतिक विकास

प्रश्न 9.
अतीत की समृद्ध दृश्य परम्पराओं को समझने के लिए यूरोपीय विद्वानों द्वारा किये गये प्रयासों की विवेचना कीजिए।
उत्तर:
अतीत की समृद्ध दृश्य-परम्पराओं को समझने के प्रयास अतीत की समृद्ध दृश्य पराम्पराएँ ईंट और पत्थर से निर्मित स्थापत्य कला, मूर्तिकला और चित्रकला के रूप में हमारे सामने आई हैं। इनमें से कुछ कलाकृतियाँ नष्ट हो गई हैं। परन्तु जो बची हैं, और संरक्षित हैं वे हमें इन सुन्दर कलाकृतियों के निर्माताओं – कलाकारों, मूर्तिकारों, राजगीरों और वास्तुकारों के दृष्टिकोण से परिचित कराती हैं। परन्तु उनके दृष्टिकोण को समझना सरल नहीं है। हम कभी भी यह बात पूर्ण रूप से नहीं समझ सकते कि इन प्रतिकृतियों को देखने और पूजने वाले लोगों के लिए इनका क्या महत्त्व था।

यूरोपीय विद्वानों के प्रयास – उन्नीसवीं शताब्दी में यूरोपीय विद्वानों ने जब देवी-देवताओं की मूर्तियाँ देखीं तो वे उनकी पृष्ठभूमि और महत्त्व को नहीं समझ पाए। कई सिरों, हाथों वाली या मनुष्य और जानवर के रूपों को मिलाकर बनाई गई मूर्तियाँ उन्हें खराब लगती थीं और कई बार उन्हें इन मूर्तियों से घृणा होने लगती थी। प्राचीन यूनान की कला परम्परा से तुलना करना- इन प्रारम्भिक यूरोपीय विद्वानों ने ऐसी विचित्र मूर्तियों का अभिप्राय समझने के लिए उनकी तुलना प्राचीन यूनान की कला परम्परा से की। ये विद्वान् यूनानी परम्परा से परिचित थे।

यद्यपि वे प्रारम्भिक भारतीय मूर्तिकला को यूनान की कला से निम्न स्तर का मानते थे, फिर भी वे बुद्ध और बोधिसत्त की मूर्तियों की खोज से काफी प्रोत्साहित हुए। इसका कारण यह था कि ये मूर्तियाँ यूनानी आदर्शों से प्रभावित थीं। ये मूर्तियाँ अधिकतर उत्तर-पश्चिम के नगरों तक्षशिला और पेशावर में मिली थीं। इन प्रदेशों में ईसा से दो सौ वर्ष पहले भारतीय यूनानी शासकों ने अपने राज्य स्थापित किए थे। ये मूर्तियाँ यूनानी मूर्तियों से काफी मिलती- जुलती थीं। चूंकि ये विद्वान यूनानी परम्परा से परिचित थे, इसलिए उन्होंने इन्हें भारतीय मूर्तिकला का सर्वश्रेष्ठ नमूना माना।

कलाकृतियों के महत्त्व को समझने के लिए लिखित ग्रन्थों से जानकारी प्राप्त करना मूर्तियों के महत्त्व और संदर्भ को समझने के लिए कला के इतिहासकार प्राय: लिखित ग्रन्थों से जानकारी प्राप्त करते हैं भारतीय मूर्तियों की यूनानी मूर्तियों से तुलना कर निष्कर्ष निकालने की अपेक्षा यह तरीका अधिक अच्छा है। परन्तु यह बहुत सरल तरीका नहीं है।

कला – इतिहासकारों व पुराणों में इस कथा को पहचानने के लिए काफी खोजबीन की है। परन्तु उनमें काफी मतभेद हैं। कुछ इतिहासकारों के अनुसार इस कथा में गंगा नदी के स्वर्ग से अवतरण का चित्रण है। उनका कहना है कि चट्टान की सतह के बीच प्राकृतिक दरार शायद नदी को दर्शा रही है। यह कथा महाकाव्यों और पुराणों में वर्णित है। परन्तु अन्य विद्वानों की मान्यता है कि यहाँ पर दिव्यास्य प्राप्त करने के लिए महाभारत में वर्णित अर्जुन की तपस्या का चित्रण है। उनका कहना है कि मूर्तियों के बीच एक साधु की मूर्ति केन्द्र में रखी गई है।

प्रश्न 10.
लौकिक सुखों से आगे, वर्धमान महावीर की शिक्षाओं का विस्तार से वर्णन कीजिए।
उत्तर:
वर्धमान महावीर की शिक्षाएँ –
(1) सारा संसार संजीव है-सम्पूर्ण विश्व प्राणवान है। यह जैन धर्म की सबसे प्रमुख अवधारणा है। वर्धमान महावीर के अनुसार सम्पूर्ण विश्व में कुछ भी निर्जीव नहीं है। यहाँ तक कि पत्थर, चट्टान, जल में भी जीवन होता है। यह भगवान महावीर की गहन अन्तर्दृष्टि थी।

(2) अहिंसा-अहिंसा का सिद्धान्त जैन दर्शन का केन्द्रबिन्दु है। जीवों के प्रति दयाभाव रखना चाहिए, हमें किसी भी जानवर, मनुष्य, पेड़-पौधे, यहाँ तक कि कीट- पतंगों को भी नहीं मारना चाहिए। महावीर के अनुसार आत्मा केवल मनुष्यों में ही नहीं बल्कि पशुओं, कीड़ों, पेड़-पौधों आदि में भी होती हैं।

JAC Class 12 History Important Questions Chapter 4 विचारक, विश्वास और इमारतें : सांस्कृतिक विकास

(3) जीवन चक्र और कर्मवाद जैन दर्शन के अनुसार जन्म और पुनर्जन्म का चक्र कर्मानुसार निर्धारित होता है। ‘कर्मों के अनुरूप ही पुनर्जन्म होता है।

(4) तप आवागमन के बन्धन से छुटकारा पाने का एक ही मार्ग त्याग और तपस्या है। मुक्ति का प्रयास संसार को त्याग कर विहारों में निवास कर तपस्या द्वारा ही फलीभूत होगा।

(5) पंच महाव्रत- जैन धर्म में पंचमहाव्रतों का सिद्धान्त दिया गया है; जैसे कि अहिंसा, अमृषा, अस्तेय, अपरिग्रह, इन्द्रिय निग्रह।

(6) त्रि-रत्न-पंचमहाव्रत के अतिरिक्त वर्धमान महावीर ने निर्वाण प्राप्ति हेतु त्रिरत्न नामक तीन सिद्धान्तों- सत्य, विश्वास, सत्यज्ञान और सत्यकार्य को अपनाने की शिक्षा भी दी है।

(7) ईश्वर की अवधारणा से मुक्ति वर्धमान महावीर ईश्वर के अस्तित्व में बिल्कुल भी विश्वास नहीं करते थे। वह यह नहीं मानते थे कि ईश्वर ने संसार की रचना की है। जैन धर्म आत्मा के अस्तित्व में विश्वास रखता है।

(8) वेदों में विश्वास जैन धर्म को मानने वाले वेदों को ईश्वरी ज्ञान नहीं मानते वे वेदों में मुक्ति हेतु दिए साधनों यह जप, तप, हवन आदि को व्यर्थ समझते हैं।

(9) जाति पाँति में अविश्वास जैन धर्म के अनुयायी जाति पाँति में विश्वास नहीं करते हैं।

(10) स्याद्वाद – जैन धर्म में स्याद्वाद प्रमुख है; कोई भी मनुष्य सम्पूर्ण सत्य के ज्ञान का दावा नहीं कर सकता। सत्यं बहुत व्यापक है, इसके अनेक पक्ष हैं। देश, काल और परिस्थिति के अनुसार मनुष्य को सत्य का आंशिक ज्ञान प्राप्त होता है।

प्रश्न 11.
बौद्ध कैसे की जाती थी? महात्मा धर्म का लेखन और इनकी सुरक्षा
उत्तर:
बुद्ध तथा उनके अनुयायी लोगों में वार्तालाप व वाद-विवाद द्वारा मौखिक रूप से अपनी शिक्षाओं का प्रसार करते थे। महात्मा बुद्ध के जीवन काल में वक्तव्यों का लेखन नहीं किया गया। महात्मा बुद्ध के देह त्याग के उपरान्त पाँचवीं चौथी सदी ईसा पूर्व में उनके शिष्यों ने वैशाली में एक सभा का आयोजन किया। शिक्षाओं का संकलन पुस्तकों के रूप में किया गया; जिन्हें त्रिपिटक, जिसका अर्थ है विभिन्न प्रकार के ग्रन्थों को रखने की तीन टोकरियाँ कहा गया। तदुपरान्त बौद्ध धर्म के विद्वानों द्वारा इन पर टिप्पणियाँ लिखी गयीं।
त्रिपिटक त्रिपिटक में तीन पिटक सम्मिलित हैं –

(1) विनयपिटक विनयपिटक में भिक्षु और भिक्षुणी जो कि बौद्ध मठों या संघों में रहते थे उनके लिए आचार संहिता थी। उन्हें किस प्रकार का आचरण करना चाहिए। इस सम्बन्ध में विनयपिटक में व्यापक नियम दिये गये हैं।

JAC Class 12 History Important Questions Chapter 4 विचारक, विश्वास और इमारतें : सांस्कृतिक विकास

(2) सुत्तपिटक – सुतपिटक में महात्मा बुद्ध की शिक्षाएँ दी गई हैं, जो उन्होंने समाज के प्रत्येक पक्ष को ध्यान में रखते हुए दी हैं।

(3) अभिधम्मपिटक अभिधम्मपिटक में दर्शनशास्त्र से सम्बन्धित विषयों की गहन व्याख्याएँ सम्मिलित हैं।
(i) नए ग्रन्थ धीरे-धीरे बौद्ध धर्म का विस्तार श्रीलंका तक फैल गया तो नए ग्रन्थों जैसे दीपवंश और महावंश नामक ग्रन्थों की रचना की गई। इन ग्रन्थों में क्षेत्र विशेष से सम्बन्धित बौद्ध साहित्य प्राप्त होता है। कई रचनाओं में महात्मा बुद्ध की जीवनी का भी समावेश है।
(ii) ग्रन्थों की सुरक्षा बौद्ध धर्म का प्रचार-प्रसार जब पूर्व एशिया तक फैल गया तो इससे आकर्षित होकर फा-शिएन और श्वेन त्सांग नामक चीनी तीर्थयात्री बौद्ध ग्रन्थों की खोज में चीन से भारत आए।

वे इनमें से कई ग्रन्थों को अपने साथ चीन ले गए और वहाँ इसका अनुवाद चीन की भाषा में किया। भारतीय बौद्ध धर्म के प्रचारक भी देश-विदेश में बौद्ध धर्म की शिक्षाओं का प्रचार-प्रसार करने के लिए कई ग्रन्थों को साथ ले गए। एशिया में फैले विभिन्न बौद्ध विहारों में यह पाण्डुलिपियाँ वर्षों तक संरक्षित रहीं। बौद्ध धर्म के प्रमुख केन्द्र तिब्बत के ल्हासा मठ में बौद्ध धर्म की पालि संस्कृत, चीनी, तिब्बती भाषा की तमाम पाण्डुलिपियाँ आज भी संरक्षित हैं।

JAC Class 12 History Important Questions Chapter 3 बंधुत्व, जाति तथा वर्ग : आरंभिक समाज

Jharkhand Board JAC Class 12 History Important Questions Chapter 3 बंधुत्व, जाति तथा वर्ग : आरंभिक समाज Important Questions and Answers.

JAC Board Class 12 History Important Questions Chapter 3 बंधुत्व, जाति तथा वर्ग : आरंभिक समाज

बहुविकल्पीय प्रश्न (Multiple Choice Questions)

1. कौरव और पांडव किस वंश से सम्बन्धित थे –
(अ) पांचाल
(ब) हर्यंक
(स) लिच्छवि
(द) कुरु
उत्तर:
(द) कुरु

2. वर्तमान महाभारत में श्लोक हैं –
(अ) दस हजार से अधिक
(ब) एक लाख से अधिक
(स) दो लाख से अधिक
(द) पचास हजार से अधिक
उत्तर:
(ब) एक लाख से अधिक

3. ‘धर्म सूत्र’ और ‘धर्मशास्त्र’ विवाह के कितने प्रकारों को अपनी स्वीकृति देते हैं-
(अ) चार
(स) आठ
(ब) दो
(द) सात
उत्तर:
(स) आठ

4. पितृवंशिकता में पिता का महत्त्वपूर्ण धार्मिक कर्तव्य माना गया था –
(अ) कन्या को शिक्षा दिलाना
(ब) कन्यादान
(द) पुत्र का विवाह
(स) पुत्र की देखभाल
उत्तर:
(ब) कन्यादान

5. किस उपनिषद् में कई लोगों को उनके मातृनामों से निर्दिष्ट किया गया था?
(अ) बृहदारण्यक
(स) कठ
(ख) प्रश्न
(द) छान्दोग्य
उत्तर:
(अ) बृहदारण्यक

JAC Class 12 History Important Questions Chapter 3 बंधुत्व, जाति तथा वर्ग : आरंभिक समाज

6. किन शासकों को उनके मातृनाम से चिह्नित किया जाता था?
(अ) कुषाण
(स) मौर्य
(ब) शुंग
(द) सातवाहन
उत्तर:
(द) सातवाहन

7. किस सातवाहन शासक ने स्वयं को अनूठा ब्राह्मण एवं क्षत्रियों के दर्प का हनन करने वाला बताया था?
(अ) शातकर्णी प्रथम
(ब) गौतमीपुत्र शातकर्णी
(स) वसिष्ठीपुत्र शातकर्णी
(द) पुलुमावि शातकर्णी
उत्तर:
(ब) गौतमीपुत्र शातकर्णी

8. एकलव्य किस वर्ग से जुड़ा हुआ था ?
(अ) कृषक
(ब) पशुपालक
(स) व्यवसायी वर्ग
(द) निषाद वर्ग
उत्तर:
(द) निषाद वर्ग

9. महाभारत की मूल भाषा है-
(अ) प्राकृत
(ब) पालि
(स) तमिल
(द) संस्कृत
उत्तर:
(द) संस्कृत

JAC Class 12 History Important Questions Chapter 3 बंधुत्व, जाति तथा वर्ग : आरंभिक समाज

10. प्रसिद्ध इतिहासकार वी.एस. सुकथांकर एक प्रसिद्ध विद्वान् थे-
(अ) संस्कृत के
(ब) मराठी के
(स) हिन्दी के
(द) अंग्रेजी के
उत्तर:
(अ) संस्कृत के

11. महाभारत के समालोचनात्मक संस्करण से सम्बन्धित एक अत्यन्त महत्त्वाकांक्षी परियोजना किस वर्ष आरम्भ हुई?
(अ) 1905 ई. में
(ब) 1911 ई. में
(द) 1942 ई. में
(स) 1919 ई. में
उत्तर:
(स) 1919 ई. में

12. महाभारत एक है-
(अ) खण्डकाव्य
(स) उपन्यास
(ब) महाकाव्य
(द) इनमें से कोई नहीं
उत्तर:
(ब) महाकाव्य

13. धर्मसूत्रों और धर्मशास्त्रों के अनुसार ब्राह्मणों का कार्य था –
(अ) वेदों की शिक्षा देना
(ब) व्यापार करना
(स) न्याय करना
(द) सेवा करना
उत्तर:
(अ) वेदों की शिक्षा देना

14. महाभारत के समालोचनात्मक संस्करण का कार्य किसके नेतृत्व में प्रारम्भ किया गया था?
(अ) रोमिला थापर
(ब) वी.एस. सुकथांकर
(स) उमा चक्रवर्ती
(द) आर.एस. शर्मा
उत्तर:
(ब) वी.एस. सुकथांकर

15. महाभारत के रचयिता किसे माना जाता है?
(अ) विशाखदत्त
(ब) बाणभट्ट
(स) कालिदास
(द) वेदव्यास
उत्तर:
(द) वेदव्यास

JAC Class 12 History Important Questions Chapter 3 बंधुत्व, जाति तथा वर्ग : आरंभिक समाज

रिक्त स्थानों की पूर्ति कीजिए।

1. कुंती ओ निषादी नामक लघु कथा की रचना…….ने की है।
2. …………… महाभारत का उपदेशात्मक अंश है।
3. संस्कृत ग्रन्थों में, …………. शब्द का प्रयोग परिवार के लिए और ………….. का बाँधयों के बड़े समूह के लिए होता है।
4. छठी शताब्दी ई.पू. से राजवंश …………… प्रणाली का अनुसरण करते थे।
5. जहाँ वंश परम्परा माँ से जुड़ी होती है वहाँ ………… शब्द का इस्तेमाल होता है।
6. गोत्र से बाहर विवाह करने को …………… कहते है।
7. ……………. में चाण्डालों के कर्त्तव्यों की सूची मिलती है।
8. मनुस्मृति के अनुसार पुरुषों के लिए धन अर्जित करने के …………… तरीके हैं।
उत्तर:
1. महाश्वेता देवी
2. भगवद्गीता
3. कुल, जाति
4. पितृवंशिकता
5. मातृवंशिकता
6. बहिर्विवाह
7. मनुस्मृति
8. सात

अतिलघूत्तरात्मक प्रश्न

प्रश्न 1.
किस राजवंश के राजाओं को उनके मातृ नाम से चिन्हित किया जाता था?
उत्तर:
सातवाहन राजवंश।

प्रश्न 2.
किस राजवंश में राजाओं के नाम से पूर्व माताओं का नाम लिखा जाता था।
उत्तर:
सातवाहन राजवंश।

प्रश्न 3.
महाभारत का समालोचनात्मक संस्करण तैयार करने का कार्य कब व किसके नेतृत्व में प्रारम्भ हुआ?
उत्तर:
(1) 1919 ई. में
(2) बी. एस. सुकथांकर के नेतृत्व में

प्रश्न 4.
बहिर्विवाह पद्धति किसे कहते हैं? यह अन्तर्विवाह पद्धति से किस प्रकार भिन्न है?
उत्तर:
गोत्र से बाहर विवाह करने को बहिर्विवाह पद्धति कहते हैं। परन्तु अन्तर्विवाह समूह ( गोत्र, कुल, जाति) के बीच होते हैं।

प्रश्न 5.
किस राज्य में राजाओं के नाम के पूर्व माताओं का नाम लिखा जाता था ?
उत्तर:
सातवाहन राज्य में करना।

प्रश्न 6.
धर्मशास्त्रों के अनुसार क्षत्रियों के किन्हीं दो आदर्श कार्यों का उल्लेख कीजिये।
उत्तर:

  • युद्ध करना
  • लोगों को सुरक्षा प्रदान

प्रश्न 7.
‘कुल’ और ‘जात’ में क्या भिन्नता है?
उत्तर:
कुल परिवार को तथा जात बांधवों के बड़े समूह को कहते हैं।

प्रश्न 8.
महाश्वेता देवी ने किस लघु कथा की रचना
उत्तर:
कुंती ओ निषादी।

JAC Class 12 History Important Questions Chapter 3 बंधुत्व, जाति तथा वर्ग : आरंभिक समाज

प्रश्न 9.
पितृवेशिकता का अर्थ स्पष्ट कीजिये।
उत्तर:
वह वंश परम्परा जो पिता के पुत्र, पौत्र, प्रपौत्र से चलती है।

प्रश्न 10.
कुल और जाति में क्या अन्तर है?
उत्तर:
संस्कृत ग्रन्थों में कुल शब्द का प्रयोग परिवार के लिए तथा जाति का प्रयोग बांधवों के बड़े समूह के लिए होता है।

प्रश्न 11.
‘अन्तर्विवाह’ किसे कहते हैं ?
उत्तर:
अन्तर्विवाह समूह (गोत्र, कुल या जाति) के बीच होते हैं।

प्रश्न 12.
वंश से क्या आशय है?
उत्तर:
पीढ़ी-दर-पीढ़ी किसी भी कुल के पूर्वज सम्मिलित रूप में एक ही वंश के माने जाते हैं।

प्रश्न 13.
महाभारत का युद्ध किन दो दलों के मध्य और क्यों हुआ था?
उत्तर:
महाभारत का युद्ध कौरवों और पाण्डवों के मध्य सत्ता पर नियन्त्रण स्थापित करने हेतु हुआ था।

प्रश्न 14.
ब्राह्मणों के अनुसार वर्ण व्यवस्था की उत्पत्ति एक दैवीय व्यवस्था है। इसे सिद्ध करने के लिए वे किस मन्त्र को ‘उद्धृत करते थे?
उत्तर:
ऋग्वेद के ‘पुरुषसूक्त’ मन्त्र को।

प्रश्न 15.
चाण्डाल कौन थे?
उत्तर:
चाण्डाल शवों की अन्त्येष्टि करते थे, मृत-पशुओं को उठाते थे।

प्रश्न 16.
‘मनुस्मृति’ के अनुसार चाण्डाल के दो कर्तव्य बताइये।
उत्तर:

  • गाँव के बाहर रहना
  • मरे हुए लोगों के वस्त्र पहनना।

प्रश्न 17.
महाभारत किस भाषा में रचित है?
उत्तर:
संस्कृत भाषा में।

JAC Class 12 History Important Questions Chapter 3 बंधुत्व, जाति तथा वर्ग : आरंभिक समाज

प्रश्न 18.
कौरवों और पाण्डवों का सम्बन्ध किस वंश से था?
उत्तर:
कौरवों और पाण्डवों का सम्बन्ध कुरुवंश से

प्रश्न 19.
इतिहास का क्या अर्थ है?
उत्तर:
इतिहास का अर्थ है ऐसा ही था।

प्रश्न 20.
चार वर्णों की उत्पत्ति का उल्लेख किस वैदिक ग्रन्थ में मिलता है?
उत्तर:
ऋग्वेद के ‘पुरुषसूक्त’ में।

प्रश्न 21.
गुरु द्रोणाचार्य ने एकलव्य को धनुर्विद्या की शिक्षा देने से क्यों इनकार कर दिया था?
उत्तर:
क्योंकि एकलव्य निषाद (शिकारी समुदाय)

प्रश्न 22.
आरम्भिक संस्कृत परम्परा में महाभारत को किस श्रेणी में रखा गया है?
उत्तर:
इतिहास की श्रेणी में

प्रश्न 23.
किस सातवाहन शासक ने अपने को ‘अनूठा ब्राह्मण’ तथा क्षत्रियों के दर्प का हनन करने वाला बतलाया था?
उत्तर:
गौतमीपुत्र शातकर्णी।

प्रश्न 24.
दो राजवंशों के नाम लिखिए जिनके राजा ब्राह्मण थे।
उत्तर:
शुंग और कण्व वंश के राजा ब्राह्मण थे।

प्रश्न 25.
भीम ने किस राक्षस कन्या से विवाह किया
उत्तर:
हिडिम्बा से

JAC Class 12 History Important Questions Chapter 3 बंधुत्व, जाति तथा वर्ग : आरंभिक समाज

प्रश्न 26.
धर्मसूत्र व धर्मशास्त्र नामक ग्रन्थ किस भाषा में लिखा गया है?
उत्तर:
धर्मसूत्र व धर्मशास्य नामक ग्रन्थ संस्कृत भाषा में लिखा गया है।

प्रश्न 27.
अधिकतर राजवंश पितृवेशिकता प्रणाली का अनुसरण करते थे या मातृवंशिकता प्रणाली का ?
उत्तर:
पितृवंशिकता प्रणाली का।

प्रश्न 28.
धृतराष्ट्र हस्तिनापुर के सिंहासन पर क्यों आसीन नहीं हुए?
उत्तर:
क्योंकि यह नेत्रहीन थे।

प्रश्न 29.
महाभारत के अनुसार किस माता ने अपने ज्येष्ठ पुत्र दुर्योधन को युद्ध न करने की सलाह दी थी?
उत्तर:
गान्धारी ने।

प्रश्न 30.
सातवाहन राजाओं से विवाह करने वाली रानियों के नाम किन गोत्रों से उद्भूत थे?
उत्तर:
गौतम तथा वशिष्ठ गोत्रों से।

प्रश्न 31.
हस्तिनापुर नामक गाँव में उत्खनन कार्य किसके नेतृत्व में किया गया और कब?
उत्तर:
(1) बी.बी. लाल के नेतृत्व में
(2) 1951

प्रश्न 32.
‘सूत’ कौन थे?
उत्तर:
महाभारत की मूलकथा के रचयिता भाट सारथी ‘सूत’ कहलाते थे।

प्रश्न 33.
महाभारत की विषय-वस्तु को किन दो मुख्य भागों में बाँटा गया है?
उत्तर:
(1) आख्यान तथा
(2) उपदेशात्मक।

JAC Class 12 History Important Questions Chapter 3 बंधुत्व, जाति तथा वर्ग : आरंभिक समाज

प्रश्न 34.
किस क्षेत्र में दानशील व्यक्ति का सम्मान किया जाता था तथा कृपण व्यक्ति को घृणा का पात्र समझा जाता था?
उत्तर:
प्राचीन तमिलकम में।

प्रश्न 35.
उन दो चीनी यात्रियों के नाम लिखिए जिन्होंने लिखा है कि चाण्डालों को नगर से बाहर रहना पड़ता था।
उत्तर:

  • फा-शिक्षन
  • श्वैन-त्सांग।

प्रश्न 36.
मनुस्मृति के अनुसार समाज का कौनसा वर्ग पैतृक संसाधन में हिस्सेदारी की माँग नहीं कर सकता था?
उत्तर:
स्त्री वर्ग।

प्रश्न 37.
प्रारम्भ में महाभारत में कितने श्लोक थे?
उत्तर:
लगभग 10,000

प्रश्न 38.
वर्तमान में महाभारत में कितने श्लोक हैं ?
उत्तर:
लगभग एक लाख श्लोक।

प्रश्न 39.
ब्राह्मणीय व्यवस्था में जाति व्यवस्था किस पर आधारित थी?
उत्तर:
जन्म पर।

प्रश्न 40.
किस शक शासक ने सुदर्शन झील का जीर्णोद्धार करवाया था ?
उत्तर:
रुद्रदामन

प्रश्न 41.
लोगों को गोशों में वर्गीकृत करने की ब्राह्मणीय पद्धति कब प्रचलन में आई ?
उत्तर:
लगभग 1000 ई. पूर्व के बाद से।

प्रश्न 42.
मनुस्मृति के आधार पर पुरुषों के लिए सम्पत्ति अर्जन के कोई चार तरीके बताइये।
उत्तर:

  • विरासत
  • खोज
  • खरीद
  • विजित करके।

प्रश्न 43.
महाभारतकालीन स्वियों की विभिन्न समस्याएँ लिखिए।
उत्तर:

  • पैतृक संसाधनों में स्थियों की हिस्सेदारी न होना
  • बहुपति प्रथा का प्रचलन
  • जुएँ में स्वियों को दाँव पर लगाना।

प्रश्न 44.
वर्ग शब्द से आपका क्या अभिप्राय है?
उत्तर:
वर्ग शब्द एक विशिष्ट सामाजिक श्रेणी का प्रतीक है।

JAC Class 12 History Important Questions Chapter 3 बंधुत्व, जाति तथा वर्ग : आरंभिक समाज

प्रश्न 45.
बहुपत्नी प्रथा एवं बहुपति प्रथा से क्या अभिप्राय है?
उत्तर:
बहुपत्नी प्रथा एक पुरुष की अनेक पत्नियाँ होने की परिपाटी है, बहुपति प्रथा एक स्वी होने की पद्धति है।

प्रश्न 46.
‘धर्मसूत्र’ एवं ‘धर्मशास्त्र’ ग्रन्थों से क्या अभिप्राय है?
उत्तर:
ब्राह्मणों ने समाज के लिए विस्तृत जो आचार संहिताएं तैयार कीं, वे धर्मसूत्र’ एवं ‘धर्मशास्त्र’ कहलाते हैं।

प्रश्न 47.
मनुस्मृति से आप क्या समझते हैं?
उत्तर:
मनुस्मृति प्राचीन भारत का सबसे प्रसिद्ध विधि- ग्रन्थ है। यह धर्मसूत्रों एवं धर्मशास्त्रों में सबसे महत्त्वपूर्ण है।

प्रश्न 48.
धर्मसूत्रों एवं धर्मशास्त्रों में विवाह के कितने प्रकारों को स्वीकृति दी गई है? इनमें कौन से विवाह ‘उत्तम’ तथा कौनसे विवाह ‘निन्दित’ माने गए हैं?
उत्तर:

  • विवाह के आठ प्रकारों को स्वीकृति दी गई है।
  • पहले चार विवाह ‘उत्तम’ तथा शेष को ‘निन्दित’ माना गया है।

प्रश्न 49.
बहिर्विवाह पद्धति की कोई दो विशेषताएँ बताइये।
उत्तर:

  • अपने गोत्र से बाहर विवाह करने को बहिर्विवाह कहते हैं।
  • इसमें ऊँची प्रतिष्ठा वाले परिवारों की कम आयु की कन्याओं और स्वियों का जीवन सावधानी से नियन्त्रित किया जाता था।

प्रश्न 50.
गोत्रों का प्रचलन किस प्रकार हुआ ?
उत्तर:
प्रत्येक गोत्र एक वैदिक ऋषि के नाम पर होता था। उस गोत्र के सदस्य ऋषि के वंशज माने जाते थे।

प्रश्न 51.
गोत्र प्रणाली के दो नियमों का उल्लेख कीजिए।
अथवा
गोत्रों के दो महत्त्वपूर्ण नियमों का उल्लेख कीजिए।
उत्तर:

  • विवाह के पश्चात् स्वियों को पति के गोत्र का माना जाता था।
  • एक ही गोत्र के सदस्यों में विवाह वर्जित था।

प्रश्न 52.
‘वर्ण’ और ‘जाति’ के कोई दो अन्तर बताइये।
उत्तर:

  • वर्ण मात्र चार थे, परन्तु जातियों की कोई निश्चित संख्या नहीं थी।
  • वर्ण व्यवस्था में अन्तर्विवाह आवश्यक नहीं है, परन्तु जाति व्यवस्था में अन्तर्विवाह अनिवार्य होता है।

प्रश्न 53.
सातवाहन शासकों का देश के किन भागों पर शासन था ?
उत्तर:
सातवाहन शासकों का पश्चिमी भारत तथा दक्यान के कुछ भागों पर शासन था।

JAC Class 12 History Important Questions Chapter 3 बंधुत्व, जाति तथा वर्ग : आरंभिक समाज

प्रश्न 54.
सातवाहनों में अन्तर्विवाह पद्धति प्रचलित थी। इसका उदाहरण दीजिए।
उत्तर:
कुछ सातवाहन रानियाँ एक ही गोत्र से थीं। यह उदाहरण सातवाहनों में अन्तर्विवाह पद्धति को दर्शाता है।

प्रश्न 55.
कुछ रानियों ने सातवाहन राजाओं से विवाह करने के बाद भी अपने पिता के गोत्र नाम को ही कायम रखा था। उदाहरण देकर स्पष्ट कीजिए।
उत्तर:
सातवाहन राजाओं से विवाह करने वाली रानियों के नाम गौतम तथा वसिष्ठ गोत्रों से उद्भूत थे, जो उनके पिता के गोत्र थे।

प्रश्न 56.
सातवाहन काल में माताएँ क्यों महत्त्वपूर्ण थीं?
उत्तर:
सातवाहन राजाओं को उनके मातृनाम ( माता के नाम से उद्भूत) से चिह्नित किया जाता था।

प्रश्न 57.
ब्राह्मणीय सामाजिक व्यवस्था में पहला दर्जा तथा सबसे निम्न दर्जा किसे प्राप्त था ?
उत्तर:
इस सामाजिक व्यवस्था में ब्राह्मणों को पहला दर्जा तथा शूद्रों को सबसे निम्न दर्जा प्राप्त था।

प्रश्न 58.
ऋग्वेद के पुरुष सूक्त’ के अनुसार चार वर्णों की उत्पत्ति कैसे हुई ?
उत्तर:
ब्राह्मण की उत्पत्ति परमात्मा के मुख से क्षत्रिय की भुजाओं से वैश्य की जंघा से शूद्र की पैरों से हुई।

प्रश्न 59.
‘स्त्री धन’ को परिभाषित कीजिए।
उत्तर:
विवाह के समय मिले उपहारों पर स्वियों का स्वामित्व माना जाता था और इसे ‘स्वी धन’ कहा जाता था।

प्रश्न 60.
मनुस्मृति के अनुसार पुरुषों के लिए धन अर्जित करने के कौनसे तरीके हैं?
उत्तर:

  • विरासत
  • खोज
  • खरीद
  • विजित करके
  • निवेश
  • कार्य द्वारा तथा
  • सज्जनों द्वारा दी गई भेंट को स्वीकार करके।

प्रश्न 61.
ब्राह्मणीय ग्रन्थों के अनुसार सम्पत्ति पर अधिकार के कौनसे आधार थे?
उत्तर:
ब्राह्मणीय ग्रन्थों के अनुसार सम्पत्ति पर अधिकार के दो आधार थे –

  • लिंग का आधार
  • वर्ण का आधार।

प्रश्न 62.
महाकाव्य काल में सबसे धनी व्यक्ति किन वर्णों के लोग होते थे?
उत्तर:
महाकाव्य काल में ब्राह्मण और क्षत्रिय सबसे धनी व्यक्ति होते थे।

JAC Class 12 History Important Questions Chapter 3 बंधुत्व, जाति तथा वर्ग : आरंभिक समाज

प्रश्न 63.
तमिलकम के सरदारों की दानशीलता का एक उदाहरण दीजिए।
उत्तर:
तमिलकम के सरदार अपनी प्रशंसा गाने वाले चारणों एवं कवियों के आश्रयदाता थे। वे उन्हें उदारतापूर्वक दान दिया करते थे।

प्रश्न 64.
महाभारत को व्यापक स्तर पर लोगों द्वारा क्यों समझा जाता था ?
उत्तर:
क्योंकि महाभारत में प्रयुक्त संस्कृत वेदों अथवा प्रशस्तियों की संस्कृत से कहीं अधिक सरल है।

प्रश्न 65.
महाभारत की विषयवस्तु को किन दो मुख्य शीर्षकों में रखा जाता है?
उत्तर:

  • आख्यान – इसमें कहानियों का संग्रह है।
  • उपदेशात्मक – इसमें सामाजिक आचार-विचार के मानदंडों का चित्रण है।

प्रश्न 66.
‘भगवद्गीता’ से आप क्या समझते हैं?
उत्तर:
‘भगवद्गीता’ महाभारत का सबसे महत्त्वपूर्ण उपदेशात्मक अंश है। इसमें कुरुक्षेत्र के युद्ध में श्रीकृष्ण अर्जुन को उपदेश देते हैं।

JAC Class 12 History Important Questions Chapter 3 बंधुत्व, जाति तथा वर्ग : आरंभिक समाज

प्रश्न 67.
सातवाहन शासकों की शासनावधि क्या थी?
उत्तर:
सातवाहन शासकों ने लगभग दूसरी शताब्दी ई. पूर्व से दूसरी शताब्दी ईसवी तक शासन किया।

प्रश्न 68.
मनुस्मृति का संकलन (रचना) कब किया गया?
उत्तर:
मनुस्मृति का संकलन (रचना) लगभग 200 ईसा पूर्व से 200 ईसवी के बीच हुआ।

प्रश्न 69.
पितृवंशिकता के आदर्श को किसने सुदृढ़ किया?
उत्तर:
महाभारत ने पितृवंशिकता के आदर्श को सुदृद किया।

प्रश्न 70.
अधिकांश राजवंश किस प्रणाली का अनुसरण करते थे?
उत्तर:
अधिकांश राजवंश पितृवशिकता प्रणाली का अनुसरण करते थे।

प्रश्न 71.
सातवाहन काल में माताएँ क्यों महत्वपूर्ण थीं?
उत्तर:
सातवाहन काल में राजाओं को उनके मातृनाम से जाना जाता था।

प्रश्न 72.
‘आदर्श जीविका’ से सम्बन्धित नियमों का उल्लेख किन ग्रन्थों में मिलता है?
उत्तर:
धर्मसूत्रों और धर्मशास्त्रों में चारों वर्णों के लिए ‘आदर्श जीविका’ से सम्बन्धित नियमों का उल्लेख मिलता है।

प्रश्न 73.
ब्राह्मणों द्वारा ‘आदर्श जीविका’ से सम्बन्धित नियमों का पालन करवाने के लिए अपनाई गई दो नीतियों का उल्लेख कीजिए।
उत्तर:

  • ब्राह्मणों ने वर्ण व्यवस्था की उत्पत्ति को एक दैवीय व्यवस्था बतलाया।
  • वे शासकों को इन नियमों का अनुसरण करने का उपदेश देते थे।

प्रश्न 74.
अर्जुन ने गुरु द्रोणाचार्य को कौनसा प्रण याद दिलाया ?
उत्तर:
द्रोणाचार्य ने अर्जुन को वचन दिया था कि वह उनके सभी शिष्यों में अद्वितीय तीरन्दाज बनेगा।

प्रश्न 75.
किस तथ्य से यह ज्ञात होता है कि शक्तिशाली मलेच्छ राजा भी संस्कृतीय परिपाटी से परिचित थे?
उत्तर:
प्रसिद्ध शक राजा रुद्रदामन ने सुदर्शन झील का पुनरुद्धार करवाया था।

JAC Class 12 History Important Questions Chapter 3 बंधुत्व, जाति तथा वर्ग : आरंभिक समाज

प्रश्न 76.
ब्राह्मणों द्वारा स्थापित सामाजिक व्यवस्था में पहला दर्जा एवं सबसे निम्न दर्जा किसे प्राप्त था ?
उत्तर:
पहला दर्जा ब्राह्मणों को तथा निम्न दर्जा शूद्रों को प्राप्त था।

प्रश्न 77.
किस अभिलेख में रेशम के बुनकरों की एक श्रेणी का उल्लेख मिलता है? यह अभिलेख कब का है ?
उत्तर:

  • मन्दसौर (मध्यप्रदेश) से प्राप्त अभिलेख में।
  • यह अभिलेख लगभग पाँचवीं शताब्दी ईसवी का है।

प्रश्न 78.
ब्राह्मणीय व्यवस्था के अन्तर्गत किन नये समुदायों का जाति में वर्गीकरण कर दिया गया ?
उत्तर:
ब्राह्मणीय व्यवस्था के अन्तर्गत निषाद (शिकारी समुदाय) तथा सुवर्णकार जैसे व्यावसायिक वर्ग का जातियों में वर्गीकरण कर दिया गया।

प्रश्न 79.
मनुस्मृति के अनुसार स्त्रियों के लिए धन अर्जित करने के कोई दो तरीके बताइए।
उत्तर:

  • स्नेह के प्रतीक के रूप में।
  • माता-पिता द्वारा दिए गए उपहार के रूप में।

प्रश्न 80.
महाकाव्य काल में सबसे धनी व्यक्ति किस वर्ग के लोग होते थे?
उत्तर:
ब्राह्मण और क्षत्रिय वर्ग के लोग सबसे धनी होते थे।

प्रश्न 81.
महाभारत में दुर्योधन के माता-पिता का क्या नाम था?
उत्तर:
धृतराष्ट्र तथा गान्धारी।

प्रश्न 82.
मज्झिमनिकाय किस भाषा में लिखा गया है?
उत्तर:
मज्झिमनिकाय पालि भाषा में लिखा गया बौद्ध ग्रन्थ है।

JAC Class 12 History Important Questions Chapter 3 बंधुत्व, जाति तथा वर्ग : आरंभिक समाज

प्रश्न 83.
बौद्ध भिक्षु फा शिएन चीन से भारत कब आया था?
उत्तर:
लगभग पाँचवीं शताब्दी ईसवी

प्रश्न 84.
चीनी तीर्थयात्री श्वेन त्सांग भारत कब आया
उत्तर:
लगभग सातवीं शताब्दी ईसवी।

लघुत्तरात्मक प्रश्न

प्रश्न 1.
600 ई. पूर्व से 600 ई. तक के मध्य आर्थिक और राजनीतिक जीवन में हुए परिवर्तनों के समकालीन समाज पर क्या प्रभाव हुए?
उत्तर:
600 ई. पूर्व से 600 ई. तक के मध्य आर्थिक और राजनीतिक जीवन में हुए परिवर्तनों के समकालीन समाज पर निम्नलिखित प्रभाव हुए-

  • वन क्षेत्रों में कृषि का विस्तार हुआ, जिससे वहाँ रहने वाले लोगों की जीवन शैली में परिवर्तन हुआ।
  • शिल्प-विशेषज्ञों के एक विशिष्ट सामाजिक समूह का उदय हुआ।
  • सम्पत्ति के असंगत वितरण से सामाजिक विषमताओं में वृद्धि हुई।

प्रश्न 2.
आरम्भिक समाज में प्रचलित आचार- व्यवहार और रिवाजों का इतिहास लिखने के लिए किन बातों का ध्यान रखना चाहिए?
उत्तर:
(1) प्रत्येक ग्रन्थ किसी समुदाय विशेष के दृष्टिकोण से लिखा जाता था अतः यह याद रखना आवश्यक. था कि ये ग्रन्थ किसने लिखे इन ग्रन्थों में क्या लिखा गया, किनके लिए इन ग्रन्थों की रचना हुई।
(2) यह बात भी ध्यान में रखनी चाहिए कि इन ग्रन्थों की रचना में किस भाषा का प्रयोग हुआ तथा इनका प्रचार- प्रसार किस प्रकार हुआ। इन ग्रन्थों का सावधानीपूर्वक प्रयोग करने से आचार व्यवहार और रिवाजों का इतिहास लिखा जा सकता है।

प्रश्न 3.
परिवार के बारे में आप क्या जानते हैं? क्या सभी परिवार एक जैसे होते हैं?
उत्तर:
संस्कृत ग्रन्थों में ‘कुल’ शब्द का प्रयोग परिवार के लिए होता है। पीढ़ी-दर-पीढ़ी किसी भी कुल के पूर्वज इकट्ठे रूप में एक ही वंश के माने जाते हैं परन्तु सभी परिवार एक जैसे नहीं होते। पारिवारिक जनों की गिनती, एक-दूसरे से उनका रिश्ता और उनके क्रियाकलापों में भी भिन्नता होती है। कई बार एक ही परिवार के लोग भोजन और अन्य संसाधनों का आपस में
करते हैं। एक साथ रहते और काम मिल बाँट कर प्रयोग करते हैं।

प्रश्न 4.
परिवार पर एक संक्षिप्त टिप्पणी लिखिए।
उत्तर:
परिवार एक बड़े समूह का हिस्सा होता है जिन्हें हम सम्बन्धी कहते हैं। तकनीकी भाषा में हम सम्बन्धियों को जातिसमूह कह सकते हैं। पारिवारिक रिश्ते ‘नैसर्गिक’ और ‘रक्त सम्बद्ध’ माने जाते हैं। कुछ समाजों में भाई-बहिन (चचेरे मौसेरे से रक्त का रिश्ता माना जाता है परन्तु अन्य समाज ऐसा नहीं मानते।

JAC Class 12 History Important Questions Chapter 3 बंधुत्व, जाति तथा वर्ग : आरंभिक समाज

प्रश्न 5.
नये नगरों के उद्भव से आई सामाजिक जटिलता की चुनौती का किस प्रकार समाधान किया हैं?
उत्तर:
नये नगरों में रहने वाले लोग एक-दूसरे से विचार-विमर्श करने लगे और आरम्भिक विश्वासों तथा व्यवहारों पर प्रश्न चिह्न लगाने लगे। इस चुनौती के उत्तर में ब्राह्मणों ने समाज के लिए विस्तृत आचारसंहिताएं तैयार कीं। ब्राह्मणों तथा अन्य लोगों को इनका अनुसरण करना पड़ता था। लगभग 500 ई. पूर्व से इन मानदण्डों का संकलन धर्मसूत्रों एवं धर्मशास्त्रों में किया गया। इनमें मनुस्मृति सर्वाधिक महत्त्वपूर्ण थी।

प्रश्न 6.
धर्मसूत्रों और धर्मशास्त्रों के ब्राह्मण लेखकों का मानना था कि उनका दृष्टिकोण सार्वभौमिक है और उनके बनाए नियमों का सब लोगों के द्वारा पालन होना चाहिए। क्या यह सम्भव था?
उत्तर:
यह सम्भव नहीं था क्योंकि उपमहाद्वीप में फैली क्षेत्रीय विभिन्नता तथा संचार की बाधाओं के कारण से ब्राह्मणों का प्रभाव सार्वभौमिक नहीं हो सकता था। धर्मसूत्रों और धर्मशास्त्रों ने विवाह की आठ प्रणालियों को स्वीकृति दी है। इनमें से पहले चार विवाह उत्तम तथा शेष विवाह निंदित माने गए। सम्भव है कि ये विवाह प्रणालियाँ उन लोगों में प्रचलित थीं, जो ब्राह्मणीय नियमों को अस्वीकार करते थे।

प्रश्न 7.
पितृवंशिकता के आदर्श को स्पष्ट कीजिए। महाभारत ने इस आदर्श को कैसे सुदृढ़ किया?
उत्तर:
पितृवशकता का अर्थ है वह वंश परम्परा जो फिर पौत्र, प्रपौत्र आदि से चलती है। महाभारत पिता के पुत्र, में बान्धवों के दो दलों – कौरवों तथा पांडवों के बीच भूमि और सत्ता को लेकर हुए युद्ध का वर्णन है, जिसमें पांडव विजयी हुए। इसके बाद पितृवंशिक उत्तराधिकार की उद्घोषणा की गई। महाभारत की मुख्य कथावस्तु ने पितृवंशिकता के आदर्श को और सुदृढ़ किया।

प्रश्न 8.
पितृवंशिकता प्रणाली में पाई जाने वाली विभिन्नता का उल्लेख कीजिए।
उत्तर:
लगभग छठी शताब्दी ई. पूर्व से अधिकतर राजवंश पितृवंशिकता प्रणाली का अनुसरण करते थे, परन्तु इस प्रथा में निम्नलिखित भिन्नताएँ थीं-

  • कभी-कभी पुत्र के न होने पर एक भाई दूसरे का उत्तराधिकारी हो जाता था।
  • कभी-कभी बन्धु बान्धव सिंहासन पर अपना अधिकार जमा लेते थे।
  • कुछ विशिष्ट परिस्थितियों में स्विर्ण जैसे प्रभावती गुप्त सिंहासन पर अधिकार कर लेती थीं।

प्रश्न 9.
ब्राह्मणीय पद्धति में लोगों को गोत्रों में किस प्रकार वर्गीकृत किया गया? क्या इन नियमों का सामान्यतः अनुसरण होता था?
उत्तर:
लगभग 1000 ई. पूर्व के बाद से प्रचलन में आई एक ब्राह्मणीय पद्धति ने लोगों को विशेष रूप से ब्राह्मणों को गोत्रों में वर्गीकृत किया। प्रत्येक गोत्र एक वैदिक ऋषि के नाम पर होता था। उस गोत्र के सदस्य ऋषि के वंशज माने जाते थे। गोत्रों के दो नियम महत्त्वपूर्ण थे –

  • विवाह के पश्चात् स्त्रियों को पिता के स्थान पर पति के गोत्र का माना जाता था तथा
  • एक ही गोत्र के सदस्य आपस में विवाह सम्बन्ध नहीं रख सकते थे।

प्रश्न 10.
क्या ब्राह्मणीय पद्धति के गोत्र सम्बन्धी नियमों का सम्पूर्ण भारत में पालन किया जाता था?
उत्तर:
इन नियमों का सम्पूर्ण उपमहाद्वीप में पालन नहीं होता था कुछ सातवाहन राजाओं की एक से अधिक पलियाँ थीं। सातवाहन राजाओं से विवाह करने वाली रानियों के नाम गौतम तथा वसिष्ठ गोत्रों से लिए गए थे जो उनके पिता के गोत्र थे। इससे ज्ञात होता है कि विवाह के बाद भी अपने पति कुल के गोत्र को ग्रहण करने की अपेक्षा, उन्होंने पिता का गोत्र नाम ही बनाए रखा। यह ब्राह्मणीय व्यवस्था के विपरीत था। कुछ सातवाहन रानियाँ एक ही गोत्र से थीं यह बात बहिर्विवाह पद्धति के विरुद्ध थी।

JAC Class 12 History Important Questions Chapter 3 बंधुत्व, जाति तथा वर्ग : आरंभिक समाज

प्रश्न 11.
अन्तर्विवाह, बहिर्विवाह, बहुपत्नी प्रथा तथा बहुपति प्रथा को स्पष्ट कीजिए।
उत्तर:

  • अन्तर्विवाह अन्तर्विवाह में वैवाहिक सम्बन्ध समूह के बीच ही होते हैं यह समूह एक गोत्र कुल अथवा एक जाति या फिर एक ही स्थान पर बसने वालों का हो सकता है।
  • बहिर्विवाह अहिर्विवाह गोत्र से बाहर विवाह करने को कहते हैं।
  • बहुपत्नी प्रथा बहुपत्नी प्रथा एक पुरुष की अनेक पत्नियाँ होने की सामाजिक परिपाटी है।
  • बहुपति प्रथा बहुपति प्रथा एक स्त्री के अनेक पति होने की पद्धति है।

प्रश्न 12.
विभिन्न वर्णों के लिए धर्मसूत्रों और धर्मशास्त्रों में वर्णित आदर्श जीविका लिखिए। क्या वह जीविका आज भी विद्यमान है?
अथवा
सूत्रों तथा धर्मशास्त्रों में चारों वर्गों के लिए ‘आदर्श जीविका’ से जुड़े कई नियम मिलते हैं? विवेचना कीजिए।
उत्तर:

  • ब्राह्मणों का कार्य अध्ययन, वेदों की शिक्षा, यज्ञ करना और करवाना, दान देना तथा दान लेना था।
  • क्षत्रियों का कार्य युद्ध करना, लोगों को सुरक्षा प्रदान करना, न्याय करना, वेदों का अध्ययन करना, यज्ञ करवाना तथा दान-दक्षिणा देना था।
  • वैश्यों का कार्य कृषि, गोपालन तथा व्यापार का कार्य करना, वेद पढ़ना, यज्ञ करवाना तथा दान-दक्षिणा देना था।
  • शूद्रों का कार्य तीनों वर्णों की सेवा करना था। यह जीविका आज विद्यमान नहीं है।

प्रश्न 13.
ब्राह्मणों ने धर्मसूत्रों और धर्मशास्त्रों में वर्णित आदर्श जीविका सम्बन्धी व्यवस्था को बनाये रखने के लिए क्या उपाय किये?
उत्तर:
ब्राह्मणों ने इस व्यवस्था को लागू करने के लिए निम्नलिखित नीतियाँ अपनाई –

  • ब्राह्मणों ने लोगों को बताया कि वर्ण व्यवस्था की उत्पत्ति एक दैवीय व्यवस्था है।
  • वे शासकों को यह उपदेश देते थे कि वे इस व्यवस्था के नियमों का अपने राज्यों में पालन करें।
  • उन्होंने लोगों को यह विश्वास दिलाया कि उनको प्रतिष्ठा जन्म पर आधारित है।

प्रश्न 14.
महाभारत में गान्धारी ने अपने ज्येष्ठ पुत्र दुर्योधन को पाण्डवों के विरुद्ध युद्ध न करने की क्या सलाह दी थी? क्या दुर्योधन ने अपनी माता की सलाह मानी थी?
उत्तर:
गान्धारी ने दुर्योधन को युद्ध न करने की सलाह देते हुए कहा कि युद्ध में कुछ भी शुभ नहीं होता, न धर्म और अर्थ की प्राप्ति होती है और न ही प्रसन्नता की यह भी आवश्यक नहीं कि युद्ध के अन्त में तुम्हें सफलता मिले। अतः मैं तुम्हें सलाह देती हूँ कि तुम बुद्ध करने का विचार त्याग दो। परन्तु दुर्योधन ने अपनी माता गान्धारी की सलाह नहीं मानी।

प्रश्न 15.
ब्राह्मणों द्वारा वर्ण व्यवस्था को दैवीय व्यवस्था क्यों बताया गया ?
उत्तर:
ब्राह्मणों की मान्यता थी कि वर्ण-व्यवस्था में उन्हें पहला दर्जा प्राप्त है। अतः समाज में अपनी सर्वोच्च स्थिति को बनाये रखने के लिए उन्होंने वर्ण व्यवस्था को एक दैवीय व्यवस्था बताया। इसे प्रमाणित करने के लिए वे ऋग्वेद के ‘पुरुषसूक्त’ मन्त्र को उद्धृत करते हैं। पुरुषसूक्त के अनुसार ब्राह्मण ब्रह्मा के मुख से क्षत्रिय भुजाओं से वैश्य जंघाओं से तथा शूद्र चरणों से उत्पन्न हुए।

JAC Class 12 History Important Questions Chapter 3 बंधुत्व, जाति तथा वर्ग : आरंभिक समाज

प्रश्न 16.
क्या सभी राजवंशों की उत्पत्ति क्षत्रिय वर्ण से ही हुई थी?
उत्तर:
शास्त्रों के अनुसार केवल क्षत्रिय ही राजा हो सकते थे परन्तु अनेक महत्वपूर्ण राजवंशों की उत्पत्ति अन्य वर्णों से भी हुई थी। यद्यपि बौद्ध ग्रन्थ मौर्य सम्राटों को क्षत्रिय बताते हैं, परन्तु ब्राह्मणीय शास्त्र उन्हें ‘निम्न’ कुल का मानते हैं मौचों के उत्तराधिकारी शुंग और कण्व ब्राह्मण थे सातवाहन वंश के सबसे प्रसिद्ध शासक गौतमीपुत्र ‘शातकर्णी ने स्वयं को अनूठा ब्राह्मण और क्षत्रियों के दर्प को कुचलने वाला बताया है।

प्रश्न 17.
“जाति प्रथा के भीतर आत्मसात् होना एक जटिल सामाजिक प्रक्रिया थी।” स्पष्ट कीजिये।
उत्तर:
सातवाहन वंश के शासक स्वयं को ब्राह्मण वर्ग का बताते थे, जबकि ब्राह्मणीय शास्त्र के अनुसार राजा को क्षत्रिय होना चाहिए। यद्यपि सातवाहन चतुर्वर्णी व्यवस्था की मर्यादा को बनाए रखना चाहते थे, परन्तु साथ ही वे उन लोगों से वैवाहिक सम्बन्ध भी स्थापित करते थे जो इस वर्ण व्यवस्था से ही बाहर थे। इस प्रकार वे अन्तर्विवाह प्रणाली का पालन करते थे, न कि बहिर्विवाह प्रणाली का जो ब्राह्मणीय ग्रन्थों में प्रस्तावित है।

प्रश्न 18.
ब्राह्मणीय व्यवस्था में जाति पर आधारित सामाजिक वर्गीकरण किस प्रकार किया गया?
उत्तर:
ब्राह्मणीय सिद्धान्त में वर्ण की भाँति जाति भी जन्म पर आधारित थी। परन्तु वर्ण जहाँ केवल चार थे, वहीं जातियों की कोई निश्चित संख्या नहीं थी। वास्तव में जिन नये समुदायों का जैसे निषादों, स्वर्णकारों आदि को चार वर्णों वाली ब्राह्मणीय व्यवस्था में शामिल करना सम्भव नहीं था, उनका जाति में वर्गीकरण कर दिया गया। कुछ जातियाँ एक ही जीविका अथवा व्यवसाय से सम्बन्धित थीं। उन्हें कभी-कभी श्रेणियों में भी संगठित किया जाता था।

प्रश्न 19.
मन्दसौर के अभिलेख से जटिल सामाजिक प्रक्रियाओं की जानकारी मिलती है। स्पष्ट कीजिये।
उत्तर:
मन्दसौर के अभिलेख से ज्ञात होता है कि रेशम के बुनकरों की एक अलग श्रेणी थी। इस अभिलेख से तत्कालीन जटिल सामाजिक प्रक्रियाओं पर प्रकाश पड़ता है। इस अभिलेख से यह भी पता चलता है कि श्रेणी के सदस्य एक व्यवसाय के अतिरिक्त और कई चीजों में भी सहभागी होते थे। सामूहिक रूप से उन्होंने अपने शिल्पकर्म से अर्जित धन को सूर्य मन्दिर के निर्माण में खर्च किया।

प्रश्न 20.
मनुस्मृति में उल्लिखित विवाह के आठ प्रकारों में से किन्हीं चार प्रकारों का वर्णन कीजिए।
उत्तर:
मनुस्मृति में उल्लिखित विवाह के चार प्रकार –

  • इसमें पिता द्वारा कन्या का दान वेदज्ञ वर को दिया जाता था।
  • इसके अन्तर्गत पिता वर का सम्मान कर उसे कन्या का दान करता था।
  • इसके अन्तर्गत वर कन्या के बान्धवों को यथेष्ट धन प्रदान करता था।
  • इस पद्धति में लड़का तथा लड़की काम से उत्पन्न हुई अपनी इच्छा से प्रेम विवाह कर लेते थे।

प्रश्न 21.
ब्राह्मणीय व्यवस्था में चार वर्णों के अतिरिक्त अन्य समुदायों की क्या स्थिति थी?
उत्तर:
ब्राह्मण ग्रन्थों ने जिस ब्राह्मणीय व्यवस्था को स्थापित किया था उसका सम्पूर्ण उपमहाद्वीप में पालन नहीं किया जाता था। उस महाद्वीप में पाई जाने वाली विविधताओं के कारण यहाँ सदा से ऐसे समुदाय रहे हैं जिन पर ब्राह्मणीय व्यवस्था का कोई प्रभाव नहीं पड़ा है। निषाद वर्ग इसी का उदाहरण हैं। ब्राह्मणीय व्यवस्था में यायावर पशुपालकों के समुदाय को भी सन्देह की दृष्टि से देखा जाता था क्योंकि उन्हें स्थायी कृषकों के समुदाय में आसानी से समाहित नहीं किया जा सकता था।

JAC Class 12 History Important Questions Chapter 3 बंधुत्व, जाति तथा वर्ग : आरंभिक समाज

प्रश्न 22.
ब्राह्मणीय व्यवस्था के अन्तर्गत कुछ वर्गों को ‘अस्पृश्य’ किन आधारों पर माना जाता था?
उत्तर:
ब्राह्मण समाज के कुछ वर्गों को ‘अस्पृश्य’ मानते थे। उनके अनुसार अनुष्ठानों के सम्पादन से जुड़े कुछ कार्य पवित्र थे। अपने को पवित्र मानने वाले लोग ‘अस्पृश्यों’ से भोजन स्वीकार नहीं करते थे कुछ कार्य ऐसे भी थे जिन्हें ‘दूषित’ माना जाता था जैसे शवों की अन्त्येष्टि करने का कार्य शवों की अन्त्येष्टि तथा मृत पशुओं को छूने वाले रा लोगों को चाण्डाल कहा जाता था। उच्च वर्ण के लोग इन  चाण्डालों का स्पर्श अथवा देखना भी अपवित्रकारी मानते इन् थे।

प्रश्न 23.
चाण्डालों के कर्त्तव्यों का वर्णन कीजिये।
उत्तर:
मनुस्मृति में चाण्डालों के कर्तव्यों का उल्लेख मिलता है। चाण्डालों को गाँव से बाहर रहना पड़ता था। वे के फेंके हुए बर्तनों का प्रयोग करते थे। मृतक लोगों के अस्व तथा लोहे के आभूषण पहनते थे। रात्रि में वे गाँवों और नगरों में ना भ्रमण नहीं कर सकते थे। उन्हें कुछ मृतकों की अन्त्येष्टि क करनी पड़ती थी तथा वधिक के रूप में भी कार्य करना पड़ता था। फाहियान के अनुसार उन्हें सड़कों पर चलते हुए करताल बजाकर अपने होने की सूचना देनी पड़ती थी, जिससे अन्य लोग उनके दर्शन न कर सकें।

प्रश्न 24.
प्राचीनकाल में सम्पत्ति पर स्त्री और पुरुष के भिन्न अधिकारों की विवेचना कीजिए।
उत्तर:
‘मनुस्मृति’ के अनुसार माता-पिता की मृत्यु के बाद पैतृक सम्पत्ति का सभी पुत्रों में समान रूप से बंटवारा किया जाना चाहिए, परन्तु ज्येष्ठ पुत्र विशेष भाग का अधिकारी था स्विय इस पैतृक संसाधन में हिस्सेदारी की माँग नहीं कर सकती थीं। परन्तु विवाह के समय मिले उपहारों पर स्त्रियों का स्वामित्व माना जाता था। इसे ‘स्वीधन’ अर्थात् ‘स्वी का धन’ कहा जाता था। इस सम्पत्ति को उनकी सन्तान विरासत के रूप में प्राप्त कर सकती थीं और इस पर उनके पति का कोई अधिकार नहीं होता था।

प्रश्न 25.
‘मनुस्मृति’ के अनुसार पुरुषों और स्त्रियों के धन अर्जित करने के तरीकों का उल्लेख कीजिए।
उत्तर:’
‘मनुस्मृति’ के अनुसार पुरुष सात तरीकों से धन अर्जित कर सकते हैं –

  1. विरासत
  2. खोज
  3. खरीद
  4. विजय प्राप्त करके
  5. निवेश
  6. कार्य द्वारा तथा
  7. सज्जनों द्वारा दी

गई भेंट को स्वीकार करके स्त्रियाँ 6 तरीकों से धन अर्जित कर सकती हैं –

  1. वैवाहिक अग्नि के सामने मिली भेंट
  2. वधू गमन के साथ मिली भेंट
  3. स्नेह के प्रतीक के रूप में मिली भेंट
  4. भ्राता, माता तथा पिता द्वारा दिए गए उपहार
  5. परवर्ती काल में मिली भेंट
  6. पति से प्राप्त भेंट।

प्रश्न 26.
“ब्राह्मणीय ग्रन्थों के अनुसार सम्पत्ति पर अधिकार का एक आधार वर्ण भी था।” विवेचना कीजिए।
उत्तर:
ब्राह्मणीय ग्रन्थों के अनुसार सम्पत्ति पर अधिकार का एक आधार वर्ण भी था शूद्रों के लिए एकमात्र ‘जीविका’ तीनों उच्चवर्णों की सेवा थी परन्तु इसमें उनकी इच्छा शामिल नहीं थी। दूसरी ओर तीनों उच्चवणों के पुरुषों के लिए विभिन्न जीविकाओं की सम्भावनाएँ रहती थीं। यदि इन नियमों को वास्तव में कार्यान्वित किया जाता, तो ब्राह्मण और क्षत्रिय सबसे अधिक धनी व्यक्ति होते यह बात कुछ अंशों तक सामाजिक व्यवस्था से मेल खाती थी।

प्रश्न 27.
बौद्धों द्वारा ब्राह्मणों द्वारा स्थापित वर्ण- व्यवस्था की आलोचनाओं का वर्णन कीजिए।
उत्तर:
कुछ परम्पराओं में इस वर्ण व्यवस्था की आलोचना की जा रही थी। लगभग छठी शताब्दी ई. पूर्व में बौद्ध ग्रन्थों में ब्राह्मणों द्वारा स्थापित वर्ण व्यवस्था की आलोचनाएँ प्रस्तुत की गई। यद्यपि बौद्धों ने इस बात को स्वीकार किया कि समाज में विषमता विद्यमान थी, परन्तु यह भेद प्राकृतिक और स्थायी नहीं थे। उन्होंने जन्म के आधार पर सामाजिक प्रतिष्ठा को मानने से इनकार कर दिया। इस प्रकार बौद्ध लोग जन्म पर आधारित वर्णीय व्यवस्था को स्वीकृति प्रदान नहीं करते।

JAC Class 12 History Important Questions Chapter 3 बंधुत्व, जाति तथा वर्ग : आरंभिक समाज

प्रश्न 28.
“प्राचीन तमिलकम में दानशील व्यक्ति का सम्मान किया जाता था। कृपण व्यक्ति घृणा का पात्र होता था।’ विवेचना कीजिए।
उत्तर:
प्राचीन तमिलकम में दानशील व्यक्ति का समाज में सम्मान किया जाता था तथा कृपण व्यक्ति घृणा का पात्र होता था। इस क्षेत्र में 2000 वर्ष पहले अनेक सरदारियाँ थीं ये सरदार अपनी प्रशंसा में गाने वाले चारणों और कवियों के आश्रयदाता होते थे। तमिल भाषा के संगम साहित्य से पता चलता है कि यद्यपि धनी और निर्धन के बीच विषमताएँ थीं, फिर भी धनी लोगों से यह अपेक्षा की जाती थी कि वे अपनी धन-सम्पत्ति का मिल बांट कर उपयोग करेंगे।

प्रश्न 29.
समाज में फैली हुई सामाजिक विषमताओं के संदर्भ में बौद्धों द्वारा प्रस्तुत की गई अवधारणा का वर्णन कीजिए।
उत्तर:
‘सुत्तपिटक’ नामक बौद्ध ग्रन्थ में एक मिथक का वर्णन है जो यह बताता है कि प्रारम्भ में सभी जीव शान्तिपूर्वक रहते थे और प्रकृति से उतना ही ग्रहण करते थे, जितनी एक समय के भोजन की आवश्यकता होती थी। परन्तु कालान्तर में लोग अधिकाधिक लालची और प्रतिहिंसक हो गए। इस स्थिति में उन्होंने समाज का नेतृत्व एक ऐसे व्यक्ति को सौंपने का निर्णय लिया जो जिसकी प्रताड़ना की जानी चाहिए उसे प्रताड़ित कर सके और जिसे निष्कासित किया जाना चाहिए, उसे निष्कासित कर सके।

प्रश्न 30.
साहित्यकार किसी ग्रन्थ का विश्लेषण करते समय किन पहलुओं पर विचार करते हैं?
अथवा
साहित्यिक स्रोतों का प्रयोग करते समय इतिहासकार को किन बातों को ध्यान में रखना चाहिए?
उत्तर:

  1. ग्रन्थों की भाषा क्या है? क्या यह आम लोगों की भाषा थी जैसे पालि, प्राकृत, तमिल अथवा पुरोहितों और किसी विशिष्ट वर्ग की भाषा थी जैसे संस्कृत।
  2. क्या ये ग्रन्थ ‘मन्त्र’ थे जो अनुष्ठानकर्त्ताओं द्वारा पढ़े जाते थे अथवा ‘कथाग्रन्थ’ थे जिन्हें लोग पढ़ और सुन सकते थे?’
  3. ग्रन्थों के लेखकों के बारे में जानकारी प्राप्त करना जिनके दृष्टिकोण और विचारों के आधार पर ग्रन्थ लिखे गए थे।
  4. श्रोताओं की अभिरुचि का ध्यान रखना।

प्रश्न 31.
महाभारत की भाषा का विवेचन कीजिए।
उत्तर:
महाभारत की भाषा- यद्यपि महाभारत की रचना अनेक भाषाओं में की गई है, परन्तु इसकी मूल भाषा संस्कृत है। परन्तु महाभारत में प्रयुक्त संस्कृत वेदों अथवा प्रशस्तियों की संस्कृत से कहीं अधिक सरल है अतः यह सम्भव है कि महाभारत को व्यापक स्तर पर समझा जाता था।

JAC Class 12 History Important Questions Chapter 3 बंधुत्व, जाति तथा वर्ग : आरंभिक समाज

प्रश्न 32.
महाभारत की विषय-वस्तु का विवेचन कीजिये।
उत्तर:
इतिहासकार महाभारत की विषय वस्तु को दो मुख्य शीर्षकों के अन्तर्गत रखते हैं –
(1) आख्यान तथा
(2) उपदेशात्मक।

‘आख्यान’ में कहानियों का संग्रह है जबकि उपदेशात्मक भाग में सामाजिक आचार-विचार के मानदण्डों का वर्णन है। परन्तु उपदेशात्मक अंशों में भी कहानियाँ होती हैं और प्रायः आख्यानों में समाज के लिए उपदेश निहित रहता है। वास्तव में महाभारत एक भाग में नाटकीय कथानक था जिसमें उपदेशात्मक अंश बाद में जोड़े
गए।

प्रश्न 33.
भगवद्गीता पर एक संक्षिप्त टिप्पणी लिखिए।
उत्तर:
भगवद्गीता महाभारत का सबसे महत्वपूर्ण उपदेशात्मक अंश है, जहाँ कुरुक्षेत्र की युद्धभूमि में श्रीकृष्ण अर्जुन को उपदेश देते हैं। इस ग्रन्थ में 18 अध्याय हैं। भगवद्गीता में ज्ञान, भक्ति एवं कर्म का समन्वय स्थापित किया गया है। अन्त में श्रीकृष्ण का उपदेश सुनकर अर्जुन अपने बान्धवों के विरुद्ध युद्ध करने के लिए तैयार हो गया। इस युद्ध में पाण्डवों की विजय हुई।

प्रश्न 34.
“महाभारत एक गतिशील ग्रन्थ है।” ‘व्याख्या कीजिए।
उत्तर:
शताब्दियों से इस महाकाव्य के अनेक पाठान्तर अनेक भाषाओं में लिखे गए अनेक कहानियाँ जिनका उद्भव एक क्षेत्र विशेष में हुआ और जिनका विशेष लोगों में प्रसार हुआ, वे सब इस महाकाव्य में सम्मिलित कर ली गई। इस महाकाव्य के मुख्य कथानक की अनेक पुनर्व्याख्याएँ प्रस्तुत की गई। महाभारत के विभिन्न प्रसंगों को मूर्तियों तथा चित्रों में भी दर्शाया गया। इस महाकाव्य ने नाटकों एवं नृत्य कलाओं के लिए भी विषय-वस्तु प्रदान की।

प्रश्न 35.
स्पष्ट कीजिए कि विशिष्ट परिवारों में पितृवंशिकता क्यों महत्त्वपूर्ण रही होगी?
उत्तर:
पूर्व वैदिक समाज के विशिष्ट परिवारों के विषय में इतिहासकारों को सुगमता के साथ सूचनाएँ प्राप्त हो जाती हैं। इतिहासकार परिवार तथा बन्धुता सम्बन्धी विचारों का विश्लेषण करते हैं, यहाँ उनका अध्ययन इसलिए महत्त्वपूर्ण हो जाता है क्योंकि इससे वैदिककालीन लोगों की विचारधारा का ज्ञान प्राप्त होता है। यहाँ महाभारत की कथा महत्त्वपूर्ण है। जिसमें बान्धवों के दो दलों, कौरव तथा पाण्डवों के मध्य भूमि तथा सत्ता को लेकर संघर्ष होता है।

दोनों एक ही कुरु वंश से सम्बन्धित थे। इनमें भीषण बुद्ध के उपरान्त पाण्डव विजयी होते हैं। इसके उपरान्त पितृवंशिक उत्तराधिकारी को उद्घोषित किया गया। लगभग छठी शताब्दी ई.पू. में अधिकांश राजवंश पितृवंशीय व्यवस्था का अनुगमन करते थे। यद्यपि इस प्रणाली में कभी-कभी भिन्नता भी देखने को मिलती है। पुत्र के न होने पर एक भाई दूसरे का उत्तराधिकारी हो जाता है। कभी बन्धु बान्धव सिंहासन पर अपना बलात् अधिकार जमा लेते थे।

प्रश्न 36.
ब्राह्मणों द्वारा लोगों को गोत्रों में वर्गीकृत करने की पद्धति कैसे प्रारम्भ हुई, इसे किस प्रकार लागू किया गया, क्या सातवाहन राजा इसके अपवाद हैं?
उत्तर:
ब्राह्मणों द्वारा लोगों को गोत्रों में वर्गीकृत करने की पद्धति लगभग 1000 ई. पू. के पश्चात् प्रारम्भ हुई। ब्राह्मणों ने लोगों का वर्गीकरण वैदिक ऋषियों के नाम पर आधारित गोत्रों के आधार पर करना प्रारम्भ कर दिया। एक ही गोत्र से सम्बन्धित सभी लोगों को उस ऋषि का वंशज समझा जाता था। गोत्र के सम्बन्ध में दो मुख्य नियम प्रचलित थे –

  • स्वियों को विवाह के पश्चात् अपने पिता का गोत्र त्यागकर अपने पति का गोत्र अपनाना पड़ता था।
  • सगोत्रीय, अर्थात् एक ही गोत्र में विवाह वर्जित था, एक गोत्र के सभी सदस्य रक्त सम्बन्धी समझे जाते थे।

सातवाहन राजाओं की रानियाँ अपने नाम के आगे अपने पिता का गोत्र लिखती थी जैसे कि गौतम और वशिष्ठ कई बार रक्त सम्बन्धियों में भी विवाह हो जाते थे। दक्षिण भारत में कहीं कहीं यह प्रथा आज भी पाई जाती है।

JAC Class 12 History Important Questions Chapter 3 बंधुत्व, जाति तथा वर्ग : आरंभिक समाज

प्रश्न 37.
चारों वर्णों से परे वर्णहीन ब्राह्मणीय व्यवस्था से अलग समुदायों की स्थिति पर टिप्पणी कीजिए।
उत्तर:
भारतीय उपमहाद्वीप की विशालता तथा विविधता और सामाजिक जटिलताओं के कारण कुछ ऐसे समुदाय भी अस्तित्व में थे, जिनका ब्राह्मणों द्वारा निश्चित किये गये वर्णों में कोई स्थान नहीं था। ऐसे समुदाय के लोगों को वर्णहीन कहा जाता था। इन वर्णहीन समुदायों पर ब्राह्मणों द्वारा निर्धारित सामाजिक नियमों का कोई प्रभाव नहीं था। संस्कृत साहित्य में जब ऐसे समुदायों का उल्लेख होता था तो इन्हें असभ्य, बेढंगे और पशु समान कहा जाता था। जंगलों में रहने वाली एक जाति निषाद की गणना इसी वर्ग में होती थी।

शिकार करना, कन्दमूल, फल – फूल एकत्र करना आदि वनों पर आधारित उत्पाद इनके जीवन निर्वाह के साधन थे। वह समाज से पूर्ण रूप से बहिष्कृत नहीं थे। समाज के अन्य वर्गों के साथ उनका संवाद होता था। उनके बीच- विचारों और मतों का आदान-प्रदान होता था महाभारत की कई कथाओं में इसका उल्लेख है।

प्रश्न 38.
प्राचीनकाल में सम्पत्ति के स्वामित्व की दृष्टि से महिलाओं की स्थिति अच्छी नहीं थी? स्पष्ट कीजिए।
अथवा
प्राचीनकाल में स्त्री और पुरुषों के मध्य सामाजिक स्थिति की भिन्नता संसाधनों पर उनके नियन्त्रण की भिन्नता के कारण ही व्यापक हुई स्पष्ट कीजिए।
उत्तर:
प्राचीनकाल में सम्पत्ति के स्वामित्व विशेषकर पैतृक सम्पत्ति के सन्दर्भ में महिलाओं की स्थिति अच्छी नहीं थी। मनुस्मृति के अनुसार पैतृक सम्पत्ति का माता- पिता की मृत्यु के पश्चात् सभी पुत्रों में समान रूप से बँटवारा किया जाना चाहिए, लेकिन ज्येष्ठ पुत्र विशेष भाग का अधिकारी होता है। स्त्रियाँ पैतृक सम्पत्ति में हिस्सेदारी की माँग नहीं कर सकतीं। किन्तु विवाह के समय मिले उपहारों पर स्वियों का स्वामित्व माना जाता था और इसे स्त्रीधन की संज्ञा दी जाती थी।

इस सम्पत्ति को उनकी संतान विरासत के रूप में प्राप्त कर सकती थी और इस पर उनके पति का कोई अधिकार नहीं होता था किन्तु मनुस्मृति स्त्रियों को पति की आज्ञा के विरुद्ध पारिवारिक सम्पत्ति अथवा स्वयं अपने बहुमूल्य धन के गुप्त संचय के विरुद्ध भी चेतावनी देती है। अधिकतर साक्ष्य भी इंगित करते हैं कि उच्च वर्ग की महिलाएँ संसाधनों पर अपनी पहुंच रखती थीं फिर भी भूमि और धन पर पुरुषों का ही नियन्त्रण था। दूसरे शब्दों में कहा जा सकता है कि स्वी और पुरुष के मध्य सामाजिक स्थिति की भिन्नता उनके नियन्त्रण की भिन्नता के कारण ही व्यापक हुई।

प्रश्न 39.
12वीं से 17वीं शताब्दी ई.पू. के मिलने वाले घरों के बारे में डॉ. बी.बी. लाल के किन्हीं एक अवलोकन का उल्लेख कीजिए।
उत्तर:
1951-52 में पुरातत्त्ववेत्ता बी. बी. लाल ने मेरठ जिले (उ.प्र.) के हस्तिनापुर नामक एक गाँव में उत्खनन किया। सम्भवतः यह महाभारत में उल्लेखित पाण्डवों की राजधानी हस्तिनापुर थी। बी. बी. लाल को यहाँ आबादी के पाँच स्तरों के साक्ष्य मिले थे जिनमें से दूसरा और तीसरा स्तर हमारे विश्लेषण के लिए महत्त्वपूर्ण है। दूसरे स्तर (लगभग बारहवीं से सातवीं शताब्दी ई.पू.) पर मिलने वाले घरों के बारे में लाल कहते हैं: “जिस सीमित क्षेत्र का उत्खनन हुआ वहाँ से आवास गृहों की कोई निश्चित परियोजना नहीं मिलती किन्तु मिट्टी की बनी दीवारें और कच्ची मिट्टी की ईटें अवश्य मिलती हैं। सरकण्डे की छाप वाले मिट्टी के पलस्तर की खोज इस बात की ओर इशारा करती है कि कुछ घरों की दीवारें सरकण्डों की बनी थीं जिन पर मिट्टी का पलस्तर चढ़ा दिया जाता था।”

JAC Class 12 History Important Questions Chapter 3 बंधुत्व, जाति तथा वर्ग : आरंभिक समाज

प्रश्न 40.
बी. बी. लाल को उत्खनन से महाभारत काल के किस पुरास्थल के साक्ष्य मिले हैं? ये साक्ष्य कितने स्तरों तक प्राप्त हुए हैं?
उत्तर:

  1. पुरातत्त्ववेत्ता बी. बी. लाल ने 1951-52 में उत्तर प्रदेश के जिले मेरठ के हस्तिनापुर नामक एक गाँव में उत्खनन किया।
  2. गंगा नदी के ऊपरी दोआब वाले क्षेत्र में इस पुरास्थल का होना जहाँ कुरू राज्य था, इस ओर इंगित करता है। यह पुरास्थल कुरुओं की राजधानी हस्तिनापुर हो सकती थी जिसका उल्लेख महाभारत में आता है।
  3. बी. बी. लाल को यहाँ की आबादी के पाँच स्तरों के साक्ष्य मिले हैं जिनमें दूसरा और तीसरा स्तर हमारे विश्लेषण के लिए महत्त्वपूर्ण है।
  4. दूसरे स्तर पर मिलने वाले घरों के बारे में लाल कहते हैं कि जिस सीमित क्षेत्र का उत्खनन हुआ वहाँ से आवासीय गृह की कोई निश्चित परियोजना नहीं मिलती है।
  5. तीसरे स्तर पर लगभग छठी से तीसरी शताब्दी ई.पू. के साक्ष्य मिले हैं। लाल के अनुसार तृतीय काल के घर कच्ची और कुछ पक्की ईंटों के बने हुए थे।

प्रश्न 41.
मंदसौर के अभिलेख से पाँचवीं शताब्दी ई. के रेशम के बुनकरों की स्थिति के विषय में क्या पता चलता है?
उत्तर:
(1) लगभग पाँचवीं शताब्दी ई. का अभिलेख मध्य प्रदेश के मंदसौर क्षेत्र से प्राप्त हुआ है। इसमें रेशम के बुनकरों की एक श्रेणी का वर्णन मिलता है जो मूलत: लाट (गुजरात) प्रदेश के निवासी थे।

(2) बाद में वे मंदसौर चले गये थे, जिसे उस समय दशपुर के नाम से जाना जाता था। यह कठिन यात्रा उन्होंने अपने बच्चों और बाँधयों के साथ सम्पन्न की।

(3) इन बुनकरों ने वहाँ के राजा की महानता के बारे में सुना था अतः वे उसके राज्य में बसना चाहते थे।

(4) यह अभिलेख जटिल सामाजिक प्रक्रियाओं की झलक देता है ये श्रेणियों के स्वरूप के विषय में अन्तर्दृष्टि प्रदान करता है। हालांकि श्रेणी की सदस्यता शिल्प में विशेषज्ञता पर निर्भर थी। कुछ सदस्य अन्य जीविका भी अपना लेते थे।

(5) इस अभिलेख से यह भी पता चलता है कि सदस्य एक व्यवसाय के अतिरिक्त और चीजों में भी सहभागी थे। सामूहिक रूप से उन्होंने शिल्पकर्म से अर्जित धन को सूर्य देवता के सम्मान में मन्दिर बनवाने पर खर्च किया।

प्रश्न 42.
मनुस्मृति के अनुसार चाण्डालों की क्या विशेषताएँ और कार्य थे? कुछ चीनी यात्रियों ने उनके बारे में क्या लिखा है?
उत्तर:
मनुस्मृति के अनुसार चाण्डालों को गाँव के बाहर रहना पड़ता था। वे फेंके हुए बर्तनों का इस्तेमाल करते थे, मरे हुए लोगों के वस्त्र तथा लोहे के आभूषण पहनते थे। रात्रि में गाँव और नगरों में चल-फिर नहीं सकते थे। सम्बन्धियों से विहीन मृतकों की उन्हें अन्त्येष्टि करनी पड़ती थी तथा अधिक के रूप में भी कार्य करना होता था। चीन से आए बौद्ध भिक्षु फा शिएन का कहना है कि अस्पृश्यों को सड़क पर चलते हुए करताल बजाकर अपने होने की सूचना देनी पड़ती थी जिससे अन्य जन उन्हें देखने के दोष से बच जाएँ एक और चीनी तीर्थयात्री हवन-त्सांग के अनुसार अधिक और सफाई करने वालों को नगर से बाहर रहना पड़ता था।

JAC Class 12 History Important Questions Chapter 3 बंधुत्व, जाति तथा वर्ग : आरंभिक समाज

प्रश्न 43.
1919 में संस्कृत विद्वान् वी.एस. शुकथांकर के नेतृत्व में किस महत्त्वाकांक्षी परियोजना की शुरुआत हुई? संक्षेप में लिखिए।
उत्तर:
1919 में प्रसिद्ध संस्कृत विद्वान् वी.एस. सुकथांकर के नेतृत्व में अनेक विद्वानों ने मिलकर महाभारत का समालोचनात्मक संस्करण तैयार करने का जिम्मा उठाया। आरम्भ में देश के विभिन्न भागों से विभिन्न लिपियों में लिखी गई महाभारत की संस्कृत पाण्डुलिपियों को एकत्रित किया गया। परियोजना पर काम करने वाले विद्वानों ने सभी पाण्डुलिपियों में पाए जाने वाले श्लोकों की तुलना करने का एक तरीका ढूँढ़ा। अंततः उन्होंने उन श्लोकों का चयन किया जो लगभग सभी पाण्डुलिपियों में पाए गए थे और उनका प्रकाशन 13,000 पृष्ठों में फैले अनेक ग्रन्थ खण्डों में किया। इस परियोजना को पूरा करने में सैंतालीस वर्ष लगे।

प्रश्न 44.
महाभारत ग्रन्थ के समालोचनात्मक संस्करण तैयार करने की प्रक्रिया में कौनसी दो विशेष बातें उभर कर आई ?
उत्तर:
महाभारत ग्रन्थ के समालोचनात्मक संस्करण तैयार करने की प्रक्रिया में निम्न दो बातें उभर कर आई –
(1) संस्कृत के कई पाठों के अनेक अंशों में समानता थी। यह इस बात से ही स्पष्ट होता है कि समूचे उपमहाद्वीप में उत्तर में कश्मीर और नेपाल से लेकर दक्षिण में केरल और तमिलनाडु तक सभी पाण्डुलिपियों में यह समानता देखने में आई।

(2) कुछ शताब्दियों के दौरान हुए महाभारत के प्रेषण में अनेक क्षेत्रीय प्रभेद भी उभर कर सामने आए। इन प्रभेदों का संकलन मुख्य पाठ की पादटिप्पणियों और परिशिष्टों के रूप में किया गया। 13,000 पृष्ठों में से आधे से भी अधिक इन प्रभेदों का ब्यौरा देते हैं।

निबन्धात्मक प्रश्न

प्रश्न 1.
महाभारत के समालोचनात्मक संस्करण को तैयार करने की परियोजना का वर्णन कीजिए।
अथवा
बीसवीं शताब्दी में किये गए महाभारत के संकलन के विभिन्न सोपानों का वर्णन कीजिये।
उत्तर:
महाभारत का समालोचनात्मक संस्करण –
(1) बी.एस. सुकथांकर के नेतृत्व में एक परियोजना का आरम्भ – 1919 में प्रसिद्ध संस्कृत विद्वान वी.एस. सुकथांकर के नेतृत्व में एक अत्यन्त महत्त्वाकांक्षी परियोजना का आरम्भ हुआ। अनेक विद्वानों ने मिलकर महाभारत का समालोचनात्मक संस्कार तैयार करने का निश्चय किया।

(2) श्लोकों का चयन – परियोजना पर काम करने वाले विद्वानों ने उन श्लोकों का चयन किया जो लगभग सभी पांडुलिपियों में पाए गए थे। उन्होंने उनका प्रकाशन 13,000 पृष्ठों में फैले अनेक ग्रन्थ-खंडों में किया। इस परियोजना को पूरा करने में सैंतालीस वर्ष लगे।

(3) दो तथ्यों का उभर कर आना –
इस सम्पूर्ण प्रक्रिया में दो तथ्य विशेष रूप से उभर कर सामने आए –

(i) संस्कृत के कई पाठों के अनेक अंशों में समानता – पहला, संस्कृत के कई पाठों में अनेक अंशों में समानता थी। यह इस बात से ही स्पष्ट होता है कि सम्पूर्ण उपमहाद्वीप में उत्तर में कश्मीर और नेपाल से लेकर दक्षिण में केरल और तमिलनाडु तक सभी पांडुलिपियों में यह समानता दिखाई दी।

JAC Class 12 History Important Questions Chapter 3 बंधुत्व, जाति तथा वर्ग : आरंभिक समाज

(ii) क्षेत्रीय प्रभेदों का उभरकर सामने आना- दूसरा तथ्य यह था कि कुछ शताब्दियों के दौरान हुए महाभारत के प्रेषण में अनेक क्षेत्रीय प्रभेद भी उभरकर सामने आएं इन प्रभेदों का संकलन मुख्य पाठ की पाद टिप्पणियों और परिशिष्टों के रूप में किया गया।

(4) प्रक्रियाओं के बारे में जानकारी इन सभी प्रक्रियाओं के बारे में हमारी जानकारी मुख्यतः उन ग्रन्थों पर आधारित है जो संस्कृत में ब्राह्मणों द्वारा उन्हीं के लिए लिखे गए। उन्नीसवीं तथा बीसवीं शताब्दी में इतिहासकारों ने पहली बार सामाजिक इतिहास के मुद्दों का अध्ययन करते हुए इन ग्रन्थों को ऊपरी तौर पर समझा।

उनका विश्वास था कि इन ग्रन्थों में जो कुछ भी लिखा गया है, वास्तव में उसी प्रकार से उसे व्यवहार में लाया जाता होगा। कालान्तर में विद्वानों ने पालि, प्राकृत और तमिल ग्रन्थों के माध्यम से अन्य परम्पराओं का अध्ययन किया। इन अध्ययनों से वे इस निष्कर्ष पर पहुंचे कि आदर्शमूलक संस्कृत ग्रन्थ सामान्यतः आधिकारिक माने जाते थे। परन्तु इन आदर्शों पर प्रश्न भी उठाए जाते थे और कभी-कभी इनकी अवहेलना भी की जाती थी।

प्रश्न 2.
मनुस्मृति से पहली, चौथी, पाँचवीं तथा छठी विवाह पद्धति के उद्धरण को प्रस्तुत कीजिये।
उत्तर:
विवाह पद्धतियों के उद्धरण प्राचीन भारत में विवाह की आठ प्रकार की पद्धतियाँ प्रचलित थीं। धर्मसूत्रों और धर्मशास्त्रों ने विवाह की आठ प्रकार की पद्धतियों को मान्यता प्रदान की है। मनुस्मृति से पहली, चौथी, पाँचव तथा छठी विवाह पद्धति के उद्धरण प्रस्तुत हैं –
(1) विवाह की पहली पद्धति इस विवाह पद्धति के अन्तर्गत कन्या का दान बहुमूल्य वस्त्रों तथा अलंकारों से विभूषित कर उसे वेदश वर को दान दिया जाए जिसे पिता ने स्वयं आमन्त्रित किया हो।

(2) विवाह की चौथी पद्धति इस विवाह पद्धति के अन्तर्गत वर की विधिपूर्वक पूजा करके कन्या का दान किया जाता था पिता वर-वधू युगल को यह कह कर सम्बोधित करता है कि “तुम साथ मिलकर अपने दायित्वों का पालन करो। गृहस्थ जीवन में दोनों मिलकर जीवनपर्यन्त धर्माचरण करो।” इसके बाद वह वर का सम्मान कर उसे
कन्या का दान करता था।

(3) विवाह की पाँचवीं पद्धति वर को वधू की प्राप्ति तब होती है, जब वह अपनी क्षमता व इच्छानुसार उसके बांधों को और स्वयं वधू को यथेष्ट धन प्रदान करता है। मनु के अनुसार इस विवाह में बान्धवों (कन्या के पिता, चाचा आदि) को कन्या के लिए यथेष्ट धन देकर स्वेच्छा से कन्या स्वीकार की जाती है।

(4) विवाह की छठी पद्धति इस पद्धति में लड़का तथा लड़की काम से उत्पन्न हुई इच्छा से प्रेम विवाह कर लेते थे। इसे गान्धर्व विवाह भी कहा जाता है। मनु ने कन्या और वर के इच्छानुसार कामुकतावश संयुक्त होने को ‘गान्धर्व विवाह’ की संज्ञा दी है। यह प्रथा प्रेम विवाह या प्रणय विवाह का सूचक है जो हिन्दू समाज में अत्यन्त प्राचीन काल से विद्यमान है।

प्रश्न 3.
क्या प्राचीन काल के भारत में केवल क्षत्रीय शासक ही शासन कर सकते थे? विवेचना कीजिये।
उत्तर:
प्राचीन भारत में क्षत्रियों एवं अक्षत्रियों द्वारा शासन करना धर्मसूत्रों एवं धर्मशास्त्रों के अनुसार केवल क्षत्रिय वर्ग के लोग ही शासन कर सकते थे। वैदिककाल के प्रायः सभी शासक क्षत्रिय वर्ग से सम्बन्धित थे परन्तु सभी शासक क्षत्रिय नहीं होते थे। प्राचीन काल में अनेक अक्षत्रिय राजाओं ने भी शासन किया।
(1) मौर्य वंश मौर्यों की उत्पत्ति के बारे में विद्वानों में मतभेद है। यद्यपि बौद्ध ग्रन्थों में मौर्यों को क्षत्रिय बताया गया है, परन्तु ब्राह्मणीय शास्त्र मौर्यो को ‘निम्न’ कुल का मानते हैं।

(2) शुंग और कण्व वंश मौर्य वंश के बाद शुंग तथा कण्व वंश के राजाओं ने शासन किया। शुंग वंश तथा कण्व वंश के राजा ब्राह्मण थे। वास्तव में योग्य, शक्तिशाली तथा साधन-सम्पन्न व्यक्ति राजनीतिक सत्ता का उपयोग कर सकता था। राजत्व क्षत्रिय कुल में जन्म लेने पर शायद ही निर्भर करता था। दूसरे शब्दों में यह आवश्यक नहीं था कि केवल क्षत्रिय कुल में जन्म लेने वाला व्यक्ति ही गद्दी पर बैठ सकता था।

(3) शक वंश ब्राह्मण मध्य एशिया से आने वाले शकों को म्लेच्छ, बर्बर अथवा विदेशी मानते थे परन्तु कुछ शक राजा भी भारतीय रीति-रिवाज और परिपाटियों का अनुसरण करते थे। संस्कृत में लिखे हुए ‘जूनागढ़ अभिलेख’ से ज्ञात होता है कि लगभग दूसरी शताब्दी ईसवी में शक- शासक रुद्रदामन ने सुदर्शन झील की मरम्मत करवाई थी जिससे लोगों को बड़ी राहत मिली थी। इससे पता चलता है कि शक्तिशाली म्लेच्छ भी भारतीय राजाओं की भाँति प्रजावत्सल थे और संस्कृतीय परिपाटी से परिचित थे।

JAC Class 12 History Important Questions Chapter 3 बंधुत्व, जाति तथा वर्ग : आरंभिक समाज

(4) सातवाहन वंश अधिकांश इतिहासकारों के अनुसार सातवाहन वंश के शासक ब्राह्मण थे। सातवाहन वंश के सबसे प्रसिद्ध शासक गौतमी पुत्र शातकर्णी ने स्वयं को ‘अनूठा ब्राह्मण’ एवं ‘क्षत्रियों के दर्प को चकनाचूर करने वाला’ बताया था। उनकी वर्ण व्यवस्था में पूर्ण आस्था थी। इसलिए उन्होंने चार वर्णों की व्यवस्था की मर्यादा को बनाये रखने का प्रयास किया।

प्रश्न 4.
ब्राह्मण किन लोगों को वर्णव्यवस्था वाली सामाजिक प्रणाली के बाहर मानते थे? क्या आप इस मत से सहमत हैं कि उन्होंने समाज के कुछ वर्गों को ‘अस्पृश्य’ घोषित कर सामाजिक विषमता को और तीव्र कर दिया?
उत्तर:
ब्राह्मणों द्वारा अस्पृश्य लोगों को वर्ण- व्यवस्था वाली सामाजिक प्रणाली के बाहर मानना
(1) समाज के कुछ वर्गों को अस्पृश्य घोषित करना – ब्राह्मण कुछ लोगों को वर्ण व्यवस्था वाली सामाजिक प्रणाली के बाहर मानते थे। उन्होंने समाज के कुछ वर्गों को ‘अस्पृश्य’ घोषित कर दिया और इस प्रकार सामाजिक विषमता को और अधिक तीव्र कर दिया।

(2) ‘अस्पृश्य’ घोषित करने के आधार ब्राह्मणों की यह मान्यता थी कि कुछ कर्म, विशेष रूप से अनुष्ठानों के सम्पादन से जुड़े हुए कर्म, पुनीत और पवित्र थे अतः अपने को पवित्र मानने वाले लोग अस्पृश्यों से भोजन स्वीकार नहीं करते थे। इसके विपरीत कुछ कार्य ऐसे थे, जिन्हें विशेष रूप से ‘दूषित’ माना जाता था। शव की अन्त्येष्टि करने वालों और मृत पशुओं को छूने वालों को चाण्डाल कहा जाता था उन्हें वर्ण व्यवस्था वाले समाज में सबसे निम्न कोटि में रखा जाता था। वे लोग, जो स्वयं को सामाजिक क्रम में सबसे ऊपर मानते थे, इन चाण्डालों का स्पर्श, यहाँ तक कि उन्हें देखना भी अपवित्रकारी मानते थे।

(3) चाण्डालों के कर्तव्य ‘मनुस्मृति’ के अनुसार चाण्डालों के निम्नलिखित कार्य थे –

  • चाण्डालों को गाँवों से बाहर रहना होता था।
  • वे फेंके हुए बर्तनों का प्रयोग करते थे तथा मृतक लोगों के वस्त्र और लोहे के आभूषण पहनते थे।
  • रात्रि में वे गाँव और नगरों में घूम नहीं सकते थे।
  • उन्हें सम्बन्धियों से विहीन मृतकों की अन्त्येष्टि करनी पड़ती थी।
  • उन्हें जल्लाद के रूप में भी कार्य करना पड़ता

(4) चीनी यात्रियों द्वारा चाण्डालों का वर्णन करना पाँचवीं शताब्दी ई. में फा-शिएन ने लिखा है कि अस्पृश्यों को सड़क पर चलते समय करताल बजाकर अपने चाण्डाल होने की सूचना देने पड़ती थी जिससे अन्य लोग उन्हें देखने के दोष से बच जाएँ इसी प्रकार लगभग सातवीं शताब्दी ई. में आने वाले चीनी यात्री श्वैन-त्सांग ने लिखा है कि जल्लाद और सफाई करने वालों को नगर से बाहर रहना पड़ता था। इस प्रकार समाज में अस्पृश्यों की स्थिति शोचनीय थी। उन्हें किसी प्रकार के अधिकार प्राप्त नहीं थे।

प्रश्न 5.
प्राचीनकाल में सम्पत्ति पर स्त्री और पुरुष के अधिकारों की विवेचना कीजिए।
उत्तर:
(1) द्यूत क्रीड़ा में युधिष्ठिर का हारना- कौरवों और पाण्डवों के बीच लम्बे समय से चली आ रही प्रतिस्पर्द्धा के फलस्वरूप दुर्योधन ने युधिष्ठिर को द्यूत क्रीड़ा के लिए आमन्त्रित किया। युधिष्ठिर अपने प्रतिद्वन्द्वी द्वारा धोखा दिए जाने के कारण इस द्यूत क्रीड़ा में सोना, हाथी, रथ, दास, सेना, कोष, राज्य तथा अपनी प्रजा की सम्पत्ति, अनुजों और फिर स्वयं को भी दाँव पर लगा कर गंवा बैठा। इसके बाद उन्होंने पांडवों की सहपत्नी द्रौपदी को भी दाँव पर लगाया और उसे भी हार गए। इससे यह ज्ञात होता है कि प्राचीन काल मैं पुरुष लोग अपनी पत्नियों को निजी सम्पति मानते थे।

JAC Class 12 History Important Questions Chapter 3 बंधुत्व, जाति तथा वर्ग : आरंभिक समाज

(2) प्राचीन काल में सम्पत्ति पर स्त्री और पुरुष के भिन्न अधिकार- मनुस्मृति के अनुसार माता-पिता की मृत्यु के पश्चात् पैतृक सम्पत्ति का सभी पुत्रों में समान रूप से बँटवारा किया जाना चाहिए। परन्तु इसमें ज्येष्ठ पुत्र विशेष भाग का अधिकारी था। स्त्रियाँ इस पैतृक संसाधन में हिस्सेदारी की मांग नहीं कर सकती थीं।

(3) स्त्रीधन यद्यपि पैतृक संसाधन में स्त्रियाँ हिस्सेदारी की माँग नहीं कर सकती थीं, परन्तु विवाह के समय मिले उपहारों पर स्वियों का स्वामित्व माना जाता था। इसे ‘स्त्रीधन’ अर्थात् ‘स्वी का धन’ कहा जाता था। इस सम्पत्ति को उनकी सन्तान विरासत के रूप में प्राप्त कर सकती थीं और इस पर उनके पति का कोई अधिकार नहीं होता था किन्तु मनुस्मृति’ स्त्रियों को पति की अनुमति के बिना पारिवारिक सम्पत्ति अथवा स्वयं अपनी बहुमूल्य वस्तुओं को गुप्त रूप से एकत्रित करने से रोकती है।

(4) सामान्यतः भूमि, पशु और धन पर पुरुषों का नियन्त्रण होना- चन्द्रगुप्त द्वितीय की पुत्री प्रभावती गुप्त का विवाह वाकाटक कुल में हुआ था। वह एक धनाढ्य महिला थी। उसने दंगुन गाँव को एक ब्राह्मण आचार्य चनालस्वामी को दान किया था परन्तु अधिकांश साक्ष्यों से ज्ञात होता है कि यद्यपि उच्चवर्ग की महिलाएँ संसाधनों पर अपनी पैठ रखती थीं, फिर भी प्रायः भूमि, पशु और धन पर पुरुषों का ही नियन्त्रण था। दूसरे शब्दों में, स्त्री और पुरुष के बीच सामाजिक स्थिति की भिन्नता संसाधनों पर उनके नियन्त्रण की भिन्नता के कारण से अधिक प्रबल हुई।

प्रश्न 6.
बी. बी. लाल के नेतृत्व में हस्तिनापुर में किए गए उत्खनन का वर्णन कीजिए।
उत्तर:
बी.बी. लाल के नेतृत्व में हस्तिनापुर में उत्खनन कार्य महाभारत में अन्य प्रमुख महाकाव्यों की भाँति युद्धों, वनों राजमहलों और बस्तियों का अत्यन्त प्रभावशाली चित्रण है। 1951-52 ई. में प्रसिद्ध पुरातत्ववेत्ता बी. बी. लाल ने मेरठ जिले (उत्तरप्रदेश) के हस्तिनापुर नामक एक गाँव में उत्खनन किया। यह दावे के साथ नहीं कहा जा सकता कि यह गाँव महाभारत में वर्णित हस्तिनापुर ही था। फिर भी नामों की समानता केवल एक संयोग हो सकता है, परन्तु गंगा के ऊपरी दोआब वाले क्षेत्र में इस पुरास्थल का होना जहाँ कुरु राज्य भी था, इस ओर संकेत करता है कि शायद यह पुरास्थल कुरुओं की राजधानी हस्तिनापुर ही था, जिसका उल्लेख महाभारत में आता है।

(1) आबादी के पाँच स्तरों के साक्ष्य-बी. बी. लाल को उत्खनन के दौरान यहाँ आबादी के पाँच स्तरों के साक्ष्य मिले थे जिनमें से दूसरा और तीसरा स्तर हमारे विश्लेषण के लिए महत्त्वपूर्ण हैं।

(2) दूसरे स्तर के साक्ष्य दूसरे स्तर (लगभग बारहवीं से सातवीं शताब्दी ई. पूर्व) पर मिलने वाले घरों के सम्बन्ध में बी.बी. लाल लिखते हैं कि “जिस सीमित क्षेत्र का उत्खनन हुआ, वहाँ से आवास गृहों की कोई निश्चित परियोजना नहीं मिलती परन्तु मिट्टी की बनी दीवार और कच्ची मिट्टी की ईंटें अवश्य मिलती हैं। सरकंडे की छाप वाले मिट्टी के पलस्तर की खोज इस बात की ओर संकेत करती है कि कुछ घरों की दीवारें सरकंडों की बनी धीं जिन पर मिट्टी का पलस्तर चढ़ा दिया जाता था।”

(3) तीसरे स्तर के साक्ष्य तीसरे स्तर (लगभग छठी से तीसरी शताब्दी ई. पूर्व) के घरों के बारे में बी.बी. लाल लिखते हैं कि “तृतीय काल के घर कच्ची और कुछ पक्की ईंटों के बने हुए थे। इनमें शोषकघट और ईंटों के नाले गंदे पानी के निकास के लिए प्रयुक्त किए जाते थे। वलय कूपों का प्रयोग, कुओं और मल की निकासी वाले गत दोनों ही रूपों में किया जाता था।

” महाभारत के आदिपर्वन में हस्तिनापुर के चित्रण से लाल द्वारा खोजे गए हस्तिनापुर के चित्रण से भिन्नता- महाभारत के आदिपर्वन में हस्तिनापुर का जो चित्रण किया गया है वह बी. बी. लाल के द्वारा खोजे गए हस्तिनापुर के चित्रण से बिल्कुल भिन्न है ऐसा प्रतीत होता है कि हस्तिनापुर नगर का यह चित्रण महाभारत के मुख्य कथानक में बाद में सम्भवतः ई. पूर्व छठी शताब्दी के बाद जोड़ा गया प्रतीत होता है, जब इस क्षेत्र में नगरों का विकास हुआ यह भी सम्भव है कि यह मात्र कवियों की कल्पना की उड़ान थी जिसकी पुष्टि किसी भी अन्य साक्ष्य से नहीं होती।

JAC Class 12 History Important Questions Chapter 3 बंधुत्व, जाति तथा वर्ग : आरंभिक समाज

प्रश्न 7.
“महाभारत की सबसे चुनौतीपूर्ण उपकथा द्रौपदी से पांडवों के विवाह की है।” स्पष्ट कीजिए तथा द्रौपदी के विवाह के बारे में विभिन्न इतिहासकारों के मतों का विवेचन कीजिए।
उत्तर:
द्रौपदी से पांडवों के विवाह की उपकथा का चुनौतीपूर्ण होना महाभारत की सबसे चुनौतीपूर्ण उपकथा द्रौपदी से पांडवों के विवाह की है। यह बहुपति विवाह का उदाहरण है जो महाभारत की कथा का अभिन्न अंग है। द्रौपदी से पांडवों के विवाह के बारे में विभिन्न इतिहासकारों ने निम्नलिखित मत प्रकट किए हैं –

(1) बहुपति विवाह की प्रथा का शासकों के विशिष्ट वर्ग में विद्यमान होना वर्तमान इतिहासकारों ने यह मत प्रकट किया है कि महाभारत के लेखक (लेखकों) द्वारा बहुपति विवाह सम्बन्ध का वर्णन इस तथ्य की ओर संकेत करता है कि सम्भवतः यह प्रथा शासकों के विशिष्ट वर्ग में किसी काल में विद्यमान थी किन्तु साथ ही इस प्रकरण के विभिन्न स्पष्टीकरण इस बात को भी व्यक्त करते हैं कि समय के साथ बहुपति प्रथा उन ब्राह्मणों में अमान्य हो गई, जिन्होंने कई शताब्दियों के दौरान इस ग्रन्थ का पुनर्निर्माण किया।

(2) बहुपति प्रथा का हिमालय क्षेत्र में प्रचलित होना- कुछ इतिहासकारों का कहना है कि यद्यपि ब्राह्मणों की दृष्टि में बहुपति प्रथा अमान्य और अवांछित थी, फिर भी यह प्रथा हिमालय क्षेत्र में प्रचलित थी और आज भी प्रचलित है। यह भी तर्क दिया जाता है कि युद्ध के समय स्त्रियों की कमी के कारण बहुपति प्रथा को अपनाया गया था। दूसरे शब्दों में यह भी कहा जा सकता है कि बहुपति प्रथा को संकट की स्थिति में अपनाया गया था।

(3) सृजनात्मक साहित्य की अपनी कथा सम्बन्धी आवश्यकताएँ होना आरम्भिक स्रोतों से इस तथ्य की पुष्टि होती है कि बहुपति प्रथा न तो एकमात्र विवाह पद्धति थी और न ही यह सबसे अधिक प्रचलित थी। फिर भी महाभारत के लेखक (लेखकों) ने इस प्रथा को ग्रन्थ के प्रमुख पात्रों के साथ अभिन्न रूप से जोड़ कर क्यों देखा ? इस सम्बन्ध में यह कहा जा सकता है कि सृजनात्मक साहित्य की अपनी कथा सम्बन्धी आवश्यकताएँ होती हैं जो सदैव समाज में विद्यमान वास्तविकताओं को उजागर नहीं करतीं।

प्रश्न 8.
महाभारत की मुख्य कथावस्तु के लिए बांग्ला लेखिका महाश्वेता देवी द्वारा खोजे गए विकल्पों का विवेचन कीजिए।
उत्तर:
महाभारत की मुख्य कथावस्तु के लिए महाश्वेता देवी द्वारा खोजे गए विकल्प प्रसिद्ध बांग्ला लेखिका महाश्वेता देवी ने महाभारत की मुख्य कथावस्तु के लिए अन्य विकल्पों की खोज की है और उन प्रश्नों की ओर ध्यान खींचा है जिन पर संस्कृत पाठ चुप है।

(1) दुर्योधन द्वारा पांडवों को छल से ‘लाक्षागृह’ मैं जलाकर मार देना- संस्कृत पाठ में दुर्योधन द्वारा पांडवों को छल से लाक्षागृह में जलाकर मार देने का उल्लेख आता है किन्तु पहले से मिली चेतावनी के कारण पांडव उस गृह में सुरंग खोदकर भाग निकलते हैं उस समय कुंती एक भोज का आयोजन करती है जिसमें ब्राह्मण आमन्त्रित होते हैं। किन्तु एक निषादी भी अपने पाँच पुत्रों के साथ उसमें आती है। भोजन करके वे गहरी नींद में सो जाते हैं और पांडवों द्वारा लगाई गई आग में वे उसी लाक्षागृह में जल कर मर जाते हैं। जब उस निषादी और उसके पुत्रों के जले हुए शव मिलते हैं तो कि पांडवों की मृत्यु हो गई है।

(2) महाश्वेता देवी की लघु कथा ‘कुंती ओ निषादी’ महाश्वेता देवी ने अपनी लघुकथा, जिसका शीर्षक ‘कुंती ओ निषादी’ है, के कथानक का प्रारम्भ वहीं से किया है, जहाँ महाभारत के इस प्रसंग का अंत होता है। उन्होंने अपनी कहानी की रचना एक वन्य प्रदेश में की है जहाँ महाभारत के युद्ध के बाद कुंती रहने लगती है। कुंती अपने अतीत के बारे में विचार करती है। वह पृथ्वी से बातें करते हुए अपनी त्रुटियों को स्वीकार करती है। प्रतिदिन वह निषादों को देखती है, जो जंगल में लकड़ी, शहद, कंदमूल आदि इकट्ठा करने आते हैं। एक निषादी प्रायः कुंती को पृथ्वी से बातें करते हुए सुनती है।

(3) निषादी द्वारा कुंती से लाक्षागृह की घटना के बारे में प्रश्न करना एक दिन आग लगने के कारण जानवर जंगल छोड़कर भाग रहे थे। कुंती ने देखा कि नियादी उन्हें घूर घूर कर देख रही थी जब निषादी ने कुंती से पूछा कि क्या उन्हें लाक्षागृह की याद है तो कुंती झेंप गई। उन्हें याद था कि उन्होंने वृद्धा निषादी तथा उसके पाँच पुत्रों को इतनी शराब पिलाई थी कि वे सब बेसुध हो गए थे जबकि वह स्वयं अपने पुत्रों के साथ बचकर लाक्षागृह से निकल गई थी।

जब कुंती ने उससे पूछा कि क्या तुम वही निषादी हो? तो उसने उत्तर दिया कि मरी हुई निषादी उसकी सास थी। साथ ही उसने यह भी कहा कि अपने अतीत पर विचार करते समय तुम्हें एक बार भी उन छ निर्दोष लोगों की याद नहीं आई जिन्हें मौत के मुंह में जाना पड़ा था, क्योंकि तुम तो अपने और अपने पुत्रों के प्राण बचाना लोग यह अनुमान लगा लेते हैं। चाहती थीं। आग की लपटें निकट आने के कारण निषादी तो अपने प्राण बचाकर निकल गई, परन्तु कुंती जहाँ खड़ी थी, वहीं रह गई।

JAC Class 12 History Important Questions Chapter 3 बंधुत्व, जाति तथा वर्ग : आरंभिक समाज

प्रश्न 9.
महाभारतकालीन सामाजिक, धार्मिक, आर्थिक एवं राजनीतिक स्थिति का वर्णन कीजिए।
उत्तर:
(1) महाभारतकालीन सामाजिक स्थिति- महाभारत काल में वर्ण व्यवस्था सुदृढ़ हो चुकी थी। समाज चार प्रमुख वर्णों में विभाजित था ब्राह्मण, क्षत्रिय, वैश्य तथा शूद्र चारों वर्णों में ब्राह्मणों को सर्वश्रेष्ठ समझा जाता था। इस युग में पुत्री की अपेक्षा पुत्र का अधिक महत्त्व था। इस युग में सजातीय विवाहों का अधिक प्रचलन था, परन्तु अन्तर्जातीय विवाहों का भी उल्लेख मिलता है बहुपति प्रथा भी प्रचलित थी। द्रौपदी का विवाह पाँच पांडवों से हुआ था। स्वयंवर प्रथा प्रचलित थी। कुल मिलाकर स्वियों की स्थिति सन्तोषजनक थी।

(2) आर्थिक स्थिति इस युग के लोगों की आजीविका का मुख्य साधन कृषि था। गेहूं, जौ, चावल, चना दाल, तिल आदि की खेती की जाती थी। राज्य की ओर से सिंचाई की व्यवस्था की जाती थी। पशुपालन भी आजीविका का एक अन्य मुख्य साधन था। इस युग में अनेक उद्योग-धन्धे विकसित थे। स्वर्णकार, लुहार, कुम्हार, बढ़ई, जुलाहे, रंगरेज, चर्मकार, रथकार आदि अनेक शिल्पकार थे। वस्त्र उद्योग काफी उन्नत अवस्था में था। इस युग में वाणिज्य तथा व्यापार भी उन्नत अवस्था में था ।

(3) धार्मिक स्थिति इस युग में ब्रह्मा, विष्णु, शिव, गणेश, पार्वती, लक्ष्मी, दुर्गा आदि देवी-देवताओं की पूजा की जाती थी। इस युग में अवतारवाद की अवधारणा प्रचलित थी। विष्णु के अवतार के रूप में राम, श्रीकृष्ण आदि की उपासना की जाती थी यज्ञों का महत्त्व भी बना हुआ था। परन्तु पशुबलि धीरे-धीरे लुप्त हो रही थी।

(4) राजनीतिक स्थिति राज का पद बड़ा प्रतिष्ठित माना जाता था उसे अग्नि, सूर्य, यम, कुबेर आदि विभिन्न रूपों में प्रतिष्ठित किया गया है। लोक कल्याण और धर्मानुसार शासन करना उसके प्रमुख कर्त्तव्य थे। राजा का पद वंशानुगत था परन्तु शारीरिक दोष होने पर बड़े पुत्र को उत्तराधिकार के अधिकार से वंचित किया जा सकता था। नेत्रहीन होने के कारण धृतराष्ट्र को गद्दी से वंचित रहना पड़ा था। राजा को सहयोग और सलाह देने के लिए मन्त्रिपरिषद् का गठन किया गया था। राजा के अन्य पदाधिकारियों में युवराज, सेनापति, द्वारपाल, दुर्गपाल आदि प्रमुख थे। सेना के चार प्रमुख अंग थे –

  • पैदल
  • अश्वारोही
  • गजारोही
  • रथारोही

महाभारत में कुछ गणराज्यों का भी उल्लेख मिलता है। गणराज्यों ने अपनी सुरक्षा के लिए अपना संघ बना रखा था।

प्रश्न 10.
ब्राह्मणों ने धर्मसूत्रों और धर्मशास्त्रों की रचना क्यों की? इसमें चार वर्गों के लिए जीविका- सम्बन्धी कौनसे नियम निर्धारित किए गए थे?
उत्तर:
(i) धर्मसूत्र में वेद विद्या के सिद्धान्त का वर्णन किया गया है। इसमें उच्च विचारों और भावों का समावेश है। धर्मसूत्र में सामाजिक नियमों का वर्णन मिलता है। धर्मशास्त्रों में मनुस्मृति सबसे प्राचीन है। इसमें प्राचीन विचारों के हिन्दुओं के जीवन के सभी पहलुओं के विषय में लिखा गया है। वर्तमान में भारतीय न्याय व्यवस्था में हिन्दुओं से सम्बन्धी कानून मनुस्मृति पर काफी हद तक आधारित हैं।

(ii) धर्मशास्त्रों और धर्मसूत्रों में एक आदर्श सामाजिक व्यवस्था का उल्लेख किया गया था। ब्राह्मणों का यह मानना था कि यह व्यवस्था जिसमें ब्राह्मणों को पहला स्थान प्राप्त है, एक दैवीय व्यवस्था है। शूद्रों और अस्पृश्यों को सबसे निचले स्तर पर रखा जाता था।

(iii) धर्मसूत्रों और धर्मशास्त्रों में चारों वर्गों के लिए आदर्श ‘जीविका’ से जुड़े कई नियम मिलते हैं। ब्राह्मणों का कार्य अध्ययन, वेदों की शिक्षा, यह करना और करवाना था तथा उनका काम दान देना और लेना था।

JAC Class 12 History Important Questions Chapter 3 बंधुत्व, जाति तथा वर्ग : आरंभिक समाज

(iv) क्षत्रियों का कर्म युद्ध करना, लोगों को सुरक्षा प्रदान करना, न्याय करना, वेद पढ़ना, यज्ञ करवाना और दान-दक्षिणा देना था। अन्तिम तीन कार्य वैश्यों के लिए भी थे साथ ही उनसे कृषि, गौ पालन और व्यापार का कर्म भी अपेक्षित था शूद्रों के लिए मात्र एक ही जीविका थी – तीनों ‘उच्च’ वर्णों की सेवा करना।

(v) इन नियमों का पालन करवाने के लिए ब्राह्मणों ने दो-तीन नीतियाँ अपनाई। पहली नीति में यह बताया गया हैं कि वर्णव्यवस्था की उत्पत्ति देवीय व्यवस्था है। दूसरा, वे शासकों को यह उपदेश देते थे कि वे इस व्यवस्था के नियमों का अपने राज्यों में अनुसरण करें। तीसरे, उन्होंने लोगों को यह विश्वास दिलाने का प्रयत्न किया कि उनकी प्रतिष्ठा जन्म पर आधारित है। किन्तु ऐसा करना आसान बात नहीं थी। अतः इन मानदण्डों को बहुधा महाभारत जैसे अनेक ग्रन्थों में वर्णित कहानियों के द्वारा बल प्रदान किया जाता था।

प्रश्न 11.
ब्राह्मणीय वर्ण व्यवस्था में अस्पृश्य वर्ग की क्या स्थिति थी? मनुस्मृति में वर्णित चाण्डालों के कर्त्तव्यों का वर्णन करें।
उत्तर:

  1. वर्ण-व्यवस्था वाली सामाजिक प्रणाली में ब्राह्मणों ने कुछ लोगों को अस्पृश्य घोषित किया था। ब्राह्मणों का मानना था कि कुछ कर्म, जिसमें विशेष रूप से ब्राह्मणों द्वारा सम्पन्न अनुष्ठान थे पुनीत और पवित्र होते थे। इन पवित्र कार्यों को करने वाले पवित्र लोग अस्पृश्यों से भोजन स्वीकार नहीं करते थे।
  2. पवित्रता के इस पहलू के ठीक विपरीत कुछ कार्य ऐसे थे जिन्हें खासतौर से ‘दृषित’ माना जाता था। शवों की अन्त्येष्टि और मृत पशुओं को छूने वालों को चाण्डाल कहा जाता था। उन्हें वर्ण व्यवस्था वाले समाज में सबसे निम्न कोटि में रखा जाता था।
  3. ब्राह्मण स्वयं को सामाजिक क्रम में सबसे ऊपर मानते थे, वे चाण्डालों का स्पर्श, यहाँ तक कि उन्हें देखना भी, अपवित्रकारी मानते थे।
  4. मनुस्मृति में चाण्डालों के ‘कर्तव्यों’ की सूची मिलती है। उन्हें गाँव के बाहर रहना होता था। वे फेंके हुए बर्तनों का इस्तेमाल करते थे, मरे हुए लोगों के वस्त्र तथा लोहे के आभूषण पहनते थे। रात्रि में वे गाँव और नगरों में चल-फिर नहीं सकते थे।
  5. सम्बन्धियों से विहीन मृतकों की उन्हें अंत्येष्टि करनी पड़ती थी तथा वधिक के रूप में भी कार्य करना होता था।
  6. लगभग पांचवीं शताब्दी ईसवी में चीन से आए बौद्ध भिक्षु फा शिएन का कहना है कि अस्पृश्यों को सड़क पर चलते हुए करताल बजाकर अपने होने की सूचना देनी पड़ती थी जिससे अन्य जन उन्हें देखने के दोष से बच जाएँ।
  7. लगभग सातवीं शताब्दी ईस्वी में भारत आए चीनी तीर्थयात्री श्वैन-त्सांग का कहना है कि वधिक और सफाई करने वालों को नगर से बाहर रहना पड़ता था।

प्रश्न 12.
ऋग्वैदिक काल में सामाजिक जीवन, पारिवारिक जीवन, विवाह, मनोरंजन के साधन आदि की क्या विशेषताएँ थीं? वर्णन कीजिए।
उत्तर:
ऋग्वैदिक काल में भारतीय आर्यों ने विभिन्न क्षेत्रों में पर्याप्त उन्नति की वे एक सभ्य और सुसंस्कृत जीवन पद्धति का विकास कर चुके थे और दूसरी ओर संसार की अन्य जातियाँ अपनी आदिम अवस्था में ही विद्यमान थीं।

इस काल की प्रमुख विशेषताएँ निम्न हैं –

(1) सामाजिक जीवन ऋग्वैदिककालीन आर्यों का जीवन पूर्णत: विकसित था, किन्तु ये लोग बहुत सादा जीवन व्यतीत करते थे। सम्पूर्ण समाज चार भागों में विभाजित था। ये वर्ण कहलाते थे। ब्राह्मण धर्म-दर्शन और विचारों के अधिष्ठाता थे विद्या का पठन-पाठन इनका मुख्य कार्य था क्षत्रिय लोगों पर समाज की रक्षा का भार था। वैश्य कृषि का कार्य करते थे तथा व्यापार व कला-कौशल में दक्षता प्राप्त करके समाज की समृद्धि की चिन्ता करते थे। शूद्रों का कार्य अन्य वर्णों की सेवा करना था। ये चारों वर्ण कर्मप्रधान थे एक ही परिवार में विभिन्न वर्गों के लोग रहते थे एक वर्ण से दूसरे में परिवर्तन सम्भव था। इन वर्णों में पारस्परिक विवाह सम्पन्न होते थे।

(2) पारिवारिक जीवन परिवार पितृप्रधान हुआ करते थे। इस काल में संयुक्त परिवार प्रणाली प्रचलित थी। पिता परिवार का मुखिया होता था। वह परिवार के सदस्यों में कार्य विभाजन करता था। पत्नी का स्थान भी महत्वपूर्ण होता था।

(3) विवाह वैदिक सभ्यता में विवाह एक धार्मिक संस्कार था। साधारणतः एक पत्नी की प्रथा प्रचलित थी। हालांकि कुलीन परिवारों में बहुपत्नी व बहुपति प्रथा के उदाहरण भी प्राप्त होते हैं। निकट सम्बन्धियों में विवाह नहीं होते थे। कन्याओं को स्वयंवर विवाह की अनुमति थी। साधारण परिवार की कन्याएँ भी स्वयंवर द्वारा स्वयं पति चुनती थीं।

JAC Class 12 History Important Questions Chapter 3 बंधुत्व, जाति तथा वर्ग : आरंभिक समाज

(4) वेश-भूषा तथा प्रसाधन-महाकाव्यकालीन लोग साधारण पोशाक ही धारण करते थे शरीर के ऊपरी भाग पर पहनने वाले वस्त्र को ऊर्ध्ववस्व तथा शरीर के निचले भाग पर धारण किए वस्त्र को अधोवस्य कहते थे। आभूषण स्वी और पुरुष दोनों ही पहनते थे। आभूषण सोने और चाँदी के होते थे जिनमें नग व मोती जड़े होते थे।

(5) मनोरंजन के साधन महाकाव्यकाल में मनोरंजन के प्रमुख साधन शिकार, नृत्य, संगीत, मल्लयुद्ध और गदायुद्ध आदि थे। जुआ भी खेला जाता था।

प्रश्न 13.
जाति प्रथा का विकास कैसा हुआ? सामाजिक विषमताओं के सन्दर्भ में इसकी व्याख्या कीजिए।
उत्तर:
प्राचीनकाल से ही भारत में किसी न किसी रूप में जाति प्रथा प्रचलित रही है। प्रारम्भ में वर्णों के आधार पर कुल चार जातियाँ थीं— ब्राह्मण, क्षत्रिय, वैश्य और शूद्र।

जाति प्रथा की उत्पत्ति के सम्बन्ध में विभिन्न मत हैं परन्तु निम्न दो मत तर्कसंगत हैं –
(1) धर्मसूत्रों और धर्म-ग्रन्थों के आधार पर ब्राह्मण ग्रन्थों के अनुसार, जैसेकि पुरुषसूक्त में वर्णन है कि जाति जन्म पर आधारित है, यह एक दैवीय व्यवस्था है। ब्रह्मा के मुख से ब्राह्मण, भुजाओं से क्षत्रिय, उदर से वैश्य तथा पैरों से शूद्रों की उत्पत्ति हुई।

(2) श्रम विभाजन के आधार पर आधुनिक इतिहासकारों का मत है कि जाति प्रथा का आरम्भ श्रम- विभाजन के कारण हुआ। शक्तिशाली तथा सामर्थ्यवान लोगों को देश की रक्षा का भार सौंपा गया; अतः योद्धाओं की एक विशेष जाति क्षत्रिय की उत्पत्ति हुई। आनुष्ठानिक धार्मिक कार्य और वेदाध्ययन करने वाले लोगों को ब्राह्मण कहा गया। व्यापारिक कार्यों तथा कृषि और पशुपालन जैसे कार्यों को करने वालों को वैश्य का दर्जा दिया गया। जो लोग इन सब कार्यों को करने का सामर्थ्य नहीं रखते थे उन्हें तीनों जातियों की सेवा का कार्य दिया गया और वे शूद्र कहलाए।

जाति प्रथा का सामाजिक विषमताओं में योगदान- आरम्भिक चरणों में जाति प्रथा इतनी कठोर नहीं थी क्योंकि यह जातीय भेद व्यवसायों पर आधारित था परन्तु बाद में जन्म आधारित मान्यता मिल जाने पर जाति प्रथा कठोर होती चली गई और इसके द्वारा समाज में बहुत-सी विषमताएँ उत्पन्न हुई –

  1. समाज में छुआछूत की भावना फैली। ब्राह्मण, क्षत्रिय, वैश्य ये तीनों वर्ण शूद्रों को स्वयं से हीन समझने लगे, यहाँ तक कि कुछ लोगों को ‘अस्पृश्य’ घोषित कर दिया गया।
  2. ब्राह्मण अपने-आपको तथा क्षत्रिय स्वयं को सभी जातियों से उच्च समझते थे, इस कारण समाज में विद्वेष की भावना सम्पन्न हुई।
  3. शिक्षा का अधिकार केवल ब्राह्मणों के पास ही था, सैनिक शिक्षा क्षत्रियों तक ही सीमित रही।
  4. विभिन्न प्रकार के कर्मकाण्ड, सामाजिक कुरीतियाँ समाज में प्रचलित हो गयीं।
  5. लोगों की राष्ट्रीय भावना में कमी आई एवं लोग अपने जातिगत स्वार्थी को पूरा करने में लग गये। इन सब कारणों से समाज की एकजुटता में कमी आई जिसका लाभ विदेशी शासकों ने उठाया।

प्रश्न 14.
महाभारत काल में लैंगिक असमानता के आधार पर सम्पत्ति के अधिकारों की व्याख्या कीजिए।
उत्तर:
महाभारत काल में स्वी की स्थिति तथा लैंगिक असमानता के कारण स्त्रियों को सामाजिक, आर्थिक, राजनीतिक आदि क्षेत्रों में पुरुषों के समान अधिकार प्राप्त युक्त नहीं थे। सम्पत्ति के अधिकारों की क्रमबद्ध व्याख्या निम्न है –

(1) वस्तु की भाँति पत्नी को निजी सम्पत्ति मानना इसका उदाहरण द्यूतक्रीड़ा के प्रकरण में मिलता है पाण्डव अपनी सारी सम्पत्ति कौरवों के साथ घृत- क्रीड़ा में चौसर के खेल में हार गए। अन्त में पाण्डवों ने अपनी साझी पत्नी द्रौपदी को भी निजी सम्पत्ति मानकर दाँव पर लगा दिया और उसे भी हार गए। स्त्री को सम्पत्ति मात्र समझने के कई उदाहरण धर्मशास्त्रों और धर्मसूत्रों में प्राप्त होते हैं।

(2) मनुस्मृति के अनुसार सम्पत्ति का बँटवारा- मनुस्मृति में सम्पत्ति के बँटवारे के सम्बन्ध में वर्णित है कि माता-पिता की सम्पत्ति में उनकी मृत्यु के पश्चात् पुत्रों का समान अधिकार होता है इसलिए पैतृक सम्पत्ति का सभी पुत्रों में समान बँटवारा करना चाहिए। माता-पिता की सम्पत्ति में पुत्रियों का कोई अधिकार नहीं था।

JAC Class 12 History Important Questions Chapter 3 बंधुत्व, जाति तथा वर्ग : आरंभिक समाज

(3) स्त्री धन स्वियों हेतु धन की व्यवस्था स्वी धन के रूप में की गई थी। स्वी को विवाह के समय जो उपहार उसे परिवार और कुटुम्बजनों से प्राप्त होते थे; उन पर उनका अधिकार होता था उस पर पति का अधिकार भी नहीं होता था।

(4) सम्पत्ति अर्जन के पुरुषों के अधिकार – मनुस्मृति के अनुसार पुरुषों के लिये धन अर्जन के सात तरीके हैं; जैसे— विरासत, खोज, खरीद, विजित करके, निवेश, काम द्वारा तथा सज्जनों से प्राप्त भेंट

(5) उच्च वर्ग की महिलाओं की विशेष स्थिति- उच्च वर्ग की महिलाओं की स्थिति बेहतर थी। उच्च वर्ग की महिलाएँ संसाधनों पर अप्रत्यक्ष रूप से अपना प्रभुत्व रखती थीं। दक्षिण भारत में महिलाओं की स्थिति रस सम्बन्ध में और भी अच्छी थी। वाकाटक वंश की रानी चन्द्रगुप्त द्वितीय की पुत्री प्रभावती गुप्त अत्यन्त प्रभावशाली महिला थी; उनके सम्पत्ति के अधिकारों के स्वामित्व का प्रमाण इस बात से मिलता है कि उन्होंने अपने अधिकार से एक ब्राह्मण को भूमिदान किया था।

JAC Class 12 Political Science Solutions Chapter 9 भारतीय राजनीति : नए बदलाव

Jharkhand Board JAC Class 12 Political Science Solutions Chapter 9 भारतीय राजनीति : नए बदलाव Textbook Exercise Questions and Answers.

JAC Board Class 12 Political Science Solutions Chapter 9 भारतीय राजनीति : नए बदलाव

Jharkhand Board Class 12 Political Science भारतीय राजनीति : नए बदलाव InText Questions and Answers

पृष्ठ 174

प्रश्न 1
‘अगर हर सरकार एक-सी नीति पर अमल करे तो मुझे नहीं लगता कि इससे राजनीति में कोई बदलाव आयेगा।’
उत्तर:
यदि सभी सरकारें या उनसे सम्बद्ध राजनीतिक दल एक ही प्रकार की नीतियाँ अपनाएं तो इससे राजनीतिक व्यवस्था स्थिर व जड़ हो जायेगी। लेकिन लोकतान्त्रिक व्यवस्था में इस प्रकार की नीति लागू होना सम्भव नहीं है क्योंकि लोकतान्त्रिक व्यवस्था में जनमत सर्वोपरि होता है और जनसामान्य के हित अलग-अलग होते हैं और यह हित समय और परिस्थितियों के अनुसार निरन्तर परिवर्तित भी होते रहते हैं, इसलिए प्रत्येक सरकार को जन इच्छाओं को ध्यान में रखते हुए कार्य करना पड़ता है।

ऐसी स्थिति में सभी सरकारें एक जैसी नीति का अनुसरण नहीं कर सकतीं। इसके अतिरिक्त भारत जैसे बहुदलीय व्यवस्था वाले देश में तो इस प्रकार की नीति लागू करना बिल्कुल भी सम्भव नहीं है क्योंकि भारत में प्रत्येक दल की विचारधारा व कार्यक्रमों में व्यापक अन्तर है और ये राजनीतिक दल सत्ता में आने पर अलग-अलग ढंग से निर्णय लेते हैं।

पृष्ठ 180

प्रश्न 2.
चलो मान लिया कि भारत जैसे देश में लोकतान्त्रिक राजनीति का तकाजा ही गठबन्धन बनाना है। लेकिन क्या इसका मतलब यह निकाला जाए कि हमारे देश में हमेशा से गठबन्धन बनते चले आ रहे हैं। अथवा, राष्ट्रीय स्तर के दल एक बार फिर से अपना बुलंद मुकाम हासिल करके दिखाएंगे।
उत्तर:
भारतीय राजनीतिक व्यवस्था में गठबन्धन का दौर हमेशा से चला आ रहा है । यह बात कुछ हद तक सहीं है, लेकिन इन गठबन्धनों के स्वरूप में व्यापक अन्तर है। पहले जहाँ एक ही पार्टी के भीतर गठबन्धन होता था अब पार्टियों के बीच गठबन्धन होता है। जहाँ तक राष्ट्रीय दलों के प्रभुत्व का सवाल है, वर्तमान दलीय व्यवस्था के बदलते दौर में अपना बुलंद मुकाम पाना बहुत कठिन है। क्योंकि वर्तमान में भारतीय दलीय व्यवस्था का स्वरूप बहुदलीय हो गया है जिसमें राष्ट्रीय दलों के साथ-साथ क्षेत्रीय दलों के महत्त्व को भी नकारा नहीं जा सकता। यही कारण है कि भारत में नब्बे के दशक से गठबन्धन सरकारों का सिलसिला चला आ रहा है।

JAC Class 12 Political Science Solutions Chapter 9 भारतीय राजनीति : नए बदलाव

प्रश्न 3.
मुझे इसकी चिन्ता नहीं है कि सरकार किसी एक पार्टी की है या गठबन्धन की। मसला तो यह है कि कोई सरकार काम कौनसे कर रही है? क्या गठबन्धन सरकार में ज्यादा समझौते करने पड़ते हैं? क्या गठबन्धन सरकार साहसी और कल्पनाशील नीतियाँ नहीं अपना सकती?
उत्तर:
सरकारों का स्वरूप चाहे कैसा भी हो। चाहे वह गठबन्धन सरकार हो या एक ही दल की सरकार हो लेकिन सरकार जनता की कसौटियों पर खरी उतरे वही सफल सरकार है। गठबन्धन सरकारों में विभिन्न दल आपसी समझौतों या शर्तों के आधार पर सरकार का गठन करते हैं। इन दलों के सभी के अपने-अपने हित एवं स्वार्थ होते हैं जिन्हें पूरा करने हेतु निरन्तर प्रयास करते रहते हैं।

जहाँ आपसी हितों में रुकावट या टकराव आता है वहीं दल अलग हो जाते हैं और सरकारें गिर जाती हैं। इस प्रकार गठबन्धन सरकारों में स्थिरता बहुत कम पायी जाती है। इस प्रकार की सरकारों में स्वतन्त्र निर्णय लेना सम्भव नहीं है। इस प्रकार गठबन्धन सरकार साहसी और कल्पनाशील नीतियाँ नहीं अपना सकती।

पृष्ठ 183

प्रश्न 4.
क्या इससे पिछड़े और दलित समुदायों के सभी नेताओं को लाभ होगा या इन समूहों के भीतर मौजूद कुछ ताकतवर जातियाँ और परिवार ही सारे फायदे अपनी मुट्ठी में कर लेंगे?
उत्तर:
इससे पिछड़े और दलित समुदायों के सभी नेताओं का लाभ तो शायद ही हो अपितु हमने यह देखा है कि ऐसे संगठनों में मौजूद कुछ नेता अत्यधिक ताकतवर और रसूखवाले हो जाते हैं।

प्रश्न 5.
असल मुद्दा नेताओं का नहीं, जनता का है! क्या इस बदलाव से सचमुच के वंचितों के लिए बेहतर नीतियाँ बनेंगी और उन पर कारगर तरीके से अमल होगा या फिर यह सारा कुछ एक राजनीतिक खेल मात्र बनकर रह जाएगा?
उत्तर:
हम इस बात को पूरी तरह नकार नहीं सकते कि ऐसे बदलावों से वंचितों के लिए कोई बेहतर नीतियाँ नहीं बनीं और उन पर कारगर तरीके से अमल होगा। परंतु कई बार नेताओं ने ऐसे नीतियों का फायदा जरूरतमंदों तक पूरा-पूरा नहीं पहुँचने दिया।

पृष्ठ 188

प्रश्न 6.
क्या हम सुनिश्चित कर सकते हैं कि जो लोग ऐसे जनसंहार की योजनाएँ बनाएँ, अमल करें और उसे समर्थन दें। वे कानून के हाथों से बच न पाएँ? ऐसे लोगों को कम-से-कम राजनीतिक रूप से तो सबक सिखाया ही जा सकता है।
उत्तर:
भारत में धर्म, जाति व सम्प्रदाय के नाम पर अनेक बार साम्प्रदायिक दंगे हुए। इन साम्प्रदायिक दंगों के पीछे प्रत्यक्ष या अप्रत्यक्ष रूप से विभिन्न राजनीतिक दलों का हाथ रहा है। ये दल अपने राजनीतिक स्वार्थों की पूर्ति हेतु दुष्प्रचार करते हैं जिससे जनसामान्य इनके बहकावे में आकर गलत कदम उठाते हैं। भारत में 1984 के सिख दंगे हों, अयोध्या की घटना हो या 2002 में गुजरात का गोधरा काण्ड, इन सभी घटनाओं के पीछे राजनीतिक दलों की स्वार्थपूर्ण नीति रही है।

इन घटनाओं को रोकने के लिए आवश्यक है कि ऐसे राजनीतिक दल व उनके नेतृत्वकर्ता जिनका आपराधिक रिकार्ड रहा है या साम्प्रदायिक दंगों में लिप्त रहे हैं उन पर चुनावी प्रक्रिया में भाग लेने पर आजीवन प्रतिबन्ध लगा दिया जाना चाहिए। जो लोग नरसंहार की योजना बनाएँ या उस पर अमल करें उन्हें कानून द्वारा सख्त सजा दी जानी चाहिए।

Jharkhand Board Class 12 Political Science भारतीय राजनीति : नए बदलाव TextBook Questions and Answers

प्रश्न 1.
उन्नी – मुन्नी ने अखबार की कुछ कतरनों को बिखेर दिया है। आप इन्हें कालक्रम के अनुसार व्यवस्थित करें।
(क) मंडल आयोग की सिफारिशें और आरक्षण विरोधी हंगामा
(ख) जनता दल का गठन
(ग) बाबरी मस्जिद का विध्वंस
(घ) इन्दिरा गांधी की हत्या
(ङ) राजग सरकार का गठन
(च) संप्रग सरकार का गठन
(छ) गोधरा की दुर्घटना और उसके परिणाम
उत्तर:
(क) इन्दिरा गांधी की हत्या (1984)
(ख) जनता दल का गठन (1988)
(ग) मण्डल आयोग की सिफारिश और आरक्षण विरोधी हंगामा (1990)
(घ) बाबरी मस्जिद का विध्वंस (1992)
(ङ) राजग सरकार का गठन (1999)
(च) गोधरा दुर्घटना और उसके परिणाम (2002)
(छ) संप्रग सरकार का गठन (2004)।

JAC Class 12 Political Science Solutions Chapter 9 भारतीय राजनीति : नए बदलाव

प्रश्न 2.
निम्नलिखित में मेल करें-

(क) सर्वानुमति की राजनीति (i) शाहबानो मामला
(ख) जाति आधारित दल (ii) अन्य पिछड़ा वर्ग का उभार
(ग) पर्सनल लॉ और लैंगिक न्याय (iii) गठबन्धन सरकार
(घ) क्षेत्रीय पार्टियों की बढ़ती ताकत (iv) आर्थिक नीतियों पर सहमति

उत्तर:

(क) सर्वानुमति की राजनीति (iv) आर्थिक नीतियों पर सहमति
(ख) जाति आधारित दल (ii) अन्य पिछड़ा वर्ग का उभार
(ग) पर्सनल लॉ और लैंगिक न्याय (i) शाहबानो मामला
(घ) क्षेत्रीय पार्टियों की बढ़ती ताकत (iii) गठबन्धन सरकार

प्रश्न 3.
1989 के बाद की अवधि में भारतीय के आपसी जुड़ाव के क्या रूप सामने आये हैं?
उत्तर:
1989 के बाद भारतीय राजनीति के मुख्य मुद्दे – 1989 के बाद भारतीय राजनीति में कई बदलाव आये जिनमें कांग्रेस का कमजोर होना, मण्डल आयोग की सिफारिशें एवं आन्दोलन, आर्थिक सुधारों को लागू करना, राजीव गांधी की हत्या तथा अयोध्या मामला प्रमुख हैं । इन स्थितियों में भारतीय राजनीति में जो मुद्दे प्रमुख रूप से उभरे उनसे राजनीतिक दलों का आपसी जुड़ाव निम्न रूप में सामने आया-

  1. कांग्रेस ने स्थिर सरकार का मुद्दा उठाकर कहा कि देश में स्थिर सरकार कांग्रेस ही दे सकती है।
  2. भाजपा ने राम मंदिर बनाने का मुद्दा उठाकर हिन्दू मतों को अपने पक्ष में कर अपने जनाधार को ग्रामीण क्षेत्रों में व्यापक करने का प्रयास किया।
  3. लोकदल व जनता दल ने मंडल आयोग की सिफारिशों का मुद्दा उठाकर पिछड़ी जातियों को अपने पक्ष में लामबंद करने का प्रयास किया ।

प्रश्न 4.
“गठबन्धन की राजनीति के इस दौर में राजनीतिक दल विचारधारा को आधार मानकर गठजोड़ नहीं करते हैं।” इस कथन के पक्ष या विपक्ष में आप कौन-कौनसे तर्क देंगे?
उत्तर:
पक्ष में तर्क- वर्तमान युग में गठबन्धन की राजनीति का दौर चल रहा है। इस दौर में राजनीतिक दल विचारधारा को आधार बनाकर गठजोड़ नहीं कर रहे बल्कि अपने निजी राजनैतिक हितों की पूर्ति के लिए गठजोड़ करते हैं। इस बात के समर्थन में निम्नलिखित तर्क दिये जा सकते हैं

  1. 1977 में जे. पी. नारायण के आह्वान पर जो जनता दल बना था उसमें कांग्रेस के विरोधी प्रायः सी. पी. आई. को छोड़कर अधिकांश विपक्षी दल शामिल थे। इन सभी दलों को हम एक ही विचारधारा वाले दल नहीं कह सकते।
  2. जनता दल की सरकार गिरने के बाद केन्द्र में राष्ट्रीय मोर्चा बना जिसमें एक ओर जनता पार्टी के वी. पी. सिंह तो दूसरी ओर उन्हें समर्थन देने वाले सी. पी. एम. वामपंथी और भाजपा जैसे तथाकथित हिन्दुत्व समर्थक गांधीवादी राष्ट्रवादी दल भी थे।
  3. कांग्रेस की सरकार, 1991 से 1996 तक नरसिंह राव के नेतृत्व में अल्पमत होते हुए भी इसलिए चलती रही क्योंकि उसे अनेक ऐसे दलों का समर्थन प्राप्त था जिससे तथाकथित साम्प्रदायिक शक्तियाँ सत्ता में न आ सकें।
  4. अटल बिहारी वाजपेयी के नेतृत्व में बनी जनतांत्रिक गठबन्धन ( एन. डी.ए) की सरकार को अकालियों, तृणमूल कांग्रेस, बीजू पटनायक कांग्रेस, समता दल, जनता पार्टी तथा अनेक क्षेत्रीय दलों ने भी सहयोग और समर्थन दिया।

विपक्ष में तर्क- उपर्युक्त कथन के विपक्ष में निम्नलिखित तर्क दिये जा सकते हैं-

  1. गठबन्धन की राजनीति के नए दौर में भी वामपंथी दलों ने भारतीय जनता पार्टी से हाथ नहीं मिलाया, वे उसे अब भी राजनीतिक दृष्टि से अस्पर्शीय पार्टी मानते हैं।
  2. समाजवादी पार्टी, वामपंथी मोर्चा, डी. पी. के. जैसे क्षेत्रीय दल किसी भी उस प्रत्याशी को खुला समर्थन नहीं देना चाहते जो एन. डी. ए. अथवा भाजपा का प्रत्याशी हो।
  3. कांग्रेस पार्टी ने अधिकांश मोर्चों पर बीजेपी विरोधी और बीजेपी ने कांग्रेस विरोधी रुख अपनाया है

JAC Class 12 Political Science Solutions Chapter 9 भारतीय राजनीति : नए बदलाव

प्रश्न 5.
आपातकाल के बाद के दौर में भाजपा एक महत्त्वपूर्ण शक्ति के रूप में उभरी। इस दौर में इस पार्टी के विकास क्रम का उल्लेख करें।
उत्तर:
भारतीय जनता पार्टी का विकास-क्रम – आपातकाल के पश्चात् भाजपा की विकास यात्रा को निम्न प्रकार से समझा जा सकता है-

  1. जनता पार्टी सरकार के पतन के बाद जनता पार्टी के भारतीय जनसंघ घटक ने वर्ष 1980 में अटल बिहारी वाजपेयी की अध्यक्षता में भारतीय जनता पार्टी का गठन किया।
  2. वर्ष 1984 के चुनावों में भाजपा को लोकसभा में केवल दो सीटें प्राप्त हुईं।
  3. सन् 1989 के चुनावों में भाजपा ने चुनाव में राम मंदिर बनवाने के नारे को उछाला। फलतः इस चुनाव में भाजपा को आशा से अधिक सफलता मिली। भाजपा ने वी. पी. सिंह को बाहर से समर्थन देकर संयुक्त मोर्चा सरकार का गठन करने में सहयोग दिया।
  4. 1991 के चुनाव में भी राम मंदिर निर्माण का नारा भाजपा के लिए विशेष लाभदायक सिद्ध हुआ।
  5. 1996 के चुनावों में यह लोकसभा में सबसे बड़ी पार्टी के रूप में उभरी।
  6. सन् 1998 के चुनावों में इसने कुछ क्षेत्रीय दलों से गठबंधन कर सरकार बनाई तथा 1999 के चुनावों में भाजपानीत गठबंधन ने फिर सत्ता प्राप्त की। राष्ट्रीय जनतांत्रिक गठबंधन के काल में अटल बिहारी वाजपेयी प्रधानमंत्री बने।
  7. सन् 2004 में तथा 2009 के चुनावों में भाजपा को पुनः अपेक्षित सफलता नहीं मिल पायी । फिर भी कांग्रेस के बाद आज यह दूसरी सबसे बड़ी पार्टी है।
  8. 2014 के लोकसभा चुनावों में भाजपा को लोकसभा में स्पष्ट बहुमत प्राप्त हुआ तथा नरेन्द्र मोदी के नेतृत्व में सरकार बनाई।

प्रश्न 6.
कांग्रेस के प्रभुत्व का दौर समाप्त हो गया है। इसके बावजूद देश की राजनीति पर कांग्रेस का असर लगातार कायम है। क्या आप इस बात से सहमत हैं? अपने उत्तर के पक्ष में तर्क दीजिए।
उत्तर:
देश की राजनीति से यद्यपि कांग्रेस का प्रभुत्व समाप्त हो गया है परन्तु अभी कांग्रेस का असर कायम है, क्योंकि अब भी भारतीय राजनीति कांग्रेस के इर्द-गिर्द घूम रही है तथा सभी राजनीतिक दल अपनी नीतियाँ एवं योजनाएँ कांग्रेस को ध्यान में रखकर बनाते हैं। अभी भी कांग्रेस पार्टी दूसरी सबसे बड़ी राष्ट्रीय पार्टी है। 2004 के 14वें लोकसभा के चुनावों में इसने अन्य दलों के सहयोग से केन्द्र सरकार बनाई।

इसके साथ-साथ जुलाई, 2007 में हुए राष्ट्रपति के चुनाव में भी इस दल की महत्त्वपूर्ण भूमिका रही। वर्ष 2009 के आम चुनावों में पहले से काफी अधिक सीटों पर जीत प्राप्त कर गठबंधन की सरकार बनाई । 2014 के लोकसभा चुनावों में यद्यपि उसे 44 ही सीटें प्राप्त हुईं तथापि उसके मत प्रतिशत में बहुत अधिक अन्तर नहीं आया है। अतः कहा जा सकता है कि कमजोर होने के बावजूद भी कांग्रेस का असर भारतीय राजनीति पर कायम है।

प्रश्न 7.
अनेक लोग सोचते हैं कि सफल लोकतन्त्र के लिए दो दलीय व्यवस्था जरूरी है। पिछले बीस सालों के भारतीय अनुभवों को आधार बनाकर एक लेख लिखिए और इसमें बताइए कि भारत की मौजूदा बहुदलीय व्यवस्था के क्या फायदे हैं?
उत्तर:
कुछ लोगों का मानना है कि सफल लोकतन्त्र के लिए दो दलीय व्यवस्था जरूरी है। इनका मानना है कि द्विदलीय व्यवस्था में साधारण बहुमत के दोष समाप्त हो जाते हैं, सरकार स्थायी होती है, भ्रष्टाचार कम फैलता है, निर्णय शीघ्रता से लिये जा सकते हैं। भारत में बहुदलीय प्रणाली भारत में बहुदलीय प्रणाली है।

कई विद्वानों का मत है कि भारत में बहुदलीय प्रणाली उचित ढंग से कार्य नहीं कर पा रही है तथा यह भारतीय लोकतन्त्र के लिए बाधा उत्पन्न कर रही है अतः भारत को द्वि-दलीय पद्धति अपनानी चाहिए परन्तु पिछले बीस सालों के अनुभव के आधार पर यह कहा जा सकता है कि बहुदलीय प्रणाली से भारतीय राजनीतिक व्यवस्था को अग्रलिखित फायदे हुए हैं-

  1. विभिन्न मतों का प्रतिनिधित्व: बहुदलीय प्रणाली के कारण भारतीय राजनीति में सभी वर्गों तथा हितों को प्रतिनिधित्व मिल जाता है।
  2. मतदाताओं को अधिक स्वतन्त्रता: अधिक दलों के कारण मतदाताओं को अपने वोट का प्रयोग करने के लिए अधिक विकल्प प्राप्त होते हैं।
  3. राष्ट्र दो गुटों में नहीं बँटता:  बहुदलीय प्रणाली होने के कारण भारत कभी भी दो विरोधी गुटों में विभाजित नहीं हुआ।
  4. मन्त्रिमण्डल की तानाशाही स्थापित नहीं होती: बहुदलीय प्रणाली के कारण भारत में मन्त्रिमण्डल तानाशाह नहीं बन सकता।
  5. अनेक विचारधाराओं का प्रतिनिधित्व:  बहुदलीय प्रणाली में व्यवस्थापिका में देश की अनेक विचारधाराओं का प्रतिनिधित्व हो सकता है

JAC Class 12 Political Science Solutions Chapter 9 भारतीय राजनीति : नए बदलाव

प्रश्न 8.
निम्नलिखित अवतरण को पढ़ें और इसके आधार पर पूछे गये प्रश्नों के उत्तर दें-
भारत की दलगत राजनीति ने कई चुनौतियों का सामना किया है। कांग्रेस प्रणाली ने अपना खात्मा ही नहीं किया बल्कि कांग्रेस के जमावड़े के बिखर जाने से आत्म-प्रतिनिधित्व की नयी प्रवृत्ति का भी जोर बढ़ा। इससे दलगत व्यवस्था और विभिन्न हितों की समाई करने की इसकी क्षमता पर भी सवाल उठे राजव्यवस्था के सामने एक महत्त्वपूर्ण काम एक ऐसी दलगत व्यवस्था खड़ी करने अथवा राजनीतिक दलों को गढ़ने की है, जो कारगर तरीके से विभिन्न हितों को मुखर और एकजुट करें – जोया हसन
(क) इस अध्याय को पढ़ने के बाद क्या आप दलगत व्यवस्था की चुनौतियों की सूची बना सकते हैं?
(ख) विभिन्न हितों का समाहार और उनसे एकजुटता का होना क्यों जरूरी है?
(ग) इस अध्याय में आपने अयोध्या विवाद के बारे में पढ़ा। इस विवाद ने भारत के राजनीतिक दलों
की समाहार की क्षमता के आगे क्या चुनौती पेश की?
उत्तर:
(क) इस अध्याय में दलगत व्यवस्था की निम्नलिखित चुनौतियाँ उभर कर सामने आती हैं-

  1. गठबन्धन की राजनीति को चलाना।
  2. क्षेत्रीय दलों का उभरना तथा राजनीतिक भ्रष्टाचार का बढ़ना।
  3. पिछड़े वर्गों की राजनीति का उभरना।
  4. अयोध्या विवाद का उभरना।
  5. गैर सैद्धान्तिक राजनीतिक समझौते का होना।
  6. साम्प्रदायिक दंगे होना।

(ख) विभिन्न हितों का समाहार और उनमें एकजुटता का होना जरूरी है, क्योंकि तभी भारत अपनी एकता और अखण्डता को बनाए रखकर विकास कर सकता है।

(ग) अयोध्या विवाद ने भारत में राजनीतिक दलों के सामने साम्प्रदायिकता की चुनौती पेश की तथा भारत में साम्प्रदायिक आधार पर राजनीतिक दलों की राजनीति बढ़ गई।

भारतीय राजनीति : नए बदलाव JAC Class 12 Political Science Notes

→ 1990 का दशक:
1984 में इन्दिरा गांधी की हत्या के बाद राजीव गांधी प्रधानमंत्री बने तथा 1984 के लोकसभा चुनावों में श्रीमती गांधी की हत्या के बाद सहानुभूतिस्वरूप भारतीय जनता ने कांग्रेस पार्टी को भारी बहुमत से विजयी बनाया। 1980 के दशक के आखिर के सालों में देश में पाँच ऐसे बदलाव आये जिनका देश की राजनीति पर गहरा प्रभाव पड़ा।

  • 1990 के दशक में एक महत्त्वपूर्ण घटना 1989 के चुनावों में कांग्रेस की हार है। 1989 में ही ‘कांग्रेस प्रणाली’ की समाप्ति हो गई।
  • दूसरा बड़ा बदलाव राष्ट्रीय राजनीति में मण्डल मुद्दे का उदय था। 1990 में राष्ट्रीय मोर्चा की नई सरकार ने मंडल आयोग की सिफारिशों को लागू किया। इन सिफारिशों के अन्तर्गत प्रावधान किया गया कि केन्द्र सरकार की नौकरियों में अन्य पिछड़ा वर्ग को आरक्षण दिया जाएगा।
  • विभिन्न सरकारों ने इस दौर में जो आर्थिक नीतियाँ अपनाईं वह बुनियादी तौर पर बदल चुकी थीं। इसे ढांचागत समायोजन कार्यक्रम अथवा नये आर्थिक प्रकार के नाम से जाना जाता है।
  • इन्हीं घटनाओं के सिलसिले में एक और घटना अयोध्या स्थित एक विवादित ढाँचे (बाबरी मस्ज़िद के रूप में प्रसिद्ध) के विध्वंस के सम्बन्ध में है। यह घटना 1992 में दिसम्बर महीने में घटी। इस घटना ने देश की राजनीति में कई परिवर्तनों को जन्म दिया।
  • मई, 1991 में राजीव गांधी की हत्या हो गई और इसके परिणामस्वरूप कांग्रेस पार्टी के नेतृत्व में परिवर्तन हुआ। 1991 के चुनावों में कांग्रेस सबसे बड़ी पार्टी के रूप में सामने आयी। राजीव गांधी के बाद नरसिम्हा राव ने कांग्रेस पार्टी की बागडोर सम्भाली।

→ गठबन्धन का युग:
1989 के चुनावों में कांग्रेस पार्टी की हार हुई। कांग्रेस की हार के बावजूद भी यह सबसे बड़ी पार्टी थी लेकिन बहुमत न मिलने के कारण उसने विपक्ष में बैठने का फैसला किया। राष्ट्रीय मोर्चा को (यह मोर्चा जनता दल और कुछ अन्य क्षेत्रीय दलों को मिलाकर बनाया) परस्पर विरुद्ध दो राजनीतिक समूहों भाजपा और वाममोर्चा ने बाहर से समर्थन दिया।

→ काँग्रेस का पतन:
काँग्रेस की हार के साथ भारत की दलीय व्यवस्था से उसका दबदबा खत्म हो गया। 1960 के दशक के अंतिम सालों में काँग्रेस के एकछत्र राज को चुनौती मिली थी, लेकिन इंदिरा गाँधी के नेतृत्व में कांग्रेस ने भारतीय राजनीति पर अपना प्रभुत्व फिर से कायम किया। 90 के दशक में काँग्रेस की अग्रणी हैसियत को एक बार फिर चुनौती मिली। इस दौर में काँग्रेस के दबदबे के खात्मे के साथ बहुदलीय शासन प्रणाली का युग शुरू हुआ। 1989 के. बाद से लोकसभा के चुनावों में कभी भी किसी पार्टी को 2014 तक पूर्ण बहुमत नहीं मिला।

→ गठबन्धन की राजनीति:
1989 के बाद लोकसभा के चुनावों में कभी भी किसी एक पार्टी को पूर्ण बहुमत नहीं मिला। इस प्रकार केन्द्र में गठबन्धन की राजनीति प्रारम्भ हुई। इसके साथ ही नब्बे का दशक ताकतवर पार्टियों और आन्दोलनों के उभार का साक्षी रहा। इन पार्टियों और आन्दोलनों ने दलित तथा पिछड़ा वर्ग (अन्य पिछड़ा वर्ग या ओ.बी.सी.) की नुमाइंदगी की। इन दलों में से अनेक ने क्षेत्रीय आकांक्षाओं की दमदार दावेदारी की। 1996 में बनी संयुक्त मोर्चा सरकार में इन पार्टियों ने अहम भूमिका निभाई।

→ भाजपा का उभार:
1991 तथा 1996 के चुनावों में भारतीय जनता पार्टी ने भी अपनी स्थिति काफी मजबूत कर ली। 1996 के चुनावों में यह सबसे बड़ी पार्टी के रूप में उभरी। इस नाते भाजपा को सरकार बनाने का निमन्त्रण मिला लेकिन अधिकांश दल भाजपा की नीतियों एवं कार्यक्रमों के खिलाफ थे । इस तरह भाजपा लोकसभा में बहुमत प्राप्त नहीं कर पायी। आखिरकार भाजपा एक गठबन्धन (राजग) के अगुआ के रूप में सत्ता में आई और 1998 के मई से 1999 के जून तक सत्ता में रही। फिर 1999 में इस गठबन्धन ने दोबारा सत्ता हासिल की। 1999 की वाजपेयी सरकार ने अपना निर्धारित कार्यकाल पूरा किया। अन्य पिछड़ा वर्ग का राजनीतिक उदय – ‘ अन्य पिछड़ा वर्ग’ में शैक्षणिक और सामाजिक रूप से पिछड़े समुदाय की गणना की जाती है।

→ मंडल का लागू होना:
1980 के दशक में अन्य पिछड़ा वर्गों के बीच लोकप्रिय ऐसे ही राजनीतिक समूहों को जनता दल ने एकजुट किया। राष्ट्रीय मोर्चा की सरकार ने मंडल आयोग की सिफारिशों को लागू करने में महत्त्वपूर्ण भूमिका निभाई। इससे अन्य पिछड़ा वर्ग की राजनीति को सुगठित रूप देने में मदद मिली।

JAC Class 12 Political Science Solutions Chapter 9 भारतीय राजनीति : नए बदलाव

→ राजनीतिक परिणाम:
1980 के दशक में दलित जातियों के राजनीतिक संगठनों का भी उभार हुआ। 1978 में बामसेफ (बेकवर्ड एवं माइनॉरिटी क्लासेज एम्पलाइज फाउंडेशन) का गठन हुआ। इस संगठन ने बहुजन यानी अनुसूचित जाति, अनुसूचित जनजाति, अन्य पिछड़ा वर्ग और अल्पसंख्यकों की राजनीतिक सत्ता की जबरदस्त तरफदारी की। कांशीराम के नेतृत्व वाली बसपा ने अपने संगठन की बुनियाद व्यवहार में बामसेफ की नीतियों पर केन्द्रित की। साम्प्रदायिकता, धर्मनिरपेक्षता, लोकतन्त्र – इस दौर में आया एक दूरगामी बदलाव धार्मिक पहचान पर आधारित राजनीति का उदय है।

इसने धर्मनिरपेक्षता और लोकतन्त्र के बारे में बहसों को सरगर्म किया। शुरू-शुरू में भाजपा ने जनसंघ की अपेक्षा कहीं ज्यादा बड़ा राजनीतिक मंच अपनाया इसने गाँधीवादी समाजवाद को अपनी विचारधारा के रूप में स्वीकार किया। लेकिन भाजपा को 1980 और 1984 के चुनावों में खास सफलता नहीं मिली। 1986 के बाद इस पार्टी ने अपनी विचारधारा में हिन्दू राष्ट्रवाद के तत्त्वों पर जोर देना शुरू किया। भाजपा ने हिन्दुत्व की राजनीति का रास्ता चुना और हिन्दुओं को लामबन्द करने की नीति अपनाई।

→ अयोध्या विवाद का निर्णय:
फैजाबाद जिला न्यायालय द्वारा फरवरी, 1986 में अयोध्या विवाद पर फैसला सुनाया गया। इस अदालत ने यह फैसला दिया कि रामजन्म भूमि बाबरी मस्जिद के अहाते का ताला खोला जाना चाहिए ताकि हिन्दू यहाँ पूजा-पाठ कर सकें क्योंकि वे इस जगह को पवित्र मानते हैं। विध्वंस और उसके बाद- राम मंदिर निर्माण का समर्थन करने वाले संगठनों ने दिसम्बर, 1992 में कारसेवा का आयोजन किया। इसके अन्तर्गत रामभक्तों का आह्वान किया गया कि वे राम मन्दिर के निर्माण में श्रमदान करें। लाखों श्रद्धालु अयोध्या की ओर चल पड़े। पूरे देश में माहौल तनावपूर्ण हो गया।

अयोध्या में तनाव अपने चरम पर था। सर्वोच्च न्यायालय ने राज्य सरकार को आदेश दिया कि वह विवादित स्थल की सुरक्षा का पूरा इंतजाम करे। लेकिन 6 दिसम्बर, 1992 को देश के विभिन्न भागों से लोग आ जुटे और इन लोगों ने विवादित ढाँचे को गिरा दिया। विवादित ढाँचे के विध्वंस की खबर से देश के कई भागों में हिन्दू और मुसलमानों के बीच झड़प हुई। जनवरी, 1993 में एक बार फिर मुम्बई में हिंसा भड़की और अगले दो हफ्तों तक जारी रही।

→ गुजरात के दंगे:
2002 के फरवरी मार्च में गुजरात में बड़े पैमाने पर हिंसा हुई। हिंसा का यह तांडव लगभग एक महीने तक चला। 2004 के लोकसभा चुनाव-2004 के चुनावों में कांग्रेस पार्टी ने भी गठबन्धन बनाकर चुनावों में भाग लिया राजग की हार हुई और संयुक्त प्रगतिशील गठबन्धन (संप्रग) की सरकार बनी। इस गठबन्धन का नेतृत्व कांग्रेस पार्टी ने किया। संप्रग को वाम मोर्चा तथा कुछ अन्य दलों का समर्थन मिला। 2004 के चुनावों में एक हद तक कांग्रेस पार्टी का पुनरुत्थान भी हुआ। बढ़ती सहमति – संप्रग सरकार में आपसी सहमति के प्रमुख बिन्दु निम्न हैं।

  • नई आर्थिक नीति की सहमति
  • पिछड़ी जातियों के राजनीतिक और सामाजिक दावे की स्वीकृति।
  • देश के शासन में प्रांतीय दलों की भूमिका की स्वीकृति।
  • विचारधारात्मक पक्ष की जगह कार्यसिद्धि पर जोर और विचारधारा सहमति के बिना राजनीतिक गठजोड़ 2009 के लोकसभा चुनाव में भी कांग्रेस गठबन्धन को बहुमत मिला

JAC Class 12 Political Science Solutions Chapter 7 जन आंदोलनों का उदय

Jharkhand Board JAC Class 12 Political Science Solutions Chapter 7 जन आंदोलनों का उदय Textbook Exercise Questions and Answers.

JAC Board Class 12 Political Science Solutions Chapter 7 जन आंदोलनों का उदय

Jharkhand Board Class 12 Political Science जन आंदोलनों का उदय InText Questions and Answers

पृष्ठ 129

प्रश्न 1.
है तो यह बड़ी शानदार बात! लेकिन कोई मुझे बताए कि हम जो यहाँ राजनीति का इतिहास पढ़ रहे हैं, उससे यह बात कैसे जुड़ती है?
उत्तर:
सामाजिक और राजनीतिक परिवर्तनों में जन आंदोलनों की शानदार उपलब्धि रही है । चिपको आंदोलन के अन्तर्गत उत्तराखण्ड के एक गाँव के स्त्री एवं पुरुषों ने पर्यावरण की सुरक्षा और जंगलों की कटाई का विरोध करने के लिए एक अनूठा प्रयास किया, जिसमें इन लोगों ने पेड़ों को अपनी बाँहों में भर लिया ताकि उनको काटने से बचाया जा सके। इस आन्दोलन को व्यापक सफलता प्राप्त हुई। जन-आंदोलन, कभी राजनीतिक तो कभी सामाजिक आंदोलन का रूप ले सकते हैं और अक्सर यह दोनों ही रूपों में नजर आते हैं। स्वतन्त्रता के तीन दशक बाद लोगों का धैर्य टूटा और जनता की बेचैनी जन-आन्दोलनों में उमड़ी। यहाँ हम राजनीति का इतिहास पढ़ रहे हैं परन्तु वह कुशल राजनीति की असफलता के उमड़े जन-आन्दोलन के उभार की अभिव्यक्ति है।

पृष्ठ 130

प्रश्न 2.
गैर-राजनीतिक संगठन ? मैं यह बात कुछ समझा नहीं! आखिर, पार्टी के बिना राजनीति कैसे की जा सकती है?
उत्तर:
सामान्यतः गैर-राजनीतिक संगठन ऐसे संगठन हैं जो दलगत राजनीति से दूर स्थानीय एवं क्षेत्रीय मुद्दों से जुड़े हुए होते हैं तथा सक्रिय राजनीति में भाग लेने की अपेक्षा एक दबाव समूह की तरह कार्य करते हैं। औपनिवेशिक दौर में अनेक स्वतंत्र सामाजिक आंदोलनों का जन्म हुआ, जैसे— जाति प्रथा विरोधी आंदोलन, किसान सभा आंदोलन और मजदूर संगठनों के आंदोलन। मुंबई, कोलकाता और कानपुर जैसे औद्योगिक शहरों में मजदूर संगठनों के आंदोलनों का मुख्य जोर आर्थिक अन्याय और असमानता के मसले पर रहा। ये सभी गैर-राजनैतिक संगठन थे और इन्होंने अपनी माँगें मनवाने के लिए आंदोलनों का सहारा लिया। इन गैर-राजनैतिक संगठनों का राजनीतिक दलों से नजदीकी सम्बन्ध रहा।

JAC Class 12 Political Science Solutions Chapter 7 जन आंदोलनों का उदय

पृष्ठ 132

प्रश्न 3.
क्या दलितों की स्थिति इसके बाद से ज्यादा बदल गई है? दलितों पर अत्याचार की घटनाओं के बारे में मैं रोजाना सुनती हूँ। क्या यह आंदोलन असफल रहा? या, यह पूरे समाज की असफलता है?
उत्तर:
दलितों की स्थिति में सुधार हेतु 1972 में शिक्षित दलित युवा वर्ग ने महाराष्ट्र में एक आन्दोलन चलाया, जिसे दलित पैंथर्स आन्दोलन के नाम से जाना जाता है। इस आन्दोलन के माध्यम से दलितों की स्थिति सुधारने हेतु अनेक प्रयास किये गये। इस आन्दोलन का उद्देश्य जाति प्रथा को समाप्त करना तथा गरीब किसान, शहरी औद्योगिक मजदूर और दलित सहित सारे वंचित वर्गों का एक संगठन तैयार करना था। लेकिन इस आन्दोलन को अपेक्षित सफलता प्राप्त नहीं हो पायी।

छुआछूत प्रथा को कानूनन समाप्त किये जाने के बावजूद भी अछूत जातियों के साथ नए दौर में भी सामाजिक भेदभाव तथा हिंसा का बरताव कई रूपों में जारी रहा। दलितों की बस्तियाँ मुख्य गाँव से अब भी दूर हैं, दलित महिलाओं के साथ यौन-अत्याचार होते हैं। जातिगत प्रतिष्ठा की छोटी-मोटी बात को लेकर दलितों पर सामूहिक जुल्म ढाये जाते हैं। इस सबके बावजूद यह आन्दोलन पूरी तरह से असफल नहीं कहा जा सकता क्योंकि धीरे-धीरे दलितों को उनके हक मिलने लगे हैं।

पृष्ठ 135

प्रश्न 4.
मुझे कोई ऐसा नहीं मिला जो कहे कि मैं किसान बनना चाहता हूँ। क्या हमें अपने देश में किसानों की जरूरत नहीं है?
उत्तर:
यद्यपि भारत एक कृषि प्रधान देश है और भारतीय अर्थव्यवस्था का अधिकांश हिस्सा कृषि पर आधारित है, लेकिन संसाधनों के अभाव व वर्षा आधारित कृषि होने के कारण किसानों को इस कार्य में पर्याप्त लाभ नहीं मिल पा रहा है। इसके अतिरिक्तं जनाधिक्य व कृषिगत क्षेत्र की सीमितता के कारण इस क्षेत्र में छिपी हुई बेरोजगारी जैसी समस्या आम बात हो गई है। इसलिए आम नागरिक को इस क्षेत्र में उज्ज्वल भविष्य नजर नहीं आ रहा है। इसलिए लोग कृषिगत कार्यों की अपेक्षा अन्य कार्यों में अधिक रुचि लेने लगे हैं। लेकिन हमें अपने देश में किसानों की अत्यन्त आवश्यकता है। यदि किसान खेती का काम करना बन्द कर देंगे, तो अन्न कहाँ से आयेगा? कृषि – आधारित उद्योग नष्ट हो जायेंगे और देश की अर्थव्यवस्था गड़बड़ा जायेगी अतः देश की अर्थव्यवस्था को सुचारू रूप से चलाने के लिए कृषक वर्ग की अत्यन्त आवश्यकता है। हाँ, कृषि सम्बन्धी समस्याओं को हल करने के लिए सरकार को प्राथमिकता से ध्यान देना चाहिए।

प्रश्न 5.
हमें इस तरह की अच्छी-अच्छी कहानियाँ तो सुनाई जाती हैं लेकिन हमें कभी यह नहीं बताया जाता कि इन कहानियों का अंत कैसा रहा। क्या यह आंदोलन शराबबंदी में सफल हो पाया? या पुरुषों ने एक समय के बाद फिर पीना शुरू कर दिया?
उत्तर:
इन कहानियों का अंत तो बहुत ही रोचक होता है। लेकिन हकीकत में इन कहानियों का अंत इतना रोचक नहीं होता। ऐसे आंदोलनों का असर पर कुछ समय के लिए ही होता है बाद में फिर पुरुष शराब पीना शुरू कर देते हैं

पृष्ठ 142

प्रश्न 6.
क्या आंदोलनों को राजनीति की प्रयोगशाला कहा जा सकता है? आंदोलनों के दौरान नए प्रयोग किए जाते हैं और सफल प्रयोगों को राजनीतिक दल अपना लेते हैं।
उत्तर:
जन-आन्दोलनों को राजनीति की प्रयोगशाला कहा जाये तो कोई अतिशयोक्ति नहीं होगी। जन-आन्दोलनों का उद्देश्य दलीय राजनीति की खामियों को दूर करना रहता है। समाज के गहरे तनावों और जनता के क्षोभ को सार्थक दिशा देकर इन आन्दोलनों ने एक तरह से लोकतन्त्र की रक्षा की है। जन-आन्दोलनों में शरीक अनेक व्यक्ति और संगठन राजनीतिक दलों में सक्रिय रूप से जुड़े। ऐसे जुड़ाव से दलगत राजनीति में विभिन्न सामाजिक तबकों की बेहतर नुमाइंदगी सुनिश्चित हुई। इस प्रकार जन-आन्दोलनों के दौरान किये जाने वाले सफल प्रयोगों को राजनीतिक दल अपना लेते हैं।

Jharkhand Board Class 12 Political Science जन आंदोलनों का उदय TextBook Questions and Answers

प्रश्न 1.
चिपको आंदोलन के बारे में निम्नलिखित में कौन-सा कथन गलत हैं-
(क) यह पेड़ों की कटाई को रोकने के लिए चला एक पर्यावरण आंदोलन था।
(ख) इस आंदोलन ने पारिस्थितिकी और आर्थिक शोषण के मामले उठाए।
(ग) यह महिलाओं द्वारा शुरू किया गया शराब-विरोधी आंदोलन था।
(घ) इस आंदोलन की माँग थी कि स्थानीय निवासियों का अपने प्राकृतिक संसाधनों पर नियंत्रण होना चाहिए । उत्तर- (ग) गलत।

JAC Class 12 Political Science Solutions Chapter 7 जन आंदोलनों का उदय

प्रश्न 2.
नीचे लिखे कुछ कथन गलत हैं। इनकी पहचान करें और जरूरी सुधार के साथ उन्हें दुरुस्त करके दोबारा लिखें-
(क) सामाजिक आंदोलन भारत के लोकतंत्र को हानि पहुँचा रहे हैं।
(ख) सामाजिक आंदोलनों की मुख्य ताकत विभिन्न सामाजिक वर्गों के बीच व्याप्त उनका जनाधार है।
(ग) भारत के राजनीतिक दलों ने कई मुद्दों को नहीं उठाया। इसी कारण सामाजिक आंदोलनों का उदय हुआ।
उत्तर:
(क) सामाजिक आन्दोलन भारत के लोकतन्त्र को बढ़ावा दे रहे हैं।
(ख) यह कथन पूर्ण रूप से सही है।
(ग) यह कथन पूर्ण रूप से सही है।

प्रश्न 3.
उत्तर प्रदेश के कुछ भागों में ( अब उत्तराखण्ड) 1970 के दशक में किन कारणों से चिपको आन्दोलन का जन्म हुआ? इस आंदोलन का क्या प्रभाव पड़ा?
उत्तर:
चिपको आंदोलन के कारण-चिपको आंदोलन के प्रमुख कारण निम्नलिखित थे

  1. चिपको आन्दोलन की शुरुआत उस समय हुई जब वन विभाग ने खेती-बाड़ी के लिए औजार बनाने के लिए गाँव वालों को ‘Ashtree’ काटने की अनुमति नहीं दी तथा खेल सामग्री बनाने वाली एक व्यावसायिक कम्पनी को यह पेड़ काटने की अनुमति प्रदान कर दी। इससे गाँव वालों में रोष उत्पन्न हुआ तथा कटाई के विरोध में यह आन्दोलन उठ खड़ा हुआ।
  2. गाँव वालों ने माँग रखी कि वन की कटाई का ठेका किसी बाहरी व्यक्ति को नहीं दिया जाना चाहिए स्थानीय लोगों का जल, जंगल, जमीन जैसे प्राकृतिक साधनों पर उपयुक्त नियंत्रण होना चाहिए।
  3. चिपको आंदोलन का एक अन्य कारण पारिस्थितिकी संतुलन बनाए रखना था। गांव वासी चाहते थे कि इस क्षेत्र के पारिस्थितिकी संतुलन को हानि पहुँचाये बिना विकास किया जाये।
  4. इस क्षेत्र में वनों की कटाई के दौरान ठेकेदार यहाँ के पुरुषों को शराब आपूर्ति का व्यवसाय भी करते थे अतः स्त्रियों ने शराबखोरी की लत के विरोध में भी आवाज बुलन्द की।

प्रभाव – इस आन्दोलन के निम्न प्रमुख प्रभाव हुए-

  1. सरकार ने 15 वर्ष के लिए हिमालयी क्षेत्रों में पेड़ों की कटाई पर रोक लगा दी।
  2. इस आन्दोलन ने अन्य आन्दोलनों को प्रभावित तथा प्रेरित किया।
  3. इस आन्दोलन के प्रभावस्वरूप ग्रामीण क्षेत्रों में व्यापक जागरूकता के आन्दोलन चलाए गए।

प्रश्न 4.
भारतीय किसान यूनियन किसानों की दुर्दशा की तरफ ध्यान आकर्षित करने वाला अग्रणी संगठन है। नब्बे के दशक में इसने किन मुद्दों को उठाया और इसे कहाँ तक सफलता मिली?
उत्तर:
1990 के दशक में भारतीय किसान यूनियन (बीकेयू) ने भारतीय किसानों की दुर्दशा सुधारने हेतु अनेक मुद्दों पर बल दिया, जिनमें प्रमुख निम्न हैं-

  1. इसने बिजली की दरों में बढ़ोतरी का विरोध किया तथा उचित दर पर गारण्टीशुदा बिजली आपूर्ति करने पर बल दिया।
  2. इसने नगदी फसलों के सरकारी खरीद मूल्यों में बढ़ोतरी की माँग की।
  3. इसने कृषि उत्पादों के अन्तर्राज्यीय आवाजाही पर लगी पाबंदियों को हटाने पर बल दिया।
  4. किसानों के लिए पेंशन का प्रावधान करने की माँग की।
  5. किसानों के बकाया कर्ज को माफ किया जाये।

सफलताएँ – भारतीय किसान यूनियन को निम्न सफलताएँ प्राप्त हुईं-

  1. बीकेयू (BKU) जैसी माँगें देश के अन्य किसान संगठनों ने भी उठायीं I
  2. 1990 के दशक में बीकेयू ने अपनी संख्या बल के आधार पर एक दबाव समूह की तरह कार्य किया तथा अन्य किसान संगठनों के साथ मिलकर अपनी मांगें मनवाने में सफलता पाई।

प्रश्न 5.
आन्ध्रप्रदेश में चले शराब-विरोधी आंदोलन ने देश का ध्यान कुछ गम्भीर मुद्दों की तरफ खींचा। ये मुद्दे क्या थे?
उत्तर:
आन्ध्रप्रदेश में चले शराब-विरोधी या ताड़ी विरोधी आन्दोलन ने देश का ध्यान निम्नलिखित गम्भीर मुद्दों की तरफ खींचा-

  1. शराब पीने से पुरुषों का शारीरिक और मानसिक स्वास्थ्य कमजोर होता है।
  2. शराब पीने से व्यक्ति की कार्यक्षमता प्रभावित होती है जिससे ग्रामीण अर्थव्यवस्था प्रभावित होती है।
  3. शराबखोरी से ग्रामीणों पर कर्ज का बोझ बढ़ता है।
  4. शराबखोरी से कामचोरी की आदत बढ़ती है।
  5. शराब माफिया के सक्रिय होने से गांवों में अपराधों को बढ़ावा मिलता है तथा अपराध और राजनीति के बीच एक घनिष्ठ सम्बन्ध बनता है।
  6. शराबखोरी से परिवार की महिलाओं से मारपीट एवं तनाव को बढ़ावा मिलता है।
  7. शराब विरोधी आंदोलन ने महिलाओं के मुद्दों दहेज प्रथा, घरेलू हिंसा, कार्यस्थल व सार्वजनिक स्थानों पर यौन उत्पीड़क, लैंगिक असमानता आदि  के प्रति समाज में जागरूकता उत्पन्न की।

JAC Class 12 Political Science Solutions Chapter 7 जन आंदोलनों का उदय

प्रश्न 6.
क्या आप शराब-विरोधी आंदोलन को महिला आंदोलन का दर्जा देंगे? कारण बताएँ।
उत्तर:
शराब-विरोधी आंदोलन को महिला आंदोलन का दर्जा दिया जा सकता है। क्योंकि शराब – 1 ब- विरोधी आंदोलन में महिलाओं ने अपने आस-पड़ौस में मदिरा या ताड़ी की बिक्री पर पाबन्दी लगाने की माँग की। यह महिलाओं का स्वतः स्फूर्त आन्दोलन था। यह आन्दोलन महिलाओं ने शराब माफिया तथा सरकार दोनों के खिलाफ चलाया। शराबखोरी से सर्वाधिक परेशानी महिलाओं को होती है। इससे परिवार की अर्थव्यवस्था चरमराने लगती है, परिवार में तनाव व मारपीट का वातावरण बनता है। तनाव के चलते पुरुष शारीरिक व मानसिक रूप से कमजोर हो जाते इन्हीं कारणों से शराब विरोधी आंदोलनों का प्रारंभ प्रायः महिलाओं द्वारा किया जाता रहा है। अत: शराब-विरोधी ‘नों को महिला आंदोलन भी कहा जा सकता है।

प्रश्न 7.
नर्मदा बचाओ आन्दोलन ने नर्मदा घाटी की बाँध परियोजना का विरोध क्यों किया?
उत्तर:
नर्मदा बचाओ आन्दोलन ने नर्मदा घाटी की बाँध परियोजना का विरोध निम्नलिखित कारणों से किया

  1. बाँध निर्माण से प्राकृतिक नदियों व पर्यावरण पर बुरा असर पड़ता है।
  2. जिस क्षेत्र में बाँध बनाये जाते हैं वहाँ रह रहे गरीबों के घर-बार, उनकी खेती योग्य भूमि, वर्षों से चले आ रहे कुटीर -धन्धों पर भी बुरा असर पड़ता है।
  3. प्रभावित गाँवों के करीब ढाई लाख लोगों के पुनर्वास का मामला उठाया जा रहा था।
  4. परियोजना पर किये जाने वाले व्यय में हेरा-फेरी के दोषों को उजागर करना भी परियोजना विरोधी स्वयंसेवकों का उद्देश्य था।
  5. आन्दोलनकर्ता प्रभावित लोगों को आजीविका और उनकी संस्कृति के साथ-साथ पर्यावरण को बचाना चाहते थे। वे जल, जंगल और जमीन पर प्रभावित लोगों का नियन्त्रण या इन्हें उचित मुआवजा और उनका पुनर्वास चाहते थे।

प्रश्न 8.
क्या आंदोलन और विरोध की कार्यवाहियों से देश का लोकतन्त्र मजबूत होता है? अपने उत्तर की पुष्टि में उदाहरण दीजिए।
उत्तर:
शांतिपूर्ण आन्दोलनों एवं विरोध की कार्यवाहियों से लोकतन्त्र मजबूत होता है। इसके समर्थन में निम्नलिखित तर्क दिये जा सकते हैं-

  1. चिपको आंदोलन अहिंसक और शांतिपूर्ण तरीके से चलाया गया। यह एक व्यापक जन आंदोलन था। इससे पेड़ों की कटाई, वनों का उजड़ना रुका। पशु-पक्षियों और जनता को जल, जंगल, जमीन और स्वास्थ्यवर्धक पर्यावरण मिला। सरकार लोकतान्त्रिक मांगों के समक्ष झुकी।
  2. वामपंथियों द्वारा चलाये गये किसान और मजदूर आंदोलनों द्वारा जन-साधारण में जागृति, राष्ट्रीय कार्यों में भागीदारी और सरकार को सर्वहारा वर्ग की उचित माँगों के लिए जगाने में सफलता मिली।
  3. दलित पैंथर्स नेताओं द्वारा चलाए गए आंदोलनों ने आदिवासी, अनुसूचित जातियों, अनुसूचित जनजातियों और पिछड़ी जातियों में चेतना पैदा की। समाज में समानता, स्वतन्त्रता, सामाजिक न्याय, आर्थिक न्याय, राजनैतिक न्याय को सुदृढ़ता मिली।
  4. ताड़ी – विरोधी आंदोलन ने नशाबंदी और मद्य-निषेध के विरोध का वातावरण तैयार किया। महिलाओं से जुड़ी अनेक समस्याएँ, जैसे यौन उत्पीड़न, घरेलू समस्या, दहेज प्रथा और महिलाओं को विधायिका में दिये जाने वाले आरक्षण के मामले उठे। महिलाओं में राजनैतिक चेतना बढ़ी।

प्रश्न 9.
दलित- पैंथर्स ने कौन-कौन-से मुद्दे उठाये?
उत्तर:
दलित पैंथर्स – दलित पैंथर्स बीसवीं शताब्दी के सातवें दशक के शुरुआती सालों में दलित शिक्षित युवा वर्ग का आन्दोलन था। इसमें ज्यादातर शहर की झुग्गी बस्तियों में पलकर बड़े हुए दलित थे। दलित पैंथर्स ने दलित समुदाय से सम्बन्धित सामाजिक असमानता, जातिगत आधार पर भेदभाव, दलित महिलाओं के साथ दुर्व्यवहार, दलितों का सामाजिक एवं आर्थिक उत्पीड़न तथा दलितों के लिए आरक्षण जैसे मुद्दे उठाये।

JAC Class 12 Political Science Solutions Chapter 7 जन आंदोलनों का उदय

प्रश्न 10.
निम्नलिखित अवतरण को पढ़ें और इसके आधार पर पूछे गए प्रश्नों के उत्तर दें-
लगभग सभी नए सामाजिक आंदोलन नयी समस्याओं जैसे – पर्यावरण का विनाश, महिलाओं की बदहाली, आदिवासी संस्कृति का नाश और मानवाधिकारों का उल्लंघन के समाधान को रेखांकित करते हुए उभरे। इनमें से कोई भी अपने आप में समाजव्यवस्था के मूलगामी बदलाव के सवाल से नहीं जुड़ा था। इस अर्थ में ये आंदोलन अतीत की क्रांतिकारी विचारधाराओं से एकदम अलग हैं। लेकिन, ये आंदोलन बड़ी बुरी तरह बिखरे हुए हैं और यही इनकी कमजोरी है सामाजिक आंदोलनों का एक बड़ा दायरा ऐसी चीजों की चपेट में है कि वह एक ठोस तथा एकजुट जन आंदोलन का रूप नहीं ले पाता और न ही वंचितों और गरीबों के लिए प्रासंगिक हो पाता है।

ये आंदोलन बिखरे-बिखरे हैं, प्रतिक्रिया के तत्त्वों से भरे हैं, अनियत हैं और बुनियादी सामाजिक बदलाव के लिए इनके पास कोई फ्रेमवर्क नहीं है। ‘इस’ या ‘उस’ के विरोध ( पश्चिम-विरोधी, पूँजीवाद – विरोधी, ‘विकास’- विरोधी, आदि) में चलने के कारण इनमें कोई संगति आती हो अथवा दबे-कुचले लोगों और हाशिए के समुदायों के लिए ये प्रासंगिक हो पाते हों- ऐसी बात नहीं। – रजनी कोठारी

(क) नए सामाजिक आंदोलन और क्रांतिकारी विचारधाराओं में क्या अंतर है?
(ख) लेखक के अनुसार सामाजिक आंदोलनों की सीमाएँ क्या-क्या हैं?
(ग) यदि सामाजिक आंदोलन विशिष्ट मुद्दों को उठाते हैं तो आप उन्हें ‘बिखरा’ हुआ कहेंगे या मानेंगे कि वे अपने मुद्दे पर कहीं ज्यादा केंद्रित हैं। अपने उत्तर की पुष्टि में तर्क दीजिए।
उत्तर:
(क) सामाजिक आन्दोलन समाज से जुड़े हुए मामलों अथवा समस्याओं को उठाते हैं; जैसे जाति भेदभाव, रंग भेदभाव, लिंग भेदभाव के विरोध में चलाये जाने वाले सामाजिक आन्दोलन। इसी प्रकार ताड़ी- विरोधी आन्दोलन, सभी को शिक्षा, सामाजिक न्याय और समानता के लिए चलाये जाने वाले आन्दोलन आदि जबकि क्रान्तिकारी विचारधारा के लोग तुरन्त सामाजिक, आर्थिक, राजनैतिक और सांस्कृतिक क्षेत्र में बदलाव लाना चाहते हैं। वे’ लक्ष्यों को ज्यादा महत्त्व देते हैं, तरीकों को नहीं।

(ख) सामाजिक आन्दोलन बिखरे हुए हैं तथा इनमें एकजुटता का अभाव पाया जाता है। सामाजिक आन्दोलनों के पास सामाजिक बदलाव के लिए कोई ढांचागत योजना नहीं है।

(ग) सामाजिक आन्दोलनों द्वारा उठाए गए विशिष्ट मुद्दों के कारण यह कहा जा सकता है कि ये आन्दोलन अपने मुद्दों पर अधिक केन्द्रित हैं।

जन आंदोलनों का उदय JAC Class 12 Political Science Notes

→ सामान्यतः
वे आन्दोलन जो दलगत राजनीति से दूर तथा जन सामान्य के हित में चलाये जाते हैं उन्हें जन- आन्दोलन कहा जाता है। भारतीय राजनीति में इस प्रकार के आन्दोलनों की विशेष भूमिका रही है।

→ चिपको आन्दोलन:
इस आन्दोलन की शुरुआत उत्तराखण्ड के दो-तीन गाँवों से हुई। 1973 में उत्तराखण्ड सरकार ने स्थानीय लोगों को कृषिगत औजार बनाने के लिए वन की कटाई करने से इनकार कर दिया, लेकिन एक खेल कम्पनी को पेड़ों की कटाई की अनुमति प्रदान कर दी। इससे स्थानीय लोगों में रोष की भावना उत्पन्न हुई और उन्होंने वनों की कटाई रोकने के लिए विशेष अभियान चलाया। इन लोगों ने पेड़ों को अपनी बाँहों में भर लिया ताकि इनको काटने से बचाया जा सके। यह विरोध आगामी दिनों में भारत के पर्यावरण आन्दोलन के रूप में परिणत हुआ और ‘चिपको आन्दोलन’ के नाम से प्रसिद्ध हुआ।

→ आन्दोलन के महत्त्वपूर्ण पक्ष:
(i) ग्रामवासियों ने माँग की कि जंगल की कटाई का कोई भी ठेका बाहरी व्यक्तियों को नहीं दिया जाना चाहिए और स्थानीय लोगों का जल, जंगल और जमीन जैसे प्राकृतिक संसाधनों पर कारगर नियन्त्रण होना चाहिए।
(ii) लोग चाहते थे कि सरकार लघु उद्योगों के लिए कम कीमत की सामग्री उपलब्ध कराए और उस क्षेत्र के पारिस्थितिकी संतुलन को नुकसान पहुँचाए बगैर यहाँ का विकास सुनिश्चित करे। आंदोलन ने भूमिहीन वन कर्मचारियों का आर्थिक मुद्दा भी उठाया और न्यूनतम मजदूरी की गारण्टी की माँग की।

→ आन्दोलन में महिलाओं की भूमिका:
चिपको आन्दोलन में महिलाओं ने सक्रिय भागीदारी की। यह आन्दोलन का एकदम नया पहलू था। इलाके में सक्रिय जंगल कटाई के ठेकेदार यहाँ के पुरुषों को शराब की आपूर्ति का व्यवसाय भी करते थे। महिलाओं ने शराबखोरी की लत के खिलाफ भी लगातार आवाज उठायी। इससे आन्दोलन का दायरा विस्तृत हुआ और उसमें कुछ और सामाजिक मसले आ जुड़े।

JAC Class 12 Political Science Solutions Chapter 7 जन आंदोलनों का उदय

→ दल – आधारित आन्दोलन:
जन-आन्दोलन कभी-कभी सामाजिक तो कभी राजनीतिक आन्दोलन का रूप ले सकते हैं और अक्सर ये आन्दोलन दोनों ही रूपों में नजर आते हैं, जैसे- भारत का स्वतन्त्रता आन्दोलन। यह मुख्य रूप से राजनीतिक आंदोलन था, लेकिन हम जानते हैं कि औपनिवेशिक दौर में सामाजिक-आर्थिक मसलों पर भी विचार मंथन चला, जिससे अनेक स्वतन्त्र सामाजिक आन्दोलनों का जन्म हुआ, जैसे- जाति प्रथा विरोधी आन्दोलन, नारी कल्याण आन्दोलन, नारी मुक्ति आन्दोलन, धर्म सुधार आन्दोलन, शिक्षा प्रसार आन्दोलन कहे जा सकते हैं। अनेक आन्दोलन 19वीं और 20वीं शताब्दी में सर्वहारा वर्गों के हितों के लिए चलाए गए। जैसे किसान सभा आन्दोलन और मजदूर संगठन आन्दोलन।

ये आन्दोलन 20वीं सदी के शुरुआती दशकों में सामने आये। इन आन्दोलनों ने सामाजिक संघर्षों के कुछ अंदरूनी मुद्दे उठाये। ऐसे कुछ आन्दोलन आजादी के बाद के दौर में चलते रहे। मुंबई, कोलकाता और कानपुर जैसे बड़े शहरों के औद्योगिक मजदूरों के बीच मजदूर संगठनों के आन्दोलनों का जोर था। बड़ी पार्टियों ने इस तबके के मजदूरों को लामबंद करने के लिए अपने-अपने मजदूर संगठन बनाए। भारत की स्वतन्त्रता के उपरान्त प्रारम्भिक वर्षों में आंध्रप्रदेश, पश्चिम बंगाल और बिहार के कुछ भागों में किसान तथा कृषि मजदूरों ने मार्क्सवादी-लेनिनवादी कम्युनिस्ट पार्टी के कार्यकर्ताओं के नेतृत्व में अपना विरोध जारी रखा। भारत के कुछ दल सीधी कार्यवाही अथवा हिंसा में यकीन करने वाले थे जिनमें मार्क्सवादी-लेनिनवादी दल प्रमुख हैं। इस काल में किसान और मजदूरों के आंदोलनों का मुख्य जोर आर्थिक पिछड़ापन, अन्याय तथा असमानता के मसले पर रहा।

→ राजनीतिक दलों से स्वतंत्र आंदोलन:
70 और 80 के दशक में कई सामाजिक तबकों का राजनीतिक दलों के. आचार-व्यवहार से मोहभंग हुआ। देश ने आजादी के बाद नियोजित विकास का मॉडल अपनाया था। इस मॉडल को अपनाने के पीछे दो लक्ष्य थे- आर्थिक संवृद्धि और आय का समतापूर्ण बँटवारा। राजनीतिक धरातल पर सक्रिय कई समूहों का विश्वास लोकतांत्रिक संस्थाओं और चुनावी राजनीति से उठ गया । ये समूह दलगत राजनीति से अलग हुए और अपने विरोध को स्वर देने के लिए इन्होंने आवाम को लामबंद करना शुरू किया। मध्यवर्ग के युवा कार्यकर्ताओं ने गाँव के गरीब लोगों के बीच रचनात्मक कार्यक्रम तथा सेवा संगठन चलाए। इन संगठनों के सामाजिक कार्यों की प्रकृति स्वयंसेवी थी इसलिए इन संगठनों को स्वयंसेवी संगठन या स्वयंसेवी क्षेत्र के संगठन कहा गया। इन संगठनों ने स्वयं को दलगत राजनीति से दूर रखा । स्थानीय या क्षेत्रीय स्तर पर ये संगठन न तो चुनाव लड़े और न ही इन्होंने किसी एक राजनीतिक दल को समर्थन दिया । इसी कारण इन संगठनों को ‘स्वतंत्र राजनीतिक संगठन’ कहा जाता है।

→ दलित पैंथर्स आन्दोलन: सातवें दशक के शुरुआती सालों से शिक्षित दलितों की पहली पीढ़ी ने अनेक मंचों से अपने हक की आवाज उठायी। इनमें ज्यादातर शहर की झुग्गी बस्तियों में पलकर बड़े हुए दलित थे। दलित हितों की | दावेदारी के इसी क्रम में महाराष्ट्र में 1972
में दलित युवाओं का संगठन ‘दलित पैन्थर्स’ बना। इस संगठन में पढ़े-लिखे दलित युवकों को सम्मिलित किया गया।

→ आंदोलन की गतिविधियाँ: महाराष्ट्र के विभिन्न इलाकों में दलितों पर बढ़ रहे अत्याचार से लड़ना दलित पैंथर्स की मुख्य गतिविधि थी। दलित पैंथर्स तथा इसके सहधर्मी संगठनों द्वारा दलितों पर बढ़ रहे अत्याचार के मुद्दे पर लगातार विरोध आंदोलन चलाया गया। इसके परिणामस्वरूप सरकार ने 1989 में एक व्यापक कानून बनाया। इस कानून के तहत दलितों पर अत्याचार करने वालों के लिए कठोर दंड का प्रावधान किया गया। दलित पैंथर्स का विचारधारात्मक एजेंडा जाति प्रथा को समाप्त करना तथा गरीब किसान, शहरी औद्योगिक मजदूर और दलित सहित सारे वंचित वर्गों का एक संगठन खड़ा करना था।

→ भारतीय किसान यूनियन: भारतीय किसान यूनियन (बीकेयू) पश्चिमी उत्तर प्रदेश और हरियाणा के किसानों का एक संगठन था। यह अस्सी के दशक के किसान आंदोलन के अग्रणी संगठनों में एक था। बीकेयू की विशेषताएँ – बीकेयू की प्रमुख विशेषताएँ निम्न हैं-

  • इसने सरकार पर अपनी माँगों को मनवाने के लिए रैली, धरना, प्रदर्शन और जेल भरो आन्दोलन का सहारा लिया। इन कार्यवाहियों में पश्चिमी उत्तरप्रदेश और उसके आस-पास के इलाकों के गांवों के हजारों किसानों ने भाग लिया।
  • बीकेयू के अधिकांश सदस्य एक खास समुदाय के थे। इस संगठन ने जातिगत समुदायों को आर्थिक मसले पर एकजुट करने के लिए ‘जाति-पंचायत’ की परम्परागत संस्था का उपयोग किया।
  • बीकेयूं के लिए धनराशि और संसाधन जातिगत संगठनों से जुटाये जाते थे और इन्हीं के सहारे बीकेयू की गतिविधियाँ भी संचालित होती थीं।
  • 1990 के शुरुआती वर्षों तक बीकेयू ने अपने को सभी राजनीतिक दलों से दूर रखा था। अपने सदस्यों की संख्या बल के दम पर यह राजनीतिक दलों में एक दबाव समूह की तरह सक्रिय था। इस संगठन ने राज्यों में मौजूद अन्य किसान संगठनों को साथ लेकर अपनी कुछ माँगें मनवाने में सफलता पाई। इस अर्थ में किसान आंदोलन अस्सी के दशक में सबसे ज्यादा सफल सामाजिक आंदोलन था।
  • किसान आंदोलन मुख्य रूप से देश के समृद्ध राज्यों में सक्रिय था। खेती को अपनी जीविका का आधार बनाने वाले अधिकांश भारतीय किसानों के विपरीत बीकेयू जैसे संगठनों के सदस्य बाजार के लिए नगदी फसल उपजाते। थे।
  • बीकेयू की तरह राज्यों के अन्य किसान संगठनों ने अपने सदस्य उन समुदायों के बीच से बनाए जिनका क्षेत्र की चुनावी राजनीति से रसूख था। महाराष्ट्र शेतकारी संगठन और कर्नाटक का रैयत संगठन ऐसे किसान संगठनों के जीवंत उदाहरण हैं।

→ ताड़ी-विरोधी आन्दोलन:
ताड़ी – विरोधी आन्दोलन की शुरुआत आंध्रप्रदेश के नेल्लौर जिले से मानी जाती है । इस आन्दोलन के अन्तर्गत ग्रामीण महिलाओं ने शराब के खिलाफ लड़ाई छेड़ी तथा शराब की बिक्री पर पाबंदी लगाने की माँग की। यह लड़ाई माफिया और सरकार दोनों के खिलाफ थी। ताड़ी – विरोधी आंदोलन के कारण दो परस्पर विरोधी गुट दृष्टिगोचर हुए। एक शराब माफिया और दूसरी ओर ताड़ी के कारण पीड़ित परिवार। शराब के ठेकेदार ताड़ी व्यापार पर एकाधिकार बनाये रखने के लिए अपराधों में व्यस्त थे। शराबखोरी से सबसे ज्यादा परेशानी महिलाओं को हो रही थी। इससे परिवार की अर्थव्यवस्था चरमराने लगी। परिवार में तनाव और मारपीट का माहौल बनने लगा।

JAC Class 12 Political Science Solutions Chapter 7 जन आंदोलनों का उदय

→ नारी मुक्ति एवं ताड़ी:
विरोधी आंदोलन – ताड़ी – विरोधी आंदोलन महिला आंदोलन का एक हिस्सा बन गया । इससे घरेलू हिंसा, दहेज प्रथा, कार्यस्थल एवं सार्वजनिक स्थानों पर यौन उत्पीड़न के खिलाफ काम करने वाले महिला समूह आमतौर पर शहरी मध्यवर्गीय महिलाओं के बीच ही सक्रिय थे और यह बात पूरे देश पर लागू होती थी। महिला समूहों के इस सतत कार्य से यह समझदारी विकसित होनी शुरू हुई कि औरतों पर होने वाले अत्याचार और लैंगिक भेदभाव का मामला खास जटिल है। इस तरह के अभियानों ने महिलाओं के मुद्दों के प्रति समाज में व्यापक जागरूकता उत्पन्न की। धीरे-धीरे महिला आंदोलन कानूनी सुधारों से हटकर सामाजिक टकराव के मुद्दों पर भी खुले तौर पर बात करने लगा। नवें दशक तक आते-आते महिला आंदोलन समान राजनीतिक प्रतिनिधित्व की बात करने लगे तथा इसी के परिणामस्वरूप संविधान के 73वें और 74वें संशोधन के अन्तर्गत महिलाओं को स्थानीय निकायों में आरक्षण प्रदान किया गया। इस व्यवस्था को राज्यों की विधानसभाओं तथा संसद में भी लागू करने की मांग की जा रही है।

→ नर्मदा बचाओ आन्दोलन:
आठवें दशक के प्रारम्भ में भारत के मध्य भाग में स्थित नर्मदा घाटी में विकास परियोजना के तहत मध्यप्रदेश, गुजरात और महाराष्ट्र से गुजरने वाली नर्मदा और उसकी सहायक नदियों पर 30 बड़े, 135 मझोले तथा 300 छोटे बाँध बनाने का प्रस्ताव रखा गया। गुजरात के सरदार सरोवर और मध्य प्रदेश के नर्मदा सागर बाँध के रूप में दो सबसे बड़ी बहुउद्देश्यीय परियोजनाओं का निर्धारण किया गया। नर्मदा नदी के बचाव में नर्मदा बचाओ आन्दोलन चला। इस आंदोलन के संचालनकर्ताओं की मुख्य माँग नर्मदा नदी पर बने इन छोटे एवं मझोले बांधों के निर्माण को रोकना है। नर्मदा बचाओ आन्दोलन इन बाँधों के निर्माण के साथ-साथ देश में चल रही विकास परियोजनाओं के औचित्य पर भी सवाल उठाता रहा है। सरदार सरोवर बाँध के निर्माण से सम्बन्धित राज्यों के 245 गाँव डूब के क्षेत्र में आ रहे थे। अतः प्रभावित गाँवों के करीब ढाई लाख लोगों के पुनर्वास का मुद्दा सबसे पहले स्थानीय कार्यकर्त्ताओं ने उठाया।

→ जन आन्दोलन के सबक:
जन आंदोलनों का राजनीतिक एवं सामाजिक व्यवस्था पर विशेष प्रभाव होता है और इन आंदोलनों से राजनीतिक व्यवस्था के संचालनकर्ताओं को नई सीख मिलती है। जन आंदोलनों के सबक निम्न हैं

  • जन आंदोलनों का इतिहास लोकतान्त्रिक राजनीति को बेहतर ढंग से समझने में मदद प्रदान करता है।
  • समाज के गहरे तनावों और जनता के क्षोभ को एक सार्थक दिशा देकर इन आंदोलनों ने लोकतन्त्र की रक्षा की है तथा भारतीय लोकतन्त्र के जनाधार को बढ़ाया है।
  • सामाजिक आंदोलनों ने समाज के उन नए वर्गों की सामाजिक-आर्थिक समस्याओं को अभिव्यक्ति दी जो अपनी दिक्कतों को चुनावी राजनीति के जरिए हल नहीं कर पा रहे थे।
  • इन आंदोलनों के आलोचकों ने यह दलील दी है कि हड़ताल, धरना और रैली जैसी सामूहिक कार्यवाहियों से सरकार के कामकाज पर बुरा असर पड़ता है। इनके अनुसार इस तरह की गतिविधियों से सरकार की निर्णय प्रक्रिया बाधित होती है तथा रोजमर्रा की लोकतान्त्रिक व्यवस्था बाधित होती है।

JAC Class 12 Political Science Solutions Chapter 8 क्षेत्रीय आकांक्षाएँ

Jharkhand Board JAC Class 12 Political Science Solutions Chapter 8 क्षेत्रीय आकांक्षाएँ Textbook Exercise Questions and Answers.

JAC Board Class 12 Political Science Solutions Chapter 8 क्षेत्रीय आकांक्षाएँ

Jharkhand Board Class 12 Political Science क्षेत्रीय आकांक्षाएँ InText Questions and Answers

पृष्ठ 149

प्रश्न 1.
क्या इसका मतलब यह हुआ कि क्षेत्रवाद साम्प्रदायिकता के समान खतरनाक नहीं है? क्या हम यह भी कह सकते हैं कि क्षेत्रवाद अपने आप में खतरनाक नहीं?
उत्तर:
सामान्यतः लोकतान्त्रिक व्यवस्था में क्षेत्रीय आकांक्षाओं को राष्ट्र विरोधी नहीं माना जाता। इसके साथ ही लोकतान्त्रिक राजनीति में इस बात के पूरे अवसर होते हैं कि विभिन्न दल और समूह क्षेत्रीय पहचान, आकांक्षा अथवा किसी खास क्षेत्रीय समस्या को आधार बनाकर लोगों की भावनाओं की नुमाइंदगी करें। लोकतान्त्रिक व्यवस्था में क्षेत्रीय राजनीति का एक अर्थ यह भी है कि क्षेत्रीय मुद्दों और समस्याओं पर नीति निर्माण की प्रक्रिया में समुचित ध्यान दिया जाए और उन्हें इसमें भागीदारी भी दी जाए। दूसरी तरफ साम्प्रदायिकता धार्मिक या भाषायी समुदाय पर आधारित एक संकीर्ण मनोवृत्ति है जो धार्मिक या भाषायी अधिकारों तथा हितों को राष्ट्रीय हितों के ऊपर रखती है इसलिए यह समाज विरोधी तथा राष्ट्रविरोधी मनोवृत्ति है। इस प्रकार यह कहा जा सकता है कि क्षेत्रवाद साम्प्रदायिकता के समान खतरनाक नहीं है।

पृष्ठ 150

प्रश्न 2.
खतरे की बात हमेशा सीमांत के राज्यों के संदर्भ में ही क्यों उठाई जाती है? क्या इस सबके पीछे विदेशी हाथ ही होता है?
उत्तर:
प्रायः यह देखा गया है कि सीमांत प्रदेशों में ही क्षेत्रवाद एवं अलगाववाद की समस्या अधिक पायी जाती है। इसका मुख्य कारण वहाँ की जनता की राजनीतिक आकांक्षाओं के साथ-साथ विदेशी ताकतों का हाथ होना माना जा सकता है। जम्मू-कश्मीर, पंजाब और पूर्वोत्तर के विभिन्न राज्यों में घटित होने वाली विभिन्न घटनाएँ इसका उदाहरण माना जा सकता है। कश्मीरियों द्वारा अलग से स्वतन्त्र राष्ट्र की और पंजाब द्वारा खालिस्तान की माँग के पीछे विदेशी ताकतों का हाथ रहा है।

JAC Class 12 Political Science Solutions Chapter 8 क्षेत्रीय आकांक्षाएँ

पृष्ठ 158

प्रश्न 3.
पर यह सारी बात तो सरकार, अधिकारियों, नेताओं और आतंकवादियों के बारे में है। कश्मीरी जनता के बारे में कोई कुछ क्यों नहीं कहता? लोकतन्त्र में तो जनता की इच्छा को महत्त्व दिया जाना चाहिए। क्यों मैं ठीक कह रही हूँ न?
उत्तर:
जम्मू-कश्मीर में अलगाववादियों का एक तबका कश्मीर को एक अलग राष्ट्र बनाना चाहता है; दूसरा कश्मीर का विलय पाकिस्तान में चाहता है तथा तीसरा तबका कश्मीर को भारत संघ का ही हिस्सा रहने देना चाहता है, लेकिन अधिक स्वायत्तता चाहता है। जम्मू और लद्दाख के लोग उपेक्षित बरताव तथा पिछड़ेपन से छुटकारा हेतु तबका, अधिक स्वायत्तता चाहते हैं। इस वजह से पूरे राज्य की स्वायत्तता की माँग जितनी प्रबल है उतनी ही प्रबल माँग इस राज्य के विभिन्न भागों में अपनी-अपनी स्वायत्तता को लेकर है।

जम्मू-कश्मीर पर प्रायः अलगाववादियों, आतंकवादियों एवं विभिन्न राजनीतिज्ञों की प्रशासनिक नीतियों का प्रभाव रहा है लेकिन वहाँ जनमत की इच्छा का सम्मान करते हुए जम्मू-कश्मीर के लोगों की इच्छानुसार प्रशासनिक निर्णय लिया जाना चाहिए अर्थात् जम्मू-कश्मीर के संदर्भ में जनमत संग्रह की नीति का पालन किया जाना चाहिए। जिससे इस क्षेत्र की समस्त समस्याओं का स्थायी समाधान हो सके।

Jharkhand Board Class 12 Political Science क्षेत्रीय आकांक्षाएँ TextBook Questions and Answers

प्रश्न 1.
निम्नलिखित में मेल करें:

(क) सामाजिक-धार्मिक पहचान के आधार पर राज्य का निर्माण (i) नगालैंड/मिजोरम
(ख) भाषायी पहचान और केंद्र के साथ तनाव (ii) झारखंड / छत्तीसगढ़
(ग) क्षेत्रीय असंतुलन के फलस्वरूप राज्य का निर्माण (iii) पंजाब
(घ) आदिवासी पहचान के आधार पर अलगाववादी माँग (iv) तमिलनाडु

उत्तर:

(क) सामाजिक-धार्मिक पहचान के आधार पर राज्य का निर्माण (iii) पंजाब
(ख) भाषायी पहचान और केंद्र के साथ तनाव (iv) तमिलनाडु
(ग) क्षेत्रीय असंतुलन के फलस्वरूप राज्य का निर्माण (i) झारखंड / छत्तीसगढ़
(घ) आदिवासी पहचान के आधार पर अलगाववादी माँग (ii) नगालैंड/मिजोरम

प्रश्न 2.
पूर्वोत्तर के लोगों की क्षेत्रीय आकांक्षाओं की अभिव्यक्ति कई रूपों में होती है। बाहरी लोगों के खिलाफ आंदोलन, ज्यादा स्वायत्तता की माँग के आंदोलन और अलग देश बनाने की माँग करना ऐसी ही कुछ अभिव्यक्तियाँ हैं। पूर्वोत्तर के मानचित्र पर इन तीनों के लिये अलग-अलग रंग भरिये और दिखाइये कि किस राज्य में कौन-सी प्रवृत्ति ज्यादा प्रबल है?
उत्तर:

  1. बाहरी लोगों के खिलाफ आन्दोलन – असम
  2. ज्यादा स्वायत्तता की माँग के आन्दोलन – मेघालय
  3. अलग देश बनाने की माँग – मिजोरम

JAC Class 12 Political Science Solutions Chapter 8 क्षेत्रीय आकांक्षाएँ 1

(अलग देश बनाने की माँग नगालैण्ड व मिजोरम ने की थी। कुछ समय पश्चात् मिजोरम स्वायत्त राज्य बनने के लिए तैयार हो गया। मेघालय, मिजोरम, अरुणाचल प्रदेश, त्रिपुरा और मणिपुर को राज्य का दर्जा दे दिया गया है।) अतः मिजोरम की माँग का समाधान हो गया है, लेकिन नगालैण्ड में अलगाववाद की समस्या का पूर्ण हल अभी तक नहीं हो पाया है।

JAC Class 12 Political Science Solutions Chapter 8 क्षेत्रीय आकांक्षाएँ

प्रश्न 3.
पंजाब समझौते के मुख्य प्रावधान क्या थे? क्या ये प्रावधान पंजाब और उसके पड़ौसी राज्यों के बीच तनाव बढ़ाने के कारण बन सकते हैं? तर्क सहित उत्तर दीजिए।
उत्तर:
पंजाब समझौता: जुलाई, 1985 में अकाली दल के तत्कालीन अध्यक्ष हरचंद सिंह लोंगोवाल और प्रधानमंत्री राजीव गांधी के बीच एक समझौता हुआ जिसे पंजाब – समझौता कहा जाता है। इस समझौते के प्रमुख प्रावधान निम्नलिखित हैं-

  1. मारे गये निरपराध व्यक्तियों के लिए मुआवजा: एक सितम्बर, 1982 के बाद हुई किसी कार्यवाही या आन्दोलन में मारे गए लोगों को अनुग्रह राशि के भुगतान के साथ सम्पत्ति की क्षति के लिए मुआवजा दिया जाएगा।
  2. सेना में भर्ती; देश के सभी नागरिकों को सेना में भर्ती का अधिकार होगा और चयन के लिए केवल योग्यता ही आधार रहेगा।
  3. सेना से निकाले हुए व्यक्तियों का पुनर्वास: सेना से निकाले हुए व्यक्तियों और उन्हें लाभकारी रोजगार दिलाने के प्रयास किए जाएंगे।
  4. विशेष सुरक्षा अधिनियम को वापस लेने पर सहमति:  पंजाब में उग्रवाद प्रभावित लोगों को मुआवजा प्रदान करने तथा उनके साथ अच्छा व्यवहार करने को तैयार हो गई तथा पंजाब से विशेष सुरक्षा अधिनियम को वापस लेने की बात पर भी सहमत हो गई।
  5. सीमा विवाद:  चण्डीगढ़ की राजधानी परियोजना क्षेत्र और सुखना ताला पंजाब को दिए जाएंगे । केन्द्र शासित प्रदेश के अन्य पंजाबी क्षेत्र पंजाब को तथा हिन्दी भाषी क्षेत्र हरियाणा को दिए जाएंगे।
    ये प्रावधान पंजाब और उसके पड़ौसी राज्यों के बीच तनाव बढ़ाने का कारण नहीं बन सकते हैं क्योंकि इसमें इन राज्यों के विवादास्पद मुद्दों को नहीं छुआ गया है।

प्रश्न 4.
आनन्दपुर साहिब प्रस्ताव के विवादास्पद होने के क्या कारण थे?
उत्तर:
आनन्दपुर साहिब प्रस्ताव के विवादास्पद होने का मुख्य कारण यह था कि इस प्रस्ताव में पंजाब सूबे के लिए अधिक स्वायत्तता की मांग की गई, जो कि परोक्ष रूप से एक अलग सिख राष्ट्र की माँग को बढ़ावा देती है। दूसरे यह प्रस्ताव ‘सिख वर्चस्व’ का ऐलान करता था।

प्रश्न 5.
आपकी राय में कश्मीर में क्षेत्रीय आकांक्षाएँ उभर आने के कारण स्पष्ट कीजिए।
अथवा
जम्मू-कश्मीर की अंदरूनी विभिन्नताओं की व्याख्या कीजिए और बताइए कि इन विभिन्नताओं के कारण इस राज्य में किस तरह अनेक क्षेत्रीय आकांक्षाओं ने सिर उठाया है।
उत्तर:
अंदरूनी विभिन्नताएँ:
जम्मू-कश्मीर में अधिकांश रूप में अंदरूनी विभिन्नताएँ पायी जाती हैं। जम्मू- कश्मीर राज्य में तीन राजनीतिक एवं सामाजिक क्षेत्र – जम्मू कश्मीर और लद्दाख शामिल हैं। जम्मू पहाड़ी क्षेत्र है, इसमें हिन्दू-मुस्लिम और सिक्ख अर्थात् कई धर्मों एवं भाषाओं के लोग रहते हैं। कश्मीर में मुस्लिम समुदाय की जनसंख्या अधिक है और यहाँ पर हिन्दू अल्पसंख्यक हैं। जबकि लद्दाख पर्वतीय क्षेत्र है, इसमें बौद्ध, मुस्लिम की आबादी है। इतनी विभिन्नताओं के कारण यहाँ पर कई क्षेत्रीय आकांक्षाओं ने सिर उठाया है। यथा

  1. इसमें पहली आकांक्षा कश्मीरी पहचान की है जिसे कश्मीरियत के रूप में जाना जाता है। कश्मीर के निवासी सबसे पहले अपने को कश्मीरी तथा बाद में कुछ और मानते थे
  2. राज्य में उग्रवाद और आतंकवाद को दूर करना भी यहाँ के लोगों की एक मुख्य आकांक्षा है।
  3. स्वायत्तता की बात जम्मू, कश्मीर और लद्दाख के लोगों को अलग-अलग ढंग से लुभाती है। यहाँ पूरे राज्य में स्वायत्तता की माँग जितनी प्रबल है, उतनी ही प्रबल माँग राज्य के विभिन्न भागों में अपनी-अपनी स्वायत्तता को लेकर है। जम्मू-कश्मीर में कई राजनीतिक दल हैं, जो जम्मू-कश्मीर के लिए स्वायत्तता की माँग करते रहते हैं। इसमें नेशनल कांफ्रेंस सबसे महत्त्वपूर्ण दल है।
  4. इसके अतिरिक्त कुछ उग्रवादी संगठन भी हैं, जो धर्म के नाम पर जम्मू-कश्मीर को भारत से अलग करना चाहते हैं

प्रश्न 6.
कश्मीर की क्षेत्रीय स्वायत्तता के मसले पर विभिन्न पक्ष क्या हैं? इनमें से कौनसा पक्ष आपको समुचित जान पड़ता है? अपने उत्तर के पक्ष में तर्क दीजिए।
उत्तर:
कश्मीर की स्वायत्तता का मसला- कश्मीर की क्षेत्रीय स्वायत्तता के मसले पर मुख्य रूप से दो पक्ष सामने आते हैं- प्रथम पक्ष वह है जो धारा 370 को समाप्त करना चाहता है, जबकि दूसरा पक्ष वह है जो इस राज्य को और अधिक स्वायत्तता देना चाहता हैं। यदि इन दोनों पक्षों का उचित ढंग से अध्ययन किया जाए तो प्रथम पक्ष अधिक उचित दिखाई पड़ता है जो धारा 370 को समाप्त करने के पक्ष में है। उनका तर्क है कि इस धारा के कारण यह राज्य भारत के साथ पूरी तरह नहीं मिल पाया है। इसके साथ-साथ जम्मू-कश्मीर को अधिक स्वायत्तता देने से कई प्रकार की राजनीतिक एवं सामाजिक समस्याएँ भी पैदा होती हैं।

JAC Class 12 Political Science Solutions Chapter 8 क्षेत्रीय आकांक्षाएँ

प्रश्न 7.
असम आन्दोलन सांस्कृतिक अभिमान और आर्थिक पिछड़ेपन की मिली-जुली अभिव्यक्ति था। व्याख्या कीजिए।
अथवा
ऑल असम स्टूडेंटस यूनियन (आ सू) द्वारा चलाये गये आंदोलनों की व्याख्या कीजिए ।
उत्तर:
असम आन्दोलन – 1979 से 1985 तक असम में बाहरी लोगों के खिलाफ चला, असम आंदोलन असम के सांस्कृतिक अभियान और आर्थिक पिछड़ेपन की मिली-जुली अभिव्यक्ति था क्योंकि-
1. असमी लोगों के मन में यह भावना घर कर गई थी कि असम में बांग्लादेश से आए विदेशी लोगों को पहचान कर उन्हें अपने देश नहीं भेजा गया तो स्थानीय असमी जनता अल्पसंख्यक हो जायेगी। यह उन्हें असमी संस्कृति पर ख़तरा दिखाई दे रहा था। अतः 1979 में ऑल असम स्टूडेंट यूनियन ने जब यह माँग की कि 1951 के बाद जितने भी लोग असम में आकर बसे हैं उन्हें असम से बाहर भेजा जाए, तो इस आंदोलन को पूरे असम में समर्थन मिला।

2. असम आंदोलन के पीछे आर्थिक मसले भी जुड़े थे। असम में तेल, चाय और कोयले जैसे प्राकृतिक संसाधनों की मौजूदगी के बावजूद व्यापक गरीबी थी। यहाँ की जनता ने माना कि असम के प्राकृतिक संसाधन बाहर भेजे जा रहे हैं और असमी लोगों को कोई फायदा नहीं हो रहा है। दूसरे, भारत के अन्य राज्यों से या किसी अन्य देश से आए लोग यहाँ के रोजगार के अवसरों को हथिया रहे हैं और यहाँ की जनता उनसे वंचित हो रही है।

प्रश्न 8.
हर क्षेत्रीय आन्दोलन अलगाववादी माँग की तरफ अग्रसर नहीं होता । इस अध्याय से उदाहरण देकर इस तथ्य की व्याख्या कीजिए।
उत्तर:
भारत के कई क्षेत्रों में काफी समय से कुछ क्षेत्रीय आन्दोलन चल रहे हैं, परन्तु सभी क्षेत्रीय आन्दोलन अलगाववादी आन्दोलन नहीं होते अर्थात् कुछ क्षेत्रीय आन्दोलन भारत से अलग नहीं होना चाहते बल्कि अपने लिए अलग राज्य की मांग करते हैं, जैसे—झारखण्ड मुक्ति मोर्चा का आन्दोलन, छत्तीसगढ़ के आदिवासियों द्वारा चलाया गया आन्दोलन तथा तेलंगाना प्रजा समिति द्वारा चलाया गया आन्दोलन इत्यादि ऐसे ही आन्दोलन रहे हैं और अपने क्षेत्र के लिए एक पृथक् राज्य के गठन के बाद ये आन्दोलन समाप्त हो गये हैं ।

प्रश्न 9.
भारत के विभिन्न भागों से उठने वाली क्षेत्रीय मांगों से ‘विविधता में एकता’ के सिद्धान्त की अभिव्यक्ति होती है। क्या आप इस कथन से सहमत हैं? तर्क दीजिए।
उत्तर:
हाँ, मैं इस कथन से सहमत हूँ कि भारत के विभिन्न भागों से उठने वाली क्षेत्रीय मांगों से विविधता में एकता के सिद्धान्त की अभिव्यक्ति होती है क्योंकि देश के विभिन्न क्षेत्रों से विभिन्न प्रकार की माँगें उठीं। तमिलनाडु में जहाँ हिन्दी भाषा के विरोध और अंग्रेजी भाषा को राष्ट्रभाषा के रूप में बनाए रखने की माँग उठी, तो असम में विदेशियों को असम से बाहर निकालने की माँग उठी, नगालैंड और मिजोरम में अलगाववाद की माँग उठी तो आंध्रप्रदेश, हरियाणा, हिमाचल प्रदेश व अन्य राज्यों में भाषा के आधार पर अलग राज्य बनाने की माँग उठी।

इसी प्रकार उत्तराखंड, छत्तीसगढ़ तथा झारखण्ड क्षेत्रों में प्रशासनिक सुविधा तथा आदिवासियों के हितों की रक्षा हेतु अलग राज्य बनाने के. आंदोलन हुए। अभी भी विभिन्न क्षेत्रों से भिन्न-भिन्न प्रकार की माँगें उठ रही हैं जिनका मुख्य आधार क्षेत्रीय पिछड़ापन है । इससे स्पष्ट होता है कि भारत के विभिन्न भागों से उठने वाली क्षेत्रीय माँगों से विविधता के दर्शन होते हैं। भारत सरकार ने इन माँगों के निपटारे के लिए लोकतांत्रिक दृष्टिकोण अपनाते हुए एकता का परिचय दिया है।

JAC Class 12 Political Science Solutions Chapter 8 क्षेत्रीय आकांक्षाएँ

प्रश्न 10.
नीचे लिखे अवतरण को पढ़ें और इसके आधार पर पूछे गए प्रश्नों के उत्तर दें:
हजारिका का एक गीत एकता की विजय पर है; पूर्वोत्तर के सात राज्यों को इस गीत में एक ही माँ की सात बेटियाँ कहा गया है. मेघालय अपने रास्ते गई अरुणाचल भी अलग हुई और मिजोरम असम के द्वार पर दूल्हे की तरह दूसरी बेटी से ब्याह रचाने को खड़ा है इस गीत का अंत असमी लोगों की एकता को बनाए रखने के संकल्प के साथ होता है और इसमें समकालीन असम में मौजूद छोटी-छोटी कौमों को भी अपने साथ एकजुट रखने की बात कही गई है। करबी और मिजिंग भाई-बहन हमारे ही प्रियजन हैं। संजीव बरुआ
(क) लेखक यहाँ किस एकता की बात कर रहा है?
(ख) पुराने राज्य असम से अलग करके पूर्वोत्तर के अन्य राज्य क्यों बनाए गए?
(ग) क्या आपको लगता है कि भारत के सभी क्षेत्रों के ऊपर एकता की यही बात लागू हो सकती है? क्यों?
उत्तर:
(क) लेखक यहाँ पर पूर्वोत्तर राज्यों की एकता की बात कर रहा है।

(ख) सभी समुदायों की सांस्कृतिक पहचान बनाए रखने के लिए तथा आर्थिक पिछड़ेपन को दूर करने के लिए पुराने राज्य असम से अलग करके पूर्वोत्तर के अन्य राज्य बनाए गए।

(ग) भारत के सभी क्षेत्रों पर एकता की यह बात लागू हो सकती है, क्योंकि भारत के सभी राज्यों में अलग- अलग धर्मों एवं जातियों के लोग रहते हैं तथा देश की एकता एवं अखण्डता के लिए उनमें एकता कायम करना आवश्यक है। दूसरे, भारत के संविधान में क्षेत्रीय विभिन्नताओं और आकांक्षाओं के लिए पर्याप्त व्यवस्था की गई है। तीसरे, देश के नेता तथा सभी राजनीतिक दल एकता के समर्थक हैं।

क्षेत्रीय आकांक्षाएँ JAC Class 12 Political Science Notes

→ क्षेत्र और राष्ट्र:
भारत में 1980 के दशक में देश के विभिन्न हिस्सों में स्वायत्तता की माँगें उठीं। लोगों ने स्वायत्तता की माँगों को लेकर संवैधानिक नियमों का उल्लंघन करते हुए हथियार तक उठाये। भारत सरकार ने इन मांगों को दबाने के लिए जवाबी कार्यवाही की, जिससे अनेक बार राजनीतिक तथा चुनावी प्रक्रिया अवरुद्ध हुई।

→ भारत सरकार का नजरिया:
राष्ट्र निर्माण की प्रक्रिया और भारत के संविधान के बारे में अध्ययन से यह ज्ञात हुआ कि देश में विभिन्न क्षेत्रों और भाषायी समूह को अपनी संस्कृति को बनाये रखने का अधिकार होगा। भारत ने एकता की भावधारा से बँधे एक ऐसे सामाजिक जीवन के निर्माण का निर्णय लिया जिससे एक समाज को आकार देने वाली तमाम संस्कृतियों की विशिष्टता बनी रहे।

→ तनाव के दायरे:
आजादी के बाद भारत में क्षेत्रीय तनाव के विभिन्न मुद्दे उभरे जिनमें प्रमुख हैं-

  • आजादी के तुरन्त बाद जम्मू-कश्मीर का मामला सामने आया। यह सिर्फ भारत और पाकिस्तान के मध्य संघर्ष का मामला नहीं था। कश्मीर घाटी के लोगों की राजनीतिक आकांक्षाओं का सवाल भी इससे जुड़ा हुआ था।
  • ठीक इसी प्रकार पूर्वोत्तर के कुछ भागों में भारत का अंग होने के मसले पर सहमति नहीं थी। पहले नगालैण्ड में और फिर मिजोरम में भारत से अलग होने की माँग करते हुए जोरदार आंदोलन चले।
  • दक्षिण भारत में द्रविड़ आन्दोलन से जुड़े कुछ समूहों ने एक समय अलग राष्ट्र की बात उठायी थी।
  • अलगाव के इन आन्दोलनों के अतिरिक्त देश में भाषा के आधार पर राज्यों के गठन की माँग करते हुए जन-आन्दोलन चले। मौजूदा कर्नाटक, महाराष्ट्र और गुजरात ऐसे ही आन्दोलनों वाले राज्यों में हैं।

→ जम्मू-कश्मीर:
जम्मू-कश्मीर भारत और पाकिस्तान के मध्य एक बड़ा मुद्दा है। 1947 से पहले जम्मू-कश्मीर में राजशाही थी। इसके हिन्दू शासक भारत में शामिल होना नहीं चाहते थे और उन्होंने अपने स्वतन्त्र राज्य के लिए भारत और पाकिस्तान के साथ समझौता करने की कोशिश की। कश्मीर समस्या की प्रमुख जड़ों को निम्न प्रकार समझा जा सकता है-

→  राज्य में नेशनल कान्फ्रेंस के शेख अब्दुल्ला के नेतृत्व में जन-आन्दोलन चला। शेख अब्दुल्ला चाहते थे कि महाराजा पद छोड़ें, लेकिन वे पाकिस्तान में शामिल होने के खिलाफ थे। नेशनल कान्फ्रेंस एक धर्मनिरपेक्ष संगठन था। इसका कांग्रेस के साथ काफी समय से गठबन्धन था। राष्ट्रीय राजनीति के कई प्रमुख नेता शेख अब्दुल्ला के मित्र थे। इनमें नेहरू भी शामिल थे।

→ अक्टूबर, 1947 में पाकिस्तान ने कबायली घुसपैठ को अपनी तरफ से कश्मीर पर कब्जा करने भेजा। ऐसे में महाराज भारतीय सेना से मदद माँगने को मजबूर हुए। भारत ने सैन्य मदद उपलब्ध करवाई और कश्मीर घाटी से घुसपैठियों को खदेड़ा। इससे पहले भारत सरकार ने महाराजा से भारत संघ में विलय के दस्तावेज पर हस्ताक्षर करा लिए। इस बारे में सहमति जताई गई कि स्थिति सामान्य होने पर जम्मू-कश्मीर की नियति का फैसला जनमत सर्वेक्षण द्वारा होगा। मार्च, 1948 में शेख अब्दुल्ला जम्मू-कश्मीर राज्य के प्रधानमन्त्री बने ( राज्य में सरकार के मुखिया को तब प्रधानमन्त्री कहा जाता था )। भारत, जम्मू एवं कश्मीर की स्वायत्तता को बनाए रखने पर सहमत हो गया। इसे संविधान में धारा 370 का प्रावधान करके संवैधानिक दर्जा दिया गया।

→ बाहरी और आन्तरिक दबाव:
जम्मू-कश्मीर की राजनीति हमेशा विवादग्रस्त एवं संघर्षयुक्त रही। इसमें बाहरी एवं आन्तरिक दोनों कारण हैं। कश्मीर समस्या का एक कारण पाकिस्तान का रवैया है। उसने हमेशा यह दावा किया है कि कश्मीर घाटी पाकिस्तान का हिस्सा होना चाहिए। 1947 में इस राज्य में पाकिस्तान ने कबायली हमला कराया। इसके परिणामस्वरूप राज्य का एक हिस्सा पाकिस्तानी नियन्त्रण में आ गया। भारत ने दावा किया कि यह क्षेत्र का अवैध अधिग्रहण है। पाकिस्तान ने इस क्षेत्र को आजाद कश्मीर कहा । 1947 के बाद कश्मीर भारत और पाकिस्तान के बीच संघर्ष का एक बड़ा मुद्दा रहा है। आन्तरिक रूप से देखें तो भारतीय संघ में कश्मीर की हैसियत को लेकर विवाद रहा। कश्मीर को संविधान में धारा 370 के अन्तर्गत विशेष स्थान दिया गया है। धारा 370 के तहत जम्मू एवं कश्मीर को भारत के अन्य राज्यों के मुकाबले ज्यादा स्वायत्तता दी गई है।

JAC Class 12 Political Science Solutions Chapter 8 क्षेत्रीय आकांक्षाएँ

→ 1948 के बाद की राजनीति:
प्रधानमन्त्री बनने के बाद शेख अब्दुल्ला ने कश्मीर में सुधार हेतु अनेक कदम उठाये। उन्होंने भूमि सुधार व जन-कल्याण हेतु अनेक कार्यक्रम चलाये लेकिन केन्द्र सरकार से मतभेद होने के कारण उन्हें 1953 में पद से हटा दिया गया। 1953 से लेकर 1974 तक राज्य की राजनीति पर कांग्रेस का असर रहा। विभाजित हो चुकी नेशनल कांफ्रेंस कांग्रेस के समर्थन से राज्य में कुछ समय तक सत्तासीन रही लेकिन बाद में यह कांग्रेस में मिल गयी। इस तरह राज्य की सत्ता सीधे कांग्रेस के नियन्त्रण में आ गई। इस बीच शेख अब्दुल्ला और भारत सरकार के बीच सुलह की कोशिश जारी रही। आखिरकार, 1974 में इन्दिरा गाँधी के साथ शेख अब्दुल्ला ने एक समझौते पर हस्ताक्षर किए और वे राज्य के मुख्यमन्त्री बने।

→ विद्रोही तेवर और उसके बाद:
1984 में विधानसभा चुनाव हुए। आधिकारिक नतीजे बता रहे थे कि नेशनल कांफ्रेंस और कांग्रेस गठबन्धन को भारी बहुमत मिला है। फारुख अब्दुल्ला मुख्यमंत्री बने। बहराल लोग यह मान रहे थे कि चुनावों में धाँधली हुई है और चुनाव परिणाम जनता की पसंद की नुमाइंदगी नहीं कर रहे। 1980 के दशक से ही यहाँ के लोगों में प्रशासनिक अक्षमता को लेकर रोष पनप रहा था। 1989 तक राज्य उग्रवादी आन्दोलन की गिरफ्त में आ चुका था। इस आन्दोलन में लोगों को अलग कश्मीर के नाम पर लामबंद किया जा रहा था। 1990 के बाद से इस राज्य के लोगों को उग्रवादियों और सेना की हिंसा भुगतनी पड़ी। 1996 में एक बार फिर इस राज्य में विधानसभा चुनाव हुए। फारुख अब्दुल्ला के नेतृत्व में नेशनल कांफ्रेंस की सरकार बनी और उसने जम्मू-कश्मीर के लिए क्षेत्रीय स्वायत्तता की माँग की। जम्मू-कश्मीर में 2002 के चुनाव बड़े निष्पक्ष ढंग से हुए । नेशनल कांफ्रेंस को बहुमत नहीं मिल पाया। इस चुनाव में पीपुल्स डेमोक्रेटिक अलायंस (पीडीपी) और कांग्रेस की गठबन्धन सरकार सत्ता में आई।

→ 2002 और इससे आगे:
गठबंधन के अनुसार मुफ्ती मोहम्मद तीन वर्षों तक सरकार के मुखिया रहे, इसके बाद गुलामनबी आजाद मुखिया बने। 2008 में कश्मीर में राष्ट्रपति शासन लगा दिया गया और 2008 के नवंबर-दिसंबर में चुनाव करवाया गया। उमर अब्दुल्ला मुखिया बने। 2014 के चुनाव में पीडीपी और बीजेपी का गठबंधन हुआ और मिली-जुली सरकार सत्ता में आई । 2018 में बीजेपी ने अपना सहयोग वापस ले लिया। फलस्वरूप कश्मीर में राष्ट्रपति शासन लगाना पड़ा। 5 अगस्त, 2019 को जम्मू और कश्मीर पुनर्गठन अधिनियम 2019 द्वारा अनुच्छेद 370 समाप्त कर दिया गया। राज्य को पुनर्गठित कर दो केन्द्र शासित प्रदेश जम्मू और कश्मीर तथा लद्दाख बना दिये गये।

→ पंजाब:
1980 के दशक में पंजाब में भी बड़े बदलाव आए। इस प्रान्त की सामाजिक बनावट विभाजन के समय पहली बार बदली थी। 1966 में पंजाबी भाषी प्रान्त का निर्माण हुआ। सिखों की राजनीतिक शाखा के रूप में 1920 के दशक में अकाली दल का गठन हुआ था। अकाली दल ने पंजाबी सूबा के गठन का आन्दोलन चलाया। पंजाबी- भाषी सूबे में सिख बहुसंख्यक हो गये। राजनीतिक संदर्भ-पंजाबी सूबे के पुनर्गठन के बाद अकाली दल ने यहाँ 1967 और इसके बाद 1977 में सरकार बनाई। 1970 के दशक में अकालियों के एक तबके ने पंजाब के लिए स्वायत्तता की माँग उठायी। 1973 में आनंदपुर साहिब में हुए एक सम्मेलन में क्षेत्रीय स्वायत्तता की माँग उठायी गयी। 1980 में अकाली दल की सरकार बर्खास्त हो गई तो अकाली दल ने पंजाब तथा पड़ौसी राज्यों के बीच पानी के बँटवारे को लेकर आन्दोलन चलाया।

→ हिंसा का चक्र:
जल्दी ही अकाली आन्दोलन का नेतृत्व नरम पंथियों के हाथों से निकल कर चरमपंथियों के हाथों में आ गया और इस आन्दोलन ने सशस्त्र विद्रोह का रूप ले लिया। उग्रवादियों ने अमृतसर स्थित सिखों के तीर्थ स्थल स्वर्णमन्दिर में अपना मुख्यालय बनाया और स्वर्णमन्दिर एक हथियारबंद किले में तब्दील हो गया। 1984 के जून माह में भारत सरकार ने ऑपरेशन ब्लू स्टार चलाया । यह स्वर्णमन्दिर में की गई सैन्य कार्यवाही का कूट नाम था । कुछ और त्रासद घटनाओं ने पंजाब की समस्या को एक जटिल रास्ते पर खड़ा कर दिया। प्रधानमंत्री इंदिरा गाँधी की 31 अक्टूबर, 1984 के दिन उनके आवास के बाहर उन्हीं के अंगरक्षकों ने हत्या कर दी। श्रीमती इंदिरा गाँधी की हत्या के कारण दिल्ली में बड़े पैमाने पर सिख विरोधी दंगे हुए जिसमें लगभग 2000 सिख स्त्री-पुरुष एवं बच्चे मारे गये।

JAC Class 12 Political Science Solutions Chapter 8 क्षेत्रीय आकांक्षाएँ

→ शांति की ओर:
984 के चुनावों के बाद तत्कालीन प्रधानमन्त्री राजीव गाँधी ने नरमपंथी अकाली नेताओं से बातचीत की शुरुआत की। अकाली दल के तत्कालीन अध्यक्ष हरचरन सिंह लोंगोवाल के साथ 1985 की जुलाई में एक समझौता हुआ। इस समझौते को राजीव गाँधी लोंगोवाल समझौता अथवा पंजाब समझौता कहा जाता है । यह समझौता पंजाब में अमन कायम करने की क्रिया में एक महत्त्वपूर्ण कदम था। लेकिन पंजाब में हिंसा का चक्र लगभग एक दशक तक चलता रहा। केन्द्र सरकार ने पंजाब में राष्ट्रपति शासन लगा दिया। इससे सामान्य राजनीतिक तथा चुनावी प्रक्रिया बाधित हुई। 1992 में पंजाब के चुनावों में 24 प्रतिशत मतदाता मत डालने आए। 1990 के दशक के मध्यवर्ती वर्षों में पंजाब में शांति बहाल हुई तथा 1997 में चुनाव में लोगों ने बढ़ चढ़कर भाग लिया। अब पंजाब में पुनः आर्थिक विकास और सामाजिक परिवर्तन के मुद्दे प्रमुख हो गये हैं।

→ पूर्वोत्तर:
पूर्वोत्तर राज्यों में क्षेत्रीय आकांक्षाएँ 1980 के दशक में एक निर्णायक मोड़ पर आ गई थीं। क्षेत्र में सात राज्य हैं। इन्हें ‘सात बहनें’ कहा जाता है। इस क्षेत्र में देश की कुल 4 फीसदी आबादी निवास करती है। लेकिन भारत के कुल क्षेत्रफल में पूर्वोत्तर के हिस्से को देखते हुए यह आबादी दोगुनी कही जाएगी। 22 किलोमीटर लंबी एक पतली सी राहदारी इस इलाके को शेष भारत से जोड़ती है अन्यथा इस क्षेत्र की सीमाएँ चीन, म्यांमार और बांग्लादेश से लगती हैं और यह इलाका भारत के लिए एक तरह से दक्षिण-पूर्वी एशिया का प्रवेश द्वार है।

इस इलाके में 1947 के बाद अनेक परिवर्तन आये हैं। पूर्वोत्तर के पूरे इलाके का बड़े व्यापक स्तर पर राजनीतिक पुनर्गठन हुआ है। नगालैण्ड को 1963 में राज्य बनाया गया। मेघालय, मणिपुर और त्रिपुरा 1972 में राज्य बने जबकि अरुणाचल प्रदेश को 1987 में राज्य का दर्जा दिया गया। 1947 के भारत विभाजन से पूर्वोत्तर के इलाके भारत के शेष भागों से एकदम अलग-थलग पड़ गए और इसका अर्थव्यवस्था पर दुष्प्रभाव पड़ा। पूर्वोत्तर के राज्यों में राजनीति में तीन मुद्दे हावी हैं

  • स्वायत्तता की माँग,
  • अलगाव के आन्दोलन तथा
  • बाहरी लोगों का विरोध।

→ स्वायत्तता की माँग:
आजादी के वक्त मणिपुर और त्रिपुरा को छोड़ दें तो यह पूरा इलाका असम कहलाता था। गैर- असमी लोगों को जब लगा कि असम की सरकार उन पर असमी भाषा थोप रही है तो इस इलाके से राजनीतिक स्वायत्तता की माँग उठी। पूरे राज्य में असमी भाषा को लादने के खिलाफ विरोध प्रदर्शन और दंगे हुए। बड़े जनजाति समुदाय के नेता असम से अलग होना चाहते थे। इन लोगों ने ‘ईस्टर्न इंडिया ट्राइबल यूनियन’ का गठन किया जो 1960 में कहीं ज्यादा व्यापक ‘आल पार्टी हिल्स कांफ्रेंस’ में बदल गया। इन नेताओं की माँग थी कि असम से

→ अलग एक जन:
जातीय राज्य बनाया जाए। आखिरकार एक जनजातीय राज्य की जगह असम को काट कर कई जनजातीय राज्य बने। केन्द्र सरकार ने अलग-अलग वक्त पर असम को बाँटकर मेघालय, मिजोरम और अरुणाचल प्रदेश बनाया। 1972 तक पूर्वोत्तर का पुनर्गठन पूरा हो चुका था लेकिन स्वायत्तता की माँग खत्म नहीं हुई। उदाहरण के लिए, असम के बोडो, करबी और दिमसा जैसे समुदायों ने अपने लिए अलग राज्य की माँग की। अपनी माँग के पीछे उन्होंने जनमत तैयार करने के प्रयास किए, जन-आंदोलन चलाए और विद्रोही कार्यवाहियाँ भी कीं।
संघीय राजव्यवस्था के कुछ और प्रावधानों का उपयोग करके स्वायत्तता की मांग को सन्तुष्ट करने की कोशिश की गई और इन समुदायों को असम में ही रखा गया। करबी और दिमसा समुदायों को जिला परिषद् के अन्तर्गत स्वायत्तता दी गई। बोडो जनजाति को हाल ही में स्वायत्त परिषद् का दर्जा दिया गया है।

→ अलगाववादी आंदोलन:
पूर्वोत्तर राज्यों में स्वायत्तता की माँग के साथ – साथ अलगाववादी ताकतों का भी बोलबाला बढ़ा। अलगाववादी देश से अलग होकर अपने को एक अलग राष्ट्र के रूप में प्रतिस्थापित करना चाहते थे। भारत के पूर्वोत्तर के राज्य असम, अरुणाचल प्रदेश, मेघालय, मणिपुर, नगालैण्ड, मिजोरम, त्रिपुरा में इस प्रकार की माँगें उठती रहती हैं।

→ बाहरी लोगों के खिलाफ आन्दोलन:
पूर्वोत्तर में बड़े पैमाने पर अप्रवासी भारतीय आये हैं। इससे एक खास समस्या उत्पन्न हुई है। स्थानीय जनता इन्हें बाहरी समझती है और बाहरी लोगों के खिलाफ उसके मन गुस्सा है। भारत के दूसरे राज्यों अथवा किसी अन्य देश से आये लोगों को यहाँ की जनता रोजगार के अवसरों और राजनीतिक सत्ता के एतबार से एक प्रतिद्वंद्वी के रूप में देखती है। स्थानीय लोग बाहर से आये लोगों के बारे में मानते हैं कि ये लोग यहाँ की जमीन हथिया रहे हैं। पूर्वोत्तर के कई राज्यों में इस मामले ने राजनीतिक रंग ले लिया है और कभी-कभी इन बातों के कारण हिंसक घटनाएँ भी होती हैं।

→ समाहार और राष्ट्रीय अखण्डता:
आजादी के छह दशक बाद भी राष्ट्रीय अखण्डता के कुछ मामलों का समाधान पूरी तरह से नहीं हो पाया। क्षेत्रीय आकांक्षाएँ लगातार एक न एक रूप से उभरती रहीं। कभी कहीं से अलग राज्य बनाने की माँग उठी तो कहीं आर्थिक विकास का मसला उठा। कहीं-कहीं से अलगाववाद के स्वर उभरे। 1980 के बाद के दौर में भारत की राजनीति इन तनावों के घेरे में रही और समाज के विभिन्न तबके की माँगों में पटरी बैठा पाने की लोकतान्त्रिक राजनीति की क्षमता की परीक्षा हुई।

JAC Class 12 Political Science Solutions Chapter 6 लोकतांत्रिक व्यवस्था का संकट

Jharkhand Board JAC Class 12 Political Science Solutions Chapter 6 लोकतांत्रिक व्यवस्था का संकट Textbook Exercise Questions and Answers.

JAC Board Class 12 Political Science Solutions Chapter 6 लोकतांत्रिक व्यवस्था का संकट

Jharkhand Board Class 12 Political Science लोकतांत्रिक व्यवस्था का संकट InText Questions and Answers

पृष्ठ 104

प्रश्न 1.
गरीब जनता पर सचमुच भारी मुसीबत आई होगी। आखिर गरीबी हटाओ के वादे का हुआ क्या? उत्तर – श्रीमती गांधी ने 1971 के आम चुनावों में ‘गरीबी हटाओ’ का नारा दिया था लेकिन इस नारे के बावजूद भी 1971-72 के बाद के वर्षों में भी देश की सामाजिक-आर्थिक दशा में सुधार नहीं हुआ और यह नारा खोखला साबित हुआ। इसी अवधि में अन्तर्राष्ट्रीय बाजार में तेल की कीमतों में कई गुना बढ़ोतरी हुई। इससे विभिन्न चीजों की कीमतों में तेजी आई। इस तीव्र मूल्य वृद्धि से लोगों को भारी कठिनाई हुई । बांग्लादेश के संकट से भी भारत की अर्थव्यवस्था पर भारी बोझ पड़ा था। लगभग 80 लाख लोग पूर्वी पाकिस्तान की सीमा पार करके भारत आ गये थे । इसके बाद पाकिस्तान से युद्ध भी करना पड़ा। फलतः गरीबी हटाओ कार्यक्रम के लिए दिये जाने वाले अनुदान में कटौती कर दी गई और यह नारा पूर्णतः असफल साबित हुआ।

पृष्ठ 107

प्रश्न 2.
क्या ‘प्रतिबद्ध न्यायपालिका’ और ‘प्रतिबद्ध नौकरशाही’ का मतलब यह है कि न्यायाधीश और सरकारी अधिकारी शासक दल के प्रति निष्ठावान हो?
उत्तर:
प्रतिबद्ध नौकरशाही के अन्तर्गत नौकरशाही किसी विशिष्ट राजनीतिक दल के सिद्धान्तों से बंधी हुई होती है और उस दल के निर्देशन में कार्य करती है। प्रतिबद्ध नौकरशाही निष्पक्ष एवं स्वतन्त्र होकर कार्य नहीं करती; बल्कि इसका कार्य किसी दल विशेष की योजनाओं को बिना किसी प्रश्न उठाए आँखें मूंद कर लागू करना होता है। जहाँ तक प्रतिबद्ध न्यायपालिका का सवाल है यह ऐसी न्यायपालिका होती है, जो एक दल विशेष या सरकार विशेष के प्रति वफादार हो तथा सरकार के निर्देशों के अनुसार चले। इस प्रकार ऐसी व्यवस्था में न्यायपालिका व व्यवस्थापिका की स्वतन्त्रता पर प्रश्नचिह्न लग जाता है और प्रशासन निरंकुश हो जाता है अर्थात् कानून बनाने एवं फैसला या निर्णय देने की शक्ति केवल एक ही संस्था या दल के पास आ जाती है। इस प्रकार की व्यवस्था प्रायः साम्यवादी देशों में पायी जाती है।

पृष्ठ 108

प्रश्न 3.
यह तो सेना से सरकार के खिलाफ बगावत करने को कहने जैसा जान पड़ता है। क्या यह बात लोकतांत्रिक है?
उत्तर:
नहीं, यह बात लोकतंत्र के खिलाफ है।

पृष्ठ 109

प्रश्न 4.
क्या राष्ट्रपति को मन्त्रिमण्डल की सिफारिश के बगैर आपातकाल की घोषणा करनी चाहिए थी? कितनी अजीब बात है!
उत्तर:
भारतीय संविधान के अनुच्छेद 352, 356 तथा 360 में राष्ट्रपति की आपातकालीन शक्तियों का उल्लेख किया गया है। भारत में 1975 में अनुच्छेद 352 के तहत आपातकाल की घोषणा की गई जिसमें मन्त्रिमण्डल से सलाह करके आपातकालीन स्थिति की घोषणा का प्रावधान है। कुछ विशेषज्ञों का मानना है कि तत्कालीन प्रधानमंत्री श्रीमती इंदिरा गांधी ने बिना मंत्रिमण्डल की सलाह के राष्ट्रपति को आपातकाल की घोषणा करने की सलाह दी थी, मन्त्रिमण्डल की बैठक उसके बाद हुई। इस प्रकार तत्कालीन परिस्थितियों में चाहें कुछ भी हुआ हो लेकिन वर्तमान में आपातकाल के प्रावधानों में सुधार कर लिया गया है। अंब आंतरिक आपातकाल सिर्फ सशस्त्र विद्रोह की स्थिति में ही लगाया जा सकता है। इसके लिए भी आपातकाल की घोषणा की सलाह मंत्रिपरिषद् को राष्ट्रपति को लिखित में देनी होगी।

JAC Class 12 Political Science Solutions Chapter 6 लोकतांत्रिक व्यवस्था का संकट

पृष्ठ 112

प्रश्न 5.
अरे ! सर्वोच्च न्यायालय ने भी साथ छोड़ दिया! उन दिनों सबको क्या हो गया था?
उत्तर:
आपातकाल के दौरान नागरिकों की स्वतन्त्रता पर प्रतिबन्ध लगा दिये गये तथा मौलिक अधिकार निष्प्रभावी हो गये। लेकिन अनेक उच्च न्यायालयों ने फैसला दिया कि आपातकाल की घोषणा के बावजूद अदालत किसी व्यक्ति द्वारा दायर की गई ऐसी बंदी प्रत्यक्षीकरण याचिका को विचार के लिए स्वीकार कर सकती है जिसमें उसने अपनी गिरफ्तारी को चुनौती दी हो। 1976 के अप्रेल माह में सर्वोच्च न्यायालय की संवैधानिक पीठ ने उच्च न्यायालयों के फैसले को उलट दिया और सरकार की दलील मान ली। इसका आशय यह था कि सरकार आपातकाल के दौरान नागरिकों से जीवन और आजादी का अधिकार वापस ले सकती है। इस फैसले को सर्वोच्च न्यायालय के सर्वाधिक विवादास्पद फैसलों में से एक माना गया। सर्वोच्च न्यायालय के इस फैसले से नागरिकों के लिए अदालत के दरवाजे बंद हो गए अर्थात् इस काल में सर्वोच्च न्यायालय ने भी जनता का साथ छोड़ दिया था।

पृष्ठ 113

प्रश्न 6.
जिन चंद लोगों ने प्रतिरोध किया, उनकी बात छोड़ दें- बाकियों के बारे में सोचें कि उन्होंने क्या किया ! क्या कर रहे थे बड़े-बड़े अधिकारी, बुद्धिजीवी, सामाजिक-धार्मिक नेता और नागरिक …………..?
उत्तर:
आपातकाल के दौरान नागरिकों के मौलिक अधिकार समाप्त कर दिए गए थे। जिन्होंने विरोध किया उनको जेल में डाल दिया गया, कई नेताओं और बुद्धिजीवियों को नजरबंद कर दिया गया। सरकार के विरोध में कोई भी स्वर उठाता उसको जेल में यातनाएँ दी जातीं। कुछ नेता जो गिरफ्तारी से बच गए वे भूमिगत होकर सरकार के खिलाफ मुहिम जारी रखी। कुछ ने आपातकाल के विरोध में अपनी पदवी लौटा दी। अधिकारी सरकार के प्रति वफादार बने रहे और ऐसा न करने पर उनका निलंबन कर दिया जाता था।

पृष्ठ 121

प्रश्न 7.
अगर उत्तर और दक्षिण के राज्यों में मतदाताओं ने इतने अलग ढर्रे पर मतदान किया, तो हम कैसे कहें कि 1977 के चुनावों का जनादेश क्या था?
उत्तर:
1977 के चुनावों में आजादी के बाद पहली बार कांग्रेस लोकसभा का चुनाव हारी। कांग्रेस को लोकसभा की मात्र 154 सीटें मिलीं। उसे 35 प्रतिशत से भी कम वोट प्राप्त हुए। जनता पार्टी और उसके साथी दलों . को 330 सीटें प्राप्त हुईं। लेकिन तत्कालीन चुनावी नतीजों पर प्रकाश डालें तो यह एहसास होता है कि कांग्रेस देश में हर जगह चुनाव नहीं हारी थी । महाराष्ट्र, गुजरात और उड़ीसा में उसने कई सीटों पर अपना कब्जा बरकरार रखा था और दक्षिण भारत के राज्यों में तो उसकी स्थिति काफी मजबूत थी। इसका मुख्य कारण यह था कि दक्षिण के राज्यों में आपातकाल के दौरान ज्यादतियाँ नहीं हुई थीं। इस आधार पर हम कह सकते हैं कि चुनावों का जनादेश आपातकाल की ज्यादतियों व उसके दुरुपयोग के विरुद्ध था।

JAC Class 12 Political Science Solutions Chapter 6 लोकतांत्रिक व्यवस्था का संकट

पृष्ठ 122

प्रश्न 8.
मैं समझ गया! आपातकाल एक तरह से तानाशाही निरोधक टीका था। इसमें दर्द हुआ और बुखार भी आया, लेकिन अन्ततः हमारे लोकतन्त्र के भीतर क्षमता बढ़ी।
उत्तर:
आपातकाल से सबक – आपातकाल के प्रमुख सबक निम्नलिखित हैं-

  1. आपातकाल का प्रथम सबक तो यही है कि भारत से लोकतन्त्र को विदा कर पाना बहुत कठिन है।
  2. दूसरे आपातकाल से संविधान में वर्णित आपातकाल के प्रावधानों के कुछ अर्थगत उलझाव भी प्रकट हुए, जिन्हें बाद में सुधार लिया गया। अब आंतरिक आपातकाल सिर्फ सशस्त्र विद्रोह की स्थिति में लगाया जा सकता है। इसके लिए यह भी जरूरी है कि आपातकाल की घोषणा की सलाह मंत्रिपरिषद् राष्ट्रपति को लिखित में दे।
  3. तीसरे आपातकाल से हर कोई नागरिक अधिकारों के प्रति ज्यादा सचेत हुआ। आपातकाल की समाप्ति के बाद अदालतों ने व्यक्ति के नागरिक अधिकारों की रक्षा में सक्रिय भूमिका निभाई।

Jharkhand Board Class 12 Political Science लोकतांत्रिक व्यवस्था का संकट TextBook Questions and Answers

प्रश्न 1.
बताएँ कि आपातकाल के बारे में निम्नलिखित कथन सही हैं या गलत-
(क) आपातकाल की घोषणा 1975 में इंदिरा गाँधी ने की।
(ख) आपातकाल में सभी मौलिक अधिकार निष्क्रिय हो गए।
(ग) बिगड़ती हुई आर्थिक स्थिति के मद्देनजर आपातकाल की घोषणा की गई थी।
(घ) आपातकाल के दौरान विपक्ष के अनेक नेताओं को गिरफ्तार कर लिया गया।
(ङ) सी. पी. आई. ने आपातकाल की घोषणा का समर्थन किया।
उत्तर:
(क) गलत,
(ख) सही,
(ग) गलत,
(घ) सही,
(ङ) सही।

प्रश्न 2.
निम्नलिखित में से कौन-सा आपातकाल की घोषणा के संदर्भ में मेल नहीं खाता है-
(क) ‘संपूर्ण क्रांति’ का आह्वान
(ख) 1974 की रेल – हड़ताल
(ग) नक्सलवादी आंदोलन
(घ) इलाहाबाद उच्च न्यायालय का फैसला
(ङ) शाह आयोग की रिपोर्ट के निष्कर्ष
उत्तर:
(ग) नक्सलवादी आंदोलन

प्रश्न 3.
निम्नलिखित में मेल बैठाएँ-

(क) संपूर्ण क्रांति (i) इंदिरा गाँधी
(ख) गरीबी हटाओ (ii) जयप्रकाश नारायण
(ग) छात्र आंदोलन (iii) बिहार आंदोलन
(घ) रेल हड़ताल (iv) जॉर्ज फर्नांडिस

उत्तर:

(क) संपूर्ण क्रांति (ii) जयप्रकाश नारायण
(ख) गरीबी हटाओ (i) इंदिरा गाँधी
(ग) छात्र आंदोलन (iii) बिहार आंदोलन
(घ) रेल हड़ताल (iv) जॉर्ज फर्नांडिस

प्रश्न 4.
किन कारणों से 1980 के मध्यावधि चुनाव करवाने पड़े?
उत्तर:
1977 के चुनावों में जनता पार्टी को जनता ने स्पष्ट बहुमत प्रदान किया लेकिन जनता पार्टी के नेताओं में प्रधानमंत्री के पद को लेकर मतभेद हो गया। आपातकाल का विरोध जनता पार्टी को कुछ दिनों के लिए ही एकजुट रख सका। जनता पार्टी के पास किसी एक दिशा, नेतृत्व अथवा साझे कार्यक्रम का अभाव था। केवल 18 महीने में ही मोरारजी देसाई ने लोकसभा में अपना बहुमत खो दिया जिसके कारण मोरारजी देसाई को त्यागपत्र देना पड़ा। मोरारजी देसाई के बाद चरणसिंह कांग्रेस पार्टी के समर्थन से प्रधानमंत्री बने लेकिन चार महीने बाद कांग्रेस पार्टी ने समर्थन वापस ले लिया। अतः जनता पार्टी की सरकार में पारस्परिक तालमेल का अभाव, सत्ता की भूख तथा राजनैतिक अस्थिरता के कारण 1980 में मध्यावधि चुनाव करवाए गए।

प्रश्न 5.
जनता पार्टी ने 1977 में शाह आयोग को नियुक्त किया था। इस आयोग की नियुक्ति क्यों की गई थी? इसके क्या निष्कर्ष थे?
उत्तर:
शाह आयोग का गठन ” 25 जून, 1975 के दिन घोषित आपातकाल के दौरान की गई कार्यवाही तथा सत्ता के दुरुपयोग, अतिचार और कदाचार के विभिन्न आरोपों के विविध पहलुओं” की जाँच के लिए किया गया था । आयोग ने विभिन्न प्रकार के साक्ष्यों की जांच की और हजारों गवाहों के बयान दर्ज किए। शाह आयोग ने अपनी जाँच के दौरान पाया कि इस अवधि में बहुत सारी ‘अति’ हुई। भारत सरकार ने आयोग द्वारा प्रस्तुत दो अंतरिम रिपोर्टों और तीसरी तथा अंतिम रिपोर्ट की सिफारिशी पर्यवेक्षणों और निष्कर्षों को स्वीकार किया ।

JAC Class 12 Political Science Solutions Chapter 6 लोकतांत्रिक व्यवस्था का संकट

प्रश्न 6.
1975 में राष्ट्रीय आपातकाल की घोषणा करते हुए सरकार ने इसके क्या कारण बताए थे?
उत्तर:
1975 में राष्ट्रीय आपातकाल की घोषणा करते हुए सरकार ने इसके निम्नलिखित कारण बताये थे-

  1. सरकार ने कहा कि विपक्षी दलों द्वारा लोकतन्त्र को रोकने की कोशिश की जा रही है तथा सरकार को उचित ढंग से कार्य नहीं करने दिया जा रहा है। आन्दोलनों के कारण हिंसक घटनाएँ हो रही हैं तथा हमारी सेना तथा पुलिस को बगावत के लिए उकसाया जा रहा है। (ii) सरकार ने कहा कि विघटनकारी ताकतों का खुला खेल जारी है और साम्प्रदायिक उन्माद को हवा दी जा रही है, जिससे हमारी एकता पर खतरा मँडरा रहा है।
  2. षड्यंत्रकारी शक्तियाँ सरकार के प्रगतिशील कामों में अड़ंगे लगा रही हैं और उसे गैर-संवैधानिक साधनों के बूते सत्ता से बेदखल करना चाहती हैं।

प्रश्न 7.
1977 के चुनावों के बाद पहली दफा केन्द्र में विपक्षी दल की सरकार बनी। ऐसा किन कारणों से संभव हुआ?
उत्तर:
1977 के चुनावों के बाद केन्द्र में पहली बार विपक्षी दल की सरकार बनने के पीछे अनेक कारण जिम्मेदार रहे, इनमें प्रमुख निम्नलिखित हैं-

  1. बड़ी विपक्षी पार्टियों का एकजुट होना: आपातकाल लागू होने से आहत विपक्षी नेताओं ने आपातकाल के बाद हुए चुनाव के पहले एकजुट होकर ‘जनता पार्टी’ नामक एक नया दल बनाया। कांग्रेस विरोधी मतों के बिखराव को रोका।
  2. जगजीवनराम द्वारा त्याग-पत्र देना: चुनाव से पहले जगजीवनराम ने कांग्रेस से त्यागपत्र दे दिया तथा कांग्रेस के कुछ अन्य नेताओं ने जगजीवनराम के नेतृत्व में एक नयी पार्टी – ‘कांग्रेस फॉर डेमोक्रेसी’ बनायी तथा बाद में यह पार्टी जनता पार्टी में शामिल हो गई ।
  3. आपातकाल की ज्यादतियाँ: आपातकाल के दौरान जनता पर अनेक ज्यादतियाँ की गईं। जैसे – संजय गाँधी के नेतृत्व में अनिवार्य नसबंदी कार्यक्रम चलाया गया; प्रेस तथा समाचार-पत्रों की स्वतंत्रता पर रोक लगा दी गई; हजारों लोगों को गिरफ्तार किया गया, आवश्यक वस्तुओं की कीमतों में अत्यधिक वृद्धि हो गई। इन सब कारणों से जनता कांग्रेस से नाराज थी और उसने कांग्रेस के विरोध में मतदान किया।
  4. जनता पार्टी का प्रचार: जनता पार्टी ने 1977 के चुनावों को आपातकाल के ऊपर जनमत संग्रह का रूप दिया तथा इस पार्टी ने चुनाव प्रचार में शासन के अलोकतांत्रिक चरित्र और आपातकाल के दौरान हुई ज्यादतियों को मुद्दा बनाया।

प्रश्न 8.
हमारी राजव्यवस्था के निम्नलिखित पक्ष पर आपातकाल का क्या असर हुआ?
1. नागरिक अधिकारों की दशा और नागरिकों पर इसका असर
2. कार्यपालिका और न्यायपालिका के संबंध
3. जनसंचार माध्यमों के कामकाज
4. पुलिस और नौकरशाही की कार्यवाहियाँ ।
उत्तर:

  1. आपातकाल के दौरान नागरिक अधिकारों को निलम्बित कर दिया गया तथा श्रीमती गाँधी द्वारा ‘मीसा कानून’ लागू किया गया जिसके अन्तर्गत किसी भी नागरिक को बिना कारण बताए कानूनी हिरासत में लिया जा सकता था।
  2. आपातकाल में कार्यपालिका एवं न्यायपालिका एक-दूसरे के सहयोगी हो गये, क्योंकि सरकार ने सम्पूर्ण न्यायपालिका को सरकार के प्रति वफादार रहने के लिए कहा तथा आपातकाल के दौरान कुछ हद तक न्यायपालिका सरकार के प्रति वफादार भी रही। इस प्रकार आपातकाल के दौरान न्यायपालिका कार्यपालिका के आदेशों का पालन करने वाली संस्था बन गई थी।
  3. आपातकाल के दौरान जनसंचार पर प्रतिबन्ध लगा दिया गया था, कोई भी समाचार पत्र सरकार के खिलाफ कोई भी खबर नहीं छाप सकता था तथा जो भी खबर अखबार द्वारा छापी जाती थी उसे
  4. पहले सरकार से स्वीकृति प्राप्त करनी पड़ती थी।
  5. आपातकाल के दौरान पुलिस और नौकरशाही, सरकार के प्रति वफादार बनी रही, यदि किसी पुलिस अधिकारी या नौकरशाही ने सरकार के आदेशों को मानने से मना किया तो उसे या तो निलम्बित कर दिया गया या गिरफ्तार कर लिया गया। इस काल में पुलिस की ज्यादतियाँ बढ़ गई थीं तथा नौकरशाही अनुशासन के नाम पर तानाशाह हो गयी थी। रिश्वतखोरी बढ़ गयी थी।

प्रश्न 9.
भारत की दलीय प्रणाली पर आपातकाल का किस तरह असर हुआ? अपने उत्तर की पुष्टि उदाहरणों से करें।
उत्तर:
आपातकाल का भारतीय दलीय व्यवस्था पर प्रतिकूल प्रभाव पड़ा क्योंकि अधिकांश विरोधी दलों को किसी प्रकार की राजनीतिक गतिविधियों की इजाजत नहीं थी। आजादी के समय से लेकर 1975 तक भारत में वैसे भी कांग्रेस पार्टी का प्रभुत्व रहा तथा संगठित विरोधी दल उभर नहीं पाया, वहीं आपातकाल के दौरान विरोधी दलों की स्थिति और भी खराब हुई। आपातकाल के बाद सरकार ने जनवरी, 1977 में चुनाव कराने का फैसला लिया। सभी बड़े विपक्षी दलों ने मिलकर एक नयी पार्टी – ‘ जनता पार्टी’ का गठन कर चुनाव लड़ा और सफलता पायी और सरकार बनाई। इस प्रकार कुछ समय के लिए ऐसा लगा कि राष्ट्रीय स्तर पर भारत की राजनैतिक प्रणाली द्वि-दलीय हो जायेगी। लेकिन 18 माह में ही जनता पार्टी का यह कुनबा बिखर गया और पुनः दलीय प्रणाली उसी रूप में आ गई।

JAC Class 12 Political Science Solutions Chapter 6 लोकतांत्रिक व्यवस्था का संकट

प्रश्न 10.
निम्नलिखित अवतरण को पढ़ें और इसके आधार पर पूछे गए प्रश्नों के उत्तर दें-
1977 के चुनावों के दौरान भारतीय लोकतंत्र, दो- दलीय व्यवस्था के जितना नजदीक आ गया था उतना पहले कभी नहीं आया। बहरहाल अगले कुछ सालों में मामला पूरी तरह बदल गया। हारने के तुरंत बाद कांग्रेस दो टुकड़ों में बँट गई: “जनता पार्टी में भी बड़ी अफरा-तफरी मची” “डेविड बटलर, अशोक लाहिड़ी और प्रणव रॉय। – पार्थ चटर्जी
(क) किन वजहों से 1977 में भारत की राजनीति दो- दलीय प्रणाली के समान जान पड़ रही थी?
(ख) 1977 में दो से ज्यादा पार्टियाँ अस्तित्व में थीं। इसके बावजूद लेखकगण इस दौर को दो- दलीय प्रणाली के नजदीक क्यों बता रहे हैं?
(ग) कांग्रेस और जनता पार्टी में किन कारणों से टूट पैदा हुई ?
उत्तर:
(क) 1977 में भारत की राजनीति दो- दलीय प्रणाली इसलिए जान पड़ती थी; क्योंकि उस समय केवल दो ही दल सत्ता के मैदान में थे, जिसमें सत्ताधारी दल कांग्रेस एवं मुख्य विपक्षी दल जनता पार्टी।

(ख) लेखकगण इस दौर को दो- दलीय प्रणाली के नजदीक इसलिए बता रहे हैं; क्योंकि कांग्रेस कई टुकड़ों. में बँट गई और जनता पार्टी में भी फूट हो गई परंतु फिर भी इन दोनों प्रमुख पार्टियों के नेता संयुक्त नेतृत्व और साझे कार्यक्रम और नीतियों की बात करने लगे। इन दोनों गुटों की नीतियाँ एक जैसी थीं। दोनों में बहुत कम अंतर था । वामपंथी मोर्चे में सी. पी. एम., सी. पी. आई., फारवर्ड ब्लॉक, रिपब्लिकन पार्टी की नीति एवं कार्यक्रमों को इनसे अलग माना जा सकता है।

(ग) 1977 के चुनावों में कांग्रेस पार्टी की हार के कारण नेताओं में निराशा पैदा हुई और इस निराशा के कारण फूट पैदा हुई, क्योंकि अधिकांश कांग्रेसी नेता श्रीमती गाँधी के चमत्कारिक नेतृत्व के महापाश से बाहर निकल चुके थे, दूसरी ओर जनता पार्टी में नेतृत्व और विचारधारा को लेकर फूट पैदा हो गई थी। प्रधानमंत्री पद के लिए उम्मीदवारों में आपसी होड़ मच गई।

लोकतांत्रिक व्यवस्था का संकट JAC Class 12 Political Science Notes

→ आपातकाल की पृष्ठभूमि:
1967 के चुनावों के बाद भारतीय राजनीति में व्यापक बदलाव आया। श्रीमती इंदिरा गाँधी एक कद्दावर नेता के रूप में उभरीं और उनकी लोकप्रियता चरम सीमा पर पहुँच गई। इस काल में दलगत प्रतिस्पर्द्धा कहीं ज्यादा तीखी और ध्रुवीकृत हो गई तथा न्यायपालिका और सरकार के सम्बन्धों में तनाव आए। इस काल में कांग्रेस के विपक्ष में जो दल थे उन्हें लग रहा था कि सरकारी प्राधिकार को निजी प्राधिकार मानकर प्रयोग किया जा रहा है और राजनीति हद से ज्यादा व्यक्तिगत होती जा रही है। कांग्रेस की टूट से इंदिरा गांधी और उनके विरोधियों के बीच मतभेद गहरे हो गये थे।

→ आर्थिक संदर्भ:
1971 में ‘गरीबी हटाओ’ का जो नारा दिया गया वह पूर्णतया खोखला साबित हुआ। बांग्लादेश के संकट से भारत की अर्थव्यवस्था पर बोझ बढ़ा। लगभग 80 लाख शरणार्थी पूर्वी पाकिस्तान की सीमा पार करके भारत आ गये। युद्ध के बाद अमरीका ने भारत की सहायता बन्द कर दी। इस अवधि में अन्तर्राष्ट्रीय बाजार में तेल की कीमतों में कई गुना वृद्धि हुई जिससे भारत की अर्थव्यवस्था पर प्रतिकूल प्रभाव पड़ा। वस्तुओं की कीमतों में जबरदस्त इजाफा हुआ।

→ गुजरात और बिहार में आंदोलन: गुजरात और बिहार में कांग्रेसी सरकार थी। इन राज्यों में हुए आन्दोलनों का प्रदेश की राजनीति पर गहरा प्रभाव पड़ा।

→ गुजरात में आंदोलन के कारण: 1974 में तेल की कीमतों व आवश्यक वस्तुओं की कीमतों में वृद्धि के विरुद्ध छात्रों ने आन्दोलन किया। इस आन्दोलन में बड़ी राजनीतिक पार्टियाँ भी शामिल हो गईं और इस आन्दोलन ने विकराल रूप धारण कर लिया। ऐसे में गुजरात में राष्ट्रपति शासन लगाना पड़ा। विपक्षी दलों ने राज्य की विधानसभा के लिए दुबारा चुनाव कराने की मांग की। कांग्रेस (ओ) के प्रमुख नेता मोरारजी देसाई ने कहा कि अगर राज्य में नए सिरे से चुनाव नहीं करवाए गए तो मैं भूख हड़ताल पर बैठ जाऊँगा। मोरारजी अपने कांग्रेस के दिनों में इंदिरा गांधी के मुख्य विरोधी रहे थे। विपक्षी दलों द्वारा समर्थित छात्र आंदोलन के गहरे दबाव में 1975 के जून में विधानसभा के चुनाव हुए। कांग्रेस इस चुनाव में हार गयी।

→ बिहार में आंदोलन के कारण:
1974 के मार्च माह में बिहार में इन्हीं मांगों को लेकर छात्रों द्वारा आन्दोलन छेड़ा गया। आंदोलन के क्रम में उन्होंने जयप्रकाश नारायण (जेपी) को बुलावा भेजा। जेपी तब सक्रिय राजनीति छोड़ चुके थे और सामाजिक कार्यों में लगे हुए थे। जेपी ने छात्रों का निमंत्रण इस शर्त पर स्वीकार किया कि आंदोलन अहिंसक रहेगा और अपने को सिर्फ बिहार तक सीमित नहीं रखेगा। इस प्रकार छात्र आंदोलन ने एक राजनीतिक चरित्र ग्रहण किया और उसके भीतर राष्ट्रव्यापी अपील आई। बिहार के इस आन्दोलन में हर क्षेत्र के लोग जुड़ने लगे। जयप्रकाश नारायण ने बिहार की कांग्रेस सरकार को बर्खास्त करने की माँग की। उन्होंने सामाजिक, आर्थिक और राजनीतिक दायरे में समग्र क्रान्ति का आह्वान किया ताकि उन्हीं के शब्दों में सच्चे लोकतन्त्र की स्थापना की जा सके।

JAC Class 12 Political Science Solutions Chapter 6 लोकतांत्रिक व्यवस्था का संकट

→ आन्दोलन का प्रभाव:
बिहार के इस आन्दोलन का प्रभाव राष्ट्रीय राजनीति पर पड़ना शुरू हुआ। जयप्रकाश नारायण चाहते थे कि यह आंदोलन देश के दूसरे हिस्सों में भी फैले। इस आंदोलन के साथ-साथ रेलवे कर्मचारियों ने भी राष्ट्रव्यापी हड़ताल का आह्वान किया, जिससे देश का सम्पूर्ण कामकाज ठप होने का खतरा उत्पन्न हो गया। विशेषज्ञों एवं विद्वानों का मानना था कि ये आन्दोलन राज्य सरकार के खिलाफ नहीं बल्कि इंदिरा गांधी के नेतृत्व के खिलाफ चलाए गए। इंदिरा गांधी का मानना था कि ये आन्दोलन उनके प्रति व्यक्तिगत विरोध से प्रेरित थे।

→ न्यायपालिका से संघर्ष:

  • न्यायपालिका के साथ इस दौर में सरकार और शासक दल के गहरे मतभेद पैदा हुए। इस क्रम में तीन संवैधानिक मामले उठे क्या संसद मौलिक अधिकारों में कटौती कर सकती है? सर्वोच्च न्यायालय का जवाब था कि संसद ऐसा नहीं कर सकती।
  • दूसरा यह कि क्या संसद संविधान में संशोधन करके सम्पत्ति के अधिकार में काट-छाँट कर सकती है? इस मसले पर भी सर्वोच्च न्यायालय का यही कहना था कि सरकार संविधान में इस तरह संशोधन नहीं कर सकती कि अधिकारों की कटौती हो जाए।
  • तीसरा, संसद ने यह कहते हुए संविधान में संशोधन किया कि वह नीति-निर्देशक सिद्धान्तों को प्रभावकारी बनाने के लिए मौलिक अधिकारों में कमी कर सकती है, लेकिन सर्वोच्च न्यायालय ने इस प्रावधान को भी निरस्त कर दिया। इससे सरकार और न्यायपालिका के मध्य संघर्ष उत्पन्न हो गया।

1973 में सरकार ने तीन वरिष्ठ न्यायाधीशों की अनदेखी करके न्यायमूर्ति ए. एन. रे को मुख्य न्यायाधीश नियुक्त किया। यह निर्णय राजनीतिक रूप से विवादास्पद बना रहा क्योंकि सरकार ने जिन तीन न्यायाधीशों की वरिष्ठता की अनदेखी इस मामले में की थी उन्होंने सरकार के इस कदम के विरुद्ध फैसला दिया।

आपातकाल की घोषणा:
12 जून, 1975 के दिन इलाहाबाद उच्च न्यायालय के न्यायाधीश जगमोहन लाल सिन्हा ने एक फैसला सुनाया। इस फैसले में उन्होंने लोकसभा के लिए इंदिरा गांधी के निर्वाचन को अवैधानिक करार दिया। जयप्रकाश नारायण की अगुवाई में विपक्षी दलों ने इंदिरा गांधी के इस्तीफे के लिए दबाव डाला। इन दलों ने 25 जून, 1975 को दिल्ली के रामलीला मैदान में एक विशाल प्रदर्शन किया। जयप्रकाश नारायण ने इंदिरा गांधी से इस्तीफे की मांग करते हुए राष्ट्रव्यापी सत्याग्रह की घोषणा की। जयप्रकाश नारायण ने सेना, पुलिस और सरकारी कर्मचारियों का आह्वान किया कि वे सरकार के अनैतिक और अवैधानिक आदेशों का पालन न करें। सरकार ने इन घटनाओं के मद्देनजर जवाब में 25 जून, 1975 को आपातकाल की घोषणा कर दी। श्रीमती गाँधी की सरकार ने राष्ट्रपति फखरुद्दीन अली अहमद से आपातकाल लागू करने की सिफारिश की और राष्ट्रपति ने तुरन्त आपातकाल की उद्घोषणा कर दी।

→  परिणाम:

  • सरकार के इस फैसले से विरोध- आंदोलन एक बार रुक गया। हड़तालों पर रोक लगा दी गई अनेक विपक्षी नेताओं को जेल में डाल दिया गया। सरकार ने निवारक नजरबंदी का बड़े पैमाने पर इस्तेमाल किया।
  • आपातकाल की मुखालफत और प्रतिरोध की कई घटनाएँ घटीं। पहली लहर में जो राजनीतिक कार्यकर्ता गिरफ्तारी से रह गए थे वे ‘भूमिगत’ हो गए और सरकार के खिलाफ मुहिम चलायी। इंडियन एक्सप्रेस और स्टेट्समैन जैसे अखबारों ने प्रेस पर लगी सेंसरशिप का विरोध किया।
  • इंदिरा गांधी के मामले में इलाहाबाद उच्च न्यायालय के फैसले की पृष्ठभूमि में संविधान में संशोधन हुआ। इस संशोधन के द्वारा प्रावधान किया गया कि प्रधानमंत्री, राष्ट्रपति और उपराष्ट्रपति पद के निर्वाचन को अदालत में चुनौती नहीं दी जा सकती।
  • आपातकाल के दौरान ही संविधान का 42वां संशोधन पारित हुआ इस संशोधन के माध्यम से संविधान के अनेक हिस्सों में बदलाव किए गए। 42वें संशोधन के माध्यम से हुए अनेक बदलावों में एक था- देश की विधायिका के कार्यकाल को 5 वर्ष से बढ़ाकर 6 साल करना।

→ आपातकाल के संदर्भ में विवाद;
सरकार का तर्क था कि भारत में लोकतन्त्र है और इसके अनुकूल विपक्षी दलों को चाहिए कि वे निर्वाचित शासक दल को अपनी नीतियों के अनुसार शासन चलाने दें। देश में लगातार गैर-संसदीय राजनीति का सहारा नहीं लिया जा सकता। इससे अस्थिरता पैदा होती है। इंदिरा गांधी ने शाह आयोग को चिट्ठी में लिखा कि षड्यंत्रकारी ताकतें सरकार के प्रगतिशील कार्यक्रमों में अड़ंगे लगा रही थीं और मुझे गैर-संवैधानिक साधनों के बूते सत्ता से बेदखल करना चाहती थीं। आपातकाल के आलोचकों का तर्क था कि आजादी के आंदोलन से लेकर लगातार भारत में जन-आंदोलन का एक सिलसिला रहा है।

जयप्रकाश नारायण सहित विपक्ष के नेताओं का विचार था कि लोकतन्त्र में लोगों को सार्वजनिक तौर पर सरकार के विरोध का अधिकार होना चाहिए। लोकतान्त्रिक कार्यप्रणाली को ठप करके आपातकाल लागू करने जैसे अतिचारी कदम उठाने की जरूरत कतई नहीं थी। दरअसल खतरा देश की एकता और अखंडता को नहीं, बल्कि शासक दल और स्वयं प्रधानमंत्री को था । आलोचक कहते हैं कि देश को बचाने के लिए बनाए गए संवैधानिक प्रावधान का दुरुपयोग इंदिरा गांधी ने निजी ताकतों को बचाने के लिए किया ।

JAC Class 12 Political Science Solutions Chapter 6 लोकतांत्रिक व्यवस्था का संकट

→ आपातकाल के दौरान क्या-क्या हुआ?:
आपातकाल को सही ठहराते हुए सरकार ने कहा कि इसके जरिए वो कानून व्यवस्था को बहाल करना चाहती थी, कार्यकुशलता बढ़ाना चाहती थी और गरीबों के हित के कार्यक्रम लागू करना चाहती थी । इस उद्देश्य से सरकार ने एक बीससूत्रीय कार्यक्रम की घोषणा की और इसे लागू करने का अपना दृढ़ संकल्प दोहराया। आपातकाल की घोषणा के बाद, शुरुआती महीनों में मध्यवर्ग इस बात से खुश था कि विरोध- आंदोलन समाप्त हो गया और सरकारी कर्मचारियों पर अनुशासन लागू हुआ। आपातकाल के दौरान कई नेताओं की गिरफ्तारी हुई थी। प्रेस पर कई तरह की पाबंदी लगाई गई। इसके अलावा कुछ और गंभीर आरोप लगे थे जो किसी आधिकारिक पद पर नहीं थे, लेकिन सरकारी ताकतों का इन लोगों ने इस्तेमाल किया था। आपातकाल का बुरा असर आम लोगों को भी भुगतना पड़ा। इसके दौरान पुलिस हिरासत में मौत और यातना की घटनाएँ भी सामने आईं। गरीब लोगों को मनमाने तरीके से एक जगह से उजाड़कर दूसरी जगह बसाने की घटनाएँ भी हुईं। जनसंख्या नियंत्रण के बहाने लोगों को अनिवार्य रूप से नसबंदी के लिए मजबूर किया गया।

→ आपातकाल से सबक:

  • आपातकाल से एकबारगी भारतीय लोकतन्त्र की ताकत और कमजोरियाँ उजागर हुईं। यद्यपि पर्यवेक्षक मानते हैं कि आपातकाल के दौरान भारत लोकतान्त्रिक नहीं रह गया था, लेकिन यह भी ध्यान देने योग्य बात है कि थोड़े दिनों के अंदर कामकाज फिर से लोकतान्त्रिक ढर्रे पर लौट आया। इस तरह आपातकाल का एक सबक तो यही है कि भारत से लोकतन्त्र विदा कर पाना बहुत कठिन है।
  • आपातकाल से संविधान में वर्णित आपातकाल के प्रावधानों के कुछ अर्थगत उलझाव भी प्रकट हुए, जिन्हें बाद में सुधार लिया गया। अब ‘अंदरूनी’ आपातकाल सिर्फ सशस्त्र विद्रोह की स्थिति में लगाया जा सकता है। इसके लिए जरूरी है कि आपातकाल की घोषणा की सलाह मंत्रिपरिषद राष्ट्रपति को लिखित में दे।
  • आपातकाल से हर कोई नागरिक अधिकारों के प्रति ज्यादा संचेत हुआ। आपातकाल की समाप्ति के बाद अदालतों ने व्यक्ति के नागरिक अधिकारों की रक्षा में सक्रिय भूमिका निभाई। आपातकाल के बाद की राजनीति – उत्तर भारत में आपातकाल का असर ज्यादा दिखाई दिया। यहाँ विपक्षी दलों ने लोकतन्त्र बचावों के बारे पर चुनाव लड़ा। जनादेश निर्णायक तौर पर आपातकाल के विरुद्ध था। जिन सरकारों को जनता ने लोकतन्त्र विरोधी माना उन्हें मतदान के रूप में उसने भारी दण्ड दिया।

→ लोकसभा चुनाव, 1977:
1977 के चुनावों में आपातकाल का असर व्यापक रूप से दिखाई दिया। इन चुनावों में जनादेश कांग्रेस के विरुद्ध था। कांग्रेस को लोकसभा में मात्र 154 सीटें मिलीं। उसे 35 प्रतिशत से भी कम वोट प्राप्त हुए। जनता पार्टी और उसके साथी दलों को लोकसभा की कुल 542 सीटों में से 330 सीटें मिलीं। खुद जनता पार्टी अकेले 295 सीटों पर जीती और उसे स्पष्ट बहुमत प्राप्त हुआ। कांग्रेस बिहार, उत्तर प्रदेश, दिल्ली, हरियाणा और पंजाब में एक भी सीट न पा सकी।

→ जनता सरकार:
1977 के चुनावों के बाद जनता पार्टी की सरकार बनी लेकिन इस पार्टी में तालमेल का अभाव था। मोरारजी देसाई प्रधानमंत्री बने, लेकिन पार्टी के भीतर खींचतान जारी रही। आपातकाल का विरोध जनता पार्टी को कुछ समय तक ही एक रख सका। जनता पार्टी बिखर गई और मोरारजी देसाई के नेतृत्व वाली सरकार ने 18 माह में अपना समर्थन खो दिया। इसके बाद चरण सिंह की सरकार भी केवल 4 माह तक ही चल पायी। 1980 में फिर से लोकसभा चुनाव हुए जिसमें कांग्रेस ने अपनी खोई हुई प्रतिष्ठा पुनः प्राप्त की।

→ विरासत:

  • 1969 से पहले तक कांग्रेस विविध विचारधाराओं के नेताओं व कार्यकर्ताओं को एक साथ लेकर चलती थी। अपने बदले हुए स्वभाव में कांग्रेस ने स्वयं को विशेष विचारधारा से जोड़ा। उसने अपने को देश की एकमात्र समाजवादी और गरीबों की हिमायती पार्टी बताना शुरू किया।
  • अप्रत्यक्ष रूप से 1977 के बाद पिछड़े वर्गों की भलाई का मुद्दा भारतीय राजनीति पर हावी होना शुरू हुआ। 1977 के चुनाव परिणामों पर पिछड़ी जातियों के कदम पर असर पड़ा, खासकर उत्तर भारत में।
  •  इस दौर में एक और महत्त्वपूर्ण मामला संसदीय लोकतन्त्र में जन- आंदोलन की भूमिका और उसकी सीमा को लेकर उठा स्पष्ट ही इस दौर में संस्था आधारित लोकतन्त्र में तनाव नजर आया। इस तनाव का एक कारण यह भी कहा जा सकता है कि हमारी दलीय प्रणाली जनता की आकांक्षाओं को अभिव्यक्त कर पाने में सक्षम साबित नहीं हुई।

JAC Class 12 Political Science Solutions Chapter 5 कांग्रेस प्रणाली : चुनौतियाँ और पुनर्स्थापना

Jharkhand Board JAC Class 12 Political Science Solutions Chapter 5 कांग्रेस प्रणाली : चुनौतियाँ और पुनर्स्थापना Textbook Exercise Questions and Answers.

JAC Board Class 12 Political Science Solutions Chapter 5 कांग्रेस प्रणाली : चुनौतियाँ और पुनर्स्थापना

Jharkhand Board Class 12 Political Science कांग्रेस प्रणाली : चुनौतियाँ और पुनर्स्थापना InText Questions and Answers

पृष्ठ 83

प्रश्न 1.
फ्रांस और कनाडा में ऐसी सूरत कायम हो, तो वहाँ कोई भी लोकतंत्र के असफल होने अथवा देश के टूटने की बात नहीं कहता। हम ही आखिर लगातार इतने शक में क्यों पड़े रहते हैं?
उत्तर:
फ्रांस में लोकतंत्र 1792 में स्थापित हुआ जबकि कनाडा एक संवैधानिक राजतंत्र है, जिसमें सम्राट राज्य का प्रमुख होता है। कनाडा एक संवैधानिक राजतंत्र 1867 में बना। भारत 1947 में आजाद हुआ और उसी समय भारत में लोकतंत्र कायम हुआ। भारत में नया नया लोकतंत्र कायम हुआ था इसलिए इतनी विविधताओं वाले देश में लोकतंत्र के असफल होने या देश के टूटने का शक लगातार लगा रहता है।

पृष्ठ 86

प्रश्न 2.
इन्दिरा गाँधी के लिए स्थितियाँ सचमुच कठिन रही होंगी-पुरुषों के दबदबे वाले क्षेत्र में आखिर वे अकेली महिला थीं। ऊँचे पदों पर अपने देश में ज्यादा महिलाएँ क्यों नहीं हैं?
उत्तर:
श्रीमती इन्दिरा गाँधी भारत की प्रथम महिला प्रधानमन्त्री बनीं लेकिन प्रारम्भिक काल में उनको सिण्डीकेट और प्रभावशाली वरिष्ठ कांग्रेसी नेताओं द्वारा अनेक चुनौतियाँ मिलीं, लेकिन पारिवारिक राजनीतिक विरासत और पर्याप्त राजनीतिक अनुभव के कारण उनको एक सामान्य महिला की अपेक्षा कम कठिनाइयों का सामना करना पड़ा। लेकिन भारत में ज्यादातर महिलाओं के समक्ष अनेक ऐसी चुनौतियाँ एवं समस्याएँ हैं जिनके कारण वे ऊँचे पदों पर नहीं आ पातीं। इनमें प्रमुख निम्नलिखित हैं-

  1. भारतीय समाज पुरुष-प्रधान है। अधिकांश कानून पुरुषों द्वारा बनाए गए और महिलाओं को समाज में गैर- बराबरी का दर्जा दिया गया।
  2. लड़कों की तुलना में उनके जन्म और पालन-पोषण में नकारात्मक भेद-भाव किया जाता था।
  3. सती-प्रथा, बहुपत्नी विवाह, दहेज प्रथा, पर्दा प्रथा, कन्या वध या भ्रूण हत्या, विधवा विवाह की मनाही, अशिक्षा आदि ने नारियों को समाज में पछाड़े रखा।
  4. भारत में पुरुषों की संकीर्ण मानसिकता के कारण वे महिलाओं को सरकारी नौकरियाँ विशेषकर ऊँचे पदों पर नहीं देखना चाहते।
  5. शिक्षा के प्रचार- प्रसार, नारी जागृति तथा नारी सशक्तीकरण के कारण अब धीरे-धीरे देश के उच्च पदों पर महिलाएँ कार्य कर रही हैं, लेकिन अभी भी उनकी संख्या काफी कम है।

पृष्ठ 89

प्रश्न 3.
क्या आज गैर-कांग्रेसवाद प्रासंगिक है? क्या मौजूदा पश्चिम बंगाल में वाममोर्चा के खिलाफ ऐसा ही तरीका अपनाया जा सकता है?
उत्तर:
गैर कांग्रेसवाद के जनक डॉ. राममनोहर लोहिया थे और गैर-कांग्रेसवाद आज भी उतना प्रासंगिक है जितना 1960 के दशक में था। बंगाल में वाममोर्चा के खिलाफ ऐसा तरीका अपनाया जा सकता है क्योंकि बंगाल में वाममोर्चा का एकाधिकार है जिसके कारण समाज में कुछ अलगाववादी तत्त्व सक्रिय हो गए हैं। इन सबको खत्म करने का ये तरीका कारगर हो सकता है।

JAC Class 12 Political Science Solutions Chapter 5 कांग्रेस प्रणाली : चुनौतियाँ और पुनर्स्थापना

पृष्ठ 90

प्रश्न 4.
त्रिशंकु विधानसभा और गठबन्धन सरकार की इन बातों में नया क्या है? ऐसी बातें तो हम आए दिन सुनते रहते हैं।
उत्तर:
भारत में 1967 के चुनावों से गठबन्धन की राजनीति सामने आयी। त्रिशंकु विधानसभा और गठबंधन सरकार की घटना उन दिनों नई थी क्योंकि पहली बार चुनावों में किसी भी पार्टी को स्पष्ट बहुमत प्राप्त नहीं हुआ था, इसलिए अनेक गैर-कांग्रेसी पार्टियों ने एकजुट होकर संयुक्त विधायक दल बनाया और गैर-कांग्रेसी सरकारों को समर्थन दिया। इसी कारण इन सरकारों को संयुक्त विधायक दल की सरकार कहा गया। लेकिन वर्तमान समय में भारतीय दलीय व्यवस्था का स्वरूप बदल गया है। बहुदलीय व्यवस्था होने के कारण केन्द्र और राज्यों में किसी भी दल को स्पष्ट बहुमत नहीं मिल पाता और त्रिशंकु विधान सभा या संसद का निर्माण हो रहा है। इसलिए आज गठबन्धन सरकार या त्रिशंकुं संसद आम बात हो गई है।

पृष्ठ 92

प्रश्न 5.
इसका मतलब यह है कि राज्य स्तर के नेता पहले के समय में भी ‘किंगमेकर’ थे और इसमें कोई नयी बात नहीं है। मैं तो सोचती थी कि ऐसा केवल 1990 के दशक में हुआ।
उत्तर:
पार्टी के ऐसे ताकतवर नेता जिनका पार्टी संगठन पर पूर्ण नियन्त्रण होता है उन्हें ‘किंगमेकर’ कहा जाता है प्रधानमन्त्री हो या मुख्यमन्त्री, इनकी नियुक्ति में इनकी विशेष भूमिका होती है। भारत में राज्य स्तर पर किंगमेकर्स की शुरुआत केवल 1990 के दशक में नहीं बल्कि इससे पहले भी इस प्रकार की स्थिति पायी जाती थी। भारत में पहले कांग्रेसी नेताओं के एक समूह को अनौपचारिक तौर पर सिंडिकेट के नाम से इंगित किया जाता था। इस समूह के नेताओं का पार्टी के संगठन पर नियन्त्रण था। मद्रास प्रान्त के कामराज, मुम्बई सिटी के एस.के. पाटिल, मैसूर के एस. निजलिंगप्पा, आन्ध्र प्रदेश के एन. संजीव रेड्डी और पश्चिम बंगाल के अतुल्य घोष इस संगठन में शामिल थे। लाल बहादुर शास्त्री एवं श्रीमती इन्दिरा गाँधी, दोनों ही सिंडिकेट की सहायता से प्रधानमन्त्री पद पर आरूढ़ हुए।

पृष्ठ 96

प्रश्न 6.
गरीबी हटाओ का नारा तो अब से लगभग चालीस साल पहले दिया गया था। क्या यह नारा महज एक चुनावी छलावा था?
उत्तर:
गरीबी हटाओ का नारा श्रीमती गाँधी ने तत्कालीन समय व परिस्थितियों को ध्यान में रखते हुए दिया। उन्होंने विपक्षी गठबन्धन द्वारा दिये गये इन्दिरा हटाओ नारे के विपरीत लोगों के सामने एक सकारात्मक कार्यक्रम रखा और इसे अपने मशहूर नारे ‘गरीबी हटाओ’ के जरिए एक शक्ल प्रदान की। इन्दिरा गाँधी ने सार्वजनिक क्षेत्र की संवृद्धि, ग्रामीण भू-स्वामित्व और शहरी सम्पदा के परिसीमन, आय और अवसरों की असमानता की समाप्ति तथा प्रिवीपर्स की समाप्ति पर अपने चुनाव अभियान में जोर दिया।

गरीबी हटाओ के नारे से श्रीमती गाँधी ने वंचित तबकों खासकर भूमिहीन किसान, दलित और आदिवासियों, अल्पसंख्यक महिलाओं और बेरोजगार नौजवानों के बीच अपने समर्थन का आधार तैयार करने की कोशिश की। लेकिन 1971 के भारत-पाक युद्ध और विश्व स्तर पर पैदा हुए तेल संकट के कारण गरीबी हटाओ का नारा कमजोर पड़ गया। 1971 में इन्दिरा गाँधी द्वारा दिया गया यह नारा महज पाँच साल के अन्दर ही असफल हो गया और 1977 में इन्दिरा गांधी को ऐतिहासिक पराजय का सामना करना पड़ा। इस प्रकार यह नारा महज एक चुनावी छलावा साबित हुआ।

JAC Class 12 Political Science Solutions Chapter 5 कांग्रेस प्रणाली : चुनौतियाँ और पुनर्स्थापना

पृष्ठ 99

प्रश्न 7.
यह तो कुछ ऐसा ही है कि कोई मकान की बुनियाद और छत बदल दे फिर भी कहे कि मकान वही है। पुरानी और नयी कांग्रेस में कौनसी चीज समान थी?
उत्तर:
1969 में कांग्रेस के विभाजन तथा कामराज योजना के तहत कांग्रेस पार्टी की पुनर्स्थापना करने का प्रयास किया। श्रीमती इन्दिरा गाँधी और उनके साथ अन्य युवा नेताओं ने कांग्रेस पार्टी को नया स्वरूप प्रदान करने का प्रयास किया। यह पार्टी पूर्णतः अपने सर्वोच्च नेता की लोकप्रियता पर आश्रित थी। पुरानी कांग्रेस की तुलना में इसका सांगठनिक ढाँचा कमजोर था। अब इस पार्टी के भीतर कई गुट नहीं थे। अर्थात् अब यह कांग्रेस विभिन्न मतों और हितों को एक साथ लेकर चलने वाली पार्टी नहीं थी। इस प्रकार इन्दिरा कांग्रेस के बारे में यह कहा जा सकता है कि इसकी बुनियाद और छत बदल दी गई थी, लेकिन नाम वही था। पुरानी और नई कांग्रेस में एक बात समान थी कि दोनों को ही लोकप्रियता में समान स्थान प्राप्त था। पार्टी के सर्वोच्च नेता पर आश्रितता की नीति में भी कोई परिवर्तन नहीं आया तथा कांग्रेस की मूल विचारधारा में भी कोई विशेष परिवर्तन नहीं आया।

Jharkhand Board Class 12 Political Science कांग्रेस प्रणाली : चुनौतियाँ और पुनर्स्थापना TextBook Questions and Answers

प्रश्न 1.
1967 के चुनावों के बारे में निम्नलिखित में कौन-कौन से बयान सही हैं:
(क) कांग्रेस लोकसभा के चुनाव में विजयी रही, लेकिन कई राज्यों में विधानसभा के चुनाव वह हार गई।
(ख) कांग्रेस लोकसभा के चुनाव भी हारी और विधानसभा के भी।
(ग) कांग्रेस को लोकसभा में बहुमत नहीं मिला, लेकिन उसने दूसरी पार्टियों के समर्थन से एक गठबन्धन सरकार बनाई।
(घ) कांग्रेस केन्द्र में सत्तासीन रही और उसका बहुमत भी बढ़ा।
उत्तर:
(क) सही
(ख) गलत
(ग) सही

प्रश्न 2.
निम्नलिखित का मेल करें:

(क) सिंडिकेट (i) कोई निर्वाचित जन-प्रतिनिधि जिस पार्टी के टिकट से जीता हो, उस पार्टी को छोड़कर अगर दूसरे दल में चला जाए।
(ख) दल-बदल (ii) लोगों का ध्यान आकर्षित करने वाला एक मनभावन मुहावरा।
(ग) नारा (iii) कांग्रेस और इसकी नीतियों के खिलाफ अलग-अलग विचारधाराओं की पार्टियों का एकजुट होना।
(घ) गैर-कांग्रेसवाद (iv) कांग्रेस के भीतर ताकतवर और प्रभावशाली नेताओं का एक समूह।

उत्तर:

(क) सिंडिकेट (iv) कांग्रेस के भीतर ताकतवर और प्रभावशाली नेताओं का एक समूह।
(ख) दल-बदल (i) कोई निर्वाचित जन-प्रतिनिधि जिस पार्टी के टिकट से जीता हो, उस पार्टी को छोड़कर अगर दूसरे दल में चला जाए।
(ग) नारा (ii) लोगों का ध्यान आकर्षित करने वाला एक मनभावन मुहावरा।
(घ) गैर-कांग्रेसवाद (iii) कांग्रेस और इसकी नीतियों के खिलाफ अलग-अलग विचारधाराओं की पार्टियों का एकजुट होना।

प्रश्न 3.
निम्नलिखित नारे से किन नेताओं का सम्बन्ध है?
(क) जय जवान, जय किसान
(ख) इन्दिरा हटाओ
(ग) गरीबी हटाओ।
उत्तर:
(क) लालबहादुर शास्त्री
(ख) विपक्षी गठबंधन
(ग) श्रीमती इन्दिरा गाँधी।

JAC Class 12 Political Science Solutions Chapter 5 कांग्रेस प्रणाली : चुनौतियाँ और पुनर्स्थापना

प्रश्न 4.
1971 के ‘ग्रैंड अलायंस’ के बारे में कौनसा कथन ठीक है?
(क) इसका गठन गैर- कम्युनिस्ट और गैर-कांग्रेसी दलों ने किया था।
(ख) इसके पास एक स्पष्ट राजनीतिक तथा विचारधारात्मक कार्यक्रम था।
(ग) इसका गठन सभी गैर-कांग्रेसी दलों ने एकजुट होकर किया था।
उत्तर:
(क) इसका गठन गैर- कम्युनिस्ट और गैर-कांग्रेसी दलों ने किया था।

प्रश्न 5.
किसी राजनीतिक दल को अपने अंदरूनी मतभेदों का समाधान किस तरह करना चाहिए? यहाँ कुछ समाधान दिए गए हैं। प्रत्येक पर विचार कीजिए और उसके सामने उसके फायदों और घाटों को लिखिए।
(क) पार्टी के अध्यक्ष द्वारा बताए गए मार्ग पर चलना।
(ख) पार्टी के भीतर बहुमत की राय पर अमल करना।
(ग) हरेक मामले पर गुप्त मतदान कराना।
(घ) पार्टी के वरिष्ठ और अनुभवी नेताओं से सलाह करना।
उत्तर:
(क) लाभ – पार्टी के अध्यक्ष द्वारा बताए गए मार्ग पर चलने से पार्टी में एकता और अनुशासन की भावना का विकास होगा। हानि – इससे एक व्यक्ति की तानाशाही या निरंकुशता स्थापित होने का खतरा बढ़ जाता है।

(ख) लाभ – मतभेदों को दूर करने के लिए बहुमत की राय जानने से यह लाभ होगा कि इससे अधिकांश की राय का पता चलेगा तथा पार्टी में आंतरिक लोकतंत्र बढ़ेगा। हानि – बहुमत की राय मानने से अल्पसंख्यकों की उचित बात की अवहेलना की सम्भावना बनी रहेगी। (ग) लाभ – पार्टी के मतभेदों को दूर करने के लिए गुप्त मतदान की प्रक्रिया अपनाने से प्रत्येक सदस्य अपनी बात स्वतन्त्रतापूर्वक रख सकेगा। यह पद्धति अधिक लोकतांत्रिक तथा निष्पक्ष है। हानि – गुप्त मतदान में क्रॉस वोटिंग का खतरा बना रहता है।

(घ) लाभ – पार्टी के मतभेदों को दूर करने के लिए वरिष्ठ और अनुभवी नेताओं की सलाह का विशेष लाभ होगा, क्योंकि वरिष्ठ नेताओं के पास अनुभव होता है तथा सभी सदस्य उनका आदर करते हैं। हानि – वरिष्ठ एवं अनुभवी व्यक्ति नये विचारों एवं मूल्यों को अपनाने से कतराते हैं।

प्रश्न 6.
निम्नलिखित में से किसे / किन्हें 1967 के चुनावों में कांग्रेस की हार के कारण के रूप में स्वीकार किया जा सकता है? अपने उत्तर की पुष्टि में तर्क दीजिए:
(क) कांग्रेस पार्टी में करिश्माई नेता का अभाव।
(ख) कांग्रेस पार्टी के भीतर टूट।
(ग) क्षेत्रीय, जातीय और साम्प्रदायिक समूहों की लामबंदी को बढ़ाना।
(घ) गैर-कांग्रेसी दलों के बीच एकजुटता।
(ङ) कांग्रेस पार्टी के अन्दर मतभेद।
उत्तर:
(कं) इसको कांग्रेस की हार के लिए उत्तरदायी नहीं ठहराया जा सकता क्योंकि कांग्रेस के पास अनेक वरिष्ठ और करिश्माई नेता थे

(ख) यह कांग्रेस पार्टी की हार का सबसे बड़ा कारण था क्योंकि कांग्रेस दो गुटों में बँटती जा रही थी युवा तुर्क और सिंडिकेट युवा तुर्क (चन्द्रशेखर, चरणजीत यादव, मोहन धारिया, कृष्णकान्त एवं आर. के. सिन्हा) तथा सिंडिकेट (कामराज, एस. के पाटिल, अतुल्य घोष एवं निजलिंगप्पा) के बीच आपसी फूट के कारण कांग्रेस पार्टी को 1967 के चुनावों में हार का सामना करना पड़ा।

(ग) 1967 में पंजाब में अकाली दल, तमिलनाडु में डी. एम. के. जैसे दल अनेक राज्यों में क्षेत्रीय, जातीय और साम्प्रदायिक दलों के रूप में उभरे जिससे कांग्रेस प्रभाव व विस्तार क्षेत्र में कमी आयी तथा कई राज्यों में उसे सत्ता से हाथ धोना पड़ा।

(घ) गैर-कांग्रेसी दलों के बीच पूर्णरूप से एकजुटता नहीं थी लेकिन जिन-जिन प्रान्तों में ऐसा हुआ वहाँ वामपंथियों अथवा गैर-कांग्रेसी दलों को लाभ मिला।
(ङ) कांग्रेस पार्टी के अन्दर मतभेद के कारण बहुत जल्दी ही आन्तरिक फूट कालान्तर में सभी के सामने आ गई और लोग यह मानने लगे कि 1967 के चुनाव में कांग्रेस की हार के कई कारणों में से यह कारण भी एक महत्त्वपूर्ण था।

JAC Class 12 Political Science Solutions Chapter 5 कांग्रेस प्रणाली : चुनौतियाँ और पुनर्स्थापना

प्रश्न 7.
1970 के दशक में इन्दिरा गाँधी की सरकार किन कारणों से लोकप्रिय हुई थी?
उत्तर:
1970 के दशक में श्रीमती गाँधी की लोकप्रियता के कारण – 1970 के दशक में श्रीमती गाँधी की लोकप्रियता के मुख्य कारण निम्नलिखित थे-
1. करिश्मावादी नेता:
इन्दिरा गाँधी कांग्रेस पार्टी की करिश्मावादी नेता थीं। वह भारत के प्रथम प्रधानमन्त्री की पुत्री थीं और उन्होंने स्वयं को गाँधी-नेहरू परिवार का वास्तविक राजनीतिक उत्तराधिकारी बताया। वह देश की प्रथम महिला प्रधानमन्त्री होने के कारण महिला मतदाताओं में अधिक लोकप्रिय हुईं।

2. प्रगतिशील कार्यक्रमों की घोषणा:
इन्दिरा गाँधी द्वारा 20 सूत्री कार्यक्रम प्रस्तुत करना, बैंकों का राष्ट्रीयकरण करना, प्रिवीपर्स को समाप्त करना, श्री वी.वी. गिरि जैसे मजदूर नेता को दल के घोषित प्रत्याशी के विरुद्ध चुनाव जिता कर लाना आदि ने उनकी लोकप्रियता को बढ़ाया ।

3. कुशल एवं साहसिक चुनावी रणनीति:
इंदिरा गाँधी ने एक साधारण से सत्ता संघर्ष को विचारधारात्मक संघर्ष में बदल दिया । उन्होंने अपनी वामपंथी नीतियों की घोषणा कर जनता को यह दर्शाने में सफलता पाई कि कांग्रेस का सिंडीकेट धड़ा इन नीतियों के मार्ग में बाधाएँ डाल रहा है। चुनावों में श्रीमती गाँधी को इसका लाभ मिला।

4. भूमि सुधार कानूनों का क्रियान्वयन:
श्रीमती गाँधी ने भूमि सुधार कानूनों के क्रियान्वयन के लिए जबरदस्त अभियान चलाया तथा उन्होंने भू-परिसीमन के कुछ और कानून भी बनाए। जिसका प्रभाव चुनाव में उनके पक्ष में गया।

प्रश्न 8.
1960 के दशक की कांग्रेस पार्टी के सन्दर्भ में सिंडिकेट का क्या अर्थ है? सिंडिकेट ने कांग्रेस पार्टी में क्या भूमिका निभाई?
उत्तर:
सिंडिकेट का अर्थ- कांग्रेसी नेताओं के एक समूह को एक अनौपचारिक रूप से सिंडिकेट के नाम से पुकारा जाता था। इस समूह के नेताओं का पार्टी के संगठन पर अधिकार एवं नियन्त्रण था। सिंडिकेट के अगुआ के. कामराज थे। इसमें विभिन्न प्रान्तों के ताकतवर नेता जैसे बम्बई सिटी (अब मुम्बई) के एस. के. पाटिल, मैसूर ( अब कर्नाटक) के एस. निजलिंगप्पा, आन्ध्र प्रदेश के एन. संजीव रेड्डी और पश्चिम बंगाल के अतुल्य घोष शामिल थे। भूमिका – इन्दिरा गाँधी के पहले मन्त्रिमण्डल में इस समूह की निर्णायक भूमिका रही।

इसने तब नीतियों के निर्माण और क्रियान्वयन में भी अहम भूमिका निभायी थी। कांग्रेस का विभाजन होने के बाद सिंडिकेट के नेता कांग्रेसी कांग्रेस (ओ) में ही रहे । चूँकि इन्दिरा गाँधी की कांग्रेस (आर) ही लोकप्रियता की कसौटी पर सफल रही, इसलिए ये ताकतवर नेता 1971 के बाद प्रभावहीन हो गए।

प्रश्न 9.
कांग्रेस पार्टी किन मसलों को लेकर 1969 में टूट की शिकार हुई? कांग्रेस के 1969 के विभाजन के लिए उत्तरदायी किन्हीं पाँच कारकों का परीक्षण कीजिए।
अथवा
1969 में कांग्रेस में विभाजन के क्या कारण थे?
उत्तर:
1969 में कांग्रेस पार्टी के विभाजन या टूट के कारण 1969 में कांग्रेस के विभाजन एवं टूट के निम्नलिखित कारण थे:

  1. दक्षिणपंथी और वामपंथी विचारधाराओं के समर्थकों के मध्य कलह: 1967 के चौथे आम चुनावों में कांग्रेस की हार के बाद कांग्रेस के कुछ नेता दक्षिणपंथी विचारधारा वालों के साथ मिलकर चुनाव लड़ना चाहते थे और कुछ वामपंथी विचारधारा वाले दलों के साथ। कांग्रेस के नेताओं की यह कलह उसके विभाजन का मुख्य कारण बनी।
  2. राष्ट्रपति पद के उम्मीदवार के चयन को लेकर मतभेद: 1969 में राष्ट्रपति पद के लिए कांग्रेस द्वारा समर्थित उम्मीदवार एन. संजीव रेड्डी को श्रीमती गाँधी व उनके समर्थकों ने मत न देकर एक स्वतन्त्र उम्मीदवार वी. वी. गिरि को समर्थन दिया। जिससे चुनाव में वी.वी. गिरि जीत गये। यह घटना कांग्रेस पार्टी के विभाजन का प्रमुख कारण बनी।
  3. युवा तुर्क एवं सिंडिकेट के बीच कलह: 1969 में कांग्रेस पार्टी के विभाजन का एक कारण युवा तुर्क (चन्द्रशेखर, चरणजीत यादव, मोहन धारिया, कृष्णकान्त एवं आर. के. सिन्हा) तथा सिंडिकेट (कामराज, एस. के. पाटिल, अतुल्य घोष एवं निजलिंगप्पा ) के बीच होने वाली कलह भी रही।
  4. वित्त विभाग, मोरारजी देसाई से वापस लेना: श्रीमती गाँधी की मोरारजी देसाई से वित्त विभाग को वापस लेने तथा बैंक राष्ट्रीयकरण के प्रस्ताव को मन्त्रिमण्डल में सर्वसम्मति से पारित कर देने की कार्यवाही ने भी कांग्रेस विभाजन को मुखरित किया।
  5. सिंडीकेट द्वारा श्रीमती गाँधी को पद से हटाने का प्रयास: 1969 में कांग्रेस के आधिकारिक उम्मीदवार के चुनाव हार जाने के बाद सिंडीकेट ने प्रधानमन्त्री श्रीमती इन्दिरा गाँधी को पद से हटाने का प्रयास किया परन्तु वे इसमें सफल न हो सके। उपर्युक्त घटनाओं के कारण कांग्रेस में आन्तरिक कलह इस कदर बढ़ गया कि नवम्बर, 1969 में कांग्रेस का विभाजन हो गया।

JAC Class 12 Political Science Solutions Chapter 5 कांग्रेस प्रणाली : चुनौतियाँ और पुनर्स्थापना

प्रश्न 10.
निम्नलिखित अनुच्छेद को पढ़ें और इसके आधार पर पूछे गए प्रश्नों के उत्तर दें:
इन्दिरा गाँधी ने कांग्रेस को अत्यन्त केन्द्रीकृत और अलोकतान्त्रिक पार्टी संगठन में तब्दील कर दिया, जबकि नेहरू के नेतृत्व में कांग्रेस शुरुआती दशकों में एक संघीय, लोकतान्त्रिक और विचारधाराओं के समाहार का मंच थी। नयी और लोकलुभावन राजनीति ने राजनीतिक विचारधारा को महज चुनावी विमर्श में बदल दिया। कई नारे उछाले गए, लेकिन इसका मतलब यह नहीं था कि उसी के अनुकूल सरकार की नीतियाँ भी बनानी थीं- 1970 के दशक के शुरुआती सालों में अपनी बड़ी चुनावी जीत के जश्न के बीच कांग्रेस एक राजनीतिक संगठन के तौर पर मर गई।
(क) लेखक के अनुसार नेहरू और इन्दिरा गाँधी द्वारा अपनाई गई रणनीतियों में क्या अन्तर था?
(ख) लेखक ने क्यों कहा है कि सत्तर के दशक में कांग्रेस ‘मर गई’?
(ग) कांग्रेस पार्टी में आए बदलावों का असर दूसरी पार्टियों पर किस तरह पड़ा?
उत्तर:
(क) जवाहर लाल नेहरू की तुलना में उनकी पुत्री और तीसरी प्रधानमन्त्री इन्दिरा गाँधी ने कांग्रेस पार्टी को बहुत ज्यादा केन्द्रीकृत और अलोकतान्त्रिक पार्टी संगठन के रूप में बदल दिया। नेहरू के काल में यह पार्टी संघीय लोकतान्त्रिक और विभिन्न विचारधाराओं को मानने वाले कांग्रेसी नेता और यहाँ तक कि विरोधियों को साथ लेकर चलने वाले एक मंच के रूप में कार्य करती थी।

(ख) लेखक ने ऐसा इसलिए कहा क्योंकि सत्तर के दशक में कांग्रेस की सर्वोच्च नेता श्रीमती इन्दिरा गाँधी एक अधिनायकवादी नेता थीं। उन्होंने मनमाने ढंग से मन्त्रिमण्डल और दल का गठन किया तथा पार्टी में विचार-विमर्श प्रायः मर गया।

(ग) कांग्रेस पार्टी में आए बदलाव के कारण दूसरी पार्टियों में परस्पर एकता बढ़ी। वे जनता पार्टी के रूप में लोगों के सामने आये। 1977 के चुनावों में विरोधी दलों ने कांग्रेस का सफाया कर दिया।

कांग्रेस प्रणाली : चुनौतियाँ और पुनर्स्थापना JAC Class 12 Political Science Notes

→ पण्डित नेहरू के शासन काल में देश के सभी भागों में कांग्रेस पार्टी का वर्चस्व बना रहा तथा कोई भी राजनीतिक पार्टी कांग्रेसी वर्चस्व को चुनौती देने में सक्षम नहीं थी। लेकिन नेहरू की मृत्यु के बाद कांग्रेस नेतृत्व के लिए अनेक चुनौतियाँ उत्पन्न होने लगीं।

→ राजनीतिक उत्तराधिकार की चुनौती:
राजनीतिक उत्तराधिकार की चुनौती मई, 1964 में नेहरू की मृत्यु के बाद उत्पन्न हुई जिसे लालबहादुर शास्त्री के प्रधानमन्त्री बनने के साथ ही हल कर लिया गया। शास्त्री 1964 1966 तक प्रधानमंत्री रहे। 10 जनवरी, 1966 को ताशकंद समझौते पर हस्ताक्षर के बाद शास्त्रीजी के निधन के उपरान्त फिर राजनीतिक उत्तराधिकार का मामला उठा। लालबहादुर शास्त्री की मृत्यु के बाद इन्दिरा गाँधी को देश की प्रथम महिला प्रधानमन्त्री और तीसरा प्रधानमन्त्री बनने का अवसर मिला । उन्होंने अपने प्रतियोगी कांग्रेस के वरिष्ठ नेता मोरारजी देसाई को पराजित किया। इन्दिरा गाँधी 1966 से 1977 तक और फिर 1980 से 1984 तक प्रधानमन्त्री पद पर रहीं। 1984 में उनकी हत्या कर दी गई।

→ चौथा आम चुनाव 1967:
भारतीय चुनावी राजनीति के इतिहास में 1967 के वर्ष को अत्यन्त महत्त्वपूर्ण पड़ाव माना जाता है। 1952 के बाद से पूरे देश में कांग्रेस पार्टी का जो दबदबा कायम था वह 1967 के चुनावों में समाप्त हो गया।

→ चुनावों का सन्दर्भ:
1960 के दशक में अनेक कारणों से देश गम्भीर आर्थिक संकट में था। आर्थिक स्थिति की विकटता के कारण कीमतों में तेजी से वृद्धि हुई, खाद्यान्न की कमी तथा बढ़ती बेरोजगारी से लोगों की स्थिति बदतर हो गई और लोग सरकार के विरोध में उतर आये। कांग्रेस सरकार इसे भाँप नहीं सकी। मार्क्सवादी समाजवादी दल से अलग हुए मार्क्सवादी-लेनिनवादी गुट ने सशस्त्र कृषक विद्रोह का नेतृत्व किया तथा किसानों को संगठित किया। तीसरे, इस काल में गम्भीर साम्प्रदायिक दंगे भी हुए।

JAC Class 12 Political Science Solutions Chapter 5 कांग्रेस प्रणाली : चुनौतियाँ और पुनर्स्थापना

→ गैर-कांग्रेसवाद:
कांग्रेस विरोधी वोटों को चुनाव में बंटने से रोकने के लिए सभी विरोधी दलों ने एकजुट होकर सभी राज्यों में एक कांग्रेस विरोधी मोर्चा बनाया जिसे राममनोहर लोहिया ने ‘गैर-कांग्रेसवाद’ का नारा दिया।

चुनाव का जनादेश तथा गठबन्धन सरकारें: व्यापक जन- असन्तोष और राजनीतिक दलों के ध्रुवीकरण के इस माहौल में 1967 के चौथे आम चुनाव हुए। इन चुनावों को कांग्रेस को जैसे-तैसे लोकसभा में तो बहुमत मिल गया लेकिन उसकी सीटों की संख्या में भारी गिरावट आयी। सात राज्यों में कांग्रेस को बहुमत नहीं मिला। 2 अन्य राज्यों में भी दल-बदल के कारण कांग्रेस सरकार नहीं बना सकी। मद्रास में डी.एम. के. ने सरकार बनायी तथा अन्य 8 राज्यों में गठबन्धन की सरकारें बनीं।

→ दल – बदल: 1967 से देश में राजनीति दल-बदल और ‘ आया राम-गया राम’ की राजनीति शुरू हुई, जिसकी वजह से भारतीय लोकतन्त्र को अस्थायी रूप से गहरा आघात लगा। कांग्रेस सिंडिकेट और इंडिकेट या पुरानी कांग्रेस और नई कांग्रेस के नाम से विभाजित हो गयी।

→ कांग्रेस में विभाजन:
1969 में कांग्रेस पार्टी का विभाजन हो गया, जिसके कई कारण थे, जैसे- दक्षिणपंथी एवं वामपंथी विषय पर कलह, राष्ट्रपति पद के उम्मीदवार के विषय में मतभेद, युवा तुर्क एवं सिंडीकेट के बीच कलह तथा मोरारजी से वित्त विभाग वापस लेना इत्यादि।

→ इंदिरा बनाम सिंडिकेट:
इंदिरा गाँधी को असली चुनौती विपक्ष से नहीं अपितु अपनी पार्टी के भीतर से मिली। उन्हें सिंडिकेट से निपटना पड़ा। ‘सिंडिकेट’ कांग्रेस के भीतर ताकतवर और प्रभावशाली नेताओं का एक समूह था। इस समूह के नेताओं का पार्टी के संगठन पर नियंत्रण था। इसके अगुवा मद्रास प्रांत के भूतपूर्व मुख्यमंत्री और कांग्रेस पार्टी के अध्यक्ष रह चुके के. कामराज थे। ‘सिंडिकेट’ ने इंदिरा गाँधी और लाल बहादुर शास्त्री को प्रधानमंत्री बनाने में महत्त्वपूर्ण भूमिका निभाई। इन नेताओं को उम्मीद थी कि इंदिरा गाँधी उनकी सलाहों पर अमल करेंगी लेकिन इसके विपरीत इंदिरा गाँधी ने सरकार और पार्टी के भीतर खुद का मुकाम बनाना शुरू किया। धीरे-धीरे और बड़ी सावधानी से उन्होंने सिंडिकेट को हाशिए पर ला खड़ा किया।

इस तरह इंदिरा गाँधी ने दो चुनौतियों का सामना किया। उन्हें ‘सिंडिकेट’ के प्रभाव से स्वतंत्र अपना मुकाम बनाया और 1967 के चुनावों में कांग्रेस ने जो जमीन खोयी थी उसे हासिल किया। 1967 में कांग्रेस कार्यसमिति ने दस सूत्री कार्यक्रम अपनाया। इस कार्यक्रम में बैंकों पर सामाजिक नियंत्रण, आम बीमा के राष्ट्रीयकरण, शहरी संपदा और आय के परिसीमन, खाद्यान्न का सरकारी वितरण, भूमि सुधार तथा ग्रामीण गरीबों को आवासीय भूखंड देने के प्रावधान शामिल थे।

→ राष्ट्रपति पद का चुनाव, 1969:
सिंडिकेट और इंदिरा गाँधी के बीच की गुटबाजी 1969 में राष्ट्रपति पद के चुनाव के समय सामने आ गई। तत्कालीन राष्ट्रपति जाकिर हुसैन की मृत्यु के कारण राष्ट्रपति का पद खाली था। इंदिरा गाँधी की असहमति के बावजूद सिंडिकेट ने तत्कालीन लोकसभा अध्यक्ष को कांग्रेस पार्टी की तरफ से राष्ट्रपति पद का उम्मीदवार बना दिया। इंदिरा गाँधी ने ऐसे समय में तत्कालीन उपराष्ट्रपति वी.वी. गिरि को बढ़ावा दिया कि वे राष्ट्रपति पद के लिए अपना नामांकन भरें।

JAC Class 12 Political Science Solutions Chapter 5 कांग्रेस प्रणाली : चुनौतियाँ और पुनर्स्थापना

→ 1971 का चुनाव और कांग्रेस का पुनर्स्थापन:
1969 में कांग्रेस के विभाजन के बाद यद्यपि इन्दिरा गाँधी की सरकार अल्पमत में आ गयी थी, लेकिन वह डी. एम. के. तथा भारतीय कम्युनिस्ट पार्टी के समर्थन से टिकी रही। अब श्रीमती गाँधी की सरकार ने भूमि सुधार कानूनों के क्रियान्वयन, भू-परिसीमन के कानून, बैंक राष्ट्रीयकरण तथा प्रिवी पर्स की समाप्ति आदि के द्वारा अपना समाजवादी रंग पेश किया तथा 1970 में लोकसभा भंग कर 1971 में चुनाव कराएं। 1971 के चुनावों में श्रीमती इन्दिरा गाँधी को ऐतिहासिक जीत प्राप्त हुई। श्रीमती इन्दिरा गाँधी की जीत के कई कारण थे, जैसे- श्रीमती गाँधी का चमत्कारिक नेतृत्व, समाजवादी नीतियाँ, कांग्रेसी दल पर श्रीमती गाँधी की पकड़, श्रीमती गाँधी के पक्ष में वोटों का ध्रुवीकरण तथा गरीबी हटाओ का नारा। 1971 के चुनावों में जहाँ श्रीमती गाँधी ने गरीबी हटाओ का नारा दिया, वहीं उनके विरोधियों ने इन्दिरा हटाओ का नारा दिया, जिसे मतदाताओं ने पसन्द नहीं किया तथा श्रीमती गाँधी के पक्ष में मतदान किया।

→ कांग्रेस की पुनर्स्थापना:
1971 के लोकसभा चुनावों के तुरन्त बाद पूर्वी पाकिस्तान (अब बांग्लादेश) में एक बड़ा राजनीतिक और सैन्य संकट उठ खड़ा हुआ। 1971 के चुनावों के बाद पूर्वी पाकिस्तान के मुद्दे को लेकर युद्ध छिड़ गया। इसके परिणामस्वरूप बांग्लादेश बना। इन घटनाओं से इन्दिरा गाँधी की लोकप्रियता में चार चाँद लग गए। विपक्ष के नेताओं तक ने उसके राज्य कौशल की प्रशंसा की। 1972 के राज्य विधानसभा के चुनावों में उनकी पार्टी को व्यापक सफलता मिली। उन्हें गरीबों, वंचितों के रक्षक और एक मजबूत राष्ट्रवादी नेता के रूप में देखा गया। पार्टी के अन्दर अथवा बाहर उसके विरोध की कोई गुंजाइश न बची। कांग्रेस को लोकसभा चुनावों के साथ-साथ राज्य स्तर के चुनावों में भी भारी सफलता प्राप्त हुई।

JAC Class 12 Political Science Solutions Chapter 4 भारत के विदेश संबंध

Jharkhand Board JAC Class 12 Political Science Solutions Chapter 4 भारत के विदेश संबंध Textbook Exercise Questions and Answers.

JAC Board Class 12 Political Science Solutions Chapter 4 भारत के विदेश संबंध

Jharkhand Board Class 12 Political Science भारत के विदेश संबंध InText Questions and Answers

पृष्ठ 66

प्रश्न 1.
चौथे अध्याय में एक बार फिर से जवाहरलाल नेहरू ! क्या वे कोई सुपरमैन थे, या उनकी भूमिका महिमा मंडित कर दी गई है?
उत्तर:
जवाहर लाल नेहरू वास्तव में एक सुपरमैन की भूमिका में ही थे। उन्होंने न केवल भारत के स्वाधीनता आंदोलन में महत्त्वपूर्ण भूमिका निभायी बल्कि स्वतंत्रता के पश्चात् भी उन्होंने राष्ट्रीय एजेण्डा तय करने में निर्णायक भूमिका निभायी। नेहरूजी प्रधानमन्त्री के साथ – साथ विदेश मन्त्री भी थे। उन्होंने भारत की विदेश नीति की रचना और क्रियान्वयन में महत्त्वपूर्ण भूमिका अदा की। नेहरू की विदेश नीति के तीन बड़े उद्देश्य थे-

  1. कठिन संघर्ष से प्राप्त सम्प्रभुता को बचाए रखना,
  2. क्षेत्रीय अखण्डता को बनाए रखना और
  3. तेज रफ्तार से आर्थिक विकास करना। नेहरू इन उद्देश्यों को गुटनिरपेक्षता की नीति अपनाकर हासिल करना चाहते थे।

इसके अतिरिक्त पण्डित नेहरू द्वारा अपनायी गई राष्ट्रवाद, अन्तर्राष्ट्रीयवाद व पंचशील की अवधारणा आज भी भारतीय विदेश नीति के आधार स्तम्भ माने जाते हैं।

पृष्ठ 68

प्रश्न 2.
हम लोग आज की बनिस्बत जब ज्यादा गरीब, कमजोर और नए थे तो शायद दुनिया में हमारी पहचान कहीं ज्यादा थी। हैं ना विचित्र बात?
उत्तर:
शीत युद्ध के दौरान दुनिया दो खेमों – पूँजीवादी और साम्यवादी – में विभाजित हो गई थी। लेकिन भारत ने अपने स्वतन्त्र अस्तित्व को बनाये रखने के लिए किसी भी खेमे में सम्मिलित न होने का निर्णय किया और स्वतन्त्र विदेश नीति का संचालन करते हुए गुटनिरपेक्षता की नीति अपनायी। इसी कारण से भारत की दुनिया में एक विशेष पहचान थी और दुनिया के अल्पविकसित और विकासशील देशों ने भारत की इस नीति का अनुसरण भी किया। लेकिन शीतयुद्ध के अन्त, बदलती विश्व व्यवस्था तथा वैश्वीकरण और उदारीकरण के दौर में भारत की गुटनिरपेक्षता की नीति की प्रासंगिकता पर एक प्रश्न चिह्न लग गया है तथा उसका झुकाव भी अमेरिका की ओर किसी- न-किसी रूप में दिखाई दे रहा है। इसलिए यह कहा जा सकता है कि आज की बनिस्बत हम लोग जब ज्यादा गरीब, कमजोर और नए थे तो शायद दुनिया में हमारी पहचान ज्यादा थी।

पृष्ठ 72

प्रश्न 3.
“मैंने सुना है कि 1962 के युद्ध के बाद जब लता मंगेशकर ने ‘ऐ मेरे वतन के लोगो..’ गाया तो नेहरू भरी सभा में रो पड़े थे। बड़ा अजीब लगता है यह सोचकर कि इतने बड़े लोग भी किसी भावुक लम्हे में रो पड़ते हैं!”
उत्तर:
यह बिल्कुल सत्य है कि जिन लोगों में राष्ट्रवाद व देश-प्रेम की भावना कूट-कूट कर भरी हुई हो और उनके सामने राष्ट्र का गुणगान किया जाए तो ऐसे लोगों का भावुक होना स्वाभाविक है। जहाँ तक पण्डित नेहरू का सवाल है, वे महान राष्ट्रवादी नेता थे। उन्होंने जीवन पर्यन्त राष्ट्र की सेवा की। इस गीत को सुनकर उनके आँसू इसलिए छलक पड़े क्योंकि इस गीत में देश के उन शहीदों की कुर्बानी की बात कही गयी थी जिन्होंने हँसते-हँसते अपनी जान न्यौछावर कर दी थी। उन्हीं की कुर्बानी ने देश को स्वतंत्रता दिलवायी थी।

JAC Class 12 Political Science Solutions Chapter 4 भारत के विदेश संबंध

पृष्ठ 74

प्रश्न 4.
“हम ऐसा क्यों कहते हैं कि भारत और पाकिस्तान के बीच युद्ध हुआ? नेता झगड़े और सेनाओं के बीच युद्ध हुआ। ज्यादातर आम नागरिकों को इनसे कुछ लेना-देना न था।”
उत्तर:
भारत – पाकिस्तान के बीच युद्ध हुआ। इस युद्ध में सेनाओं ने एक-दूसरी सेनाओं से संघर्ष किया तथा राजनेताओं ने इस संघर्ष की अनुमति दी। परन्तु युद्ध का प्रभाव आम जन-जीवन पर भी पड़ता है क्योंकि युद्ध के पीछे की परिस्थितियाँ आम नागरिकों पर भी प्रभाव डालती हैं। दूसरे, सैनिक भी आम नागरिकों में से ही होते हैं। शहीद सैनिकों के परिवारों को आंसुओं के घूँट पीने पड़ते हैं। जन-जीवन अस्त-व्यस्त हो जाता है। इससे स्पष्ट होता है कि युद्ध चाहे नेताओं के वैचारिक स्तर पर हो अथवा सेनाओं के बीच हो, आम जन-जीवन इससे प्रभावित हुए बिना नहीं रहता।

पृष्ठ 76

प्रश्न 5.
इस बात से तो ऐसा जान पड़ता है कि भारत सोवियत खेमे में शामिल हो गया था। क्या सोवियत संघ के साथ इस 20 वर्षीय सन्धि पर हस्ताक्षर करने के बावजूद हम कह सकते हैं कि भारत गुटनिरपेक्षता की नीति पर अमल कर रहा था?
उत्तर:
1971 में हुई भारत – सोवियत मैत्री सन्धि से गुटनिरपेक्षता के सिद्धान्तों का उल्लंघन नहीं हुआ, क्योंकि इस सन्धि के पश्चात् भी भारत गुटनिरपेक्षता के मौलिक सिद्धान्तों पर कायम रहा। वह वारसा गुट में शामिल नहीं हुआ था। उसने अपनी प्रतिरक्षा के लिए बढ़ रहे खतरे, अमेरिका तथा पाकिस्तान में बढ़ रही घनिष्ठता व सहायता के कारण रूस से 20 वर्षीय संधि की थी। उसने संधि के बावजूद गुटनिरपेक्षता की नीति पर अमल किया। यही कारण है कि जब सोवियत संघ की सेनाएँ अफगानिस्तान में पहुँचीं तो भारत ने इसकी आलोचना की। इसलिए यह कहना कि सोवियत मैत्री संधि पर हस्ताक्षर करने के बाद भारत ने गुटनिरपेक्षता की नीति पर अमल नहीं किया, गलत है।

पृष्ठ 78

प्रश्न 6.
बड़ा घनचक्कर है! क्या यहाँ सारा मामला परमाणु बम बनाने का नहीं है? हम ऐसा सीधे-सीधे क्यों नहीं कहते?
उत्तर:
भारत ने सन् 1974 में परमाणु परीक्षण किया तो इसे उसने शांतिपूर्ण परीक्षण करार दिया। भारत ने मई, 1998 में परमाणु परीक्षण किए तथा यह जताया कि उसके पास सैन्य उद्देश्यों के लिए अणु शक्ति को प्रयोग में लाने की क्षमता है। इस दृष्टि से यह मामला यद्यपि परमाणु बम बनाने का ही था तथापि भारत की परमाणु नीति में सैद्धान्तिक तौर पर यह बात स्वीकार की गई है किं भारत अपनी रक्षा के लिए परमाणु हथियार रखेगा लेकिन इन हथियारों का प्रयोग ‘पहले नहीं’ करेगा।

Jharkhand Board Class 12 Political Science भारत के विदेश संबंध TextBook Questions and Answers

प्रश्न 1.
इन बयानों के आगे सही या गलत का निशान लगाएँ-
(क) गुनिरपेक्षता की नीति अपनाने के कारण भारत, सोवियत संघ और संयुक्त राज्य अमेरिका, दोनों की सहायता हासिल कर सका।
(ख) अपने पड़ोसी देशों के साथ भारत के सम्बन्ध शुरूआत से ही तनावपूर्ण रहे।
(ग) शीतयुद्ध का असर भारत-पाक सम्बन्धों पर भी पड़ा।
(घ) 1971 की शान्ति और मैत्री की सन्धि संयुक्त राज्य अमेरिका से भारत की निकटता का परिणाम थी।
उत्तर:
(क) सही
(ख) गलत
(ग) सही
(घ) गलत।

JAC Class 12 Political Science Solutions Chapter 4 भारत के विदेश संबंध

प्रश्न 2.
निम्नलिखित का सही जोड़ा मिलाएँ:

(क) 1950-64 के दौरान भारत की विदेश नीति का लक्ष्य (i) तिब्बत के धार्मिक नेता जो सीमा पार करके भारत चले आए।
(ख) पंचशील (ii) क्षेत्रीय अखण्डता और सम्प्रभुता की रक्षा तथा अर्थिक विकास।
(ग) बांडुंग सम्मेलन (iii) शान्तिपूर्ण सह-अस्तित्व के पाँच सिद्धान्त।
(घ) दलाई
लामा
(iv) इसकी परिणति गुटनिरपेक्ष आन्दोलन में हुई।

उत्तर:

(क) 1950-64 के दौरान भारत की विदेश नीति का लक्ष्य (ii) क्षेत्रीय अखण्डता और सम्प्रभुता की रक्षा तथा अर्थिक विकास।
(ख) पंचशील (iii) शान्तिपूर्ण सह-अस्तित्व के पाँच सिद्धान्त।
(ग) बांडुंग सम्मेलन (iv) इसकी परिणति गुटनिरपेक्ष आन्दोलन में हुई।
(घ) दलाई लामा (i) तिब्बत के धार्मिक नेता जो सीमा पार करके भारत चले आए।

प्रश्न 3.
नेहरू विदेश नीति के संचालन को स्वतन्त्रता का एक अनिवार्य संकेतक क्यों मानते थे? अपने उत्तर में दो कारण बताएँ और उनके पक्ष में उदाहरण भी दें।
उत्तर:
नेहरू विदेश नीति के संचालन को स्वतन्त्रता का एक अनिवार्य संकेत इसलिए मानते थे क्योंकि स्वतन्त्रता किसी भी देश की विदेश नीति के संचालन की प्रथम एवं अनिवार्य शर्त है। स्वतंत्रता से तात्पर्य है। राष्ट्र के पास प्रभुसत्ता का होना तथा प्रभुसत्ता के दो पक्ष हैं।

  1. आंतरिक संप्रभुता और
  2. बाह्य संप्रभुता। इसमें बाह्य संप्रभुता का सम्बन्ध विदेश नीति का स्वतन्त्रतापूर्वक बिना किसी दूसरे राष्ट्र के दबाव के संचालित करना है।

दो कारण और उदाहरण:

  1. जो देश किसी दबाव में आकर अपनी विदेश नीति का निर्धारण करता है तो उसकी स्वतंत्रता निरर्थक होती है तथा एक प्रकार से दूसरे देश के अधीन हो जाता है व उसे अनेक बार अपने राष्ट्रीय हितों की भी अनदेखी करनी पड़ती है। इसी तथ्य को ध्यान में रखते हुए नेहरू ने शीत युद्ध काल में किसी भी गुट में शामिल न होने और असंलग्नता की नीति को अपनाकर दोनों गुटों के दबाव को नहीं माना।
  2. भारत स्वतन्त्र नीति को इसलिए अनिवार्य मानता था ताकि यह अन्तर्राष्ट्रीय स्तर पर उपनिवेशवाद, जातीय भेदभाव, रंग-भेदभाव का मुकाबला डटकर कर सके। भारत ने 1949 में साम्यवादी चीन को मान्यता प्रदान की तथा सुरक्षा परिषद् में उसकी सदस्यता का समर्थन किया और सोवियत संघ ने हंगरी पर जब आक्रमण किया तो उसकी निंदा की।

प्रश्न 4.
“ विदेश नीति का निर्धारण घरेलू जरूरत और अन्तर्राष्ट्रीय परिस्थितियों के दोहरे दबाव में होता है।” 1960 के दशक में भारत द्वारा अपनाई गई विदेश नीति से एक उदाहरण देते हुए अपने उत्तर की पुष्टि करें।
उत्तर:
किसी देश की विदेश नीति पर घरेलू और अन्तर्राष्ट्रीय वातावरण का असर पड़ता है। विकासशील देशों के पास अन्तर्राष्ट्रीय व्यवस्था के भीतर अपने सरोकारों को पूरा करने के लिए जरूरी संसाधनों का अभाव होता है। ऐसे देशों का जोर इस बात पर होता है कि उनके पड़ोस में अमन-चैन कायम रहे और विकास होता रहे। 1960 के दशक में भारत द्वारा गुटनिरपेक्षता की नीति अपनाना भी इसका उदाहरण माना जा सकता है। उदाहरणार्थ – तत्कालीन समय में भारत की आर्थिक स्थिति अत्यधिक दयनीय थी, इसलिए उसने शीत युद्ध के काल में किसी भी गुट का समर्थन नहीं किया और दोनों ही गुटों से आर्थिक सहायता प्राप्त करता रहा।

JAC Class 12 Political Science Solutions Chapter 4 भारत के विदेश संबंध

प्रश्न 5.
अगर आपको भारत की विदेश नीति के बारे में फैसला लेने को कहा जाए तो आप इसकी किन बातों को बदलना चाहेंगे? ठीक तरह यह भी बताएँ कि भारत की विदेश नीति के किन दो पहलुओं को आप बरकरार रखना चाहेंगे? अपने उत्तर के समर्थन में तर्क दीजिए
उत्तर:

  • भारतीय विदेश नीति में निम्न दो बदलाव लाना चाहूँगा-
    1. मैं वर्तमान एकध्रुवीय विश्व में भारत की गुटनिरपेक्षता की नीति में बदलाव लाना चाहूँगा क्योंकि वर्तमान वैश्वीकरण और उदारीकरण के युग में गुट निरपेक्षता की नीति अप्रासंगिक हो गयी है।
    2. मैं भारत की विदेश नीति में चीन एवं पाकिस्तान के साथ जिस प्रकार की नीति अपनाई जा रही है उसमें बदलाव लाना चाहूँगा, क्योंकि इसके वांछित परिणाम नहीं मिल पा रहे हैं।
  • इसके अतिरिक्त मैं भारतीय विदेश नीति के निम्नलिखित दो पहलुओं को बरकरार रखना चाहूँगा
    1. सी. टी.बी. टी. के बारे में वर्तमान दृष्टिकोण को और परमाणु नीति की वर्तमान नीति को जारी रखूँगा।
    2. मैं संयुक्त राष्ट्र संघ की सदस्यता जारी रखते हुए विश्व बैंक अन्तर्राष्ट्रीय मुद्राकोष से सहयोग जारी रखूँगा।

प्रश्न 6.
निम्नलिखित पर संक्षिप्त टिप्पणियाँ लिखिए-
(क) भारत की परमाणु नीति।
(ख) विदेश नीति के मामलों पर सर्व सहमति
उत्तर:
(क) भारत की परमाणु नीति- भारत ने मई, 1974 और 1998 में परमाणु परीक्षण करते हुए अपनी परमाणु नीति को नई दिशा प्रदान की। इसके प्रमुख पक्ष निम्नलिखित हैं-

  1. आत्मरक्षार्थ – भारत ने आत्मरक्षा के लिए परमाणु हथियारों का निर्माण किया, ताकि कोई अन्य देश भारत पर परमाणु हमला न कर सक।
  2. प्रथम प्रयोग नहीं – भारत ने परमाणु हथियारों का युद्ध में पहले प्रयोग न करने की घोषणा कर रखी है।
  3. भारत परमाणु नीति एवं परमाणु हथियार बनाकर विश्व में एक शक्तिशाली राष्ट्र बनना चाहता है तथा विश्व में प्रतिष्ठा प्राप्त करना चाहता है।
  4. परमाणु सम्पन्न राष्ट्रों की भेदभावपूर्ण नीति का विरोध करना।

(ख) विदेश नीति के मामलों पर सर्व सहमति – विदेश नीति के मामलों पर सर्वसहमति आवश्यक है, क्योंकि यदि एक देश की विदेश नीति के मामलों में सर्वसहमति नहीं होगी, तो वह देश अन्तर्राष्ट्रीय मंचों पर अपना पक्ष प्रभावशाली ढंग से नहीं रख पायेगा। भारत की विदेश नीति के महत्त्वपूर्ण बिन्दुओं जैसे गुटनिरपेक्षता, साम्राज्यवाद एवं उपनिवेशवाद का विरोध, दूसरे देशों से मित्रतापूर्ण सम्बन्ध बनाना तथा अन्तर्राष्ट्रीय शान्ति एवं सुरक्षा को बढ़ावा देना इत्यादि पर सदैव सर्वसहमति रही है।

JAC Class 12 Political Science Solutions Chapter 4 भारत के विदेश संबंध

प्रश्न 7.
भारत की विदेश नीति का निर्माण शान्ति और सहयोग के सिद्धान्तों को आधार मानकर हुआ। लेकिन, 1962-1971 की अवधि यानी महज दस सालों में भारत को तीन युद्धों का सामना करना पड़ा। क्या आपको लगता है कि यह भारत की विदेश नीति की असफलता है अथवा आप इसे अन्तर्राष्ट्रीय परिस्थितियों का परिणाम मानेंगे? अपने मंतव्य के पक्ष में तर्क दीजिए।
उत्तर:
आजादी के समय भारत ने अपनी विदेश नीति का निर्माण शान्ति और सहयोग के सिद्धान्तों के आधार पर किया। परन्तु 1962 से लेकर 1971 तक भारत को तीन युद्ध लड़ने पड़े तो इसके पीछे कुछ हद तक भारत की विदेश नीति की असफलता भी मानी जाती है तथा अन्तर्राष्ट्रीय परिस्थितियों का परिणाम भी। यथा-
1. चीन का आक्रमण ( 1962 ):
चीन और भारत के दोस्ताना रिश्ते में गंभीर विवाद तब पैदा हुआ जब 1950 में चीन ने तिब्बत पर अपना अधिकार जमा लिया। क्योंकि इस वजह से भारत और चीन के बीच ऐतिहासिक रूप से जो मध्यवर्ती राज्य बना चला आ रहा था, वह खत्म हो गया। शुरू में भारत (सरकार) ने इसका खुलकर विरोध नहीं किया था । भारत की बेचैनी तब बढ़ी जब चीन ने तिब्बत की संस्कृति को दबाना शुरू किया। भारत और चीन के बीच एक सीमा विवाद भी उठ खड़ा हुआ था। यह विवाद चीन से लगी लंबी सीमा रेखा के पश्चिमी और पूर्वी छोर के बारे में था। चीन ने भारतीय भू-क्षेत्र में पड़ने वाले अक्साई चीन और अरुणाचल प्रदेश के अधिकांश हिस्सों पर अपना दावा किया। दोनों देशों के शीर्ष नेताओं के बीच लंबी चर्चा और वार्तालाप के बावजूद इस मतभेद को सुलझाया नहीं जा सका। इसलिए भारत को संघर्ष में शामिल होना पड़ा।

2. पाकिस्तान के साथ युद्ध-कश्मीर मसले को लेकर पाकिस्तान के साथ बँटवारे के तुरंत बाद ही संघर्ष छिड़ गया था। 1947 में ही कश्मीर में भारत और पाकिस्तान की सेनाओं के बीच छाया-युद्ध छिड़ गया था। 1965 में इस छाया-युद्ध ने गंभीर किस्म के सैन्य संघर्ष का रूप ले लिया। बाद में, भारतीय प्रधानमंत्री लालबहादुर शास्त्री और संजीव पास बुक्स पाकिस्तान के जनरल अयूब खान के बीच 1966 में ताशकंद – समझौता हुआ। 1965 की लड़ाई ने भारत की कठिन आर्थिक स्थिति को और गंभीर बना दिया था।

3. बांग्लादेश युद्ध, 1971-1970 में पाकिस्तान के सामने एक गहरा अंदरूनी संकट आ खड़ा हुआ। जुल्फिकार अली भुट्टो की पार्टी पश्चिमी पाकिस्तान में विजयी रही जबकि मुजीबुर्रहमान की पार्टी ने पूर्वी पाकिस्तान में कामयाबी हासिल की। पूर्वी पाकिस्तान की बंगाली आबादी ने पश्चिमी पाकिस्तान के भेदभावपूर्ण रवैये के विरोध में मतदान किया था, परंतु यह जनादेश पाकिस्तान के शासक स्वीकार नहीं कर पा रहे थे।

1971 में पाकिस्तानी सेना ने शेख मुजीब को गिरफ्तार कर लिया और पूर्वी पाकिस्तान के लोगों पर जुल्म करना शुरू कर दिया। फलस्वरूप पर्वी पाकिस्तान की जनता ने अपने इलाके को पाकिस्तान से मुक्त कराने के लिए संघर्ष छेड़ दिया। 1971 में भारत को 80 लाख शरणार्थियों का भार पठाना पड़ा। ये शरणार्थी पूर्वी पाकिस्तान से भागकर आए थे। उपर्युक्त कारणों की वजह से भारत ने बांग्लादेश के ‘मुक्ति संग्राम’ को नैतिक समर्थन और भौतिक सहायता दी।

महीनों राजनयिक तनाव और सैन्य तैनाती के बाद 1971 के दिसंबर में भारत और पाकिस्तान के बीच पूर्ण पैमाने पर युद्ध छिड़ गया। पाकिस्तान के लड़ाकू विमानों ने पंजाब और राजस्थान पर हमला किया और उसकी सेना ने जम्मू- कश्मीर पर हमला किया। भारत ने अपनी जल, थल और वायु सेना द्वारा इस हमले का जवाब दिया। दस दिनों के भीतर भारतीय सेना ने ढाका को तीन तरफ से घेर लिया और पाकिस्तानी सेना को आत्मसमर्पण करना पड़ा। तत्पश्चात् बांग्लादेश के रूप में एक स्वतंत्र राष्ट्र के उदय के साथ भारतीय सेना ने अपनी तरफ से एकतरफा युद्ध-विराम घोषित कर दिया। बाद में 3 जुलाई, 1972 को इंदिरा गाँधी और जुल्फिकार अली भुट्टो के बीच शिमला समझौता हुआ। अधिकांश भारतीयों ने इसे गौरव की घड़ी के रूप में देखा और माना कि भारत का सैन्य-पराक्रम प्रबल हुआ है।

JAC Class 12 Political Science Solutions Chapter 4 भारत के विदेश संबंध

प्रश्न 8.
क्या भारत की विदेश नीति से यह झलकता है कि भारत क्षेत्रीय स्तर की महाशक्ति बनना चाहता है? 1971 के बांग्लादेश युद्ध के सन्दर्भ में इस प्रश्न पर विचार करें।
उत्तर:
भारत की विदेश नीति से यह बिल्कुल नहीं झलकता कि भारत क्षेत्रीय स्तर की महाशक्ति बनना चाहता है। 1971 के बांग्लादेश युद्ध के सन्दर्भ में देखें तो भी ऐसा प्रतीत नहीं होता। बांग्लादेश के निर्माण के लिए स्वयं पाकिस्तान की पूर्वी पाकिस्तान के प्रति उपेक्षापूर्ण नीतियाँ, बंगालियों की अपनी भाषा, संस्कृति के प्रति अटूट प्यार व गहरी आस्था को माना जा सकता है। दूसरे, भारत ने कभी भी अपने छोटे-छोटे पड़ौसी देशों को अमेरिका की तरह अपने गुट में सम्मिलित करने का प्रयास नहीं किया बल्कि 1965 तथा 1971 में जीते गये क्षेत्रों को भी वापस कर दिया। इस प्रकार भारत बलपूर्वक किसी देश पर बात मनवाना नहीं चाहता।

प्रश्न 9.
किसी राष्ट्र का राजनीतिक नेतृत्व किस तरह उस राष्ट्र की विदेश नीति पर असर डालता है? भारत की विदेश नीति के उदाहरण देते हुए इस प्रश्न पर विचार कीजिए।
उत्तर:
किसी भी देश की विदेश नीति के निर्धारण में उस देश के राजनीतिक नेतृत्व की विशेष भूमिका होती है उदाहरण के लिए, पण्डित नेहरू के विचारों से भारत की विदेश नीति पर्याप्त प्रभावित हुई। पण्डित नेहरू साम्राज्यवाद, उपनिवेशवाद व फासिस्टवाद के घोर विरोधी थे और वे समस्याओं का समाधान करने के लिए शान्तिपूर्ण मार्ग के समर्थक थे । वह मैत्री, सहयोग व सहअस्तित्व के प्रबल समर्थक थे। पण्डित जवाहरलाल नेहरू ने अपने विचारों द्वारा भारत की विदेश नीति के ढाँचे को ढाला।

इसी प्रकार श्रीमती इन्दिरा गाँधी, राजीव गाँधी और अटल बिहारी वाजपेयी के नेतृत्व व व्यक्तित्व की भी भारत की विदेश नीति पर स्पष्ट छाप दिखाई देती है। श्रीमती इन्दिरा गांधी ने बैंकों का राष्ट्रीयकरण किया, गरीबी हटाओ का नारा दिया और रूस के साथ दीर्घ अनाक्रमण सन्धि की। इसी तरह अटल बिहारी वाजपेयी की विदेश नीति में देश में परमाणु शक्ति का विकास हुआ। उन्होंने ‘जय जवान, जय किसान और जय विज्ञान’ का नारा दिया। इस प्रकार देश का राजनीतिक नेतृत्व उस राष्ट्र की विदेश नीति पर प्रभाव डालता है।

प्रश्न 10.
निम्नलिखित अवतरण को पढ़ें और इसके आधार पर पूछे गये प्रश्नों के उत्तर दीजिए : गुटनिरपेक्षता का व्यापक अर्थ है अपने को किसी भी सैन्य गुट में शामिल नहीं करना… इसका अर्थ होता है चीजों को यथासम्भव सैन्य दृष्टिकोण से न देखना और इसकी कभी जरूरत आन पड़े तब भी किसी सैन्य गुट के नजरिए को अपनाने की जगह स्वतन्त्र रूप से स्थिति पर विचार करना तथा सभी देशों के साथ दोस्ताना रिश्ते कायम करना…..
(क) नेहरू सैन्य गुटों से दूरी क्यों बनाना चाहते थे? – जवाहरलाल नेहरू
(ख) क्या आप मानते हैं कि भारत – सोवियत मैत्री की सन्धि से गुटनिरपेक्षता के सिद्धान्तों का उल्लंघन हुआ? अपने उत्तर के समर्थन में तर्क दीजिए।
(ग) अगर सैन्य – गुट न होते तो क्या गुटनिरपेक्षता की नीति बेमानी होती?
उत्तर:
(क) नेहरू सैन्य गुटों से दूरी बनाना चाहते थे क्योंकि वे किसी भी सैनिक गुट में शामिल न होकर एक स्वतन्त्र विदेश नीति का संचालन करना चाहते थे।

(ख) भारत – सोवियत मैत्री सन्धि से गुटनिरपेक्षता के सिद्धान्तों का उल्लंघन नहीं हुआ, क्योंकि इस सन्धि के पश्चात् भी भारत गुटनिरपेक्षता के मौलिक सिद्धान्तों पर कायम रहा तथा जब सोवियत संघ की सेनाएँ अफगानिस्तान में पहुँचीं तो भारत ने इसकी आलोचना की ।

(ग) यदि विश्व में सैनिक गुट नहीं होते तो भी गुटनिरपेक्षता की प्रासंगिकता बनी रहती, क्योंकि गुटनिरपेक्ष आन्दोलन की स्थापना शान्ति एवं विकास के लिए की गई थी तथा शान्ति एवं विकास के लिए चलाया गया कोई भी आन्दोलन कभी भी अप्रासंगिक नहीं हो सकता।

भारत के विदेश संबंध JAC Class 12 Political Science Notes

→ अंतर्राष्ट्रीय संदर्भ:
भारत बड़ी विकट और चुनौतीपूर्ण अन्तर्राष्ट्रीय परिस्थितियों में आजाद हुआ था, जैसे महायुद्ध की तबाही, पुनर्निर्माण की समस्या, उपनिवेशवाद की समाप्ति, लोकतन्त्र को कायम करना तथा एक अन्तर्राष्ट्रीय संस्था का निर्माण करना आदि की चुनौतियाँ विश्व के समक्ष थीं। इसके अतिरिक्त भारत के समक्ष अनेक विषम घरेलू परिस्थितियाँ भी थीं, जैसे बंटवारे का दबाव, गरीबी तथा अशिक्षा आदि। ऐसे में भारत ने अपनी विदेश नीति में अन्य सभी देशों की सम्प्रभुता का सम्मान करने और शान्ति कायम करके अपनी सुरक्षा सुनिश्चित करने का लक्ष्य सामने रखा।

→ संवैधानिक सिद्धान्त:
भारतीय संविधान के अनुच्छेद 51 में अन्तर्राष्ट्रीय शान्ति और सुरक्षा को बढ़ावा देने के लिए राज्य के नीति-निर्देशक सिद्धान्तों के हवाले से कहा गया है कि राज्य को विदेशों से सम्बन्ध बनाये रखने हेतु निम्न निर्देशों का पालन करना अनिवार्य है-

  • अन्तर्राष्ट्रीय शान्ति और सुरक्षा में अभिवृद्धि करना।
  • राष्ट्रों के बीच न्यायसंगत और सम्मानपूर्ण सम्बन्धों को बनाये रखना।
  • संगठित लोगों के एक-दूसरे से व्यवहारों में अन्तर्राष्ट्रीय विधि और संधि – बाध्यताओं के प्रति आदर की भावना रखना।
  • अन्तर्राष्ट्रीय विवादों को मध्यस्थता द्वारा निपटाने का प्रयास करना।

→ गुटनिरपेक्षता की नीति:
द्वितीय विश्व-युद्ध के बाद दुनिया दो खेमों (सैनिक गुटों) में विभाजित हो गई । एक खेमे का अगुआ संयुक्त राज्य अमेरिका और दूसरे का सोवियत संघ। दोनों खेमों के बीच विश्व स्तर पर आर्थिक, राजनीतिक और सैन्य टकराव जारी रहा। इसी दौर में संयुक्त राष्ट्रसंघ भी अस्तित्व में आया; परमाणु हथियारों का निर्माण शुरू हुआ; चीन में कम्युनिस्ट शासन की स्थापना हुई । अनौपनिवेशीकरण की प्रक्रिया भी इसी दौर में आरंभ हुई थी। नेहरू की भूमिका – पण्डित जवाहरलाल नेहरू ने भारत के प्रधानमन्त्री व विदेश मन्त्री के रूप में भारत की विदेश नीति की रचना की और इसके क्रियान्वयन में भी महत्त्वपूर्ण भूमिका अदा की। नेहरू की विदेश नीति के तीन बड़े उद्देश्य थे

  • कठिन संघर्ष से प्राप्त सम्प्रभुता को बचाये रखना,
  • क्षेत्रीय अखण्डता को बनाये रखना, और
  • तेज रफ्तार से आर्थिक विकास करना नेहरू इन उद्देश्यों को गुटनिरपेक्षता की नीति अपनाकर हासिल करना चाहते थे।

JAC Class 12 Political Science Solutions Chapter 4 भारत के विदेश संबंध

→ दोनों खेमों से दूरी:
आजाद भारत की विदेश नीति में शान्तिपूर्ण विश्व का सपना था और इसके लिए भारत ने गुटनिरपेक्षता की नीति का पालन किया। इसके लिए भारत ने शीतयुद्ध से उपजे तनाव को कम करने की कोशिश की और संयुक्त राष्ट्र संघ के शान्ति अभियानों में अपनी सेना भेजी। भारत शीतयुद्ध के दौरान किसी भी खेमे में शामिल नहीं हुआ।

→ एफ्रो-एशियाई एकता:
भारत के आकार, अवस्थिति और शक्ति सम्भावनाओं को देखते हुए नेहरू ने विश्व के मामलों, विशेषकर एशियाई मामलों में भारत के लिए बड़ी भूमिका निभाने का स्वप्न देखा। नेहरू के दौर में भारत ने एशिया और अफ्रीका के नव- स्वतन्त्र देशों के साथ सम्पर्क बनाए। 1940 और 1950 के दशकों में नेहरू ने बड़े मुखर स्वर में एशियाई एकता की पेशकश की। नेहरू की अगुआई में भारत में मार्च, 1947 में एशियाई सम्बन्ध सम्मेलन का आयोजन किया गया। भारत ने इंडोनेशिया के स्वतंत्रता संग्राम के समर्थन में एक अंतर्राष्ट्रीय सम्मेलन किया।

भारत अनौपनिवेशीकरण की प्रक्रिया का प्रबल समर्थक था और उसने पूरी दृढ़ता से नस्लवाद का, खासकर दक्षिण अफ्रीका में हो रहे रंगभेद का विरोध किया । इण्डोनेशिया के शहर बांडुंग में एफ्रो-एशियाई सम्मेलन 1955 में हुआ। इसी सम्मेलन में गुटनिरपेक्ष आंदोलन की नींव पड़ी। गुटनिरपेक्ष आंदोलन का पहला सम्मेलन 1961 के सितंबर में बेलग्रेड में हुआ।

→ चीन के साथ शान्ति और संघर्ष:
आजाद भारत ने चीन के साथ अपने रिश्तों की शुरूआत बड़े मैत्रीपूर्ण ढंग से की। चीनी क्रान्ति 1949 में हुई थी। इस क्रान्ति के बाद भारत, चीन की कम्युनिस्ट सरकार को मान्यता देने वाला पहला देश था। इसके अतिरिक्त शान्तिपूर्ण सहअस्तित्व के पाँच सिद्धान्तों यानी पंचशील की घोषणा भारत के प्रधानमन्त्री नेहरू और चीन के प्रमुख नेता चाऊ एन लाई ने संयुक्त रूप से 29 अप्रैल, 1954 को की। दोनों देशों के बीच मजबूत सम्बन्ध की दिशा में यह एक अगला कदम था। भारत और चीन के नेता एक-दूसरे के देश का दौरा करते थे और उनके स्वागत के लिए भारी भीड़ जुटती थी।

→ चीन का आक्रमण, 1962:
चीन के साथ भारत के दोस्ताना रिश्ते में दो कारणों से खटास आई। चीन ने 1950 में तिब्बत पर कब्जा कर लिया। तिब्बत की संस्कृति को कुचलने की खबर आने पर भारत की बेचैनी भी बढ़ी। तिब्बत के धार्मिक नेता दलाई लामा ने भारत से राजनीतिक शरण माँगी और 1959 में भारत ने उन्हें शरण दे दी। इससे कुछ दिनों पहले भारत और चीन के बीच सीमा विवाद भी उठा था। मुख्य विवाद चीन से लगी लंबी सीमा-रेखा के पश्चिमी और पूर्वी छोर के बारे में था। चीन ने भारतीय भू-क्षेत्र में पड़ने वाले दो इलाकों जम्मू-कश्मीर के लद्दाख वाले हिस्से के अक्साई चीन और अरुणाचल प्रदेश के अधिकांश हिस्सों पर अपना अधिकार जताया। 1957. से 1959 के बीच चीन ने अक्साई चीन इलाके पर कब्जा कर लिया और इस इलाके में उसने रणनीतिक बढ़त हासिल करने के लिए एक सड़क बनाई।

जिस समय पूरे विश्व का ध्यान दो महाशक्तियों की तनातनी से पैदा इस संकट की तरफ लगा हुआ था, उसी समय चीन ने 1962 के अक्टूबर में दोनों विवादित क्षेत्रों पर बड़ी तेजी तथा व्यापक स्तर पर हमला किया। पहले हमले में चीनी सेना ने अरुणाचल प्रदेश के कुछ महत्त्वपूर्ण इलाकों पर कब्जा कर लिया। दूसरे हमले में लद्दाख से लगे पश्चिमी मोर्चे पर भारतीय सेना ने चीनी सेना को रोक लिया था परंतु पूर्व में चीनी सेना आगे बढ़ते हुए असम के मैदानी हिस्से तक पहुँच गई। आखिरकार, चीन ने एकतरफा युद्ध विराम घोषित किया और चीन की सेना अपने पुराने जगह पर वापस चली गई। चीन-युद्ध से भारत की छवि को देश और विदेश दोनों ही जगह धक्का लगा। इस युद्ध से भारतीय राष्ट्रीय स्वाभिमान को चोट पहुँची परंतु इसके साथ-साथ राष्ट्र – भावना भी बलवती हुई।

इस संघर्ष का असर विपक्षी दलों पर भी हुआ। चीन के साथ हुए युद्ध ने भारत के नेताओं को पूर्वोत्तर की डाँवाडोल स्थिति के प्रति संचेत किया। यह इलाका अत्यंत पिछड़ी दशा में था और अलग-थलग पड़ गया था। राष्ट्रीय एकता और अखंडता के लिहाज से भी यह इलाका चुनौतीपूर्ण था। चीन – युद्ध के तुरंत बाद इस इलाके को नयी तरतीब में ढालने की कोशिशें शुरू की गईं। नागालैंड को प्रांत का दर्जा दिया गया। मणिपुर और त्रिपुरा केन्द्र शासित प्रदेश थे लेकिन उन्हें अपनी विधानसभा के निर्वाचन का अधिकार मिला।

→ पाकिस्तान के साथ युद्ध और शान्ति:
पाकिस्तान की स्थापना सन् 1947 में भारत के विभाजन के परिणामस्वरूप हुई। आरम्भ से ही भारत तथा पाकिस्तान के आपसी सम्बन्ध तनावपूर्ण रहे। सन् 1947 में कश्मीर मुद्दे को लेकर दोनों देशों की सेनाओं के मध्य छाया- युद्ध हुआ । बहरहाल यह संघर्ष पूर्णव्यापी युद्ध का रूप न ले सका। यह मसला संयुक्त राष्ट्र संघ के हवाले कर दिया गया। सन् 1965 में पाकिस्तान ने भारत पर फिर आक्रमण किया परन्तु भारत ने इसका मुँहतोड़ जवाब दिया और पाकिस्तान को बुरी तरह पराजित किया। अन्त में संयुक्त राष्ट्रसंघ के माध्यम से दोनों देशों के बीच ताशकन्द समझौता हुआ, जिसके परिणामस्वरूप भारत ने पाकिस्तान के जीते हुए क्षेत्र वापस लौटा दिए।

JAC Class 12 Political Science Solutions Chapter 4 भारत के विदेश संबंध

→ बांग्लादेश युद्ध, 1971:
सन् 1971 में बांग्लादेश की समस्या को लेकर भारत तथा पाकिस्तान के बीच पुनः युद्ध छिड़ गया। इस युद्ध में पाकिस्तान पुनः पराजित हुआ और पाकिस्तान का एक बहुत बड़ा भाग उसके हाथ से छिन गया अर्थात् बांग्लादेश नामक एक स्वतन्त्र राज्य की स्थापना हुई।

→ करगिल संघर्ष:
1999 – सन् 1999 में पाकिस्तान ने भारत के नियन्त्रण रेखा के कई ठिकानों जैसे द्रास, मश्कोह, काकसर और बतालिक क्षेत्र पर कब्जा कर लिया। भारतीय सेना ने जवाबी कार्यवाही की इससे दोनों देशों के मध्य संघर्ष छिड़ गया। इसे करगिल की लड़ाई के नाम से जाना जाता है। 26 जुलाई, 1999 तक भारतीय सेना ने पाकिस्तानी सेना को खदेड़ते हुए पुनः अपने क्षेत्रों पर अधिकार कर लिया । पाकिस्तान के इस आक्रमण की अन्तर्राष्ट्रीय जगत में तीव्र आलोचना की गई। भारत एवं पाकिस्तान के मध्य तनावपूर्ण सम्बन्धों को सुधारने हेतु अनेक प्रयास किये गये जिसमें रेल व बस यात्राएँ शुरू करना, द्विपक्षीय वार्ताएँ की गईं लेकिन अभी तक कोई स्थायी समाधान नहीं निकल पाया है।

→ भारत की परमाणु नीति:
भारत मई, 1974 में पहला परमाणु परीक्षण कर परमाणु आयुध सम्पन्न राष्ट्रों की श्रेणी में आया लेकिन इसकी शुरूआत 1940 के दशक के अन्तिम सालों में होमी जहाँगीर भाभा के निर्देशन में हो चुकी थी। भारत शान्तिपूर्ण उद्देश्यों में इस्तेमाल के लिए अणु ऊर्जा का उपयोग करने के पक्ष में है। भारत ने परमाणु अप्रसार के लक्ष्य को ध्यान में रखकर की गई सन्धियों का विरोध किया क्योंकि ये सन्धियाँ उन्हीं देशों पर लागू होती थीं जो परमाणु शक्तिहीन राष्ट्र थे। मई, 1998 में भारत ने परमाणु परीक्षण किए और दुनिया को यह जताया कि उसके पास भी सैन्य उद्देश्यों के लिए अणु-शक्ति के इस्तेमाल की क्षमता है। भारत की परमाणु नीति में सैद्धान्तिक तौर पर यह बात स्वीकार की गई है कि भारत अपनी रक्षा के लिए परमाणु हथियार रखेगा लेकिन वह हथियारों का प्रयोग पहले नहीं करेगा। इसके साथ ही भारत वैश्विक स्तर पर लागू और भेदभावहीन परमाणु निःशस्त्रीकरण के प्रति वचनबद्ध है ताकि परमाणु हथियारों से मुक्त विश्व की रचना हो।

JAC Class 12 Political Science Solutions Chapter 3 नियोजित विकास की राजनीति

Jharkhand Board JAC Class 12 Political Science Solutions Chapter 3 नियोजित विकास की राजनीति Textbook Exercise Questions and Answers

JAC Board Class 12 Political Science Solutions Chapter 3 नियोजित विकास की राजनीति

Jharkhand Board Class 12 Political Science नियोजित विकास की राजनीति InText Questions and Answers

पृष्ठ 49

प्रश्न 1.
क्या आप यह कह रहे हैं कि ‘आधुनिक’ बनने के लिए ‘पश्चिमी’ होना जरूरी नहीं है? क्या यह सम्भव है?
उत्तर:
आधुनिक बनने के लिए पश्चिमी होना जरूरी नहीं है। प्रायः आधुनिकीकरण का सम्बन्ध पाश्चात्यीकरण से माना जाता है लेकिन यह सही नहीं है, क्योंकि आधुनिक होने का अर्थ मूल्यों एवं विचारों में समस्त परिवर्तन से लगाया जाता है और यह परिवर्तन समाज को आगे की ओर ले जाने वाले होने चाहिए। आधुनिकीकरण में परिवर्तन केवल विवेक पर ही नहीं बल्कि सामाजिक मूल्यों पर भी आधारित होते हैं। इसमें वस्तुतः समाज के मूल्य साध्य और लक्ष्य भी निर्धारित करते हैं कि कौनसा परिवर्तन अच्छा है और कौनसा बुरा है; कौनसा परिवर्तन आगे की ओर ले जाने वाला है और कौनसा अधोगति में पहुँचाने वाला है। जबकि पाश्चात्यीकरण मूल्यमुक्तता पर बल देता है, इसका न कोई क्रम होता है और न कोई दिशा इस प्रकार आधुनिक बनने के लिए पश्चिमी होना जरूरी नहीं है।

पृष्ठ 58

प्रश्न 2.
अरे! मैं तो भूमि सुधारों को मिट्टी की गुणवत्ता सुधारने की तकनीक समझता था।
उत्तर:
भूमि सुधार का तात्पर्य मिट्टी की गुणवत्ता सुधारने की तकनीक तक ही सीमित नहीं है बल्कि इसके अन्तर्गत अनेक तत्त्वों को सम्मिलित किया जाता है-

  1. जमींदारी एवं जागीरदारी व्यवस्था को समाप्त करना।
  2. जमीन के छोटे-छोटे टुकड़ों को एक साथ करके कृषिगत कार्य को अधिक सुविधाजनक बनाना।
  3. बेकार एवं बंजर भूमि को उपजाऊ बनाने हेतु व्यवस्था करना।
  4. कृषिगत भू- जोतों की उचित व्यवस्था करना।
  5. सिंचाई के साधनों का विकास करना।
  6. अच्छे खाद व उन्नत बीजों की व्यवस्था करना।
  7. किसानों द्वारा उत्पन्न खाद्यान्नों की उचित कीमत दिलाने का प्रयास करना।
  8. किसानों को समय-समय पर विभिन्न प्रकार के ऋण व विशेष अनुदानों की व्यवस्था करना आदि।

Jharkhand Board Class 12 Political Science नियोजित विकास की राजनीति TextBook Questions and Answers

प्रश्न 1
‘बॉम्बे प्लान’ के बारे में निम्नलिखित में कौनसा बयान सही नहीं है?
(क) यह भारत के आर्थिक भविष्य का एक ब्लू-प्रिंट था।
(ख) इसमें उद्योगों के ऊपर राज्य के स्वामित्व का समर्थन किया गया था।
(ग) इसकी रचना कुछ अग्रणी उद्योगपतियों ने की थी।
(घ) इसमें नियोजन के विचार का पुरजोर समर्थन किया गया था।
उत्तर:
(ख) इसमें उद्योगों के ऊपर राज्य के स्वामित्व का समर्थन किया गया था।

JAC Class 12 Political Science Solutions Chapter 3 नियोजित विकास की राजनीति

प्रश्न 2.
भारत ने शुरूआती दौर में विकास की जो नीति अपनाई उसमें निम्नलिखित में से कौनसा विचार शामिल नहीं था?
(क) नियोजन
(ख) उदारीकरण
(ग) सहकारी खेती
(घ) आत्मनिर्भरता ।
उत्तर:
(ख) उदारीकरण।

प्रश्न 3.
भारत में नियोजित अर्थव्यवस्था चलाने का विचार ग्रहण किया गया था:
(क) बॉम्बे प्लान से
(ख) सोवियत खेमे के देशों के अनुभवों से
(ग) समाज के बारे में गाँधीवादी विचार से
(घ) किसान संगठनों की मांगों से

(क) सिर्फ ख और घ
(ग) सिर्फ घ और ग
(ख) सिर्फ क और ख
(घ) उपर्युक्त सभी
उत्तर:
उपर्युक्त सभी।

प्रश्न 4.
निम्नलिखित का मेल करें:

(क) चरणसिंह (i) औद्योगीकरण
(ख) पी.सी. महालनोबिस (ii) जोनिंग
(ख) सिर्फ क और ख (iii) किसान
(घ) उपर्युक्त सभी (iv) सहकारी डेयरी

उत्तर:

(क) चरणसिंह (iii) किसान
(ख) पी.सी. महालनोबिस (i) औद्योगीकरण
(ग) बिहार का अकाल (ii) जोनिंग
(घ) वर्गीज कुरियन (iv) सहकारी डेयरी

JAC Class 12 Political Science Solutions Chapter 3 नियोजित विकास की राजनीति

प्रश्न 5.
आजादी के समय विकास के सवाल पर प्रमुख मतभेद क्या थे? क्या इन मतभेदों को सुलझा लिया गया?
उत्तर:
आजादी के समय विकास के सवाल पर मतभेद -आजादी के समय विकास के सवाल पर विभिन्न मतभेद थे। यथा

  1. आर्थिक संवृद्धि के साथ सामाजिक न्याय को सुनिश्चित करने के लिए सरकार कौन- सी भूमिका निभाये ? इस सवाल पर मतभेद था।
  2. विकास के दो मॉडलों – उदारवादी – पूँजीवादी मॉडल और समाजवादी मॉडल में से किस मॉडल को अपनाया जाये ? इस बात पर मतभेद था।
  3. कुछ लोग औद्योगीकरण को उचित रास्ता मानते थे तो कुछ की नजर में कृषि का विकास करना और ग्रामीण क्षेत्र की गरीबी को दूर करना सर्वाधिक जरूरी था।
  4. कुछ अर्थशास्त्री केन्द्रीय नियोजन के पक्ष में थे जबकि अन्य कुछ विकेन्द्रित नियोजन को विकास के लिए आवश्यक मानते थे।

मतभेदों को सुलझाना – उपर्युक्त में से कुछ मतभेदों को सुलझा लिया गया है परन्तु कुछ को सुलझाना अभी भी शेष है, जैसे

  1. सभी में इस बात पर सहमति बनी कि देश के व्यापार उद्योगों और कृषि को क्रमशः व्यापारियों, उद्योगपतियों और किसानों के भरोसे पूर्णत: नहीं छोड़ा जा सकता है।
  2. लगभग सभी इस बात पर सहमत थे कि विकास का अर्थ आर्थिक संवृद्धि और सामाजिक-आर्थिक न्याय दोनों ही है।
  3. इस बात पर भी सहमति हो गई कि गरीबी मिटाने और सामाजिक-आर्थिक पुनर्वितरण के काम की जिम्मेदारी सरकार की होगी।
  4. विकास के लिए मिश्रित अर्थव्यवस्था को अपनाने पर सहमति बनी।

प्रश्न 6.
पहली पंचवर्षीय योजना का किस चीज पर सबसे ज्यादा जोर था ? दूसरी पंचवर्षीय योजना पहली से किन अर्थों में अलग थी?
उत्तर:
पहली पंचवर्षीय योजना में देश में लोगों को गरीबी के जाल से निकालने का प्रयास किया गया और इस योजना में ज्यादा जोर कृषि क्षेत्र पर दिया गया। इसी योजना के अन्तर्गत बाँध और सिंचाई के क्षेत्र में निवेश किया गया। भाखड़ा नांगल जैसी विशाल परियोजनाओं के लिए बड़ी धनराशि आवंटित की गई। इस योजना में भूमि सुधार पर जोर दिया गया और इसे देश के विकास की बुनियादी चीज माना गया।

दोनों में अन्तर:

  1. प्रथम पंचवर्षीय एवं द्वितीय पंचवर्षीय योजना में प्रमुख अन्तर यह था कि जहाँ प्रथम पंचवर्षीय योजना में कृषि क्षेत्र पर अधिक बल दिया गया, वहीं दूसरी योजना में भारी उद्योगों के विकास पर अधिक जोर दिया गया।
  2. पहली पंचवर्षीय योजना का मूलमंत्र था – -धीरज, जबकि दूसरी पंचवर्षीय योजना तेज संरचनात्मक परिवर्तन पर बल देती थी।

प्रश्न 7.
हरित क्रान्ति क्या थी? हरित क्रान्ति के दो सकारात्मक और दो नकारात्मक परिणामों का उल्लेख करें।
उत्तर:

  • हरित क्रान्ति का अर्थ – “हरित क्रान्ति से अभिप्राय कृषिगत उत्पादन की तकनीक को सुधारने तथा कृषि उत्पादन में तीव्र वृद्धि करने से है।” इसके तीन तत्व थे
    1. कृषि का निरन्तर विस्तार
    2. दोहरी फसल लेना
    3. अच्छे बीजों का प्रयोग इस प्रकार हरित क्रांति का अर्थ है – सिंचित और असिंचित कृषि क्षेत्रों में अधिक उपज देने वाली किस्मों को आधुनिक कृषि पद्धति से उगाकर उत्पादन बढ़ाना।
  • हरित क्रान्ति के दो सकारात्मक परिणाम- हरित क्रांति के निम्न सकारात्मक परिणाम निकले-
    1. हरित क्रान्ति से खेतिहर पैदावार में सामान्य किस्म का इजाफा हुआ ( ज्यादातर गेहूँ की पैदावार बढ़ी) और देश में खाद्यान्न की उपलब्धता में बढ़ोत्तरी हुई।
    2. हरित क्रांति के कारण कृषि में मँझोले दर्जे के किसानों यानी मध्यम श्रेणी के भू-स्वामित्व वाले किसानों का उभार हुआ।
  • हरित क्रान्ति के नकारात्मक परिणाम – हरित क्रान्ति के दो नकारात्मक परिणाम निम्न हैं-
    1. इस क्रान्ति से गरीब किसानों और भू-स्वामियों के बीच का अंतर बढ़ गया।
    2. इससे पंजाब, हरियाणा और पश्चिमी उत्तरप्रदेश जैसे इलाके कृषि की दृष्टि से समृद्ध हो गए जबकि बाकी इलाके खेती के मामले में पिछड़े रहे।

प्रश्न 8.
“नियोजन के शुरुआती दौर में ‘कृषि बनाम उद्योग’ का विवाद रहा।” इस कथन को स्पष्ट कीजिए।
अथवा
दूसरी पंचवर्षीय योजना के दौरान औद्योगिक विकास बनाम कृषि विकास का विवाद चला था। इस विवाद में क्या-क्या तर्क दिए गए थे?
उत्तर:
दूसरी पंचवर्षीय योजना के दौरान औद्योगिक विकास और कृषि विकास में किस क्षेत्र के विकास पर जोर दिया जाय, का विवाद चला। इस विवाद के सम्बन्ध में विभिन्न तर्क दिये गये। यथा

  1. कृषि क्षेत्र का विकास करने वाले विद्वानों का यह तर्क था कि इससे देश आत्मनिर्भर बनेगा तथा किसानों की दशा में सुधार होगा, जबकि औद्योगिक विकास का समर्थन करने वालों का यह तर्क था कि औद्योगिक विकास से देश में रोजगार के अवसर बढ़ेंगे तथा देश में बुनियादी सुविधाएँ बढ़ेंगी।
  2. अनेक लोगों का मानना था कि दूसरी पंचवर्षीय योजना में कृषि के विकास की रणनीति का अभाव था और इस योजना के दौरान उद्योगों पर जोर देने के कारण खेती और ग्रामीण इलाकों को चोट पहुँचेगी।
  3. कई अन्य लोगों का सोचना था कि औद्योगिक उत्पादन की वृद्धि दर को तेज किए बगैर गरीबी के मकड़जाल से मुक्ति नहीं मिल सकती । कृषि विकास हेतु तो अनेक कानून बनाये जा चुके हैं, लेकिन औद्योगिक विकास की दिशा में कोई विशेष प्रयास नहीं हुए हैं।

JAC Class 12 Political Science Solutions Chapter 3 नियोजित विकास की राजनीति

प्रश्न 9.
” अर्थव्यवस्था में राज्य की भूमिका पर जोर देकर भारतीय नीति-निर्माताओं ने गलती की। अगर शुरूआत से ही निजी क्षेत्र को खुली छूट दी जाती तो भारत का विकास कहीं ज्यादा बेहतर तरीके से होता ।” इस विचार के पक्ष या विपक्ष में अपने तर्क दीजिए।
उत्तर:
उपर्युक्त विचार के पक्ष व विपक्ष में निम्नलिखित तर्क दिये जा सकते हैं- पक्ष में तर्क-

  1. भारत ने 1990 के दशक से ही नई आर्थिक नीति अपनायी है। तब से यह तेजी से उदारीकरण और वैश्वीकरण की ओर बढ़ रहा है। यदि यह नीति प्रारम्भ से ही अपना ली गई होती तो भारत का विकास अधिक बेहतर होता।
  2. अन्तर्राष्ट्रीय मुद्राकोष और विश्व बैंक जैसी बड़ी अन्तर्राष्ट्रीय संस्थाएँ भी उन्हीं देशों को अधिक ऋण व निवेश के संसाधन प्रदान करती हैं जहाँ निजी क्षेत्र को खुली छूट दी जाती है। जिन बड़े कार्यों के लिए सरकार पूँजी जुटाने में असमर्थ होती है उन कार्यों में अन्तर्राष्ट्रीय संस्थाएँ, बहुराष्ट्रीय कम्पनियाँ और देशी बड़े-बड़े पूँजीपति लोग पूँजी लगा सकते हैं।
  3. अन्तर्राष्ट्रीय स्तर पर प्रतियोगिता में भारत तभी ठहर सकता था जब निजी क्षेत्र को छूट दे दी गई होती।

विपक्ष में तर्क- निजी क्षेत्र के विपक्ष में निम्न तर्क दिये जा सकते हैं

  1. भारत में कृषिगत और औद्योगिक क्षेत्र का विस्तार सरकारी वर्ग की प्रभावशाली नीतियों व कार्यक्रमों से हुआ। यदि ऐसा नहीं होता तो भारत पिछड़ जाता।
  2. भारत में विकसित देशों की तुलना में जनसंख्या ज्यादा है। यहाँ बेरोजगारी है, गरीबी है। यदि पश्चिमी देशों की होड़ में भारत सरकारी हिस्से को अर्थव्यवस्था में कम कर दिया जाएगा तो बेरोजगारी बढ़ेगी, गरीबी फैलेगी, धन और पूँजी कुछ ही बहुराष्ट्रीय कम्पनियों के हाथों में केन्द्रित हो जाएगी जिससे आर्थिक विषमता और बढ़ जाएगी।
  3. भारत कृषि-प्रधान देश है। वह विकसित देशों के कृषि उत्पादन से मुकाबला नहीं कर सकता। कृषि क्षेत्र में सरकारी सहयोग राशि के बिना कृषि का विकास रुक जायेगा।

प्रश्न 10.
निम्नलिखित अवतरण को पढ़ें और इसके आधार पर पूछे गए प्रश्नों के उत्तर दें- आजादी के बाद के आरंभिक वर्षों में कांग्रेस पार्टी के भीतर दो परस्पर विरोधी प्रवृत्तियाँ पनपीं। एक तरफ राष्ट्रीय पार्टी कार्यकारिणी ने राज्य के स्वामित्व का समाजवादी सिद्धांत अपनाया, उत्पादकता को बढ़ाने के साथ- साथ आर्थिक संसाधनों के संकेंद्रण को रोकने के लिए अर्थव्यवस्था के महत्त्वपूर्ण क्षेत्रों का नियंत्रण और नियमन किया। दूसरी तरफ कांग्रेस की राष्ट्रीय सरकार ने निजी निवेश के लिए उदार आर्थिक नीतियाँ अपनाईं और उसके बढ़ावे के लिए विशेष कदम उठाए। इसे उत्पादन में अधिकतम वृद्धि की अकेली कसौटी पर जायज ठहराया गया। – फ्रैंकिन फ्रैंकल
(क) यहाँ लेखक किस अंतर्विरोध की चर्चा कर रहा है? ऐसे अंतर्विरोध के राजनीतिक परिणाम क्या होंगे?
(ख) अगर लेखक की बात सही है तो फिर बताएँ कि कांग्रेस इस नीति पर क्यों चल रही थी? क्या इसका संबंध विपक्षी दलों की प्रकृति से था?
(ग) क्या कांग्रेस पार्टी के केंद्रीय नेतृत्व और इसके प्रांतीय नेताओं के बीच भी कोई अंतर्विरोध था?
उत्तर:
(क) उपर्युक्त अवतरण में लेखक कांग्रेस पार्टी के अन्तर्विरोध की चर्चा कर रहा है जो क्रमशः वामपंथी विचारधारा से और दूसरा खेमा दक्षिणपंथी विचारधारा से प्रभावित था अर्थात् जहाँ कांग्रेस पार्टी की राष्ट्रीय कार्यकारिणी समाजवादी सिद्धान्तों में विश्वास रखती थी, वहीं राष्ट्रीय कांग्रेस की राष्ट्रीय सरकार निजी निवेश को बढ़ावा दे रही थी। इस प्रकार के अन्तर्विरोध से देश में राजनीतिक अस्थिरता फैलने की सम्भावना रहती है।

(ख) कांग्रेस इस नीति पर इसलिए चल रही थी, कि कांग्रेस में सभी विचारधाराओं के लोग शामिल थे तथा सभी लोगों के विचारों को ध्यान में रखकर ही कांग्रेस पार्टी इस प्रकार का कार्य कर रही थी। इसके साथ-साथ कांग्रेस पार्टी ने इस प्रकार की नीति इसलिए भी अपनाई ताकि विपक्षी दलों के पास आलोचना का कोई मुद्दा न रहे।

(ग) कांग्रेस के केन्द्रीय नेतृत्व एवं प्रान्तीय नेताओं में कुछ हद तक अन्तर्विरोध पाया जाता था। जहाँ केन्द्रीय नेतृत्व राष्ट्रीय एवं अन्तर्राष्ट्रीय मुद्दों को महत्त्व देता था, वहीं प्रान्तीय नेता प्रान्तीय एवं स्थानीय मुद्दों को महत्त्व देते थे। परिणामस्वरूप कांग्रेस के प्रभावशाली क्षेत्रीय नेताओं ने आगे चलकर अपने अलग-अलग राजनीतिक दल बनाए, जैसे- चौधरी चरण सिंह ने क्रांति दल या भारतीय लोकदल बनाया तो उड़ीसा में बीजू पटनायक ने उत्कल कांग्रेस का गठन किया।

नियोजित विकास की राजनीति JAC Class 12 Political Science Notes

→ नियोजित विकास का सम्बन्ध उचित रीति से सोच-विचार कर कदम उठाने से है। प्रत्येक क्रिया नियोजित विकास की क्रिया कहलाती है जो विभिन्न कार्यों को सम्पन्न करने हेतु दूरदर्शिता, विचार-विमर्श तथा उद्देश्यों एवं उनकी प्राप्ति हेतु प्रयुक्त होने वाले साधनों की स्पष्टता पर आधारित हो।

→ राजनीतिक फैसले और विकास-

  • भारत में नियोजित विकास की राजनीति आरम्भ करने से पहले विभिन्न तरह से विचार-विमर्श किये गये तथा तत्कालीन बाजार शक्तियों, लोगों की आकांक्षाओं एवं पर्यावरण पर विकास के पड़ने वाले तथ्यों को ध्यान में रखा गया।
  • अच्छे नियोजन के लक्ष्य को प्राप्त करने हेतु नियोजन के लक्ष्य एवं उद्देश्य स्पष्ट होने चाहिए इसके साथ ही साधनों की व्यवस्था, लचीलापन, समन्वय, व्यावहारिकता, पद- सोपान तथा निरन्तरता जैसे विषयों पर भी ध्यान दिया जाना आवश्यक है।
  • भारत में उड़ीसा राज्य की सरकार ने राज्य में इस्पात की बहुतायत को देखते हुए इन क्षेत्रों में उद्योग विकसित करने का मानस बनाया। लेकिन इस क्षेत्र में आदिवासियों के विस्थापन व पर्यावरण जैसी समस्या उभरकर सामने आई जिसको हल करना सरकार की पहली प्राथमिकता थी। इस प्रकार विकास योजनाओं को लागू करने से पहले पर्याप्त सोच-विचार की आवश्यकता है।

→ राजनीतिक टकराव:
विकास से जुड़े निर्णयों से प्रायः सामाजिक – समूह के हितों को दूसरे सामाजिक – समूह के हितों की तुलना में तौला जाता है। साथ ही मौजूदा पीढ़ी के हितों और आने वाली पीढ़ी के हितों को भी लाभ-हानि की तुला पर मापना पड़ता है। किसी भी लोकतन्त्र में ऐसे फैसले जनता द्वारा लिए जाने चाहिए या कम से कम इन फैसलों पर विशेषज्ञों की स्वीकृति की मुहर होनी चाहिए। आजादी के बाद आर्थिक संवृद्धि और सामाजिक न्याय की स्थापना हो इसके लिए सरकार कौन – सी भूमिका निभाए? इस सवाल को लेकर पर्याप्त मतभेद था।

क्या कोई ऐसा केन्द्रीय संगठन जरूरी है जो पूरे देश के लिए योजनाएँ बनाये ? क्या सरकार को कुछ महत्त्वपूर्ण उद्योग और व्यवसाय खुद चलाने चाहिए? अगर सामाजिक न्याय आर्थिक संवृद्धि की जरूरतों के आड़े आता हो तो ऐसी सूरत में सामाजिक न्याय पर कितना जोर देना उचित होगा ? इनमें से प्रत्येक सवाल पर टकराव हुए जो आज तक जारी हैं

JAC Class 12 Political Science Solutions Chapter 3 नियोजित विकास की राजनीति

→ विकास की धारणाएँ:
भारत ने तत्कालीन परिस्थितियों को ध्यान में रखते हुए मिश्रित मॉडल को अपनाया स्वतन्त्रता के समय भारत के सामने विकास के दो मॉडल थे। पहला उदारवादी – पूँजीवादी मॉडल था। यूरोप के अधिकतर हिस्सों और संयुक्त राज्य अमरीका में यही मॉडल अपनाया गया था। दूसरा समाजवादी मॉडल था, इसे सोवियत संघ ने अपनाया था। भारत ने इन दोनों मॉडलों के तत्त्वों को सम्मिलित करते हुए मिश्रित अर्थव्यवस्था के मॉडल को अपनाया। आजादी के आंदोलन के दौरान ही एक सहमति बन गई थी और नेताओं की इस पसंद में यही सहमति प्रतिबिंबित हो रही थी।

राष्ट्रवादी नेताओं के मन में यह बात बिल्कुल साफ थी कि आजाद भारत की सरकार के आर्थिक सरोकार अंग्रेजी हुकूमत के आर्थिक सरोकारों से एकदम अलग होंगे। आजाद भारत की सरकार अंग्रेजी हुकूमत की तरह संकुचित व्यापारिक हितों की पूर्ति के लिए काम नहीं करेगी। आजादी के आन्दोलन के दौरान ही यह बात भी साफ हो गई थी कि गरीबी मिटाने और सामाजिक-आर्थिक पुनर्वितरण के काम का मुख्य जिम्मा सरकार का होगा।

→ नियोजन:
नियोजन के विचार को 1940 और 1950 के दशक में पूरे विश्व में जनसमर्थन मिला था। यूरोप महामंदी का शिकार होकर सबक सीख चुका था; जापान और चीन ने युद्ध की विभीषिका झेलने के बाद अपनी अर्थव्यवस्था फिर खड़ी कर ली थी और सोवियत संघ ने 1930 तथा 1940 के दशक में भारी कठिनाइयों के बीच शानदार आर्थिक प्रगति की थी। इन सभी बातों के कारण नियोजन के पक्ष में दुनियाभर में हवा बह रही थी।

→ योजना आयोग:
योजना आयोग की स्थापना मार्च, 1950 में भारत – सरकार ने एक सीधे-सादे प्रस्ताव के जरिये की। यह आयोग एक सलाहकार की भूमिका निभाता है और इसकी सिफारिशें तभी प्रभावकारी हो पाती हैं जब मन्त्रिमण्डल उन्हें मंजूर करे।

→ शुरुआती कदम:
सोवियत संघ की तरह भारत ने भी पंचवर्षीय योजनाओं का विकल्प चुना। योजना के अनुसार केन्द्र सरकार और सभी राज्य सरकारों के बजट को दो हिस्सों में बाँटा गया। एक हिस्सा गैर योजना- व्यय का था। इसके अंतर्गत सालाना आधार पर दैनंदिन मदों पर खर्च करना था। दूसरा हिस्सा योजना-व्यय का था।

→ प्रथम पंचवर्षीय योजना:
1951 में प्रथम पंचवर्षीय योजना का प्रारूप जारी हुआ इस योजना की कोशिश देश को गरीबी के मकड़जाल से निकालने की थी। पहली पंचवर्षीय योजना में कृषि क्षेत्र पर जोर दिया गया था। इसी योजना के अंतर्गत बाँध और सिंचाई के क्षेत्र में निवेश किया गया । भाखड़ा नांगल जैसी विशाल परियोजनाओं के लिए बड़ी धनराशि आबंटित की गई।

→ औद्योगीकरण की तेज रफ्तार:
दूसरी पंचवर्षीय योजना में भारी उद्योगों के विकास पर जोर दिया गया। पी.सी. महालनोबिस के नेतृत्व में अर्थशास्त्रियों और योजनाकारों के एक समूह ने यह योजना तैयार की थी। हरा योजना के अंतर्गत सरकार ने देशी उद्योगों को संरक्षण देने के लिए आयात पर भारी शुल्क लगाया। संरक्षण की इस नीति से निजी और सार्वजनिक क्षेत्र के उद्योगों को आगे बढ़ने में मदद मिली। औद्योगीकरण पर दिए गए इस जोर ने भारतीय अर्थव्यवस्था के विकास को एक नया आयाम दिया। तीसरी पंचवर्षीय योजना दूसरी योजना से कुछ खास अलग नहीं थी। आलोचकों के अनुसार दूसरी पंचवर्षीय योजना से इनकी रणनीतियों में शहरों की तरफदारी नजर आने लगी थी।

→ विकेन्द्रित नियोजन:
जरूरी नहीं है कि नियोजन केन्द्रीकृत ही हो, केरल में विकेन्द्रित नियोजन अपनाया गया जिसे ‘केरल मॉडल’ कहा जाता है। इसमें इस बात के प्रयास किये गए कि लोग पंचायत, प्रखण्ड और जिला स्तर की योजनाओं को तैयार करने में शामिल हों। इसमें शिक्षा, स्वास्थ्य, भूमि सुधार, गरीबी उन्मूलन पर जोर दिया गया।

→ मुख्य विवाद: विकास से जुड़े दो मुख्य विवाद या प्रश्न निम्न उठे

  • कृषि बनाम उद्योग
  • निजी क्षेत्र बनाम सार्वजनिक क्षेत्र

भारत जैसी पिछड़ी अर्थव्यवस्था में सरकार देश के ज्यादा संसाधन किस क्षेत्र अर्थात् कृषि में लगाये या उद्योग में तथा विकास के जो दो नये मॉडल थे, भारत ने उनमें से किसी को नहीं अपनाया। पूँजीवादी मॉडल में विकास का काम पूर्णतया निजी क्षेत्र के भरोसे होता है। भारत ने यह रास्ता नहीं अपनाया। भारत ने विकास का समाजवादी मॉडल भी नहीं अपनाया जिसमें निजी सम्पत्ति को समाप्त कर दिया जाता है और हर तरह के उत्पादन पर राज्य का नियन्त्रण होता है। इन दोनों ही मॉडलों की कुछ बातों को ले लिया गया और अपने-अपने देश में इन्हें मिले-जुले रूप में लागू किया गया। इसी कारण भारतीय अर्थव्यवस्था को मिश्रित अर्थव्यवस्था कहा जाता है। इस मॉडल की आलोचना दक्षिणपंथी और वामपंथी दोनों खेमों से हुई।

→ मुख्य परिणाम: नियोजित विकास की शुरुआती कोशिशों को देश के आर्थिक विकास और सभी नागरिकों की भलाई के लक्ष्य में आंशिक सफलता मिली। नियोजित आर्थिक विकास के निम्न प्रमुख परिणाम निकले

  • बुनियाद: इस दौर में भारत के आगामी विकास की बुनियाद पड़ी। इस काल में भारत की बड़ी विकास- परियोजनाएँ, सार्वजनिक क्षेत्र के कुछ भारी उद्योग, परिवहन तथा संचार का आधारभूत ढाँचा आदि की स्थापना हुई।
  • भूमि सुधार: इस अवधि में भूमि सुधार के गम्भीर प्रयत्न हुए। जमींदारी प्रथा को समाप्त किया गया, . छोटी जोतों को एक-साथ किया गया तथा भूमि की अधिकतम सीमा तय की गई।
  • खाद्य संकट: इस काल में सूखे, युद्ध, विदेशी मुद्रा संकट तथा खाद्यान्न की भारी कमी की समस्या का सामना करना पड़ा। खाद्य संकट के कारण सरकार को गेहूँ का आयात करना पड़ा।

→ हरित क्रान्ति: भारत में हरित क्रान्ति की शुरुआत 1960 के दशक में हुई। हरित क्रान्ति के तीन तत्त्व थे-

  • कृषि का निरन्तर विस्तार
  • दोहरी फसल का उद्देश्य तथा
  • अच्छे बीजों का प्रयोग

हरित क्रान्ति से उत्पादन में रिकार्ड वृद्धि हुई, विशेषकर कृषिगत क्षेत्र में वृद्धि हुई तथा औद्योगिक विकास एवं बुनियादी ढाँचे में भी विकास हुआ। हरित क्रान्ति की कुछ कमियाँ भी रही हैं, जैसे- खाद्यान्न संकट बने रहना तथा केवल उत्तरी राज्यों को ही लाभ मिलना तथा धनी किसानों को ही लाभ मिलना इत्यादि।

→ बाद के बदलाव:
1960 के दशक में श्रीमती गाँधी के नेतृत्व में सरकार ने यह फैसला किया कि अर्थव्यवस्था के नियंत्रण और निर्देशन में राज्य बड़ी भूमिका निभायेगा। 14 निजी बैंकों का राष्ट्रीयकरण कर दिया गया लेकिन सरकारी नियंत्रण वाली अर्थव्यवस्था के पक्ष में बनी सहमति अधिक दिनों तक कायम नहीं रही क्योंकि

  • भारत की आर्थिक प्रगति दर 3-3.5 प्रतिशत ही रही।
  • सार्वजनिक क्षेत्र के उद्योगों में भ्रष्टाचार, अकुशलता का जोर बढ़ा।
  • नौकरशाही के प्रति लोगों का विश्वास टूट गया।

फलतः 1980 के दशक के बाद से अर्थव्यवस्था में राज्य की भूमिका को कम कर दिया गया।

JAC Class 12 History Important Questions Chapter 2 राजा, किसान और नगर : आरंभिक राज्य और अर्थव्यवस्थाएँ

Jharkhand Board JAC Class 12 History Important Questions Chapter 2 राजा, किसान और नगर : आरंभिक राज्य और अर्थव्यवस्थाएँ Important Questions and Answers.

JAC Board Class 12 History Important Questions Chapter 2 राजा, किसान और नगर : आरंभिक राज्य और अर्थव्यवस्थाएँ

बहुविकल्पीय प्रश्न (Multiple Choice Questions)

1. ब्राह्मी और खरोष्ठी लिपियों को पढ़ने वाला था –
(अ) जॉन मार्शल
(स) व्हीलर
(ब) कनिंघम
(द) प्रिंसेप
उत्तर:
(द) प्रिंसेप

2. बौद्ध और जैन धर्म के आरम्भिक ग्रन्थों में कितने महाजनपदों का उल्लेख मिलता है?
(अ) 8
(स) 16
(ब) 18
(द) 14
उत्तर:
(स) 16

3. अशोक के अधिकांश अभिलेख किस भाषा में हैं?
(अ) संस्कृत
(ब) पालि
(स) हिन्दी
(द) प्राकृत
उत्तर:
(द) प्राकृत

4. ‘अर्थशास्त्र’ का रचयिता कौन था?
(अ) बाणभट्ट
(ब) चाणक्य
(स) कल्हण
(द) मेगस्थनीज
उत्तर:
(ब) चाणक्य

JAC Class 12 History Important Questions Chapter 2 राजा, किसान और नगर : आरंभिक राज्य और अर्थव्यवस्थाएँ

5. ‘इण्डिका’ की रचना किसने की थी ?
(अ) चाणक्य
(ब) कर्टियस
(स) मेगस्थनीज
(द) प्लूटार्क
उत्तर:
(स) मेगस्थनीज

6. अशोक ने धम्म के प्रचार के लिए किस अधिकारी की नियुक्ति की ?
(अ) धर्माध्यक्ष
(ब) दानाध्यक्ष
(स) सैन्य महामात
(द) धर्म महामात
उत्तर:
(द) धर्म महामात

7. कौनसे शासक अपने नाम के आगे ‘देवपुत्र’ की उपाधि लगाते थे?
(अ) मौर्य शासक
(ब) शुंग शासक
(स) सातवाहन शासक
(द) कुषाण शासक
उत्तर:
(द) कुषाण शासक

8. ‘प्रयाग प्रशस्ति’ की रचना किसने की थी?
(अ) हरिषेण
(स) कालिदास
(ब) बाणभट्ट
(द) कल्हण
उत्तर:
(अ) हरिषेण

9. अभिलेखों के अध्ययन को कहते हैं –
(अ) मुखाकृति
(ब) पुरालिपि
(स) अभिलेखशास्त्र
(द) शिल्पशास्त्र
उत्तर:
(स) अभिलेखशास्त्र

10. निम्न में से सबसे महत्त्वपूर्ण जनपद था –
(अ) कुरु
(ब) मत्स्य
(स) गान्धार
(द) मगध
उत्तर:
(द) मगध

JAC Class 12 History Important Questions Chapter 2 राजा, किसान और नगर : आरंभिक राज्य और अर्थव्यवस्थाएँ

11. मगध महाजनपद की प्रारम्भिक राजधानी थी –
(अ) पटना
(ब) राजगाह
(स) संस्कृत
(द) हिन्दी
उत्तर:
(ब) राजगाह

12. मौर्य साम्राज्य का सबसे महत्त्वपूर्ण राजनीतिक केन्द्र
(अ) तक्षशिला
(ब) उज्जयिनी
(स) पाटलिपुत्र
(द) सुवर्णगिरि
उत्तर:
(स) पाटलिपुत्र

13. किस शासक को बीसवीं शताब्दी के राष्ट्रवादी नेताओं ने प्रेरणा का स्रोत माना है?
(अ) अशोक
(ब) कनिष्क
(स) समुद्रगुप्त
(द) चन्द्रगुप्त मौर्य
उत्तर:
(अ) अशोक

14. प्रयाग प्रशस्ति की रचना की –
(अ) कल्हण
(ब) हरिषेण
(स) अशोक
(द) विल्हण
उत्तर:
(ब) हरिषेण

JAC Class 12 History Important Questions Chapter 2 राजा, किसान और नगर : आरंभिक राज्य और अर्थव्यवस्थाएँ

15. आरम्भिक भारत का सबसे प्रसिद्ध विधिग्रन्थ है –
(अ) ऋग्वेद
(स) रामायण
(ब) मनुस्मृति
(द) महाभारत
उत्तर:
(ब) मनुस्मृति

16. छठी शताब्दी में सबसे पहले ढाले एवं प्रयोग किए गए चाँदी और ताँबे के सिक्के कहलाते हैं –
(अ) आहत
(स) सिक्का
(अ) मुहर
(द) रुपया
उत्तर:
(अ) आहत

17. सर्वप्रथम सोने के सिक्के किन शासकों ने जारी किए?
(अ) शक
(स) मौर्य
(ब) कुषाण
(द) गुप्त
उत्तर:
(ब) कुषाण

18. किस शासक के लिए देवानांपिय उपाधि का प्रयोग हुआ है?
(अ) समुद्रगुप्त
(स) अकबर
(ब) चन्द्रगुप्त
(द) अशोक
उत्तर:
(द) अशोक

19. जनपद शब्द का प्रयोग हमें किन दो भाषाओं में मिल जाता है?
(अ) पालि और प्राकृत
(ब) प्राकृत और पालि
(स) प्राकृत और संस्कृत
(द) पालि और संस्कृत
उत्तर:
(स) प्राकृत और संस्कृत

JAC Class 12 History Important Questions Chapter 2 राजा, किसान और नगर : आरंभिक राज्य और अर्थव्यवस्थाएँ

20. अजातशत्रु तथा अशोक जैसे शासकों के नाम किन अभिलेखों से प्राप्त हुए हैं?
(अ) प्राकृत अभिलेख
(ब) पालि अभिलेख
(स) संस्कृत अभिलेख
(द) तमिल अभिलेख
उत्तर:
(अ) प्राकृत अभिलेख

रिक्त स्थानों की पूर्ति कीजिए।

1. …………. का अर्थ एक ऐसा भूखण्ड है जहाँ कोई जन अपना पाँव रखता है अथवा बस जाता है।
2. प्राचीनतम अभिलेख ………….. भाषा में लिखे जाते थे।
3. जहाँ सत्ता पुरुषों के एक समूह के हाथ में होती है …………… कहते हैं।
4. …………. आधुनिक बिहार के राजगीर का प्राकृत नाम है।
5. धम्म के प्रचार के लिए ……………. नामक विशेष अधिकारियों की नियुक्ति की गई।
6. सिलप्यादिकारम् ………….. भाषा का महाकाव्य है।
7. प्रयाग प्रशस्ति की रचना …………… राजकवि …………. ने की थी।
उत्तर:
1. जनपद
2. प्राकृत
3. ओलीगार्की, समूह शासन
4. राजगाह
5. धम्म महामात
6. तमिल
7. समुद्रगुप्त, हरिषेण।

अतिलघूत्तरात्मक प्रश्न

प्रश्न 1.
साँची में कौनसी प्रसिद्ध पुरातात्विक इमारत स्थित है?
उत्तर:
साँची में अशोक द्वारा बनवाया गया बृहद् तथा प्रसिद्ध स्तूप स्थित है।

प्रश्न 2.
शासकों की प्रतिमा और नाम के सिक्के सर्वप्रथम किस राजवंश के राजाओं ने जारी किये थे?
उत्तर:
कुषाण राजवंश के राजाओं ने।

JAC Class 12 History Important Questions Chapter 2 राजा, किसान और नगर : आरंभिक राज्य और अर्थव्यवस्थाएँ

प्रश्न 3.
मौर्य शासकों की राजधानी पाटलिपुत्र का वर्तमान नाम क्या है?
उत्तर:
मौर्य शासकों की राजधानी पाटलिपुत्र का वर्तमान नाम पटना है।

प्रश्न 4.
भारत में सोने के सिक्के सबसे पहले कब और किस वंश के राजाओं ने जारी किए थे?
उत्तर:
प्रथम शताब्दी ईसवी में कुषाण राजाओं ने जारी किए थे।

प्रश्न 5.
लगभग दूसरी शताब्दी ई. में सुदर्शन सरोवर का जीर्णोद्धार किस शक राजा ने करवाया था?
उत्तर:
रुद्रदामन ने।

प्रश्न 6.
अभिलेख किसे कहते हैं?
उत्तर:
अभिलेख पत्थर, धातु या मिट्टी के बर्तन पर खुदे होते हैं।

प्रश्न 7.
अशोक के अधिकांश अभिलेखों और सिक्कों पर उसका क्या नाम लिखा है?
उत्तर:
अशोक के अधिकांश अभिलेखों और सिक्कों पर ‘पियदस्सी’ नाम लिखा है।

प्रश्न 8.
कलिंग पर विजय प्राप्त करने वाला मौर्य सम्राट् कौन था?
उत्तर:
सम्राट् अशोक।

प्रश्न 9.
छठी शताब्दी ई. पूर्व के चार महाजनपदों के नाम उनकी राजधानी सहित लिखिए।
उत्तर:
(1) अंग (चम्पा)
(2) मगध (राजगीर)
(3) काशी (वाराणसी)
(4) कोशल (श्रावस्ती)।

प्रश्न 10.
चौथी शताब्दी ई. पूर्व किसे मगध की राजधानी बनाया गया?
उत्तर:
चौथी शताब्दी ई. पूर्व में पाटलिपुत्र को मगध की राजधानी बनाई गयी।

प्रश्न 11.
मगध महाजनपद के सबसे शक्तिशाली होने के दो कारण बताइये।
उत्तर:
(1) खेती की अच्छी उपज होना
(2) लोहे की खदानों का उपलब्ध होना।

JAC Class 12 History Important Questions Chapter 2 राजा, किसान और नगर : आरंभिक राज्य और अर्थव्यवस्थाएँ

प्रश्न 12.
मौर्य साम्राज्य के इतिहास की जानकारी के लिए दो प्रमुख स्रोतों का उल्लेख कीजिए।
उत्तर:
(1) कौटिल्य कृत ‘अर्थशास्त्र’
(2) मेगस्थनीज द्वारा लिखित ‘इण्डिका।

प्रश्न 13.
मौर्य साम्राज्य के पाँच प्रमुख राजनीतिक केन्द्रों का उल्लेख कीजिए।
उत्तर:
राजधानी पाटलिपुत्र तथा चार प्रान्तीय केन्द्र- तक्षशिला, उज्जयिनी, तोसलि, सुवर्णगिरि

प्रश्न 14.
सिल्लपादिकारम् क्या है?
उत्तर:
सिल्लपादिकारम् तमिल महाकाव्य है। प्रश्न 15. प्रयाग प्रशस्ति का ऐतिहासिक महत्त्व बताइये। उत्तर- प्रयाग प्रशस्ति से समुद्रगुप्त की विजयों, चारित्रिक विशेषताओं की जानकारी मिलती है।

प्रश्न 16.
मौर्य स्वम्राज्य का संस्थापक कौन था? उसने मौर्य साम्राज्य की स्थापना कब की ?
उत्तर:
(1) चन्द्रगुप्त मौर्य
(2) 321 ई. पूर्व

प्रश्न 17.
गुप्त साम्राज्य के इतिहास की जानकारी के प्रमुख स्रोतों का उल्लेख कीजिये।
उत्तर:

  • साहित्य
  • सिक्के
  • अभिलेख
  • हरिषेण द्वारा लिखित ‘प्रयाग प्रशस्ति’।

प्रश्न 18.
जातक ग्रन्थों का क्या महत्त्व है?
उत्तर:
इनसे राजाओं और प्रजा के सम्बन्धों की जानकारी मिलती है।

प्रश्न 19.
‘मनुस्मृति’ के बारे में आप क्या जानते हैं ?
उत्तर:
‘मनुस्मृति’ आरम्भिक भारत का सबसे प्रसिद्ध विधिग्रन्थ है।

JAC Class 12 History Important Questions Chapter 2 राजा, किसान और नगर : आरंभिक राज्य और अर्थव्यवस्थाएँ

प्रश्न 20.
प्रभावती गुप्त कौन थी?
उत्तर:
प्रभावती गुप्त चन्द्रगुप्त द्वितीय (375-415 ई.) की पुत्री थी।

प्रश्न 21.
जातक कथाएँ अथवा जातक कहानी क्या
उत्तर:
महात्मा बुद्ध के पूर्व जन्मों की कहानियों के संकलन को जातक कथाओं के नाम से जाना जाता है।

प्रश्न 22.
गुप्त वंश की किस शासिका ने भूमिदान किया था और क्यों?
उत्तर:
प्रभावती गुप्त ने भूमिदान किया था क्योंकि वह रानी थी।

प्रश्न 23.
‘हर्षचरित’ की रचना किसने की थी?
उत्तर:
बाणभट्ट ने।

प्रश्न 24.
‘ श्रेणी’ से क्या अभिप्राय है?
उत्तर:
उत्पादकों और व्यापारियों के संघ’ श्रेणी’ कहलाते

प्रश्न 25.
सोने के सबसे भव्य सिक्के प्रचुर मात्रा में किन सम्राटों ने जारी किये?
उत्तर:
गुप्त सम्राटों ने।

प्रश्न 26.
‘अभिलेख शास्त्र’ किसे कहते हैं?
उत्तर:
अभिलेखों के अध्ययन को ‘अभिलेख शास्त्र’ कहते हैं।

प्रश्न 27.
गण और संघ नामक राज्यों में कौन शासन करता था?
उत्तर:
लोगों का समूह।

JAC Class 12 History Important Questions Chapter 2 राजा, किसान और नगर : आरंभिक राज्य और अर्थव्यवस्थाएँ

प्रश्न 28.
महाजनपद के राजा का प्रमुख कार्य क्या
उत्तर:
किसानों, व्यापारियों तथा शिल्पकारों से कर तथा भेंट वसूल करना।

प्रश्न 29.
मगध राज्य के तीन शक्तिशाली राजाओं के नाम लिखिए।
उत्तर:

  • बिम्बिसार
  • अजातशत्रु
  • महापद्मनन्द।

प्रश्न 30.
अशोक के प्राकृत के अधिकांश अभिलेख किस लिपि में लिखे गए थे?
उत्तर:
ब्राह्मी लिपि में।

प्रश्न 31.
‘तमिलकम’ में किन सरदारियों (राज्यों) का उदय हुआ?
उत्तर:
‘तमिलकम’ में चोल, चेर तथा पाण्ड्य सरदारियों का उदय हुआ।

प्रश्न 32.
दक्षिणी भारत के सरदारों तथा सरदारियों की जानकारी किससे मिलती है?
उत्तर:
प्राचीन तमिल संगम ग्रन्थों में संग्रहित कविताओं

प्रश्न 33.
‘देवपुत्र’ की उपाधि किन शासकों ने धारण की थी?
उत्तर:
कुषाण शासकों ने।

प्रश्न 34.
प्रयाग प्रशस्ति अभिलेख किस नाम से प्रसिद्ध
उत्तर:
‘इलाहाबाद स्तम्भ अभिलेख’ के नाम से।

प्रश्न 35.
प्राचीन भारत में राजाओं और प्रजा के बीच रहने वाले तनावपूर्ण सम्बन्धों की जानकारी किससे मिली है?
उत्तर:
जातक कथाओं से।

प्रश्न 36.
अग्रहार’ से आप क्या समझते हैं ?
उत्तर:
अग्रहार वह भू-भाग था, जो ब्राह्मणों को दान किया जाता था।

JAC Class 12 History Important Questions Chapter 2 राजा, किसान और नगर : आरंभिक राज्य और अर्थव्यवस्थाएँ

प्रश्न 37.
श्रेणी से क्या आशय है?
उत्तर:
उत्पादक एवं व्यापारियों के संघ को श्रेणी कहा जाता है।

प्रश्न 38.
दानात्मक अभिलेखों से क्या अभिप्राय है?
उत्तर:
इनमें धार्मिक संस्थाओं को दिए दान का विवरण होता था।

प्रश्न 39.
इस युग में भारत से रोमन साम्राज्य को किन वस्तुओं का निर्यात किया जाता था ?
उत्तर:
कालीमिर्च जैसे मसालों, कपड़ों, जड़ी-बूटियों

प्रश्न 40.
‘पेरिप्लस ऑफ एभ्रियन सी’ नामक ग्रन्थ का रचयिता कौन था?
उत्तर:
एक यूनानी समुद्री यात्री

प्रश्न 41.
छठी शताब्दी ई. पूर्व में सबसे पहले किन सिक्कों का प्रयोग हुआ?
उत्तर:
चांदी और ताँबे के आहत सिक्कों का।

प्रश्न 42.
मुद्राशास्व’ किसे कहते हैं?
उत्तर:
‘मुद्राशास्त्र’ सिक्कों का अध्ययन है।

प्रश्न 43.
पंजाब और हरियाणा में प्रथम शताब्दी ई. में किन कबाइली गणराज्यों ने सिक्के जारी किये?
उत्तर:
यौधेय गणराज्यों ने।

प्रश्न 44.
महाजनपद की दो प्रमुख विशेषताएँ क्या था?
उत्तर:
(1) अधिकांश महाजनपदों पर राजा का शासन होता था।
(2) प्रत्येक महाजनपद की एक राजधानी होती थी जो

प्रश्न 45.
अशोक के धम्म के कोई दो सिद्धान्त लिखिए।
उत्तर:
(1) बड़ों के प्रति आदर।
(2) संन्यासियों तथा ब्राह्मणों के प्रति उदारता।

JAC Class 12 History Important Questions Chapter 2 राजा, किसान और नगर : आरंभिक राज्य और अर्थव्यवस्थाएँ

प्रश्न 46.
अशोक के अभिलेखों की लिपियों को किस विद्वान ने पढ़ने में सफलता प्राप्त की और कब ?
उत्तर:
1838 ई. में जेम्स प्रिंसेप को अशोक के अभिलेखों की लिपियों को पढ़ने में सफलता मिली।

प्रश्न 47.
अशोक के अभिलेख किन भाषाओं में लिखे गए?
उत्तर:
अशोक के अधिकांश अभिलेख प्राकृत में हैं तथा पश्चिमोत्तर में मिले अभिलेख अरामेइक तथा यूनानी भाषा में हैं।

प्रश्न 48.
मौर्यवंश के इतिहास की जानकारी के प्रमुख स्रोतों का उल्लेख कीजिए।
उत्तर:

  • कौटिल्यकृत ‘अर्थशास्त्र’
  • मेगस्थनीज द्वारा रचित ‘इण्डिका’
  • अशोककालीन अभिलेख
  • जैन, बौद्ध और पौराणिक ग्रन्थ।

प्रश्न 49.
मेगस्थनीज ने सैनिक गतिविधियों के संचालन के लिए कितनी समितियों का उल्लेख किया है?
उत्तर:
मेगस्थनीज ने सैनिक गतिविधियों के संचालन के लिए एक समिति तथा 6 उपसमितियों का उल्लेख किया है।

प्रश्न 50.
बीसवीं सदी के राष्ट्रवादी नेताओं ने अशोक को प्रेरणा का स्रोत क्यों माना है?
उत्तर:
अन्य राजाओं की अपेक्षा अशोक एक बहुत शक्तिशाली, परिश्रमी, विनम्र, उदार एवं दानशील था।

प्रश्न 51.
सुदर्शन झील का निर्माण किसने करवाया था तथा किस शक राजा ने इसकी मरम्मत करवाई थी ?
उत्तर:
सुदर्शन झील का निर्माण मौर्यकाल में एक स्थानीय राज्यपाल ने करवाया था तथा शक राजा रुद्रदामन ने इसकी मरम्मत करवाई थी।

प्रश्न 52.
किस महिला शासिका द्वारा किया गया भूमिदान का उदाहरण विरला उदाहरण माना जाता है ?
उत्तर:
वाकाटक वंश की शासिका प्रभावती गुप्त द्वारा किया गया भूमिदान विरला उदाहरण माना जाता है।

प्रश्न 53.
शासकों द्वारा किए जाने वाले भूमिदान के दो कारण लिखिए।
उत्तर:
(1) भूमिदान कृषि को नए क्षेत्रों में प्रोत्साहित करने की एक रणनीति थी।
(2) भूमिदान से दुर्बल होते राजनीतिक प्रभुत्व का संकेत मिलता है।

JAC Class 12 History Important Questions Chapter 2 राजा, किसान और नगर : आरंभिक राज्य और अर्थव्यवस्थाएँ

प्रश्न 54.
नगरों की दो प्रमुख विशेषताओं का उल्लेख कीजिये।
उत्तर:
(1) अधिकांश नगर महाजनपदों की राजधानियाँ
(2) प्राय: सभी नगर संचार मार्गों के किनारे बसे थे।

प्रश्न 55.
‘हर्षचरित’ के बारे में आप क्या जानते हैं?
उत्तर:
हर्षचरित कनौज के शासक हर्षवर्धन की जीवनी है। इसके लेखक बाणभट्ट थे, जो हर्षवर्धन के राजकवि थे।

प्रश्न 56.
आहत या पंचमार्क क्या है?
उत्तर:
6 वीं शताब्दी ईसा पूर्व में चाँदी के बने सिक्कों को आहत अथवा पंचमार्क कहा जाता है।

प्रश्न 57.
अशोक के ‘धम्म’ से आप क्या समझते हैं ?
उत्तर:
अशोक ने अपनी प्रजा की उन्नति के लिए कुछ नैतिक नियमों के पालन पर बल दिया। इन्हें ‘धम्म’ कहते हैं।

प्रश्न 58.
जातक कथाओं के अनुसार राजा और प्रजा के बीच तनावपूर्ण सम्बन्धों के दो कारण लिखिए।
उत्तर:
(1) राजा अपनी प्रजा पर भारी कर लगाते थे।
(2) लुटेरों की लूटमार से लोगों में असन्तोष व्याप्त था।

प्रश्न 59.
गहपति कौन था?
उत्तर:
गहपति घर का मुखिया होता था और घर में रहने वाली स्वियों, बच्चों, नौकरों और दासों पर नियन्त्रण करता था।

प्रश्न 60.
आरम्भिक जैन और बौद्ध विद्वानों के अनुसार मगध के सबसे शक्तिशाली महाजनपद बनने के क्या कारण थे?
उत्तर:
मगध के शासक बिम्बिसार, अजातशत्रु, महापद्मनन्द अत्यन्त महत्त्वाकांक्षी थे तथा उनके मन्त्री उनकी नीतियाँ लागू करते थे।

प्रश्न 61.
ऐसे पाँच महाजनपदों के नाम लिखिए जिनके नाम जैन और बौद्ध ग्रन्थों में मिलते हैं।
उत्तर:

  • वज्जि
  • मगध
  • कोशल
  • कुरु
  • पांचाल

प्रश्न 62.
गणराज्यों की दो विशेषताएँ बताइये।
उत्तर:
(1) गणराज्य में कई लोगों का समूह शासन करता था।
(2) इस समूह का प्रत्येक व्यक्ति राजा कहलाता था।

JAC Class 12 History Important Questions Chapter 2 राजा, किसान और नगर : आरंभिक राज्य और अर्थव्यवस्थाएँ

प्रश्न 63.
अशोक के धम्म के चार सिद्धान्तों का उल्लेख कीजिये।
उत्तर:

  • बड़ों के प्रति आदर
  • संन्यासियों और ब्राह्मणों के प्रति उदारता
  • सेवकों तथा दासों के साथ उदार व्यवहार
  • दूसरों के धर्मों का आदर करना।

प्रश्न 64.
अभिलेखों में किनके क्रियाकलापों एवं उपलब्धियों का उल्लेख होता है?
उत्तर:
अभिलेखों में उन लोगों की उपलब्धियों, क्रियाकलापों एवं विचारों का उल्लेख होता है, जो उन्हें बनवाते हैं।

प्रश्न 65.
प्राचीन भारतीय इतिहास में छठी शताब्दी ई. पूर्व को एक महत्त्वपूर्ण परिवर्तनकारी काल क्यों माना जाता है? दो विशेषताएँ लिखिए।
उत्तर:
(1) इस काल में जैन धर्म एवं बौद्ध धर्म का उदय हुआ।
(2) इस काल में 16 महाजनपदों का भी उदय हुआ।

प्रश्न 66.
दो लिपियों का जिक्र करें जिन्हें सम्राट अशोक के अभिलेखों में उपयोग किया गया है?
अथवा
अशोक के अभिलेख किन लिपियों में लिखे गए थे?
उत्तर:
अशोक के प्राकृत के अधिकांश अभिलेख ब्राह्मी लिपि में तथा पश्चिमोत्तर के कुछ अभिलेख खरोष्ठी लिपि में लिखे गए थे।

प्रश्न 67.
मौर्य साम्राज्य के अधिकारियों द्वारा किये जाने वाले दो कार्यों का उल्लेख कीजिये।
उत्तर:
(1) नदियों की देख-रेख करना
(2) भूमि मापन का काम करना।

प्रश्न 68.
मौर्य साम्राज्य क्यों महत्वपूर्ण था? दो बातों का उल्लेख कीजिये।
उत्तर:
(1) मौर्य साम्राज्य एक विशाल साम्राज्य था, जिसकी सम्भावना बड़ी चुनौतीपूर्ण थी।
(2) मौर्यकाल में वास्तुकला एवं मूर्तिकला का अत्यधिक विकास हुआ।

प्रश्न 69.
मौर्य साम्राज्य की दो त्रुटियों का उल्लेख कीजिए।
उत्तर:
(1) मौर्य साम्राज्य केवल 150 वर्षों तक ही अस्तित्व में रहा।
(2) यह साम्राज्य उपमहाद्वीप के सभी क्षेत्रों में नहीं फैल पाया।

प्रश्न 70.
अर्थशास्त्र का रचयिता कौन था? इसका क्या ऐतिहासिक महत्त्व है?
उत्तर:
(1) कौटिल्य
(2) इसमें मौर्यकालीन सैनिक और प्रशासनिक संगठन तथा सामाजिक अवस्था के बारे में विस्तृत विवरण मिलते हैं।

प्रश्न 71.
यूनानी इतिहासकारों के अनुसार मौर्य सम्राट चन्द्रगुप्त के पास कितनी सेना थी?
उत्तर:
चन्द्रगुप्त मौर्य के पास 6 लाख पैदल सैनिक, 30 हजार घुड़सवार तथा 9 हजार हाथी थे।

प्रश्न 72.
दक्षिणी भारत में ‘सरदार’ कौन होते थे?
उत्तर:
सरदार एक शक्तिशाली व्यक्ति होता था, जिसका पद वंशानुगत भी हो सकता था और नहीं भी।

प्रश्न 73.
सरदार के दो कार्यों का उल्लेख कीजिये।
उत्तर:
(1) विशेष अनुष्ठान का संचालन करना
(2) युद्ध के समय सेना का नेतृत्व करना।

प्रश्न 74.
प्राचीन भारत में उपज बढ़ाने के तरीकों का उल्लेख कीजिये।
उत्तर:
(1) हल का प्रचलन
(2) कुओं, तालाबों, नहरों के माध्यम से सिंचाई करना।

JAC Class 12 History Important Questions Chapter 2 राजा, किसान और नगर : आरंभिक राज्य और अर्थव्यवस्थाएँ

प्रश्न 75.
600 ई.पूर्व से 600 ई. के काल में ग्रामीण समाज में पाई जाने वाली विभिन्नताओं का उल्लेख कीजिये।
उत्तर:
इस युग में भूमिहीन खेतिहर मजदूर, किसान और बड़े-बड़े जमींदार अस्तित्व में थे।

प्रश्न 76.
इस युग में भूमिकर के सम्बन्ध में ब्राह्मणों को छोटे कौनसी सुविधाएँ प्राप्त थीं?
उत्तर:
(1) ब्राह्मणों से भूमिकर या अन्य कर नहीं वसूले जाते थे।
(2) ब्राह्मणों को स्थानीय लोगों से कर वसूलने का अधिकार था।

प्रश्न 77.
अशोक के अभिलेखों की क्या महत्ता है ?
उत्तर:
अशोक के अभिलेखों से अशोक की शासन- व्यवस्था उसके साम्राज्य विस्तार, धम्म प्रचार आदि कार्यों के बारे में जानकारी मिलती है।

प्रश्न 78.
‘राजगाह’ का शाब्दिक अर्थ बताइये।
उत्तर:
राजगाह’ का शाब्दिक अर्थ है ‘राजाओं का

प्रश्न 79.
जैन और बौद्ध ग्रन्थों में पाए जाने वाले महाजनपदों के नाम लिखिए।
उत्तर:

  1. वज्जि
  2. मगध
  3. कोशल
  4. कुरु
  5. पांचाल
  6. गान्धार
  7. अवन्ति।

प्रश्न 80.
अभिलेखों से क्या तात्पर्य है?
उत्तर:
अभिलेखों में उन लोगों की उपलब्धियाँ, क्रियाकलाप या विचार लिखे जाते हैं, जो उन्हें बनवाते हैं।

प्रश्न 81.
जैन एवं बौद्ध लेखकों के अनुसार मगध महाजनपद के प्रसिद्ध सम्राट कौन थे?
उत्तर:
जैन एवं बौद्ध लेखकों के अनुसार बिम्बिसार, अजातशत्रु तथा महापद्मनन्द मगध महाजनपद के प्रसिद्ध सम्राट थे।

प्रश्न 82.
मगध महाजनपद के सबसे शक्तिशाली होने के क्या कारण थे?
उत्तर:

  1. मगध क्षेत्र में खेती की उपज अच्छी थी
  2. यहाँ लोहे की अनेक खदानें थीं
  3. जंगली क्षेत्र में हाथी उपलब्ध थे।

प्रश्न 83.
सामान्यतः अशोक को अभिलेखों में किस नाम से सम्बोधित किया गया है?
उत्तर:
अधिकांशतः अशोक को अभिलेखों में देवनामप्रिय प्रियदर्शी अथवा पियदस्सी नाम से सम्बोधित किया गया है।

प्रश्न 84.
प्राकृत भाषा से आप क्या समझते हैं?
उत्तर:
प्राकृत उन भाषाओं को कहा जाता था, जो जनसामान्य की भाषाएँ होती थीं।

प्रश्न 85.
अभिलेखों में किनका ब्यौरा होता है?
उत्तर:
अभिलेखों में राजाओं के क्रियाकलापों तथा महिलाओं और पुरुषों द्वारा धार्मिक संस्थाओं को दिए गए दान का ब्यौरा होता है।

प्रश्न 86.
अधिकांश महाजनपदों तथा ‘गण’ और ‘संघ’ नामक राज्यों में कौन शासन करता था?
उत्तर:
अधिकांश महाजनपदों में राजा तथा ‘गण’ और ‘संघ’ नामक राज्यों में लोगों का समूह शासन करता था।

JAC Class 12 History Important Questions Chapter 2 राजा, किसान और नगर : आरंभिक राज्य और अर्थव्यवस्थाएँ

प्रश्न 87.
मनुस्मृति क्या है? इसकी रचना कब की
उत्तर:
(1) मनुस्मृति प्रारम्भिक भारत का सबसे प्रसिद्ध विधि ग्रन्थ है
(2) इसकी रचना दूसरी शताब्दी ई. पूर्व और दूसरी शताब्दी ई. के बीच की गई।

प्रश्न 88.
छठी शताब्दी ई. पूर्व में पाटलिपुत्र और उज्जयिनी किन संचार मार्गों के किनारे बसे थे?
उत्तर:
पाटलिपुत्र नदी मार्ग के किनारे तथा उम्जयिनी भूतल मार्गों के किनारे बसे थे।

प्रश्न 89.
ओलीगार्की या समूह शासन से आपका क्या तात्पर्य है?
उत्तर:
ओलीगार्की या समूह शासन उसे कहते हैं जहाँ सत्ता पुरुषों के एक समूह के हाथ में होती है।

प्रश्न 90.
कलिंग का आधुनिक नाम क्या है?
उत्तर:
उड़ीसा

प्रश्न 91.
अपने अधिकारियों और प्रजा के लिए प्राकृतिक पत्थरों और पॉलिश किए हुए स्तम्भों पर सन्देश लिखवाने वाले पहले सम्राट् कौन थे?
उत्तर:
अशोक वह पहले सम्राट् थे जिन्होंने अपने अधिकारियों और प्रजा के लिए प्राकृतिक पत्थरों और पॉलिश किए हुए स्तम्भों पर सन्देश लिखवाए।

प्रश्न 92.
चीनी शासक स्वयं को किस नाम से सम्बोधित करते थे?
उत्तर:
चीनी शासक स्वयं को स्वर्गपुत्र के नाम से सम्बोधित करते थे।

प्रश्न 93.
गुप्त शासकों का इतिहास किसकी सहायता से लिखा गया है?
उत्तर:
गुप्त शासकों का इतिहास साहित्य, सिक्कों और अभिलेखों की सहायता से लिखा गया है।

प्रश्न 94.
गंदविन्दु जातक नामक कहानी में क्या बताया गया है?
उत्तर:
गंदविन्दु जातक नामक एक कहानी में यह बताया गया है कि एक कुटिल राजा की प्रजा किस प्रकार दुःखी रहती है।

प्रश्न 95.
पालि भाषा में गहपति का प्रयोग किनके लिए किया जाता है?
उत्तर:
पालि भाषा में गहपति का प्रयोग छोटे किसानों और जमींदारों के लिए किया जाता है।

प्रश्न 96.
उत्तरी कृष्ण मार्जित पात्र से क्या अभिप्राय
उत्तर:
नगरों में मिले चमकदार कलई वाले मिट्टी के कटोरे, थालियाँ आदि उत्तरी कृष्ण मार्जित पात्र कहलाते हैं।

प्रश्न 97.
पियदस्सी शब्द से आपका क्या अभिप्राय है?
उत्तर:
पियदस्सी अर्थात् देखने में सुन्दर।

प्रश्न 98.
देवानांप्रिय शब्द का क्या आशय है?
उत्तर:’
देवानांप्रिय शब्द का आशय है, देवताओं का

प्रश्न 99.
अभिलेखशास्त्रियों ने पतिवेदक शब्द का अर्थ क्या बताया ?
उत्तर:
अभिलेखशास्त्रियों ने पतिवेदक शब्द का अर्थ संवाददाता बताया।

JAC Class 12 History Important Questions Chapter 2 राजा, किसान और नगर : आरंभिक राज्य और अर्थव्यवस्थाएँ

लघुत्तरात्मक प्रश्न 

प्रश्न 1.
अशोक द्वारा अपने अधिकारियों और प्रजा को दिए गए संदेशों को वर्तमान संदर्भ में प्रासंगिकता सिद्ध कीजिए।
उत्तर:
अशोक ने अपने अधिकारियों और प्रजा को संदेश दिया कि वे बड़ों के प्रति आदर, संन्यासियों और ब्राह्मणों के प्रति उदारता, सेवकों और दासों के साथ उदार व्यवहार तथा दूसरे के धर्मों और परम्पराओं का आदर करें। वर्तमान समय में भी अशोक के संदेशों की प्रासंगिकता बनी हुई है।

प्रश्न 2.
“हड़प्पा सभ्यता के बाद डेढ़ हजार वर्षों के लम्बे अन्तराल में उपमहाद्वीप के विभिन्न भागों में कई प्रकार के विकास हुए।” स्पष्ट कीजिए।
उत्तर:
(1) इस काल में सिन्धु नदी और इसकी उपनदियों के किनारे रहने वाले लोगों ने ऋग्वेद का लेखन- कार्य किया।
(2) उत्तर भारत, दक्कन पठार क्षेत्र और कर्नाटक आदि कई क्षेत्रों में कृषक बस्तियों का उदय हुआ।
(3) ईसा पूर्व पहली सहस्राब्दी के दौरान मध्य और दक्षिण भारत में शवों के अन्तिम संस्कार के नवीन तरीके अपनाए गए। इनमें महापाषाण नामक पत्थरों के बड़े-बड़े ढाँचे मिले हैं। कई स्थानों पर शवों के साथ लोहे से बने उपकरणों एवं हथियारों को भी दफनाया गया था।

प्रश्न 3.
ईसा पूर्व छठी शताब्दी से भारत में हुए नवीन परिवर्तनों का उल्लेख कीजिए।
उत्तर:
ईसा पूर्व छठी शताब्दी से भारत में हुए नवीन परिवर्तन- ई. पूर्व छठी शताब्दी से भारत में निम्नलिखित नवीन परिवर्तन हुए –
(1) इन परिवर्तनों में सबसे प्रमुख परिवर्तन आरम्भिक राज्यों, साम्राज्यों और रजवाड़ों का विकास है। इन राजनीतिक प्रक्रियाओं के लिए कुछ अन्य परिवर्तन जिम्मेदार थे। इनकी जानकारी कृषि उपज को संगठित करने के तरीके से होती है।
(2) इस युग में लगभग सम्पूर्ण उपमहाद्वीप में नए नगरों का उदय हुआ।

प्रश्न 4.
जेम्स प्रिंसेप कौन था? उसके द्वारा किये |गए शोध कार्य का उल्लेख कीजिए।
उत्तर:
जेम्स प्रिंसेप ईस्ट इण्डिया कम्पनी में एक अधिकारी के पद पर नियुक्त था। उसने 1838 ई. में ब्राह्मी और खरोष्ठी लिपियों को पढ़ने में सफलता प्राप्त की। इन लिपियों का उपयोग सबसे आरम्भिक अभिलेखों और सिक्कों में किया गया है। प्रिंसेप को यह जानकारी हुई कि अभिलेखों और सिक्कों पर पियदस्सी अर्थात् सुन्दर मुखाकृति वाले राजा का नाम लिखा है। कुछ अभिलेखों पर राजा का नाम अशोक भी लिखा था।

प्रश्न 5.
“जेम्स प्रिंसेप के शोध कार्य से आरम्भिक भारत के राजनीतिक इतिहास के अध्ययन को एक नई दिशा मिली।” व्याख्या कीजिए।
उत्तर:
(1) भारतीय तथा यूरोपीय विद्वानों ने उपमहाद्वीप पर शासन करने वाले प्रमुख राजवंशों की पुनर्रचना के लिए विभिन्न भाषाओं में लिखे अभिलेखों और ग्रन्थों का उपयोग किया। परिणामस्वरूप बीसवीं शताब्दी के आरम्भिक दशकों तक उपमहाद्वीप के राजनीतिक इतिहास की एक सामान्य रूपरेखा तैयार हो गई।
(2) उसके पश्चात् विद्वानों को जानकारी हुई कि राजनीतिक परिवर्तनों और आर्थिक तथा सामाजिक विकासों के बीच सम्बन्ध तो थे, परन्तु सम्भवतः सीधे सम्बन्ध सदैव नहीं थे।

प्रश्न 6.
अभिलेख आज भी इतिहास की जानकारी के महत्त्वपूर्ण साधन हैं। स्पष्ट कीजिए।
अथवा
वर्तमान इतिहास लेखन में अभिलेखों की उपादेयता सिद्ध कीजिए।
अथवा
इतिहास लेखन में अभिलेखों का क्या महत्त्व है?
अथवा
अभिलेख किसे कहते हैं? उनका ऐतिहासिक महत्त्व बताइये।
उत्तर:
अभिलेख अभिलेख उन्हें कहते हैं, जो पत्थर, धातु या मिट्टी के बर्तन जैसी कठोर सतह पर खुदे होते हैं।

अभिलेखों का ऐतिहासिक महत्त्व –
(1) अभिलेखों में उन लोगों की उपलब्धियों, क्रियाकलापों तथा विचारों का उल्लेख मिलता है, जो उन्हें बनवाते हैं।
(2) इनमें राजाओं के क्रियाकलापों तथा महिलाओं और पुरुषों द्वारा धार्मिक संस्थाओं को दिए दान का विवरण होता है।
(3) अभिलेखों में इनके निर्माण की तिथि भी खुदी होती है।

प्रश्न 7.
जिन अभिलेखों पर तिथि खुदी हुई नहीं मिलती है, उनका काल निर्धारण किस प्रकार किया जाता था?
उत्तर:
जिन अभिलेखों पर तिथि खुदी हुई नहीं मिलती है, उनका काल निर्धारण आमतौर पर पुरालिपि अथवा लेखन शैली के आधार पर किया जा सकता है। उदाहरणार्थ, लगभग 250 ई. पूर्व में अक्षर अ ‘प्र’ इस प्रकार लिखा जाता था और 500 ई. में बह भ इस प्रकार लिखा जाता था।

JAC Class 12 History Important Questions Chapter 2 राजा, किसान और नगर : आरंभिक राज्य और अर्थव्यवस्थाएँ

प्रश्न 8.
प्राचीनतम अभिलेख किन भाषाओं में लिखे जाते थे?
उत्तर:
प्राचीनतम अभिलेख प्राकृत भाषाओं में लिखे जाते थे। प्राकृत उन भाषाओं को कहा जाता था जो जनसामान्य की भाषाएँ होती थीं। प्राकृत भाषा में अजातशत्रु को ‘अजातसत्तु’ तथा अशोक को ‘असोक’ लिखा जाता है। कुछ अभिलेख तमिल, पालि और संस्कृत भाषाओं में भी मिलते हैं।

प्रश्न 9.
आरम्भिक भारतीय इतिहास में छठी शताब्दी ई. पूर्व को एक महत्त्वपूर्ण परिवर्तनकारी काल क्यों माना जाता है?
उत्तर:
आरम्भिक भारतीय इतिहास में छठी शताब्दी ई. पूर्व को निम्न कारणों से एक महत्त्वपूर्ण परिवर्तनकारी काल माना जाता है-

  1. इस काल को प्रायः आरम्भिक राज्यों, नगरों, लोहे के बढ़ते प्रयोग तथा सिक्कों के विकास के साथ जोड़ा जाता है।
  2. इस काल में बौद्ध धर्म, जैन धर्म सहित विभिन्न दार्शनिक विचारधाराओं का विकास हुआ।
  3. इस काल में सोलह महाजनपदों का उदय हुआ।

प्रश्न 10.
‘जनपद’ किसे कहते हैं?
उत्तर:
जनपद का अर्थ एक ऐसा भूखण्ड है, जहाँ कोई जन (लोग, कुल या जनजाति) अपना पाँव रखता है। अथवा बस जाता है। इस शब्द का प्रयोग प्राकृत व संस्कृत दोनों भाषाओं में मिलता है प्रारम्भ में इन जनों का कोई निश्चित स्थान नहीं होता था और अपनी आवश्यकतानुसार ये स्थान बदल लिया करते थे परन्तु शीघ्र ही वे एक निश्चित स्थान पर बस गए। भिन्न-भिन्न भौगोलिक क्षेत्र ‘जनों’ के बस जाने के कारण ‘जनपद’ कहलाने लगे।

प्रश्न 11.
बौद्धकालीन 16 महाजनपदों के नाम लिखिए।
अथवा
छठी शताब्दी ई. पूर्व के सोलह महाजनपदों का उल्लेख कीजिए।
उत्तर:
छठी शताब्दी ई. पूर्व में भारत में अनेक महाजनपदों का उदय हुआ। बौद्ध और जैन धर्म के आरम्भिक ग्रन्थों में 16 महाजनपदों का उल्लेख मिलता है।

इनके नाम हैं –

  1. अंग
  2. मगध
  3. काशी
  4. कोशल
  5. वज्जि
  6. मल्ल
  7. चेदि
  8. वत्स
  9. कुरु
  10. पांचाल
  11. मत्स्य
  12. शूरसेन
  13. अश्मक
  14. अवन्ति
  15. कम्बोज तथा
  16. गान्धार।

प्रश्न 12.
मगध के शक्तिशाली महाजनपद के रूप में उदय के प्रमुख कारण लिखिए।
अथवा
मगध महाजनपद के सबसे शक्तिशाली बनने के क्या कारण थे?
थी।
उत्तर:
आधुनिक इतिहासकारों के अनुसार मगध महाजनपद के सबसे शक्तिशाली बनने के कारण –

  1. मगध क्षेत्र में खेती की उपज बहुत अच्छी होती
  2. मगध क्षेत्र में लोहे की खदानें भी सरलता से उपलब्ध थीं। अतः लोहे से उपकरण और हथियार बनाना आसान होता था।
  3. मगध के जंगलों में बड़ी संख्या में हाथी उपलब्ध थे। ये हाथी मगध राज्य की सेना के एक महत्त्वपूर्ण अंग थे।
  4. गंगा और इसकी उपनदियों से आवागमन सस्ता वसुलभ होता था।
  5. बिम्बिसार, अजातशत्रु की नीतियों मगध के विकास के लिए उत्तरदायी थीं।

प्रश्न 13.
आरम्भिक जैन एवं बौद्ध लेखकों के अनुसार मगध के सबसे शक्तिशाली महाजनपद बनने के क्या कारण थे?
उत्तर:
आरम्भिक जैन और बौद्ध लेखकों के अनुसार मगध के सबसे शक्तिशाली महाजनपद बनने के लिए मगध के पराक्रमी शासकों की नीतियाँ उत्तरदायी थीं। बिम्बिसार, अजातशत्रु, महापद्मनन्द आदि राजा अत्यन्त महत्त्वाकांक्षी एवं पराक्रमी शासक थे जिनके नेतृत्व में मगध राज्य का अत्यधिक विस्तार हुआ। इनके मन्त्री भी योग्य थे जो इनकी नीतियों को सफलतापूर्वक लागू करते थे।

JAC Class 12 History Important Questions Chapter 2 राजा, किसान और नगर : आरंभिक राज्य और अर्थव्यवस्थाएँ

प्रश्न 14.
महाजनपदों की शासन व्यवस्था का वर्णन कीजिए।
उत्तर:
अधिकांश महाजनपदों पर राजा का शासन होता था, परन्तु ‘गण’ और ‘संघ’ नामक प्रसिद्ध राज्यों में कई लोगों का समूह शासन करता था। इस समूह का प्रत्येक व्यक्ति राजा कहलाता था। बन्जि संघ की ही भाँति कुछ राज्यों में भूमि सहित अनेक आर्थिक स्रोतों पर राजा गण सामूहिक नियन्त्रण रखते थे। प्रत्येक महाजनपद की एक राजधानी होती थी जो प्रायः किले से घेरी जाती थी।

प्रश्न 15.
मगध की प्रारम्भिक राजधानी राजगाह पर एक संक्षिप्त टिप्पणी लिखिए।
उत्तर:
मगध की प्रारम्भिक राजधानी राजगाह (आधुनिक बिहार में स्थित राजगीर) थी। राजगाह का शाब्दिक अर्थ है—’राजाओं का घर’। पहाड़ियों के बीच बसा राजगाह एक किलेबन्द शहर था। बाद में चौथी शताब्दी ई. पूर्व में पाटलिपुत्र को राजधानी बनाया गया, जो अब पटना के नाम से जाना जाता है। यह गंगा के रास्ते आवागमन के मार्ग पर स्थित था।

प्रश्न 16.
अशोक कौन था?
उत्तर:
अशोक बिन्दुसार का पुत्र तथा चन्द्रगुप्त मौर्य का पौत्र था। वह आरम्भिक भारत का सर्वप्रसिद्ध सम्राट था। उसने कलिंग पर विजय प्राप्त की अशोक पहला सम्राट था, जिसने अपने अधिकारियों एवं प्रजा के लिए सन्देश प्राकृतिक पत्थरों और पालिश किए हुए स्तम्भों पर लिखवाये थे। उसने अपने अभिलेखों के माध्यम से ‘धम्म’ का प्रचार किया।

प्रश्न 17.
अशोक के अभिलेख किन भाषाओं और लिपियों में लिखे गए थे?
उत्तर:
अशोक के अभिलेखों की भाषाएँ एवं लिपियाँ – अशोक के अधिकांश अभिलेख प्राकृत भाषा में लिखे गए हैं जबकि पश्चिमोत्तर से प्राप्त अभिलेख अरामेइक और यूनानी भाषा में हैं। प्राकृत के अधिकांश अभिलेख ब्राह्मी लिपि में लिखे गए थे परन्तु पश्चिमोत्तर के कुछ अभिलेख खरोष्ठी लिपि में लिखे गए थे। अरामेइक और यूनानी लिपियों का प्रयोग अफगानिस्तान में मिले अभिलेखों में किया गया था।

प्रश्न 18.
मेगस्थनीज के द्वारा चन्द्रगुप्त मौर्य के सैनिक विभाग के प्रबन्ध के बारे में क्या विवरण दिया गया है?
उत्तर:
मेगस्थनीज ने लिखा है कि सैनिक विभाग का प्रबन्ध करने के लिए एक समिति तथा छः उपसमितियाँ बनी हुई थीं। पहली उपसमिति का काम नौसेना का संचालन करना था। दूसरी उपसमिति यातायात तथा खानपान का संचालन करती थी। तीसरी उपसमिति का काम पैदल सैनिकों का संचालन करना था। चौथी उपसमिति अश्वारोही सेना का संचालन करती थी तथा पाँचवीं उपसमिति का काम रथारोहियों का संचालन करना था। छठी उपसमिति हाथियों का संचालन करती थी।

प्रश्न 19.
मेगस्थनीज के अनुसार सैनिक गतिविधियों के संचालन के लिए नियुक्त दूसरी उपसमिति क्या कार्य करती थी?
उत्तर:
मेगस्थीज के अनुसार दूसरी उपसमिति विभिन्न प्रकार के कार्य करती थी, जैसे उपकरणों के ढोने के लिए बैलगाड़ियों की व्यवस्था करना, सैनिकों के लिए भोजन और जानवरों के लिए चारे की व्यवस्था करना तथा सैनिकों की देखभाल के लिए सेवकों और शिल्पकारों को नियुक्त करना आदि।

प्रश्न 20.
मेगस्थनीज के अनुसार मौर्य साम्राज्य के प्रमुख अधिकारियों के कार्यों का उल्लेख कीजिए।
उत्तर:
(1) कुछ अधिकारी नदियों की देख-रेख तथा भूमि मापन का कार्य करते थे।
(2) कुछ अधिकारी प्रमुख नहरों से उपनहरों के लिए छोड़े जाने वाले पानी के मुखद्वार का निरीक्षण करते थे ताकि प्रत्येक स्थान पर पानी की समान पूर्ति हो सके। यही अधिकारी शिकारियों का संचालन करते थे।
(3) ये अधिकारी करवसूली करते थे और भूमि से जुड़े सभी व्यवसायों का निरीक्षण करते थे। इसके अतिरिक्त वे लकड़हारों, बढ़ई, लोहारों तथा खननकर्ताओं का भी निरीक्षण करते थे।

JAC Class 12 History Important Questions Chapter 2 राजा, किसान और नगर : आरंभिक राज्य और अर्थव्यवस्थाएँ

प्रश्न 21.
मौर्य साम्राज्य के प्रमुख राजनीतिक केन्द्रों का उल्लेख कीजिए।
उत्तर:
मौर्य साम्राज्य के पाँच प्रमुख राजनीतिक केन्द्र थे –

  1. पाटलिपुत्र- यह मौर्य साम्राज्य की राजधानी थी
  2. तक्षशिला यह प्रान्तीय केन्द्र था।
  3. उज्जयिनी – यह भी प्रान्तीय केन्द्र था।
  4. तोसलि – यह भी एक प्रान्तीय केन्द्र था।
  5. स्वर्णगिरि – यह भी एक प्रान्तीय केन्द्र था।

मगध साम्राज्य की राजधानी पाटलिपुत्र तथा उसके आस-पास के प्रान्तीय केन्द्रों पर सबसे सुदृढ़ प्रशासनिक नियन्त्रण था।

प्रश्न 22.
मौर्य साम्राज्य के महत्त्व की विवेचना कीजिए।
उत्तर:
(1) मौर्य काल में भवन निर्माण कला, मूर्तिकला आदि की महत्त्वपूर्ण उन्नति हुई।
(2) मौर्य सम्राटों के अभिलेखों पर लिखे संदेश अन्य शासकों के अभिलेखों से भिन्न हैं।
(3) अन्य शासकों की अपेक्षा अशोक एक बहुत शक्तिशाली, प्रभावशाली और परिश्रमी शासक थे। वह बाद के राजाओं की अपेक्षा विनीत और नम्र भी थे।
(4) अशोक की उपलब्धियों से प्रभावित होकर बीसवीं सदी के राष्ट्रवादी नेताओं ने अशोक को प्रेरणा का स्रोत माना।

प्रश्न 23.
मौर्य साम्राज्य की कमजोरियों पर प्रकाश डालिए।
उत्तर:
(1) मौर्य साम्राज्य केवल 150 वर्ष तक ही अस्तित्व में रहा। इसे उपमहाद्वीप के इस लम्बे इतिहास में बहुत बड़ा काल नहीं माना जा सकता।
(2) मौर्य साम्राज्य उपमहाद्वीप के सभी क्षेत्रों में नहीं फैल पाया था। इसके अतिरिक्त मौर्य साम्राज्य की सीमाओं के अन्तर्गत भी नियन्त्रण एक समान नहीं था। अतः दूसरी शताब्दी ई. पूर्व तक उपमहाद्वीप के अनेक भागों में नए- नए शासक और रजवाड़े स्थापित होने लगे।

प्रश्न 24.
अशोक ने अपने साम्राज्य को अखण्ड बनाए रखने के लिए क्या उपाय किये?
उत्तर:
अशोक ने अपने साम्राज्य को अखण्ड बनाए रखने के लिए ‘धम्म’ के प्रचार का सहारा लिया। अशोक के ‘धम्म’ के सिद्धान्त बड़े साधारण व्यावहारिक तथा सार्वभौमिक थे। उसने धम्म के माध्यम से अपनी प्रजा की लौकिक और पारलौकिक उन्नति का प्रयास किया। उसने धम्म के प्रचार के लिए ‘धर्ममहामात्र’ नामक विशेष अधिकारी नियुक्त किए।

प्रश्न 25.
सरदार और सरदारी का अर्थ स्पष्ट कीजिए।
उत्तर:
सरदार एक शक्तिशाली और प्रभावशाली व्यक्ति होता था जिसका पद वंशानुगत भी हो सकता था और नहीं भी। उसके समर्थक उसके वंश के लोग होते थे सरदार विशेष अनुष्ठान का संचालन करता था, युद्ध के समय सेना का नेतृत्व करता था तथा लोगों के विवादों को सुलझाने में मध्यस्थता की भूमिका निभाता था। वह अपने अधीन लोगों से भेंट लेता था और उसे अपने समर्थकों में बाँट दिया करता था। सरदारी में सामान्यतया कोई स्थायी सेना या अधिकारी नहीं होते थे।

प्रश्न 26.
दक्षिण के राज्यों और उनके शासकों का वर्णन कीजिए।
उत्तर:
उपमहाद्वीप के दक्कन और उससे दक्षिण के क्षेत्र में स्थित तमिलकम (अर्थात् तमिलनाडु एवं आन्ध्रप्रदेश और केरल के कुछ भाग) में चोल, चेर और पाण्ड्य जैसी सरदारियों का उदय हुआ। ये राज्य बहुत ही. समृद्ध और स्थायी सिद्ध हुए कई सरदार और राजा लम्बी दूरी के व्यापार द्वारा राजस्व जुटाते थे। इनमें मध्य और पश्चिम भारत के क्षेत्रों पर शासन करने वाले सातवाहन और उपमहाद्वीप के पश्चिमोत्तर और पश्चिम में शासन करने वाले मध्य एशियाई मूल के शक शासक सम्मिलित थे।

प्रश्न 27.
प्राचीन भारतीय शासक देवी-देवताओं के साथ जुड़ने का प्रयास क्यों करते थे?
उत्तर:
प्राचीन भारतीय शासक उच्च स्थान प्राप्त करने के लिए देवी-देवताओं के साथ जुड़ने का प्रयास करते थे। कुषाण शासकों ने इस उपाय का सर्वश्रेष्ठ उद्धरण प्रस्तुत किया। कुषाण सम्राटों ने देवस्थानों पर अपनी विशालकाय मूर्तियाँ स्थापित कीं। वे इन मूर्तियों के माध्यम से स्वयं देवतुल्य प्रस्तुत करना चाहते थे। कुछ कुषाण शासकों ने ‘देवपुत्र’ की उपाधि धारण की।

JAC Class 12 History Important Questions Chapter 2 राजा, किसान और नगर : आरंभिक राज्य और अर्थव्यवस्थाएँ

प्रश्न 28.
‘प्रयाग प्रशस्ति’ पर एक संक्षिप्त टिप्पणी लिखिए।
उत्तर:
‘प्रयाग प्रशस्ति’ के रचयिता हरिषेण थे, जो गुप्त वंश के सम्राट समुद्रगुप्त के राजकवि थे ‘प्रयाग- प्रशस्ति’ संस्कृत में लिखी गई थी। ‘प्रयाग प्रशस्ति’ से हमें समुद्रगुप्त के जीवन, विजयों, व्यक्तिगत गुणों, तत्कालीन राजनीतिक दशा आदि के बारे में जानकारी मिलती है। ‘प्रयाग प्रशस्ति’ में समुद्रगुप्त की दिग्विजय का उल्लेख है। ‘प्रयाग प्रशस्ति’ में समुद्रगुप्त को परमात्मा पुरुष, उदारता की प्रतिमूर्ति, कुबेर, वरुण, इन्द्र और यम के तुल्य बताया गया है।

प्रश्न 29.
हरिषेण द्वारा ‘प्रयाग प्रशस्ति’ में समुद्रगुप्त की चारित्रिक विशेषताओं का किस प्रकार वर्णन किया गया है?
उत्तर:
प्रयाग प्रशस्ति’ में हरिषेण ने समुद्रगुप्त के लिए लिखा है कि “धरती पर उनका कोई प्रतिद्वन्द्वी नहीं था अनेक गुणों और शुभ कार्यों से सम्पन्न उन्होंने अपने पैर के तलवे से अन्य राजाओं के यश को मिटा दिया है। वे परमात्मा पुरुष हैं, साधु (भले) की समृद्धि और असाधु (बुरे) के विनाश के कारण हैं। वे करुणा से भरे हुए हैं। उनके मस्तिष्क की दीक्षा दीन-दुखियों विरहणियों और पीड़ितों के उद्धार के लिए की गई है। वे देवताओं में कुबेर ( धन-देव), वरुण (समुद्र-देव), इन्द्र (वर्षा के देवता) और यम (मृत्यु-देव)
के तुल्य हैं।”

प्रश्न 30.
शक- शासक रुद्रदामन के अभिलेख में सुदर्शन झील के बारे में क्या विवरण दिया गया है?
उत्तर:
शक- शासक रुद्रदामन के जूनागढ़ अभिलेख में सुदर्शन झील के बारे में वर्णन है कि जलद्वारों और तटबन्धों वाली इस झील का निर्माण मौर्यकाल में एक स्थानीय राज्यपाल द्वारा किया गया था। परन्तु एक भीषण तूफान के कारण इसके तटबन्ध टूट गए और सारा पानी बह गया। तत्कालीन शासक रुद्रदामन ने इस झील की मरम्मत अपने से करवाई थी और इसके लिए अपनी प्रजा से कर भी नहीं लिया था।

प्रश्न 31.
जनता में राजा की छवि के बारे में किस स्त्रोत से जानकारी मिलती है?
उत्तर:
जनता में राजा की छवि के बारे में जातकों एवं पंचतंत्र जैसे ग्रन्थों में वर्णित कथाओं से जानकारी मिलती है। इतिहासकारों ने पता लगाया है कि इनमें से अनेक कथाओं के स्रोत मौखिक किस्से-कहानियाँ हैं, जिन्हें बाद में लिखा गया होगा। इन जातकों से राजा के व्यवहार के कारण जनता की शोचनीय स्थिति तथा राजा और प्रजा के बीच तनावपूर्ण सम्बन्धों की जानकारी मिलती है।

प्रश्न 32.
‘गंदतिन्दु’ नामक जातक कथा से राजा और प्रजा के सम्बन्धों पर क्या प्रकाश पड़ता है?
उत्तर:
गंदविन्दु’ नामक जातक कथा से पता चलता है कि एक कुटिल राजा की प्रजा किस प्रकार से दुःखी रहती है। जब राजा अपनी पहचान बदलकर प्रजा के बीच में यह जानने के लिए गया कि लोग उसके बारे में क्या सोचते हैं, तो सभी लोगों ने अपने दुःखों के लिए राजा की आलोचना की। उनकी शिकायत थी कि रात में डाकू लोग उन पर हमला करते हैं तथा दिन में कर इकट्ठा करने वाले अधिकारी उन्हें परेशान करते हैं।

प्रश्न 33.
जातक कथाओं के अनुसार राजा और प्रजा के बीच सम्बन्ध तनावपूर्ण क्यों रहते थे?
उत्तर:
जातक कथाओं से ज्ञात होता है कि राजा और प्रजा, विशेषकर ग्रामीण प्रजा के बीच सम्बन्ध तनावपूर्ण रहते थे। इसका प्रमुख कारण यह था कि शासक अपने राजकोष को भरने के लिए प्रजा से बड़े-बड़े करों की माँग करते थे, जिससे किसानों में घोर असन्तोष व्याप्त था। जातक कथाओं से ज्ञात होता है कि राजा के अत्याचारों से बचने के लिए किसान लोग अपने घर छोड़कर जंगल की ओर भाग जाते थे। इस प्रकार राजा और ग्रामीण प्रजा के बीच तनाव बना रहता था।

प्रश्न 34.
छठी शताब्दी ई. पूर्व में उपज बढ़ाने के तरीकों का उल्लेख कीजिए।
उत्तर:
(1) भारी वर्षा होने वाले क्षेत्रों में लोहे के फाल वाले हलों के माध्यम से उर्वर भूमि की जुताई की जाती थी। कुछ क्षेत्रों में खेती के लिए कुदाल का उपयोग किया जाता था।
(2) गंगा की घाटी में धान की रोपाई किए जाने से उपज में भारी वृद्धि होने लगी।
(3) उपज बढ़ने के लिए कुओं, तालाबों तथा नहरों के माध्यम से सिंचाई की जाती थी राजाओं द्वारा कुआँ, तालाबों तथा नहरों का निर्माण करवाया जाता था।

प्रश्न 35.
ग्रामीण समाज में पाई जाने वाली विभिन्नताओं का उल्लेख कीजिए।
उत्तर:
बौद्ध ग्रन्थों से पता चलता है कि भारत में खेती से जुड़े लोगों में भेद बढ़ता जा रहा था बौद्ध कथाओं में भूमिहीन खेतिहर श्रमिकों, छोटे किसानों और बड़े-बड़े जमींदारों का उल्लेख मिलता है। बड़े-बड़े जमींदार और ग्राम प्रधान शक्तिशाली तथा प्रभावशाली माने जाते थे तथा वे किसानों पर नियन्त्रण रखते थे आरम्भिक तमिल संगम साहित्य में भी गाँवों में रहने वाले विभिन्न वर्गों के लोगों का उल्लेख है, जैसे कि वेल्लालर या बड़े जमींदार, हलवाहा या उल्चर और दास अणिमई।

JAC Class 12 History Important Questions Chapter 2 राजा, किसान और नगर : आरंभिक राज्य और अर्थव्यवस्थाएँ

प्रश्न 36.
मनुस्मृति के अनुसार सीमा सम्बन्धी विवादों के समाधान के लिए राजा को क्या सलाह दी गई है?
उत्तर:
मनुस्मृति में सीमा सम्बन्धी विवादों के समाधान के लिए राजा को निम्न सलाह दी गई है –
चूंकि सीमाओं की अनभिज्ञता के कारण विश्व में बार-बार विवाद पैदा होते हैं, इसलिए उसे सीमाओं की पहचान के लिए गुप्त निशान जमीन में गाड़ कर रखने चाहिए जैसे कि पत्थर, हड्डियाँ, गाय के बाल, भूसी, राख, खपटे, गाय के सूखे गोबर, ईंट, कोयला, कंकड़ और रेत। उसे सीमाओं पर इसी प्रकार के और तत्त्व भूमि में छुपाकर गाड़ने चाहिए जो समय के \
साथ नष्ट न हों।

प्रश्न 37.
‘हर्षचरित’ पर एक संक्षिप्त टिप्पणी लिखिए।
उत्तर:
हर्षचरित’ हर्षचरित’ संस्कृत में लिखी गई कनौज के शासक हर्षवर्धन की जीवनी है। इस ग्रन्थ की रचना हर्षवर्धन के राजकवि बाणभट्ट ने की थी। ‘हर्षचरित’ से हर्षवर्धन के जीवन, राज्यारोहण, उसकी विजयों, चारित्रिक विशेषताओं आदि की जानकारी मिलती है। इससे तत्कालीन राजनीतिक, सामाजिक, आर्थिक एवं धार्मिक अवस्थाओं तथा समाज के विभिन्न वर्गों तथा उनके व्यवसाय के बारे में भी पर्याप्त जानकारी प्राप्त होती है।

प्रश्न 38.
‘गहपति’ के बारे में आप क्या जानते हैं ?
उत्तर:
पालि भाषा में गहपति का प्रयोग छोटे किसानों और जमींदारों के लिए किया जाता था। गहपति घर का मुखिया होता था तथा घर में रहने वाली महिलाओं, बच्चों, नौकरों और दासों पर नियन्त्रण करता था। घर से जुड़े भूमि, जानवर या अन्य सभी वस्तुओं का वह मालिक होता था । कभी-कभी इस शब्द का प्रयोग नगरों में रहने वाले प्रतिष्ठित व्यक्तियों और व्यापारियों के लिए भी होता था।

प्रश्न 39.
बाणभट्ट ने अपने ग्रन्थ ‘हर्षचरित’ में विश्य क्षेत्र के जंगल के किनारे की एक बस्ती के जीवन का किस प्रकार चित्रण किया है?
उत्तर:
‘हर्षचरित’ के अनुसार, बस्ती के किनारे का अधिकांश क्षेत्र जंगल है और यहाँ धान की उपज वाली, खलिहान और उपजाऊ भूमि के हिस्सों को छोटे किसानों ने आपस में बाँट लिया है। यहाँ के अधिकांश लोग कुदाल का प्रयोग करते हैं क्योंकि घास से भरी भूमि में हल चलाना कठिन है। यहाँ लोग पेड़ की छाल के गट्ठर लेंकर चलते हैं। वे फूलों से भरे अनगिनत बोरे, अलसी और सन, भारी मात्रा में शहद, मोरपंख, मोम, लकड़ी और पास के बोझ लेकर आते जाते रहते हैं।

प्रश्न 40.
भूमिदान का उल्लेख किन में मिलता है? भूमिदान किन्हें दिया जाता था ?
उत्तर:
भूमिदान – ईसवी की आरम्भिक शताब्दियों से ही भूमिदान के प्रमाण मिलते हैं। कई भूमिदानों का उल्लेख अभिलेखों में मिलता है। इनमें से कुछ अभिलेख पत्थरों पर उत्कीर्ण थे परन्तु अधिकांश अभिलेख ताम्रपत्रों पर खुदे होते थे। इन्हें सम्भवतः उन लोगों को प्रमाण रूप में दिया जाता था जो भूमिदान लेते थे साधारणतया भूमिदान धार्मिक संस्थाओं और ब्राह्मणों को दिए गए थे।

प्रश्न 41.
अभिलेखों से प्रभावती गुप्त के भूमिदान किए जाने के बारे में क्या जानकारी मिलती है?
उत्तर:
प्रभावती गुप्त सम्राट चन्द्रगुप्त द्वितीय (लगभग 375-415 ई.) की पुत्री थी संस्कृत धर्मशास्त्रों के अनुसार महिलाओं को भूमि जैसी सम्पत्ति पर स्वतन्त्र अधिकार नहीं था। परन्तु एक अभिलेख से ज्ञात होता है कि प्रभावती गुप्त भूमि की स्वामिनी थी और उसने भूमिदान भी किया था। इसका एक कारण यह हो सकता है कि चूँकि वह एक रानी थी और इसीलिए उनका यह उदाहरण एक विरला ही रहा हो।

प्रश्न 42.
प्रभावती गुप्त के अभिलेख से हमें ग्रामीण प्रजा के बारे में क्या जानकारी मिलती है?
उत्तर:
प्रभावती गुप्त के अभिलेख से हमें तत्कालीन ग्रामीण प्रजा के बारे में जानकारी मिलती है। उस समय गाँवों में ब्राह्मण, किसान तथा अन्य प्रकार के वर्गों के लोग रहते थे। ये लोग शासकों या उनके प्रतिनिधियों को अनेक प्रकार की वस्तुएँ प्रदान करते थे। अभिलेखों के अनुसार इन लोगों को गाँव के नए प्रधान की आज्ञाओं का पालन करना पड़ता था। ये लोग अपने भुगतान उसी को ही देते थे।

प्रश्न 43.
भूमिदान के प्रभावों की विवेचना कीजिए।
उत्तर:

  1. कुछ इतिहासकारों के अनुसार भूमिदान शासक वंश द्वारा कृषि को नये क्षेत्रों में प्रोत्साहित करने की एक रणनीति थी।
  2. जब राजा का शासन सामन्तों पर कमजोर होने लगा, तो उन्होंने भूमिदान के माध्यम से अपने समर्थकों को इकट्ठा करना शुरू कर दिया।
  3. राजा स्वयं को उत्कृष्ट स्तर के मानव के रूप में प्रदर्शित करना चाहते थे।
  4. भूमिदान के प्रचलन से राज्य तथा किसानों के बीच सम्बन्ध की झांकी मिलती है।

प्रश्न 44.
अपने अभिलेख में दंगुन गाँव के दान में प्रभावती गुप्त द्वारा दी गई रियायतों का वर्णन कीजिए।
उत्तर:
अपने अभिलेख में प्रभावती गुप्त ने निम्नलिखित रियायतों की घोषणा की। इस गाँव में पुलिस या सैनिक प्रवेश नहीं करेंगे। दौरे पर आने वाले शासकीय अधिकारियों को यह गाँव पास देने और आसन में प्रयुक्त होने वाले जानवरों की खाल और कोयला देने के दायित्व से मुक्त है। साथ ही वे मदिरा खरीदने और नमक हेतु खुदाई करने के राजसी अधिकार को कार्यान्वित किए जाने से मुक्त हैं। इस गाँव को खनिज पदार्थ, खदिर वृक्ष के उत्पाद, फूल और दूध देने से भी छूट है।

प्रश्न 45.
छठी शताब्दी ई. पूर्व में उपमहाद्वीप के विभिन्न क्षेत्रों में नये नगरों के विकास का उल्लेख कीजिए।
उत्तर:
लगभग छठी शताब्दी ई. पूर्व में उपमहाद्वीप के विभिन्न क्षेत्रों में अनेक नगरों का विकास हुआ। इनमें से अधिकांश नगर महाजनपदों की राजधानियाँ थीं। प्रायः सभी नगर संचार मार्गों के किनारे बसे थे। पाटलिपुत्र जैसे कुछ नगर नदी मार्ग के किनारे बसे थे। उज्जयिनी जैसे नगर भूतल मार्गों के किनारे बसे थे। इसके अतिरिक्त पुहार जैसे नगर समुद्र तट पर थे, जहाँ से समुद्री मार्ग प्रारम्भ हुए। मथुरा जैसे अनेक शहर व्यावसायिक, सांस्कृतिक एवं राजनीतिक गतिविधियों के प्रमुख केन्द्र थे।

JAC Class 12 History Important Questions Chapter 2 राजा, किसान और नगर : आरंभिक राज्य और अर्थव्यवस्थाएँ

प्रश्न 46.
छठी शताब्दी ई. पूर्व में नगरों में रहने वाले सम्भ्रान्त वर्ग और शिल्पकार वर्गों का उल्लेख कीजिए।
उत्तर:
किलेबन्द नगरों में उत्कृष्ट श्रेणी के मिट्टी के कटोरे और थालियाँ मिली हैं जिन पर चमकदार कलई चढ़ी है। सम्भवतः इनका उपयोग धनी लोग करते होंगे। इसके साथ ही सोने, चाँदी, हाथीदाँत, कांस्य आदि के बने आभूषण, उपकरण, हथियार भी मिले हैं इनका उपयोग भी धनी लोगों द्वारा किया जाता था। दानात्मक अभिलेखों में नगरों में रहने वाले बुनकर, लिपिक, बढ़ई, स्वर्णकार, व्यापारी, अधिकारी के बारे में विवरण मिलता है।

प्रश्न 47.
पाटलिपुत्र के इतिहास पर प्रकाश डालिए।
उत्तर:
पाटलिपुत्र का विकास पाटलिग्राम नामक एक गाँव से हुआ। पाँचवीं सदी ई. पूर्व में मगध शासकों ने अपनी राजधानी राजगाह (राजगीर) से हटाकर इसे बस्ती में लाने का निर्णय किया और इसका नाम पाटलिपुत्र रखा। चौथी शताब्दी ई. पूर्व तक पाटलिपुत्र मौर्य साम्राज्य की राजधानी और एशिया के सबसे बड़े नगरों में से एक बन गया। कालान्तर में इसका महत्त्व कम हो गया। जब सातवीं शताब्दी में चीनी यात्री ह्वेनसांग यहाँ आया, तो उरो यह नगर खंडहर में परिवर्तित मिला।

प्रश्न 48.
छठी शताब्दी ई. पूर्व में उपमहाद्वीप और उसके बाहर के व्यापार मार्ग की विवेचना कीजिए।
उत्तर:
छठी शताब्दी ई. पूर्व से ही उपमहाद्वीप में नदी मार्ग और भूमार्ग विकसित हो चुके थे। ये मार्ग कई दिशाओं में विकसित हो गए थे। मध्य एशिया और उससे भी आगे तक भू-मार्ग थे। समुद्र तट पर बने अनेक बन्दरगाहों से जल मार्ग अरब सागर से होते हुए, उत्तरी अफ्रीका, पश्चिम एशिया तक फैल गया था और बंगाल की खाड़ी से यह मार्ग चीन और दक्षिणपूर्व एशिया तक फैल गया था। राजाओं ने इन मार्गों पर नियन्त्रण करने का प्रयास किया।

प्रश्न 49.
छठी शताब्दी ई. पूर्व में उपमहाद्वीप में व्यापारी वर्ग तथा आयात-निर्यात पर प्रकाश डालिये।
उत्तर:
व्यापारी वर्ग-व्यापारिक मार्गों पर चलने वाले व्यापारियों में पैदल फेरी लगाने वाले व्यापारी तथा बैलगाड़ी और घोड़े खच्चरों के दल के साथ चलने वाले व्यापारी थे। साथ ही समुद्री मार्ग से भी लोग यात्रा करते थे। आयात-निर्यात- नमक, अनाज, कपड़ा, धातु और उससे बनी वस्तुएँ, पत्थर, लकड़ी, जड़ी-बूटी आदि वस्तुएँ एक स्थान से दूसरे स्थान तक पहुँचाई जाती थीं भारत से कालीमिर्च जैसे मसालों, कपड़ों तथा जड़ी-बूटियों का निर्यात रोमन साम्राज्य को किया जाता था।

प्रश्न 50.
‘पेरिप्लस ऑफ एरीशियन सी’ नामक ग्रन्थ में एक यूनानी समुद्री यात्री ने मालाबार तट ( आधुनिक केरल) पर होने वाले आयात-निर्यात का क्या विवरण दिया है?
उत्तर:
यूनानी समुद्री यात्री के अनुसार भारी मात्रा में कालीमिर्च और दालचीनी खरीदने के लिए बाजार वाले नगरों में विदेशी व्यापारी जहाज भेजते हैं। यहाँ भारी मात्रा में सिक्कों, पुखराज, सुरमा, मूँगे, कच्चे शीशे, तांबे, टिन |और सीसे का आयात किया जाता है। इन बाजारों के आस-पास भारी मात्रा में उत्पन्न कालीमिर्च का निर्यात किया जाता है। इसके अतिरिक्त उच्च कोटि के मोतियों, हाथीदाँत, रेशमी वस्त्र, विभिन्न प्रकार के पारदर्शी पत्थरों, हीरों और काले नग तथा कछुए की खोपड़ी का आयात
होता है।

प्रश्न 51.
सिक्कों के प्रचलन से व्यापार और वाणिज्य सम्बन्धी गतिविधियों पर क्या प्रभाव पड़े ?
उत्तर:

  1. व्यापार में विनिमय कुछ सीमा तक आसान हो गया।
  2. बहुमूल्य वस्तु तथा भारी माश में अन्य वस्तुओं का विनिमय किया जाता था।
  3. दक्षिण भारत में अनेक रोमन सिक्कों के प्राप्त होने से पता चलता है कि दक्षिण भारत के रोमन साम्राज्य से व्यापारिक सम्बन्ध थे।
  4. गुप्तकाल में सिक्कों के माध्यम से व्यापार- विनिमय करने में आसानी होती थी जिससे राजाओं को भी लाभ होता था।
  5. यौधेय गणराज्यों ने भी सिक्के चलाए।

प्रश्न 52.
छठी शताब्दी ई. से सोने के सिक्के कम संख्या में मिलने से किन तथ्यों के बारे में जानकारी मिलती है?
उत्तर:
कुछ इतिहासकारों का मत है कि –

  1. इस काल में कुछ आर्थिक संकट उत्पन्न हो गया
  2. रोमन साम्राज्य के पतन के पश्चात् दूरवर्ती व्यापार में कमी आई जिससे उन राज्यों, समुदायों और क्षेत्रों की सम्पन्नता पर प्रभाव पड़ा, जिन्हें दूरवर्ती व्यापार से लाभ मिलता था।
  3. इस काल में नए नगरों और व्यापार के नये तन्त्रों का उदय होने लगा था।
  4. सिक्के इसलिए कम मिलते हैं क्योंकि वे प्रचलन में थे और उनका किसी ने भी संग्रह करके नहीं रखा था।

प्रश्न 53.
अशोककालीन ब्राह्मी लिपि का अर्थ किस प्रकार निकाला गया?
उत्तर:
ब्राह्मी लिपि का प्रयोग अशोक के अधिकांश अभिलेखों में किया गया है। अठारहवीं शताब्दी से यूरोपीय विद्वानों ने भारतीय विद्वानों की सहायता से आधुनिक बंगाली और देवनागरी लिपि में कई पाण्डुलिपियों का अध्ययन आरम्भ किया और उनके अक्षरों की प्राचीन अक्षरों के नमूनों से तुलना शुरू की। विद्वानों ने यह अनुमान लगाया कि से संस्कृत में लिखे हैं, जबकि प्राचीनतम अभिलेख वास्तव में प्राकृत में थे जेम्स प्रिंसेप को अशोककालीन ब्राह्मी लिपि का 1838 ई. में अर्थ निकालने में सफलता मिली।

JAC Class 12 History Important Questions Chapter 2 राजा, किसान और नगर : आरंभिक राज्य और अर्थव्यवस्थाएँ

प्रश्न 54.
विद्वानों द्वारा खरोष्ठी लिपि को कैसे पढ़ा गया?
उत्तर:
पश्चिमोत्तर के अभिलेखों में खरोष्ठी लिपि का प्रयोग किया गया है। इस क्षेत्र में शासन करने वाले हिन्द- यूनानी राजाओं (लगभग द्वितीय- प्रथम शताब्दी ई. पूर्व) द्वारा बनवाए गए सिक्कों में राजाओं के नाम यूनानी और खरोष्ठी में लिखे गए हैं। यूनानी भाषा पढ़ने वाले यूरोपीय विद्वानों ने अक्षरों का मेल किया। चूँकि जेम्स प्रिंसेप ने खरोष्ठी में लिखे अभिलेखों की भाषा की पहचान प्राकृत के रूप में की थी, इसलिए लम्बे अभिलेखों को पढ़ना आसान हो गया।

प्रश्न 55.
अपने अभिलेख में अशोक ने पतिवेदकों को क्या आदेश दिए हैं?
उत्तर:
अपने अभिलेख में अशोक यह कहते हैं, “अतीत में समस्याओं को निपटाने और नियमित रूप से सूचना एकत्र करने की व्यवस्थाएँ नहीं थीं। परन्तु मैंने निम्नलिखित व्यवस्था की है लोगों के समाचार हम तक ‘पतिवेदक’ (संवाददाता) सदैव पहुँचायेंगे। चाहे मैं कहीं भी हूँ, खाना खा रहा हूँ, अन्तःपुर में हूँ, विश्राम कक्ष में हूँ, गोशाला में हूँ, या फिर पालकी में मुझे ले जाया जा रहा हो अथवा वाटिका में हूँ। मैं लोगों के मसलों का निराकरण हर स्थल पर करूँगा।”

प्रश्न 56.
मौर्यकालीन इतिहास के प्रमुख स्रोतों का वर्णन कीजिए।
अथवा
मौर्य साम्राज्य के इतिहास का पुनर्निर्माण करने में इतिहासकारों द्वारा प्रयुक्त विभिन्न साधनों का विवेचन कीजिये।
उत्तर:

  1. अशोक के अभिलेखों से अशोक के शासन, उसके धम्म आदि के बारे में जानकारी मिलती है।
  2. कौटिल्यकृत ‘अर्थशास्त्र’ से मौयों की शासन- व्यवस्था तथा मौर्यकालीन समाज पर प्रकाश पड़ता है।
  3. मेगस्थनीज की पुस्तक ‘इण्डिका’ से चन्द्रगुप्त मौर्य की शासन व्यवस्था आदि के बारे में जानकारी मिलती है।
  4. जैन, बौद्ध पौराणिक ग्रन्थों और मूर्तियों, स्तम्भों आदि से भी मौर्यकालीन इतिहास पर प्रकाश पड़ता है।

प्रश्न 57.
कलिंग युद्ध के परिणामों की विवेचना कीजिए।
उत्तर:

  1. इस युद्ध में लगभग एक लाख सैनिक मारे गए, 11⁄2 लाख बन्दी बनाए गए और इससे भी ज्यादा लोग मौत के मुँह में चले गए।
  2. इस युद्ध के बाद अशोक ने साम्राज्यवादी नीति का परित्याग कर दिया।
  3. अब अशोक ने धम्म का प्रचार-प्रसार करना शुरू कर दिया।
  4. कलिंग युद्ध के बाद अशोक बौद्ध धर्म का अनुयायी बन गया।
  5. इस युद्ध में हुए भारी नर संहार से अशोक को अत्यधिक पश्चाताप हुआ।

प्रश्न 58.
गुप्त शासकों का इतिहास लिखने में सहायक स्त्रोतों का वर्णन कीजिये।
अथवा
गुप्त वंश के इतिहास के प्रमुख स्रोतों का वर्णन कीजिए।
उत्तर:

  1. पुराणों, स्मृतियों आदि से गुप्त वंश के इतिहास पर काफी प्रभाव पड़ता है।
  2. विभिन्न साहित्यिक रचनाओं से गुप्तकालीन शासन पद्धति, सामाजिक रीति-रिवाजों, राजनीतिक अवस्था आदि के बारे में जानकारी मिलती है।
  3. ‘प्रयाग प्रशस्ति’ से समुद्रगुप्त की विजयों, जीवन- चरित्र के बारे में जानकारी मिलती है।
  4. गुप्तकालीन सिक्कों से गुप्त शासकों के धर्म, साम्राज्य की सीमाओं के बारे में जानकारी मिलती है।

प्रश्न 59.
समुद्रगुप्त के चरित्र का मूल्यांकन कीजिए।
उत्तर:

  1. समुद्रगुप्त एक वीर योद्धा, कुशल सेनापति तथा महान विजेता था ।
  2. ‘प्रयाग प्रशस्ति’ के अनुसार समुद्रगुप्त परमात्मा पुरुष हैं। वह साधु की समृद्धि और असाधु के विनाश के कारण हैं। ये उदारता की प्रतिमूर्ति हैं। वे देवताओं में कुबेर, वरुण, इन्द्र और यम के तुल्य हैं।
  3. समुद्रगुप्त योग्य शासक भी थे।
  4. वह उच्च कोटि के विद्वान तथा साहित्य एवं कला के संरक्षक थे।

प्रश्न 60.
गुप्तकालीन सिक्कों पर एक संक्षिप्त टिप्पणी लिखिए।
उत्तर:
गुप्त सम्राटों के अधिकांश सिक्के सोने के बने हैं। परन्तु कुछ गुप्त शासकों ने चाँदी तथा ताँबे के सिक्के भी चलाए। सोने के सबसे भव्य सिक्कों में से कुछ गुप्त – शासकों ने जारी किए। इन सिक्कों के माध्यम से दूर देशों से व्यापार विनिमय करने में आसानी होती थी जिससे शासकों को भी लाभ होता था। स्कन्दगुप्त ने भी सोने तथा चाँदी के सिक्के चलाए। उसके समय में सिक्कों की शुद्धता तथा सुन्दरता में कमी आ गई।

JAC Class 12 History Important Questions Chapter 2 राजा, किसान और नगर : आरंभिक राज्य और अर्थव्यवस्थाएँ

प्रश्न 61.
अशोक के ‘धम्म’ पर संक्षिप्त टिप्पणी लिखें।
अथवा
अशोक के धम्म के मुख्य सिद्धान्तों का वर्णन कीजिये।
उत्तर:
अशोक के धम्म के सिद्धान्त बहुत ही साधारण और सार्वभौमिक थे। उसके अनुसार धम्म के माध्यम से लोगों का जीवन इस लोक में तथा परलोक में अच्छा रहेगा। उसके धम्म के सिद्धान्त थे –

  1. बड़ों के प्रति आदर
  2. संन्यासियों तथा ब्राह्मणों के प्रति उदारता
  3. सेवकों तथा दासों के प्रति उदार व्यवहार
  4. दूसरे के धर्मों और परम्पराओं का आदर
  5. क्रोध, उग्रता, निष्ठुरता, ईर्ष्या तथा अभिमान का परित्याग ।

प्रश्न 62.
मौर्य सम्राट् के अधिकारी क्या-क्या कार्य करते थे?
उत्तर:
मेगस्थनीज के द्वारा लिखे गए विवरण से हमें मौर्य सम्राट् के अधिकारियों के कार्यों का विवरण मिलता है –
साम्राज्य के महान् अधिकारियों में से कुछ नदियों की देख-रेख और भूमि मापन का कार्य करते हैं। कुछ प्रमुख नहरों से उपनहरों के लिए छोड़े जाने वाले पानी के मुखद्वार का निरीक्षण करते हैं ताकि प्रत्येक स्थान पर पानी की पूर्ति समान मात्रा में हो सके। यही अधिकारी शिकारियों का संचालन करते हैं और शिकारियों के कृत्यों के आधार पर उन्हें इनाम या दण्ड देते हैं। वे कर वसूली करते हैं और भूमि से जुड़े सभी व्यवसायों का निरीक्षण करते हैं. साथ ही लकड़हारों, बढ़ई, लोहारों और खननकर्त्ताओं का भी निरीक्षण करते हैं।

प्रश्न 63.
” भारतीय उपमहाद्वीप में ईसा पूर्व छठी शताब्दी से नए परिवर्तन के प्रमाण मिलते हैं।” कथन को स्पष्ट कीजिए।
उत्तर:
भारतीय उपमहाद्वीप में ईसा पूर्व छठी शताब्दी से निम्नलिखित परिवर्तन हुए –
(1) भारतीय उपमहाद्वीप में आरम्भिक राज्यों, साम्राज्यों और रजवाड़ों का तीव्र गति से विकास हुआ है। इन राजनीतिक प्रक्रियाओं के पीछे कुछ अन्य परिवर्तन जिम्मेदार थे। इसके बारे में जानकारी का कृषि उपज को संगठित करने के तरीके से पता चलता है।

(2) इस काल में लगभग सम्पूर्ण उपमहाद्वीप में नए नगरों का उदय हुआ। इतिहासकार इस प्रकार के विकास का आकलन करने के लिए अभिलेखों, ग्रन्थों, सिक्कों तथा चित्रों जैसे विभिन्न प्रकार के स्रोतों का अध्ययन करते हैं।

प्रश्न 64.
महाजनपदों का उद्भव कैसे हुआ? संक्षेप में समझाइए।
उत्तर:
मध्य गंगा घाटी में 1000 ई.पू. के मध्य प्रथमतः लोहे के साक्ष्य प्राप्त होना प्रारम्भ हो जाते हैं। इस काल में लोहे के आरम्भ के फलस्वरूप उन्नत कृषि के औजार तथा हलों का प्रयोग आरम्भ हुआ। इस प्रयोग के कारण प्रचुर मात्रा में खेती की पैदावार हुई। कृषि में इस क्रान्ति के परिणामस्वरूप न केवल स्थायी जीवन को बढ़ावा मिला बल्कि राज्य को भरपूर राजस्व की भी प्राप्ति हुई। अधिक राजस्व की प्राप्ति के कारण राज्य को स्थायी सेना रखना सुगम हो गया। इस स्थायी सेना के माध्यम से राजाओं ने अपने क्षेत्र में कानून व्यवस्था स्थापित की, साथ ही साथ अपने समीपवर्ती क्षेत्रों को विजित करके अपने क्षेत्र तथा राज्य को विस्तृत बनाया। अंततः यही विस्तृत क्षेत्र सोलह महाजनपदों के रूप में स्थापित हुए।

प्रश्न 65.
मौर्य साम्राज्य की स्थापना किसने की? भारतीय इतिहास में मौर्य साम्राज्य की स्थापना का महत्त्व लिखिए।
उत्तर:
चन्द्रगुप्त मौर्य एक महान् विजेता था। चन्द्रगुप्त मौर्य ने 321 ई. पू. में मगध के शासक घनानंद को हराकर मगध को अपने अधिकार में कर लिया। मगध प्रदेश भारत में मौर्य साम्राज्य की स्थापना का आधार बना। मौर्य साम्राज्य की स्थापना के साथ ही भारत में छोटे-मोटे राज्य समाप्त हो गए और उनके स्थान पर एक विशालकाय साम्राज्य की स्थापना हुई। मौर्य साम्राज्य की स्थापना के पूर्व छोटे राज्यों का कोई क्रमबद्ध इतिहास नहीं था। मौर्य साम्राज्य की स्थापना के पश्चात् भारतीय इतिहास का क्रमबद्ध आधार बना मौर्य साम्राज्य के दौरान विदेश व्यापार में खूब उन्नति हुई, भारत का विदेशों से व्यापक सम्पर्क स्थापित हुआ, इसके साथ ही भारत से विदेशी सत्ता का अन्त भी हुआ।

JAC Class 12 History Important Questions Chapter 2 राजा, किसान और नगर : आरंभिक राज्य और अर्थव्यवस्थाएँ

प्रश्न 66.
यूनानी राजदूत मेगस्थनीज द्वारा वर्णित चन्द्रगुप्त मौर्य की सैन्य व्यवस्था पर संक्षेप में टिप्पणी लिखिए।
उत्तर:
यूनानी राजदूत मेगस्थनीज ने चन्द्रगुप्त मौर्य की | सैन्य व्यवस्था का विधिवत वर्णन किया है। मेगस्थनीज के अनुसार सैन्य व्यवस्था के समुचित संचालन हेतु एक समिति और छह उपसमितियों का गठन किया गया था। इन समितियों को पृथक् पृथक् सैन्य गतिविधियों के संचालन की जिम्मेदारी दी गई थी। जैसे एक समिति नौसैनिक गतिविधियों का संचालन करती थी तो दूसरी समिति सैनिकों की भोजन व्यवस्था का संचालन करती थी तीसरी समिति पैदल सेना का संचालन, चौथी समिति अश्वारोहियों की सेना का, पाँचवीं समिति रथारोहियों की सेना तथा छठी समिति हाथियों की सेना का संचालन करती थी।

प्रश्न 67.
मौर्य साम्राज्य केवल 150 वर्षों तक ही चल सका क्यों?
उत्तर:
मौर्य साम्राज्य के केवल 150 वर्षों तक चलने के निम्न कारण हैं –

  1. अशोक की मृत्यु के पश्चात् मौर्य साम्राज्य की बागडोर निर्बल शासकों के हाथ में आई जो मौर्य साम्राज्य के विस्तृत क्षेत्रों को सम्भालने में सक्षम नहीं हुए।
  2. बौद्ध धर्म के अनुयायी होने के कारण अशोक को ब्राह्मण समाज के क्रोध का भाजन बनना पड़ा, वे मौर्य वंश के विरुद्ध हो गए।
  3. कलिंग विजय के पश्चात् अशोक पश्चात्ताप के भाव से भर उठे और उन्होंने अहिंसा की नीति अपनाई।
  4. साम्राज्य की सीमा के अन्तर्गत नियन्त्रण कमजोर होने के कारण अनेक राजे-रजवाड़ों ने अपनी स्वतन्त्र सत्ता घोषित कर दी।
    इस प्रकार धीरे-धीरे मौर्य साम्राज्य का अन्त हो गया।

प्रश्न 68.
दैविक राजा से क्या तात्पर्य है? संक्षिप्त विवरण दीजिए।
उत्तर:
कुषाण शासक ने अपने राज्य को अक्षुण्ण बनाए रखने तथा प्रजा से पूजनीय तथा उच्च स्थिति प्राप्त करने के लिए अपने आपको ईश्वर के रूप में प्रस्तुत किया। इसका सर्वोत्तम प्रमाण उनकी मूर्तियों और सिक्कों से प्राप्त होता है। कई कुषाण शासक अपने नाम के आगे देवपुत्र की उपाधि लगाते थे। अनुमान लगाया जाता है कि उन्होंने चीनी शासकों का अनुसरण किया जो अपने नाम के आगे ‘स्वर्गपुत्र’ की उपाधि लगाते हैं। कुषाण और शकों ने इस बात का प्रचार किया कि राजा को शासन करने का दैवीय अधिकार परमात्मा से प्राप्त है और यह अधिकार वंशानुगत है।

प्रश्न 69
प्रशस्तियों से आप क्या समझते हैं? इलाहाबाद स्तम्भ अभिलेख किसके सम्बन्ध में है?
उत्तर:
प्रशस्तियों का तात्पर्य राजाओं के यशोगान से है। तत्कालीन साहित्यिक कवियों द्वारा अपने राजाओं की प्रशंसा में प्रशस्ति लिखी जाती थी। गुप्त साम्राज्य के सबसे शक्तिशाली सम्राट् समुद्रगुप्त के राजकवि हरिषेण ने प्रयाग प्रशस्ति के नाम से एक रचना की जिसे इलाहाबाद स्तम्भ अभिलेख कहा जाता है। यह प्रशस्ति संस्कृत में लिखी गई थी। हरिषेण ने सम्राट् समुद्रगुप्त की प्रशंसा में लिखा है-

“धरती पर उनका कोई प्रतिद्वन्द्वी नहीं है, अनेक गुणों और शुभ कार्यों से सम्पन्न उन्होंने अपने पैर के तलवे से अन्य राजाओं के यश को मिटा दिया है। वे परमात्मा पुरुष हैं, साधु की समृद्धि और असाधु के विनाश के कारण हैं। वे अजेय हैं। उनके कोमल हृदय को भक्ति और विनय से ही वश में किया जा सकता है। वे करुणा से भरे हुए हैं। वे अनेक सहस्र गायों के दाता हैं। उनके मस्तिष्क की दीक्षा दीन-दुखियों, बिरहिणियों और पीड़ितों के उद्धार के लिए की गई है। वे मानवता के लिए दिव्यमान उदारता की प्रतिमूर्ति हैं। वे देवताओं में कुबेर, वरुण, इन्द्र और यम के तुल्य हैं। ”

प्रश्न 70.
अभिलेखों से भूमिदान के सम्बन्ध में क्या विवरण प्राप्त होते हैं?
उत्तर:
भूमिदान से सम्बन्धित अभिलेख देश के कई हिस्सों से प्राप्त हुए हैं, कहीं-कहीं भूमि के छोटे टुकड़े तो कहीं बड़े-बड़े क्षेत्रों के दान का उल्लेख है। कुछ अभिलेख पत्थरों पर लिखे गए थे। अधिकांश अभिलेख ताम्रपत्रों पर उत्कीर्ण हैं। साधारणतया भूमिदान ब्राह्मणों और सामाजिक संस्थाओं को दिया जाता था। अभिलेख मुख्यतया संस्कृत में लिखे जाते थे, कुछ अभिलेख तमिल और तेलुगु भाषा में भी हैं। कुछ महिलाओं द्वारा भी भूमिदान के साक्ष्य प्राप्त हैं। वाकाटक वंश के राजा की रानी प्रभावती ने ब्राह्मण को भूमिदान किया था।

प्रश्न 71.
नए नगरों के विकास पर संक्षेप में टिप्पणी लिखिए।
उत्तर:
उपमहाद्वीप के विभिन्न क्षेत्रों में छठी शताब्दी ईसा पूर्व कई नगरों का विकास हुआ प्रमुख नगर महाजनपदों की राजधानियों के रूप में विकसित हुए। इन नगरों के विकास में प्राकृतिक रूप से संचार मार्गों की भूमिका रही है। पाटलिपुत्र गंगा नदी के किनारे, दक्षिण भारत में पुहार जैसे महाजनपद समुद्र तट के किनारे जहाँ समुद्री मार्ग प्रारम्भ हुए तथा मथुरा जैसे अनेक शहर व्यावसायिक गतिविधियों के प्रमुख केन्द्र थे। उज्जयिनी जैसे नगर भूतल परिवहन मार्गों से देश के सभी भागों से जुड़ा हुआ था। इसके अतिरिक्त तक्षशिला, कन्नौज, वाराणसी, श्रावस्ती, वैशाली, राजगीर, पैठन, सोपारा, विदिशा, धान्यकंटक, कोडूमनाल जैसे प्रमुख अन्य नए शहर विकसित हुए।

JAC Class 12 History Important Questions Chapter 2 राजा, किसान और नगर : आरंभिक राज्य और अर्थव्यवस्थाएँ

प्रश्न 72.
ब्राह्मी लिपि तथा खरोष्ठी लिपि का अध्ययन कैसे किया गया?
उत्तर:
ब्राह्मी लिपि भारत की सभी भाषा लिपियों की जनक है। यूरोपीय और भारतीय विद्वानों ने बंगाली और देवनागरी शब्दों को पढ़कर उनकी पुराने अभिलेखों के शब्दों से तुलना की आरम्भिक अभिलेखों का अध्ययन करने वाले विद्वानों ने कई बार यह अनुमान लगाया कि इन अभिलेखों की भाषा संस्कृत थी, वास्तव में अभिलेखों की भाषा प्राकृत थी।

1830 के दशक में जेम्स प्रिंसेप ने अथक प्रयासों से अशोककालीन ब्राह्मी लिपि को पढ़ने और उसका अर्थ निकालने में सफलता प्राप्त की। पश्चिमोत्तर भाग से प्राप्त होने वाली खरोष्ठी लिपि को पढ़ने की विधि अलग थी। यूनानी विद्वान् जो यूनानी लिपि को पढ़ने में समर्थ थे, ने खरोष्ठी भाषा के शब्दों को उसके
साथ मिलान करके पढ़ने में सफलता प्राप्त की।

प्रश्न 73.
गुजरात की सुदर्शन झील पर संक्षेप में टिप्पणी लिखिए।
उत्तर:
सुदर्शन झील एक कृत्रिम जलाशय था। हमें इस जलाशय के बारे में जानकारी लगभग दूसरी शताब्दी ई. के संस्कृत के एक पाषाण अभिलेख से होती है। इस अभिलेख में कहा गया है कि जलद्वारों और तटबन्धों वाली इस झील का निर्माण मौर्यकाल में एक स्थानीय राज्यपाल द्वारा किया गया था। लेकिन एक भीषण तूफान के कारण इसके तटबन्ध टूट गए और सारा पानी बह गया। बताया जाता है कि तत्कालीन शासक रुद्रदमन ने इस झील की मरम्मत अपने खर्चे से करवाई थी, और इसके लिए अपनी प्रजा से कर भी नहीं लिया था। इसी पाषाण खण्ड पर एक और अभिलेख है जिसमें कहा गया है कि गुप्त वंश के एक शासक ने एक बार फिर इस झील की मरम्मत करवाई थी।

निबन्धात्मक प्रश्न 

प्रश्न 1.
छठी शताब्दी ई. पूर्व के सोलह महाजनपदों का वर्णन कीजिए।
उत्तर:
छठी शताब्दी ई. पूर्व के सोलह महाजनपद
(1) जनपद-‘जनपद’ का अर्थ एक ऐसा भूखण्ड है, जहाँ कोई जन (लोग, कुल या जनजाति) अपना पाँव रखता है अथवा बस जाता है। बड़ा और शक्तिशाली जनपद ‘महाजनपद’ कहलाता था।

(2) सोलह महाजनपद-छठी शताब्दी ई. पूर्व में भारत में अनेक महाजनपदों का उदय हुआ। बौद्ध और जैन धर्म के आरम्भिक ग्रन्थों में सोलह महाजनपदों का उल्लेख मिलता है। यद्यपि महाजनपदों के नाम की सूची इन ग्रन्थों में एक समान नहीं है, परन्तु वज्जि, मगध, कोशल, कुरु, पांचाल, गान्धार और अवन्ति जैसे नाम प्रायः मिलते हैं। इससे ज्ञात होता है कि उक्त महाजनपद सबसे महत्त्वपूर्ण महाजनपदों में गिने जाते हैं।

छठी शताब्दी ई. पूर्व में भारत में निम्नलिखित सोलह महाजनपद अस्तित्व में थे –

  1. अंग
  2. मगध
  3. क्राशी
  4. कोशल
  5. वज्जि
  6. मल्ल
  7. चेदि
  8. वत्स
  9. कुरु
  10. पांचाल
  11. मत्स्य
  12. शूरसेन
  13. अश्मक
  14. अवन्ति
  15. कम्बोज
  16. गान्धार

(3) शासन व्यवस्था अधिकांश महाजनपदों पर राजा का शासन होता था, परन्तु ‘गण’ और ‘संघ’ के नाम से प्रसिद्ध राज्यों में कई लोगों का समूह शासन करता था। इस समूह का प्रत्येक व्यक्ति राजा कहलाता था। वज्जि संघ की ही भाँति कुछ राज्यों में भूमि सहित अनेक आर्थिक स्रोतों पर राजा गण सामूहिक नियन्त्रण रखते थे।

(4) महाजनपद की राजधानी- प्रत्येक महाजनपद की एक राजधानी होती थी, जो प्रायः किले से घेरी जाती थी। किलेबन्द राजधानियों की देखभाल और प्रारम्भिक सेनाओं और नौकरशाही के लिए भारी आर्थिक स्रोतों की आवश्यकता होती थी। शासकों का कार्य किसानों, व्यापारियों और शिल्पकारों से कर तथा भेंट वसूलना माना जाता था। इसके अतिरिक्त पड़ोसी राज्यों पर आक्रमण करके धन इकट्ठा करना भी सम्पत्ति जुटाने का एक वैध उपाय माना जाता था। धीरे-धीरे कुछ राज्यों ने अपनी स्थायी सेनाएँ और नौकरशाही तन्त्र तैयार कर लिए।

प्रश्न 2.
मौर्य साम्राज्य कितना महत्त्वपूर्ण था? विवेचना कीजिये।
उत्तर:
मौर्य साम्राज्य का महत्त्व उन्नीसवीं शताब्दी में जब इतिहासकारों ने भारत का आरम्भिक इतिहास लिखना शुरू किया, तो उन्होंने मौर्य- साम्राज्य को इतिहास का एक प्रमुख काल स्वीकार किया। इस समय भारत ब्रिटिश साम्राज्य के अधीन एक औपनिवेशिक देश था। उन्नीसवीं तथा बीसवीं सदी के भारतीय इतिहासकारों को प्राचीन भारत में एक ऐसे साम्राज्य की सम्भावना बहुत चुनौतीपूर्ण तथा उत्साहवर्धक लगी।

JAC Class 12 History Important Questions Chapter 2 राजा, किसान और नगर : आरंभिक राज्य और अर्थव्यवस्थाएँ

संक्षेप में मौर्य साम्राज्य के महत्व का विवेचन निम्नलिखित बिन्दुओं के अन्तर्गत किया जा सकता है –
(1) कला के क्षेत्र में उन्नति मौर्य काल में भवन- निर्माण कला तथा मूर्तिकला की महत्त्वपूर्ण उन्नति हुई। खुदाई में प्राप्त मौर्यकालीन सभी पुरातत्व एक उच्चकोटि की कला के प्रमाण हैं।

(2) मौर्य सम्राटों के अभिलेख अशोक ने अनेक शिलाओं, गुफाओं, स्तम्भों आदि पर अभिलेख उत्कीर्ण करवाये। ये अभिलेख जनता की भलाई के लिए थे। इतिहासकारों के अनुसार इन अभिलेखों पर लिखे सन्देश अन्य शासकों के अभिलेखों से भिन्न थे।

(3) अशोक की महानता इतिहासकारों के अनुसार अन्य शासकों की अपेक्षा सम्राट अशोक एक बहुत शक्तिशाली और न्यायप्रिय शासक था। वह परवर्ती राजाओं की अपेक्षा विनीत और नम्र भी था, जो अपने नाम के साथ बड़ी बड़ी उपाधियाँ जोड़ते थे इसलिए बीसवीं शताब्दी के राष्ट्रवादी नेताओं ने भी अशोक को प्रेरणा का स्रोत माना

(4) मौर्य साम्राज्य की कमजोरियाँ – यद्यपि मौर्य- साम्राज्य अत्यन्त गौरवशाली था, परन्तु उसमें कुछ कमजोरियाँ भी थीं जिनका वर्णन निम्नानुसार है-

  • मौर्य साम्राज्य का केवल 150 वर्ष तक ही अस्तित्व में रहना- मौर्य साम्राज्य केवल 150 वर्ष तक ही अस्तित्व में रहा। इसे भारतीय उपमहाद्वीप के इस लम्बे इतिहास में बहुत बड़ा काल नहीं माना जा सकता।
  • मौर्य साम्राज्य का सम्पूर्ण उपमहाद्वीप में विस्तार न होना- मौर्य साम्राज्य का उपमहाद्वीप के सभी क्षेत्रों में विस्तार नहीं हो सका साम्राज्य की सीमा के अन्तर्गत भी सभी प्रदेशों पर नियन्त्रण एकसमान नहीं था अतः दूसरी शताब्दी ई. पूर्व तक उपमहाद्वीप के अनेक भागों में नये-नये शासकों और रजवाड़ों की स्थापना होने लगी।

प्रश्न 3.
दक्षिण भारत में सरदारों और सरदारियों के उद्भव की विवेचना कीजिए।
उत्तर:
दक्षिण भारत में सरदारों और सरदारियों का उद्भव 600 ई. पूर्व से 600 ई. के काल में भारत के दक्कन और उससे दक्षिण के क्षेत्र में स्थित तमिलकम अर्थात् तमिलनाडु, आंध्रप्रदेश एवं केरल के कुछ भाग में चोल, चेर और पाण्ड्य जैसी सरदारियों का उदय हुआ ये राज्य बहुत ही समृद्ध और स्थायी सिद्ध हुए।
(1) सरदार और सरदारी सरदार एक शक्तिशाली व्यक्ति होता था जिसका पद वंशानुगत भी हो सकता था और नहीं भी हो सकता था। उसके सहयोगी और समर्थक उसके वंश के लोग होते थे सरदारों के प्रमुख कार्य थे –

  •  विशेष अनुष्ठानों का संचालन करना
  • युद्ध के समय सेना का नेतृत्व करना
  • विवादों का निपटारा करने में मध्यस्थता की भूमिका निभाना सरदार अपनी अधीन प्रजा से भेंट लेता था और उसे अपने समर्थकों में बाँट देता था। राजा लोग विभिन्न करों की वसूली करते थे सरदारी में प्राय: कोई स्थायी सेना या अधिकारी नहीं होते थे।

(2) जानकारी के स्रोत हमें दक्षिण के इन राज्यों के विषय में विभिन्न प्रकार के स्रोतों से जानकारी मिलती है। उदाहरणार्थ- प्राचीन तमिल संगम ग्रन्थों में संकलित कविताएँ सरदारों का विवरण देती हैं।

(3) व्यापार द्वारा राजस्व प्राप्त करना-दक्षिण भारत के अनेक सरदार और राजा लम्बी दूरी के व्यापार द्वारा राजस्व प्राप्त करते थे इन राजाओं में मध्य और पश्चिम भारत के क्षेत्रों पर शासन करने वाले सातवाहन (लगभग द्वितीय शताब्दी ई. पूर्व से द्वितीय शताब्दी ईसवी तक ) और उपमहाद्वीप के पश्चिमोत्तर तथा पश्चिम में शासन करने वाले मध्य एशियाई मूल के शक शासक सम्मिलित थे।

(4) उच्च सामाजिक दर्जे का दावा करना – यद्यपि सातवाहनों की जाति के बारे में विद्वानों में मतभेद है, परन्तु सत्तारूढ़ होने के बाद उन्होंने उच्च सामाजिक दर्जे का दावा कई प्रकार से किया। अधिकांश विद्वानों के अनुसार सातवाहन ब्राह्मणवंशीय थे।

प्रश्न 4.
छठी शताब्दी ई. पूर्व से छठी शताब्दी ईसवी तक के काल में भारत के राजाओं तथा ग्रामीण प्रजा के बीच सम्बन्धों की विवेचना कीजिये।
उत्तर:
राजाओं तथा ग्रामीण प्रजा के बीच सम्बन्ध छठी शताब्दी ई. पूर्व से छठी शताब्दी ईसवी तक के काल में भारत में राजाओं तथा ग्रामीण प्रजा के बीच पाए जाने वाले सम्बन्धों के बारे में जातकों तथा पंचतंत्र जैसे ग्रन्थों से पर्याप्त जानकारी मिलती है। जातक कथाओं की रचना पहली सहस्राब्दी ई. के मध्य में पालि भाषा में की गई थी। ग्रामीण प्रजा की दयनीय दशा- जातक कथाओं से पता चलता है कि इस काल में ग्रामीण जनता की दशा बड़ी दयनीय थी। ‘गंदतिन्दु’ नामक जातक में वर्णित एक कथा में बताया गया है कि राजा की प्रजा बड़ी दुःखी थी। इन दुःखी लोगों में वृद्ध महिलाएँ, पुरुष, किसान, पशुपालक, ग्रामीण बालक आदि सम्मिलित थे।

एक बार राजा वेश बदलकर लोगों के बीच में यह जानने के लिए गया कि लोग उसके बारे में क्या सोचते हैं। उन सभी लोगों ने अपने दुःखों के लिए राजा को उत्तरदायी ठहराया तथा उसकी कटु आलोचना की। उन्होंने अपना दुःख व्यक्त करते हुए राजा को बताया कि रात्रि में डाकू लोग उन पर हमला करते थे तथा दिन में कर एकत्रित करने वाले अधिकारी उन्हें प्रताड़ित करते थे। इस परिस्थिति से मुक्ति पाने के लिए लोग अपने-अपने गाँव छोड़कर जंगलों में बस गए। राजा और प्रजा के बीच तनावपूर्ण सम्बन्ध-उपर्युक्त कथा से ज्ञात होता है कि इस काल में राजा और प्रजा, विशेषकर ग्रामीण प्रजा के बीच सम्बन्ध तनावपूर्ण रहते थे।

JAC Class 12 History Important Questions Chapter 2 राजा, किसान और नगर : आरंभिक राज्य और अर्थव्यवस्थाएँ

इसका प्रमुख कारण यह था कि शासक अपने राजकोष को भरने के लिए लोगों से विभिन्न प्रकार के करों की माँग करते थे। इससे किसानों में पोर असन्तोष व्याप्त रहता था। इस जातक कथा से ज्ञात होता है कि इस संकट से बचने के लिए ग्रामीण लोग अपने घरों को छोड़ कर जंगलों में भाग जाते थे। इस प्रकार इस काल की जनता में राजा की छवि बड़ी खराब थी क्योंकि वे जनता का शोषण करते थे और उसके कष्टों एवं समस्याओं को दूर करने के लिए प्रयत्न नहीं करते थे।

प्रश्न 5.
छठी शताब्दी ई. पूर्व से छठी शताब्दी ईसवी तक के काल में किये जाने वाले भूमिदानों का वर्णन कीजिये। भूमिदानों के क्या प्रभाव हुए?
उत्तर:
भूमिदान प्राचीन भारत में शासकों, सामन्तों एवं धन-सम्पन्न लोगों द्वारा भूमिदान किये जाते थे अनेक भूमिदानों का उल्लेख अभिलेखों में मिलता है। इनमें से कुछ अभिलेख पत्थरों पर लिखे गए थे परन्तु अधिकांश अभिलेख ताम्र- पत्रों पर उत्कीर्ण होते थे जिन्हें उन लोगों को प्रमाण रूप में दिया जाता था, जो भूमिदान लेते थे।
(1) प्रभावती गुप्त द्वारा भूमिदान देना प्रभावती गुप्त गुप्त वंश के प्रसिद्ध सम्राट चन्द्रगुप्त द्वितीय ( लगभग 375-415 ई.) की पुत्री थी। उसका विवाह दक्कन पठार के वाकाटक वंश के शासक रुद्रसेन द्वितीय के साथ हुआ था प्रभावती गुप्त ने दंगुन गाँव को भूमिदान के रूप में प्रसिद्ध आचार्य चनालस्वामी को दान किया था।

(2) भूमिदान अभिलेख से ग्रामीण प्रजा की जानकारी प्रभावती गुप्त के भूमिदान अभिलेख से हमें ग्रामीण प्रजा के बारे में जानकारी मिलती है। उस समय गाँवों में ब्राह्मण, किसान तथा अन्य प्रकार के वर्गों के लोग रहते थे। ये लोग शासकों या उनके प्रतिनिधियों को अनेक प्रकार की वस्तुएँ प्रदान करते थे।

(3) भूमिदान की भूमि में अन्तर भूमिदान की भूमि की माप एक जैसी नहीं थी। कुछ लोगों को भूमि के छोटे- छोटे टुकड़े दिए गए थे तो कुछ लोगों को बड़े-बड़े क्षेत्र दान में दिए गए थे। इसके अतिरिक्त भूमिदान में दान प्राप्त करने वाले लोगों के अधिकारों में भी क्षेत्रीय परिवर्तन मिलते हैं।

(4) भूमिदान के प्रभाव-भूमिदान के निम्नलिखित प्रभाव हुए –

  • कुछ इतिहासकारों के अनुसार भूमिदान शासक- वंश के द्वारा कृषि को नये क्षेत्रों में प्रोत्साहित करने की एक रणनीति थी।
  • कुछ इतिहासकारों का मत है कि भूमिदान से दुर्बल होते हुए राजनीतिक प्रभुत्व की जानकारी मिलती है।
  • कुछ इतिहासकारों के अनुसार राजा स्वयं को उत्कृष्ट स्तर के मानव के रूप में प्रदर्शित करना चाहते थे।

प्रश्न 6.
छठी शताब्दी ई. पूर्व के नये नगरों तथा नगरों में रहने वाले सम्भ्रान्त वर्ग और शिल्पकारों का वर्णन कीजिये।
उत्तर:
(1) छठी शताब्दी ई. पूर्व के नये नगर- छठी शताब्दी ई. पूर्व में भारत के विभिन्न क्षेत्रों में नये नगरों का उदय हुआ। इनमें से अधिकांश नगर महाजनपदों की राजधानियाँ थीं। प्रायः सभी नगर संचार मार्गों के किनारे बसे थे। पाटलिपुत्र जैसे कुछ नगर नदी मार्ग के किनारे बसे थे। उज्जयिनी जैसे अन्य नगर भूतल मार्गों के किनारे बसे थे। इसके अतिरिक्त पुरहार जैसे नगर समुद्रतट पर स्थित थे, जहाँ से समुद्री मार्ग प्रारम्भ हुए मथुरा जैसे अनेक नगर व्यावसायिक, सांस्कृतिक और राजनीतिक गतिविधियों के प्रमुख केन्द्र थे।

(2) नगरों में रहने वाले सम्भ्रान्त वर्ग के लोग और शिल्पकार – शासक वर्ग और राजा किलेबन्द नगरों में रहते थे। खुदाई में इन नगरों में उत्कृष्ट श्रेणी के मिट्टी के कटोरे तथा थालियाँ मिली हैं, जिन पर चमकदार कलई चढ़ी हुई है। इन्हें ‘उत्तरी कृष्ण मार्जित पात्र’ कहा जाता है। सम्भवतः इन पात्रों का उपयोग धनी लोग किया करते होंगे। इसके अतिरिक्त सोने, चाँदी, कांस्य, ताँबे, हाथीदांत, शीशे जैसे विभिन्न प्रकार के पदार्थों के बने आभूषण, उपकरण, हथियार, वर्तन आदि मिले हैं। इनका उपयोग भी अमीर लोग करते थे। दानात्मक अभिलेखों में नगरों में रहने वाले बुनकर, लिपिक, बढ़ई, कुम्हार, स्वर्णकार, लौहकार, धोबी, अधिकारी, धार्मिक गुरु, व्यापारी आदि शिल्पकारों के बारे में जानकारी मिलती है।

(3) श्रेणी – शिल्पकारों और व्यापारियों ने अपने संघ बना लिए थे, जिन्हें ‘श्रेणी’ कहा जाता है। ये श्रेणियाँ सम्भवतः पहले कच्चे माल को खरीदती थीं, फिर उनसे सामान तैयार कर बाजार में बेच देती थीं। यह सम्भव है कि शिल्पकारों ने नगरों में रहने वाले सम्भ्रान्त वर्ग के लोगों की बढ़ती मांग को पूरा करने के लिए कई प्रकार के लौह उपकरणों का प्रयोग किया हो।

JAC Class 12 History Important Questions Chapter 2 राजा, किसान और नगर : आरंभिक राज्य और अर्थव्यवस्थाएँ

प्रश्न 7.
प्राचीन काल में भारतीय शासकों द्वारा प्रचलित सिक्कों का वर्णन कीजिये। सिक्कों के व्यापार और वाणिज्य सम्बन्धी गतिविधियों पर क्या प्रभाव पड़े?
उत्तर:
प्राचीन काल में भारतीय शासकों द्वारा प्रचलित सिक्के प्राचीन काल में भारतीय शासकों द्वारा सोने, चाँदी, ताँबे आदि के सिक्के जारी किये गए। छठी शताब्दी ई. पूर्व में चाँदी और ताँबे के आहत सिक्कों का सबसे पहले प्रयोग किया गया। सम्पूर्ण भारत में उत्खनन में इस प्रकार के सिक्के मिले हैं।

(1) कुषाण राजाओं द्वारा प्रचलित सिक्के सोने के सिक्के सबसे पहले प्रथम शताब्दी ईसवी में कुषाण राजाओं द्वारा जारी किए गए थे। उत्तर और मध्य भारत के अनेक पुरास्थलों पर कुषाणों द्वारा प्रचलित सिक्के मिले हैं। सोने के सिक्कों के व्यापक प्रयोग से पता चलता है कि बहुमूल्य वस्तुओं तथा भारी मात्रा में अन्य वस्तुओं का विनिमय किया जाता था। इसके अतिरिक्त दक्षिण भारत के अनेक पुरास्थलों से बड़ी संख्या में रोमन सिक्के मिले हैं। इनसे जात होता है कि दक्षिण भारत के रोमन साम्राज्य से व्यापारिक सम्बन्ध स्थापित थे।

(2) यौधेय शासकों द्वारा जारी किए गए सिक्के- पंजाब और हरियाणा के यौधेय (प्रथम शताब्दी ई.) कबायली गणराज्यों ने भी सिक्के जारी किये थे उत्खनन में यौधेय शासकों द्वारा जारी किए गए ताँबे के सिक्के हजारों की संख्या में मिले हैं, जिनसे पता चलता है कि यौधेय शासकों की व्यापारिक गतिविधियों में बड़ी रुचि और सहभागिता थी।

(3) गुप्त शासकों द्वारा जारी किये गए सिक्के-सोने के सबसे उत्कृष्ट सिक्कों में से कुछ सिक्के गुप्त शासकों द्वारा जारी किए गए थे। गुप्त शासकों के आरम्भिक सिक्कों में प्रयुक्त सोना अत्यन्त उत्कृष्ट कोटि का था। सोने के कम सिक्के मिलने के कारण छठी शताब्दी ई. से सोने के सिक्के मिलने कम हो गए। इसके कारण निम्नलिखित थे प्रमुख

  • कुछ इतिहासकारों के अनुसार रोमन साम्राज्य के पतन के बाद दूरवर्ती व्यापार में कमी आई जिससे इन राज्यों समुदायों और प्रदेशों की सम्पन्नता क्षीण हो गई, जिन्हें दूरवर्ती व्यापार से लाभ मिलता था।
  • कुछ अन्य इतिहासकारों का मत है कि इस काल में नये नगरों और व्यापार के नवीन तन्त्रों का उदय होने लगा था।
  • यद्यपि इस काल के सोने के सिक्कों का मिलना तो कम हो गया, परन्तु अभिलेखों और ग्रन्थों में सिक्कों का उल्लेख होता रहा है।

प्रश्न 8.
“अशोक भारत का एक महान सम्राट था।” इस कथन की विवेचना कीजिए।
अथवा
अशोक को महान क्यों कहा जाता है ?
उत्तर:
अशोक की गिनती भारत के महान सम्राटों में की जाती है।
अशोक के महान् सम्राट कहे जाने के आधार –
(1) महान् विजेता- गद्दी पर बैठते ही अशोक ने अपने पितामह चन्द्रगुप्त मौर्य की भाँति साम्राज्यवादी नीति अपनाई। कल्हण की ‘राजतरंगिणी’ से ज्ञात होता है कि अशोक ने कश्मीर को जीत कर मौर्य साम्राज्य में सम्मिलित कर लिया। अशोक के अभिलेखों से ज्ञात होता है कि अशोक ने कलिंग को जीतकर उसे अपने साम्राज्य में सम्मिलित कर लिया। इस प्रकार अशोक ने अपने पूर्वजों से प्राप्त मौर्य साम्राज्य को सुरक्षित ही नहीं रखा, बल्कि उसका विस्तार भी किया।

(2) योग्य शासक अशोक केवल एक विजेता ही नहीं, अपितु एक योग्य शासक भी था वह एक प्रजावत्सल सम्राट था। वह अपनी प्रजा को अपनी सन्तान के तुल्य समझता था तथा उसकी नैतिक और भौतिक उन्नति के लिए सदैव प्रयत्नशील रहता था। उसने प्रशासनिक क्षेत्र में अनेक सुधार किये और अपनी प्रशासकीय प्रतिभा का परिचय दिया।

(3) धम्म – विजेता- कलिंग युद्ध के बाद अशोक ने साम्राज्यवादी एवं युद्धवादी नीति का परित्याग कर दिया। उसने बुद्ध घोष के स्थान पर धम्म घोष की नीति अपनाई । उसने धम्म के प्रचार के लिए अपना तन-मन-धन समर्पित कर दिया।

(4) बौद्ध धर्म को संरक्षण देना बौद्ध धर्म के प्रचारक एवं संरक्षक के रूप में भी प्राचीन भारतीय इतिहास में कि अशोक का महत्त्वपूर्ण स्थान है। उसने बौद्ध धर्म के सिद्धान्तों ल को शिलाओं, स्तम्भों पर उत्कीर्ण करवाया। उसने भारत के च विभिन्न प्रदेशों एवं विदेशों में बौद्ध धर्म के प्रचार के लिए अनेक धर्म प्रचारक भेजे।

(5) कला का संरक्षक अशोक कला प्रेमी था। उसने रा अनेक राजप्रासादों, स्तम्भों, स्तूपों, विहारों आदि का निर्माण ि करवाया। उसके द्वारा निर्मित पाषाण स्तम्भ मौर्यकालीन कला के सर्वोत्कृष्ट नमूने हैं। बौद्धग्रन्थों के अनुसार अशोक ने 84,000 स्तूपों का निर्माण करवाया।

(6) अशोक की चारित्रिक विशेषताएँ- अशोक के स अभिलेखों का अध्ययन करने वाले इतिहासकारों को यह प्रतीत हुआ कि अन्य राजाओं की अपेक्षा अशोक एक बहुत से ही शक्तिशाली तथा परिश्रमी शासक था। इसके अतिरिक्त वह बाद के उन राजाओं की अपेक्षा विनीत भी था जो अपने नाम के साथ बड़ी-बड़ी उपाधियाँ जोड़ते थे। इसलिए बीसवीं सदी के राष्ट्रवादी नेताओं ने अशोक को प्रेरणा का स्रोत माना है।

JAC Class 12 History Important Questions Chapter 2 राजा, किसान और नगर : आरंभिक राज्य और अर्थव्यवस्थाएँ

प्रश्न 9.
मौयों की आर्थिक व्यवस्था का वर्णन कीजिए।
उत्तर:
मौर्यों की आर्थिक व्यवस्था मौर्यो की आर्थिक व्यवस्था का विवेचन निम्नलिखित बिन्दुओं के अन्तर्गत किया जा सकता है –
(1) कृषि- इस काल के लोगों का मुख्य व्यवसाय कृषि था। कृषकों को राज्य का संरक्षण प्राप्त था। युद्ध के समय भी कृषकों को हानि नहीं पहुंचाई जाती थी। खेती हल और बैलों से की जाती थी। राज्य की ओर से सिंचाई की सुविधा दी जाती थी। अकाल और बाढ़ के समय कृषकों की सहायता की जाती थी मौर्य काल में गेहूं, जौ, चावल, चना, उड़द, मूंग, मसूर, सरसों, कपास, बाजरा आदि की खेती की जाती थी। मेगस्थनीज के अनुसार भारत के कृषक सदा प्रायः दो फसलें काटते थे। चन्द्रगुप्त मौर्य के गवर्नर पुष्यगुप्त ने सौराष्ट्र में सुदर्शन झील का निर्माण करवाया था। इससे ज्ञात होता है कि शासक सिंचाई के साधनों के विकास के लिए सदैव प्रयत्नशील रहते थे।

(2) पशुपालन इस युग में पशुपालन भी एक प्रमुख व्यवसाय था। पशुओं में गाय-बैल, भेड़-बकरी, भैंस, गधे, ऊँट, सूअर, कुत्ते आदि प्रमुखता से पाले जाते थे।

(3) उद्योग-धन्धे इस काल में अनेक उद्योग-धन्धे विकसित थे। इस युग में सूती, ऊनी, रेशमी सभी प्रकार के वस्व जुने जाते थे। वस्यों का निर्यात विदेशों को किया जाता था। उच्चकोटि की मलमल बड़ी मात्रा में दक्षिण भारत से आयात की जाती थी। इस युग में विभिन्न प्रकार के अस्त्र-शस्त्रों का निर्माण किया जाता था। इस समय जलपोतों का भी निर्माण होने लगा था, जिससे व्यापार का काफी विकास हुआ था। चमड़े का उद्योग भी उन्नत था।

(4) श्रेणियाँ मौर्यकाल में शिल्पियों तथा व्यवसायियों की अपनी श्रेणियाँ (संगठन) होती थीं। इन श्रेणियों को राज्य की ओर से सहायता दी जाती थी और उन्हें सरकारी नियमों के अनुसार कार्य करना पड़ता था।

(5) व्यापार वाणिज्य मौर्यकाल में आन्तरिक और वैदेशिक व्यापार दोनों उन्नत अवस्था में थे। बड़े-बड़े नगर सड़कों से जुड़ गए थे। आन्तरिक व्यापार के लिए देश में सुरक्षित स्थल मार्ग थे। इनमें पाटलिपुत्र से तक्षशिला तक जाने वाला राजपथ अत्यधिक महत्त्वपूर्ण था। राज्य की ओर से मार्गों की सुरक्षा का प्रबन्ध किया जाता था। आन्तरिक व्यापार देश की नदियों के मार्ग से भी होता था। मौर्य युग में विदेशी व्यापार भी उन्नत था लंका, बर्मा, पश्चिमी एशिया, मिस्र आदि के साथ व्यापारिक सम्बन्ध थे भारत से मिस्र को हाथी, कछुए सीपियाँ, मोती, रंग, नील और बहुमूल्य लकड़ी निर्यात होती थी।

(6) मुद्रा – मौर्यकाल में नियमित सिक्कों का प्रचलन हो गया था। सिक्के सोने, चाँदी तथा तांबे के बने होते थे। ‘अर्थशास्त्र’ में सुवर्ण (स्वर्ण मुद्रा), कार्यापण, पण या धरण (चाँदी के सिक्के), माषक (ताँबे के सिक्के), काकणी (ताम्र मुद्रा) आदि का उल्लेख मिलता है। मुद्राओं को सरकारी टकसालों में ढाला जाता था।

प्रश्न 10.
समुद्रगुप्त के चरित्र एवं व्यक्तित्व पर प्रकाश डालिए।
अथवा
समुद्रगुप्त के चरित्र का मूल्यांकन कीजिए। उत्तर- समुद्रगुप्त का चरित्र एवं व्यक्तित्व (मूल्यांकन) –
समुद्रगुप्त प्राचीन भारत का एक महान् सम्राट था। उसके चरित्र एवं व्यक्तित्व की प्रमुख विशेषताओं का वर्णन के निम्नानुसार किया गया है –
(1) गुणसम्पन्न व्यक्ति समुद्रगुप्त एक गुणसम्पन्न व्यक्ति था। वह उदार, दयालु और दानशील व्यक्ति था। वह साधु की समृद्धि और असाधु के विनाश का कारण था।

(2) वीर योद्धा तथा महान विजेता-समुद्रगुप्त एक ये वीर योद्धा, कुशल सेनापति तथा महान विजेता था। उसके कि सिक्कों पर अंकित ‘पराक्रमांक’, ‘व्याघ्रपराक्रम’ जैसी उपाधियाँ न्ता उसके अद्वितीय पराक्रम तथा शूरवीरता की परिचायक हैं।

(3) योग्य शासक समुद्रगुप्त एक योग्य शासक भी था। उसका प्रशासन अत्यन्त सुदृढ़ एवं सुव्यवस्थित था। वह एक प्रजावत्सल शासक था तथा अपनी प्रजा की नैति एवं भौतिक उन्नति के लिए सदैव प्रयत्नशील रहता था।

(4) कुशल राजनीतिज्ञ समुद्रगुप्त एक कुश राजनीतिज्ञ भी था। उसने देशकाल के अनुकूल भिन्न-भिन्न नीतियों का अनुसरण किया। जहाँ उसने आर्यावर्त के राज्य को जीतकर उन्हें अपने साम्राज्य में सम्मिलित किया, वह उसने दक्षिणापथ के राजाओं को पराजित कर उनके राज्य वापस लौटा दिये तथा उनसे केवल कर लेना ही उचि
समझा।

JAC Class 12 History Important Questions Chapter 2 राजा, किसान और नगर : आरंभिक राज्य और अर्थव्यवस्थाएँ

(5) साहित्य का संरक्षक समुद्रगुप्त साहित्य क संरक्षक था। वह स्वयं एक उच्च कोटि का विद्वान था औ विद्वानों, कवियों तथा साहित्यकारों का संरक्षक था। समुद्रगुप्त कविता भी करता था और अपनी काव्य निपुणता के कारण ‘कविराज’ की उपाधि से प्रसिद्ध था।

(6) कला का संरक्षक समुद्रगुप्त कला का भी संरक्षक था संगीत से उसे विशेष प्रेम था। वह स्वयं एक अच्छा संगीतज्ञ था। उसके कुछ सिक्कों पर उसे वीणा बजाते हुए दिखाया गया है, जिससे सिद्ध होता है कि वह संगीत प्रेमी था।

(7) धर्म-सहिष्णु समुद्रगुप्त वैष्णव धर्म का अनुवायी था वैष्णव धर्म का प्रबल समर्थक होते हुए भी उसका धार्मिक दृष्टिकोण उदार तथा न्यायपरक था उसने अपने राजदरबार में प्रसिद्ध बौद्ध विद्वान वसुबन्धु को आश्रय प्रदान किया था।

प्रश्न 11.
अभिलेखशास्त्री एवं इतिहासकार अशोक के अभिलेखों से ऐतिहासिक साक्ष्य किस प्रकार प्राप्त करते हैं?
उत्तर:
अभिलेखों से ऐतिहासिक साक्ष्य प्राप्त करना –
(1) अभिलेखों का परीक्षण करना अशोक के एक अभिलेख में यह लिखा है कि “राजन देवानामपिय पियदस्सी यह कहते हैं।” इस अभिलेख में शासक अशोक का नाम नहीं लिखा है। इसमें अशोक द्वारा अपनाई गई उपाधियों (‘देवानाम्पिय’ तथा ‘पियदस्सी) का प्रयोग किया गया है परन्तु अन्य अभिलेखों में अशोक का नाम मिलता है।

जिसमें उसकी उपाधियाँ भी लिखी हैं। इन अभिलेखों का परीक्षण करने के पश्चात् अभिलेखशास्त्रियों ने पता लगाया है कि उनके विषय, शैली, भाषा और पुरालिपि विज्ञान, सबमें समानता है। अतः उन्होंने यह निष्कर्ष निकाला कि इन अभिलेखों को एक ही शासक ने बनवाया था। अभिलेखशास्त्रियों तथा इतिहासकारों को इन अभिलेखों का अध्ययन करने के बाद यह पता लगाना पड़ता है कि अशोक ने अपने अभिलेखों में जो कहा है, वह कहाँ तक सही है।

(2) ब्रैकेट में लिखे शब्दों का अर्थ ढूँढ़ना- अशोक के कुछ अभिलेखों में कुछ शब्द ब्रैकेट में लिखे हुए हैं जैसे एक अभिलेख में अशोक कहता है कि “मैंने निम्नलिखित (व्यवस्था) की है ………….. ” इसी प्रकार अशोक ने अपने तेरहवें शिलालेख में कलिंग युद्ध मैं हुए विनाश का वर्णन करते हुए कहा है कि “इस युद्ध में एक लाख व्यक्ति मारे गए, डेढ़ लाख व्यक्ति बन्दी बनाए गए तथा इससे भी कई गुना व्यक्ति (महामारी, भुखमरी आदि) से मारे गए।” अभिलेखशास्त्री प्रायः वाक्यों के अर्थ स्पष्ट करने के लिए इन ब्रैकेटों का प्रयोग करते हैं। इसमें अभिलेखशास्त्रियों को बड़ी सावधानी बरतनी पड़ती है जिससे कि लेखक का मूल अर्थ बदल न जाए।

(3) युद्ध के प्रति मनोवृत्ति में परिवर्तन को दर्शाना- अशोक के तेरहवें शिलालेख से कलिंग के युद्ध के बारे में जानकारी मिलती है। इससे अशोक की वेदना प्रकट होती है। इसके साथ ही यह युद्ध के प्रति अशोक की मनोवृत्ति में परिवर्तन को भी दर्शाता है।

प्रश्न 12.
“अभिलेखों से प्राप्त जानकारी की भी सीमा होती है।” विवेचना कीजिए
अथवा
“राजनीतिक और आर्थिक इतिहास का पूर्ण ज्ञान मात्र अभिलेख शास्त्र से ही नहीं मिलता है।” विवेचना कीजिये।
उत्तर:
अभिलेखों से प्राप्त जानकारी की सीमा प्राचीन भारतीय इतिहास के निर्माण के लिए अभिलेखों से प्राप्त जानकारी पर्याप्त नहीं होती है। अभिलेखों से प्राप्त जानकारी की भी सीमा होती है। कभी-कभी इसकी तकनीकी सीमा होती है जैसे अक्षरों को हल्के ढंग से उत्कीर्ण किया जाता है, जिन्हें पढ़ पाना कठिन होता है। इसके अतिरिक्त -अभिलेख नष्ट भी हो सकते हैं, जिनसे अक्षर लुप्त हो जाते हैं। अभिलेखों के शब्दों के वास्तविक अर्थ के बारे में भी पूर्ण रूप से जानकारी प्राप्त करना भी काफी कठिन होता है क्योंकि कुछ अर्थ किसी विशेष स्थान या समय से सम्बन्धित होते हैं।

खुदाई में कई हजार अभिलेख प्राप्त हुए हैं। परन्तु सभी के अर्थ नहीं निकाले जा सके हैं। इसके अतिरिक्त ऐसे अनेक अभिलेख भी रहे होंगे, जो कालान्तर में सुरक्षित नहीं बचे हैं। इसलिए जो अभिलेख अभी उपलब्ध हैं, वे सम्भवतः कुल अभिलेखों के अंशमात्र हैं। यह भी आवश्यक नहीं है कि जिसे हम आज राजनीतिक और आर्थिक रूप से महत्त्वपूर्ण मानते हैं, उन्हें अभिलेखों में अंकित ही किया गया हो।

उदाहरणार्थ, खेती की वन दैनिक प्रक्रियाएँ और जीवन के हर रोज के सुख-दुःख का उल्लेख अभिलेखों में नहीं मिलता है क्योंकि प्रायः अभिलेख पर बड़े और विशेष अवसरों का वर्णन करते हैं। इसके अतिरिक्त अभिलेख सदैव उन्हीं लोगों के विचारों को प्रकट करते प हैं, जो उन्हें बनवाते थे। इसलिए इन अभिलेखों में प्रकट किये गए विचारों की समीक्षा अन्य विचारों के परिप्रेक्ष्य में करनी होगी ताकि अपने अतीत का उचित ज्ञान हो सके। इस प्रकार स्पष्ट है कि राजनीतिक और आर्थिक इतिहास त का पूर्ण ज्ञान मात्र अभिलेख शास्त्र में ही नहीं मिलता है।

JAC Class 12 History Important Questions Chapter 2 राजा, किसान और नगर : आरंभिक राज्य और अर्थव्यवस्थाएँ

प्रश्न 13.
प्रारम्भिक राज्यों के रूप में महाजनपदों के राजनीतिक इतिहास का विस्तार से वर्णन कीजिए।
उत्तर:
प्रारम्भिक राज्यों के रूप में महाजनपदों के राजनीतिक इतिहास का वर्णन निम्न बिन्दुओं के अन्तर्गत किया जा सकता है –
(1) प्रारम्भिक भारतीय इतिहास में छठी शताब्दी ई.पू. को एक महत्त्वपूर्ण परिवर्तनकारी काल माना जाता है। इस काल का सम्बन्ध प्रायः आधुनिक राज्यों, नगरों, लोहे के बढ़ते प्रयोग एवं सिक्कों के विकास के साथ स्थापित किया जाता है। इसी काल के दौरान बौद्ध एवं जैन धर्म सहित विभिन्न दार्शनिक विचारधाराओं का जन्म एवं विकास हुआ।

बौद्ध एवं जैन धर्म के प्रारम्भिक ग्रन्थों में महाजनपद नाम से 16 राज्यों का उल्लेख मिलता है। यद्यपि इन ग्रन्थों में महाजनपदों के सभी नाम एक जैसे नहीं हैं। लेकिन मगध, कोसल, वज्जि, कुरु, पांचाल, गांधार एवं अवन्ति जैसे नाम प्राय: एक समान देखने को मिलते हैं। इससे स्पष्ट होता है कि अपने समय में इन सभी महाजनपदों की गिनती महत्त्वपूर्ण महाजनपदों में होती होगी।

(2) अधिकांश महाजनपदों का शासन राजा द्वारा संचालित होता था, लेकिन गण एवं संघ के नाम से प्रसिद्ध राज्यों में कई लोगों का समूह शासन करता था। इस समूह का प्रत्येक व्यक्ति राजा कहलाता था। भगवान बुद्ध और भगवान महावीर इन्हीं गणों से सम्बन्ध रखते थे। हालांकि इन राज्यों के इतिहास स्रोतों के अभाव के कारण नहीं ‘लिखे जा सके हैं, लेकिन यह सम्भावना है कि ऐसे कई राज्य लगभग एक हजार वर्षों तक अस्तित्व में रहे होंगे।

(3) प्रत्येक महाजनपद अपनी एक राजधानी रखता था। इस राजधानी के चारों तरफ किले का निर्माण किया जाता था। इन किलेबन्द राजधानी शहर के रख-रखाव एवं सेवाओं व नौकरशाही के खर्चों के लिए बहुत बड़ी मात्रा में धन की आवश्यकता पड़ती थी।

(4) इस काल में शासकों का कार्य कृषकों, व्यापारियों एवं शिल्पकारों से कर एवं भेंट वसूलना माना जाता था। वनवासियों एवं चरवाहों से भी कर लिए जाने की कोई जानकारी प्राप्त नहीं हुई थी। इस काल में पड़ौसी राज्यों पर आक्रमण करके भी धन एकत्रित किया जाता था।

प्रश्न 14.
मौर्य साम्राज्य की प्रशासनिक व्यवस्थाओं पर प्रकाश डालिए।
उत्तर;
मौर्य साम्राज्य की प्रशासनिक व्यवस्थाओं की चर्चा हम निम्न बिन्दुओं के अन्तर्गत कर सकते हैं –
(1) केन्द्रीय शासन राजकीय सत्ता का प्रमुख केन्द्र तथा प्रशासन का सर्वोच्च सम्राट् होता था। सम्राट् के पास असीमित शक्तियाँ होती थीं। सम्राट् नियमों का निर्माता, सर्वोच्च न्यायाधीश, सेनानायक एवं मुख्य कार्यकारिणी का अध्यक्ष होता था।

(2) राजा के अधिकारियों के दायित्व – राजा ने राज्य कार्यों में सहायता एवं परामर्श हेतु मन्त्रिपरिषद् की स्थापना की थी। मन्त्रिपरिषद् में चरित्रवान तथा बुद्धिमान व्यक्तियों की नियुक्ति की जाती थी, परन्तु मन्त्रिपरिषद् का निर्णय राजा के लिए मानना अनिवार्य नहीं था कुछ अन्य अधिकारी भी होते थे, जैसे-अमात्य, महामात्य तथा अध्यक्ष आदि अमात्य मन्त्रियों के अधीन विभागों का काम संभालते थे।

(3) प्रान्तीय शासन संचालन की कुशल व्यवस्था हेतु मौर्य शासकों ने साम्राज्य को पाँच विभिन्न प्रान्तों में विभाजित किया था। प्रान्तीय शासन का प्रमुख राजपरिवार से सम्बन्धित होता था। उसका मुख्य कार्य प्रान्त में शान्ति एवं व्यवस्था बनाए रखना था।

(4) नगर की प्रशासनिक व्यवस्था उचित प्रशासनिक व्यवस्था हेतु नगर को कई भागों में बाँटा गया था। प्रत्येक नगर की अपनी नगरपालिका तथा न्यायपालिका थी। न्यायपालिका न्यायाधीश की सहायता हेतु न्यायाधिकारी न्य होते थे।

(5) ग्रामीण प्रशासनिक व्यवस्था-ग्राम प्रशासन की ह व्यवस्था, ग्राम पंचायतों के द्वारा की जाती थी। ग्राम का र मुखिया ‘ग्रामिक’ अथवा ‘ग्रामिणी’ कहलाता था। के

(6) दण्ड विधान दण्ड विधान काफी कठोर था। वहीं चोरी करने, डाका डालने एवं हत्या जैसे जघन्य अपराधों हेतु मृत्यु दण्ड का विधान था अपराधी का अंग-भंग भी ” किया जाता था। दीवानी तथा फौजदारी दो प्रकार के न्यायालय ता थे। सर्वोच्च न्यायाधीश स्वयं राजा होता था।

(7) सेना प्रणाली मौर्य शासकों ने शक्तिशाली सेना एवं का गठन किया था चन्द्रगुप्त मौर्य की सेना में 6 लाख पैदल, 30,000 घुड़सवार तथा 9 हजार हाथी और 8 हजार
रथ थे।

(8) समाज, कला, शिक्षा तथा आर्थिक स्थिति- चा। कृषि लोगों का मुख्य व्यवसाय था। कृषि आधारित पशुपालन, शिल्प तथा उद्योग काफी उन्नत अवस्था में थे। साहित्य और शिक्षा का व्यापक प्रचार-प्रसार था अशोक पहला सम्राट् था जिसके बनवाये स्तम्भ शिल्पकला के उदाहरण हैं।

प्रश्न 15.
मौर्यकालीन इतिहास की जानकारी के प्रमुख स्रोतों का वर्णन कीजिए।
उत्तर:
लगभग 321 ई. पू. में चन्द्रगुप्त मौर्य ने मौर्य साम्राज्य की स्थापना की। मौर्य वंश के उदय के साथ ही भारत का इतिहास रोचक बन जाता है क्योंकि इस काल में इतिहासकारों को तमाम ऐसे ऐतिहासिक साक्ष्य और विवरण प्राप्त होते हैं। प्रमुख स्रोतों का वर्णन निम्न है –
(1) मैगस्थनीज द्वारा रचित इंडिका मेगस्थनीज सम्राट् चन्द्रगुप्त मौर्य के दरबार में 302 ई.पू. से 298 ई.पू. तक रहा। उसने अपनी पुस्तक इंडिका में भारत में देखे गए वृतान्त का प्रत्यक्षदर्शी विवरण लिखा है। मेगस्थनीज ने सम्राट् चन्द्रगुप्त की शासन व्यवस्था, सैनिक गतिविधियों का संचालन तथा मौर्यकालीन सामाजिक व्यवस्था के बारे में विस्तृत रूप से लिखा है। मेगस्थनीज द्वारा इंडिका में दिया गया वर्णन काफी हद तक प्रामाणिक माना जाता है।

JAC Class 12 History Important Questions Chapter 2 राजा, किसान और नगर : आरंभिक राज्य और अर्थव्यवस्थाएँ

(2) चाणक्य का अर्थशास्त्र- चाणक्य द्वारा रचित अर्थशास्त्र मौर्यकालीन इतिहास का अन्य प्रमुख स्रोत है। मौर्यकालीन सामाजिक, धार्मिक एवं आर्थिक दशाओं का अर्थशास्त्र में व्यापक वर्णन है।

(3) जैन, बौद्ध तथा पौराणिक ग्रन्थ जैन, बौद्ध तथा पौराणिक ग्रन्थों से भी मौर्यकालीन इतिहास की जानकारी प्राप्त होती है। जैन विद्वानों ने प्राकृत, संस्कृत, तमिल जैसी भाषाओं में साहित्य का सूजन किया है।

(4) पुरातात्त्विक प्रमाण पुरातात्त्विक प्रमाण भी मौर्यकालीन इतिहास पर प्रकाश डालते हैं। उत्खनन में प्राप्त मूर्तियाँ, राजप्रासाद, स्तम्भ, सिक्के आदि उस काल के इतिहास को जानने में सहायक सिद्ध हुए हैं।

(5) अभिलेख सम्राट् अशोक ने अपने शासन काल में अनेक अभिलेख उत्कीर्ण करवाए। अभिलेखों से मौर्यकालीन शासन व्यवस्था, नैतिक आदर्शों, धम्मपद के सिद्धान्तों तथा जनकल्याण के लिए किए गए कार्यों की व्यापक जानकारी प्राप्त होती है। जूनागढ़ के अभिलेख तथा अशोक द्वारा लिखवाये गये अभिलेख इतिहास के महत्त्वपूर्ण स्रोत हैं। मौर्यकालीन शासकों के सिक्के भी इस काल का इतिहास जानने में सहायक सिद्ध हुए।

प्रश्न 16.
राजधर्म के नवीन सिद्धान्तों की विवेचना कीजिए एवं दक्षिण के राजा तथा सरदारों, दक्षिणी राज्यों के उदय पर प्रकाश डालिए।
उत्तर:
मौर्य साम्राज्य उपमहाद्वीप के सभी भागों में नहीं फैल पाया। साम्राज्य की सीमा के अन्तर्गत भी प्रशासन का नियन्त्रण एक समान नहीं था दूसरी शताब्दी ई.पू. आते-आते राजधर्म के नवीन सिद्धान्त स्थापित होने शुरू हो गए थे। उपमहाद्वीप के दक्कन और उससे दक्षिण के क्षेत्रों में जिनमें आन्ध्र प्रदेश, केरल राज्य तथा तमिलनाडु के कुछ भाग शामिल थे जिसे संयुक्त रूप से तमिल कम कहा जाता था, में नयी सरदारियों का उदय हुआ। इन सरदारियों में चोल, चेर तथा पांड्य प्रमुख थीं तीसरी शताब्दी के आरम्भ में मध्य भारत तथा दक्षिण में एक नया वंश जिसे वाकाटक कहते थे, बहुत प्रसिद्ध था।

सरदार एक प्रभावशाली व्यक्ति होता था। प्रायः सरदार का पद वंशानुगत होता था। सरदारियों की राज्य व्यवस्था, साम्राज्यों की व्यवस्था से भिन्न होती थी उनकी व्यवस्था में कराधान नहीं था साम्राज्य व्यवस्था में लोगों को कर देना पड़ता था। सरदारी में साधारण रूप से कोई स्थायी सेना या अधिकारी नहीं होते थे सरदार का कार्य विशेष अनुष्ठानों का संचालन करना था तथा विवादों को सुलझाने में मध्यस्थता करना भी था।

कई सरदार और राजा लम्बी दूरी के व्यापार द्वारा राजस्व एकत्र करते थे। मध्य एशियाई मूल के इन शासकों के सामाजिक उद्गम के बारे में विशेष विवरण प्राप्त नहीं है, लेकिन सत्ता में आने के बाद इन्होंने राजधर्म में नवीन सिद्धान्त की स्थापना की और राजत्व के अधिकार को दैवीय अधिकार से जोड़ा। कुषाण शासकों ने अपने आपको देवतुल्य प्रस्तुत करने के लिए देवस्थानों पर अपनी विशालकाय मूर्तियाँ लगवाई।

चौथी शताब्दी ई. में गुप्त साम्राज्य सहित कई बड़े- बड़े साम्राज्य के साक्ष्य प्राप्त हुए। इस काल को इतिहास में विशेष स्थान प्राप्त है। इस काल में सामन्ती प्रथा का उदय हुआ। कई साम्राज्य सामन्तों पर निर्भर थे, वे अपने निर्वाह स्थानीय संसाधनों द्वारा करते थे, जिसमें भूमि पर नियन्त्रण भी शामिल था जो सामन्त शक्तिशाली होते थे वे राजा बन जाते थे और जो राजा दुर्बल होते थे वे बड़े शासकों के अधीन हो जाते थे। इस प्रकार हम देखते हैं कि मौर्य साम्राज्य के पतन के बाद राजधर्म के नवीन सिद्धान्तों का चलन प्रारम्भ हुआ। सरदार और सरदारी, सामन्त प्रथा, राजत्व का दैवीय अधिकार इन सिद्धान्तों के महत्त्वपूर्ण उदाहरण हैं।